12.01.2013 Views

PES Skill Sheets.book - Capital High School

PES Skill Sheets.book - Capital High School

PES Skill Sheets.book - Capital High School

SHOW MORE
SHOW LESS

Create successful ePaper yourself

Turn your PDF publications into a flip-book with our unique Google optimized e-Paper software.

Chapter and Section<br />

Chapter 1: Measurement<br />

1.1 Measurement • Lab Safety • Using Your<br />

Text<strong>book</strong><br />

1.2 Time and Distance • Measuring Length • Averaging • SQ3R Reading and<br />

Study Method<br />

1.3 Converting Units • Dimensional<br />

Analysis<br />

• Fractions Review<br />

• Significant Digits<br />

1.4 Graphing • Creating<br />

Scatterplots<br />

• What’s the Scale?<br />

Chapter 2: The Scientific Process<br />

2.1 Inquiry and the Scientific<br />

Method<br />

2.2 Experiments and<br />

Variables<br />

2.3 The Nature of Science<br />

and Technology<br />

• Recording<br />

Observations in the<br />

Lab<br />

• Lab Report Format<br />

• Using a<br />

Spreadsheet<br />

Physical, Earth, and Space Science <strong>Skill</strong> and Practice Worksheets<br />

• Study Notes<br />

• Science Vocabulary<br />

• SI Units<br />

• Scientific Notation<br />

• Stopwatch Math<br />

• Understanding<br />

Light Years<br />

• SI Unit Conversion<br />

Extra Practice<br />

• SI-English<br />

Conversions<br />

• Interpreting Graphs<br />

• Recognizing<br />

Patterns on Graphs<br />

• Scientific<br />

Processes<br />

• What’s Your<br />

Hypothesis?<br />

• Identifying Control<br />

and Experimental<br />

Variables<br />

• Indirect<br />

Measurement<br />

i


Chapter and Section<br />

Chapter 3: Mapping<br />

3.1 Maps • Position on the<br />

Coordinate Plane<br />

ii<br />

Physical, Earth, and Space Science <strong>Skill</strong> and Practice Worksheets<br />

• Latitude and<br />

Longitude<br />

• Map Scales<br />

• Vectors on a Map<br />

3.2 Topographic Maps • Topographic Maps<br />

3.3 Bathymetric Maps • Bathymetric Maps • Tanya Atwater<br />

Chapter 4: Motion<br />

4.1 Speed and Velocity • Solving Equations<br />

With One Variable<br />

• Problem Solving<br />

Boxes<br />

• Problem Solving<br />

with Rates<br />

• Percent Error<br />

4.2 Graphs of Motion • Calculating Slope<br />

From a Graph<br />

• Speed<br />

• Velocity<br />

• Analyzing Graphs<br />

of Motion With<br />

Numbers<br />

• Analyzing Graphs<br />

of Motion Without<br />

Numbers<br />

4.3 Acceleration • Acceleration<br />

• Acceleration and<br />

Speed-Time<br />

Graphs<br />

• Navigation<br />

• Acceleration Due to<br />

Gravity


Chapter and Section<br />

Chapter 5: Force<br />

5.1 Forces • Ratios and<br />

Proportions<br />

Physical, Earth, and Space Science <strong>Skill</strong> and Practice Worksheets<br />

• Internet Research<br />

• Bibliographies<br />

5.2 Friction • Friction<br />

• Mass vs. Weight<br />

• Mass, Weight, and<br />

Gravity<br />

5.3 Forces in Equilibrium • Equilibrium<br />

Chapter 6: Laws of Motion<br />

6.1 Newton’s First Law • Net Force and<br />

Newton’s First Law<br />

6.2 Newton’s Second Law • Newton’s Second<br />

Law<br />

6.3 Newton’s Third Law and<br />

Momentum<br />

• Applying Newton’s<br />

Laws<br />

• Momentum<br />

• Momentum<br />

Conservation<br />

• Collisions and<br />

Conservation of<br />

Momentum<br />

• Isaac Newton<br />

• Gravity Problems<br />

• Universal<br />

Gravitation<br />

• Rate of Change of<br />

Momentum<br />

iii


Chapter and Section<br />

Chapter 7: Work and Energy<br />

7.1 Force, Work, and<br />

Machines<br />

7.2 Energy and the<br />

Conservation of Energy<br />

iv<br />

Physical, Earth, and Space Science <strong>Skill</strong> and Practice Worksheets<br />

• Mechanical<br />

Advantage<br />

• Mechanical<br />

Advantage of<br />

Simple Machines<br />

•Work<br />

• Types of Levers<br />

• Potential and<br />

Kinetic Energy<br />

• Identifying Energy<br />

Transformations<br />

• Energy<br />

Transformations<br />

Extra Practice<br />

• Conservation of<br />

Energy<br />

7.3 Efficiency and Power • Efficiency<br />

• Power<br />

Chapter 8: Matter and Temperature<br />

8.1 The Nature of Matter<br />

8.2 Temperature • Measuring<br />

Temperature<br />

• Temperature<br />

Scales<br />

8.3 Phases of Matter • Reading a Heating/<br />

Cooling Curve<br />

• James Joule<br />

• Gear Ratios<br />

• Levers in the<br />

Human Body<br />

• Bicycle Gear Ratios<br />

Project<br />

• Power in Flowing<br />

Energy<br />

• Efficiency and<br />

Energy


Chapter and Section<br />

Chapter 9: Heat<br />

9.1 Heat and Thermal<br />

Energy<br />

Physical, Earth, and Space Science <strong>Skill</strong> and Practice Worksheets<br />

• Specific Heat<br />

Using the Heat<br />

Equation<br />

9.2 Heat Transfer Heat Transfer<br />

Chapter 10: Properties of Matter<br />

10.1 Density Measuring Mass<br />

with a Triple Beam<br />

Balance<br />

Measuring Volume<br />

Calculating Volume<br />

10.2 Properties of Solids<br />

Density<br />

10.3 Properties of Fluids Pressure in fluids<br />

Boyle’s Law<br />

10.4 Buoyancy Buoyancy<br />

Charles’ Law<br />

Pressure-<br />

Temperature<br />

Relationship<br />

Archimedes<br />

Narcis Monturiol<br />

Archimedes’<br />

Principle<br />

v


Chapter and Section<br />

Chapter 11: Weather and Climate<br />

11.1 Earth’s Atmosphere • Layers of the<br />

Atmosphere<br />

vi<br />

Physical, Earth, and Space Science <strong>Skill</strong> and Practice Worksheets<br />

11.2 Weather Variables • Gaspard-Gustave<br />

Coriolis<br />

• Degree Days<br />

11.3 Weather Patterns • Joanne Simpson • Weather Maps<br />

Chapter 12: Atoms and the Periodic Table<br />

12.1 Atomic Structure • Structure of the<br />

Atom<br />

• Atoms and Isotopes<br />

12.2 Electrons • Electrons and<br />

Energy Levels<br />

12.3 The Periodic Table of<br />

the Elements<br />

12.4 Properties of the<br />

Elements<br />

Chapter 13: Compounds<br />

13.1 Chemical Bonds and<br />

Electrons<br />

13.2 Chemical Formulas • Finding the Least<br />

Common Multiple<br />

13.3 Molecules and Carbon<br />

Compounds<br />

• The Periodic Table<br />

• Dot Diagrams<br />

• Chemical Formulas<br />

• Naming<br />

Compounds<br />

• Ernest Rutherford<br />

• Neils Bohr<br />

• Tracking A<br />

Hurricane<br />

• Families of<br />

Compounds


Chapter and Section<br />

Chapter 14: Changes in Matter<br />

14.1 Chemical Reactions • Chemical<br />

Equations<br />

14.2 Types of Reactions • Classifying<br />

Reactions<br />

14.3 Energy in Chemical<br />

Reactions<br />

Physical, Earth, and Space Science <strong>Skill</strong> and Practice Worksheets<br />

14.4 Nuclear Reactions • Lise Meitner<br />

• Marie and Pierre<br />

Curie<br />

• Rosalyn Yalow<br />

• Chien-Shiung Wu<br />

Chapter 15: Matter and Earth’s Resources<br />

15.1 Chemical Cycles<br />

15.2 Global Climate Change • Svante Arrhenius<br />

• The Avogadro<br />

Number<br />

• Formula Mass<br />

• Predicting<br />

Chemical<br />

Equations<br />

• Percent Yield<br />

• Radioactivity<br />

vii


Chapter and Section<br />

Chapter 16: Electricity<br />

16.1 Charge and Electric<br />

Circuits<br />

viii<br />

Physical, Earth, and Space Science <strong>Skill</strong> and Practice Worksheets<br />

• Open and Closed<br />

Circuits<br />

16.2 Current and Voltage • Using an Electric<br />

Meter<br />

16.3 Resistance and Ohm’s<br />

Laws<br />

• Voltage, Current,<br />

and Resistance<br />

•Ohm’s law<br />

16.4 Types of Circuits •Series Circuits<br />

• Parallel Circuits<br />

Chapter 17: Magnetism<br />

17.1 Properties of Magnets • Magnetic Earth<br />

• Benjamin Franklin<br />

• Thomas Edison<br />

• George<br />

Westinghouse<br />

• Lewis Latimer<br />

17.2 Electromagnets • Maglev Train Model<br />

Project<br />

17.3 Electric Motors and<br />

Generators<br />

17.4 Generating Electricity • Michael Faraday • Transformers<br />

• Electrical Power


Chapter and Section<br />

Chapter 18: Earth’s History and Rocks<br />

Physical, Earth, and Space Science <strong>Skill</strong> and Practice Worksheets<br />

18.1 Geologic Time • Andrew Douglass<br />

18.2 Relative Dating • Relative Dating • Nicolas Steno<br />

18.3 The Rock Cycle • The Rock Cycle<br />

Chapter 19: Matter and Earth<br />

19.1 Inside Earth • Earth’s Interior • Charles Richter<br />

• Jules Verne<br />

19.2 Plate Tectonics • Alfred Wegener<br />

•Harry Hess<br />

• John Tuzo Wilson<br />

19.3 Plate Boundaries • Earth’s Largest<br />

Plates<br />

19.4 Metamorphic Rocks • Continental U.S.<br />

Geology<br />

Chapter 20: Earthquakes and Volcanoes<br />

20.1 Earthquakes • Averaging • Finding an<br />

Earthquake<br />

Epicenter<br />

20.2 Volcanoes • Volcano Parts<br />

20.3 Igneous Rocks • Basalt and Granite<br />

ix


Chapter and Section<br />

Chapter 21: Water and Solutions<br />

21.1 Water<br />

21.2 Solutions • Concentration of<br />

Solutions<br />

• Solubility<br />

• Salinity and<br />

Concentration<br />

Problems<br />

x<br />

Physical, Earth, and Space Science <strong>Skill</strong> and Practice Worksheets<br />

21.3 Acids, Bases, and pH • Calculating pH<br />

Chapter 22: Earth’s Water Systems<br />

22.1 Water on Earth’s<br />

Surface<br />

22.2 The Water Cycle • The Water Cycle<br />

22.3 Oceans<br />

Chapter 23: How Water Shapes the Land<br />

23.1 Weathering and<br />

Erosion<br />

23.2 Rivers, Streams, and<br />

Sedimentation<br />

23.3 Glaciers<br />

23.4 Sedimentary Rocks<br />

• Groundwater and<br />

Wells Project


Chapter and Section<br />

Chapter 24: Waves and Sound<br />

24.1 Harmonic Motion • Period and<br />

Frequency<br />

Physical, Earth, and Space Science <strong>Skill</strong> and Practice Worksheets<br />

• Harmonic Motion<br />

Graphs<br />

24.2 Properties of Waves • Waves • Wave Interference<br />

24.3 Sound • Decibel Scale<br />

• The Human Ear<br />

Chapter 25: Light and the Electromagnetic Spectrum<br />

25.1 Properties of Light • The<br />

Electromagnetic<br />

Spectrum<br />

• Standing Waves<br />

• Waves and Energy<br />

• Palm Pipes Project<br />

25.2 Color and Vision • Color Mixing • The Human Eye<br />

25.3 Optics • Measuring Angles • Using Ray<br />

Diagrams<br />

• Reflection<br />

• Refraction<br />

• Drawing Ray<br />

Diagrams<br />

xi


Chapter and Section<br />

Chapter 26: The Solar System<br />

26.1 Motion in the Solar<br />

System<br />

26.2 Motion and<br />

Astronomical Cycles<br />

26.3 Objects in the Solar<br />

System<br />

Chapter 27: Stars<br />

xii<br />

Physical, Earth, and Space Science <strong>Skill</strong> and Practice Worksheets<br />

• Astronomical Units<br />

Gravity Problems<br />

Universal<br />

Gravitation<br />

Touring the Solar<br />

System<br />

27.1 The Sun The Sun:<br />

A Cross-Section<br />

27.2 Stars Inverse Square<br />

Law<br />

27.3 Life Cycles of Stars<br />

Chapter 28: Exploring the Universe<br />

28.1 Tools of Astronomers Scientific Notation Understanding<br />

Light Years<br />

Parsecs<br />

Nicolaus<br />

Copernicus<br />

Galileo Galilei<br />

Johannes Kepler<br />

Benjamin Banneker<br />

Arthur Walker<br />

Edwin Hubble<br />

Measuring the<br />

Moon’s Diameter<br />

28.2 Galaxies Light Intensity Henrietta Leavitt Calculating<br />

Luminosity<br />

28.3 Theories About the<br />

Universe<br />

Doppler Shift


Name: Date:<br />

1.1 Lab Safety<br />

What can I do to protect myself and others in the lab?<br />

Science equipment and supplies are fun to use.<br />

However, these materials must always be used<br />

with care. Here you will learn how to be safe in<br />

a science lab.<br />

Follow these basic safety guidelines<br />

Your teacher will divide the class into groups. Each group should create a poster-sized display of one of the<br />

following guidelines. Hang the posters in the lab. Review these safety guidelines before each investigation.<br />

1. Prepare for each investigation.<br />

a. Read the investigation sheets carefully.<br />

b. Take special note of safety instructions.<br />

2. Listen to your teacher’s instructions before, during, and after the investigation. Take notes to help you<br />

remember what your teacher has said.<br />

3. Get ready to work: Roll long sleeves above the wrist. Tie back long hair. Remove dangling jewelry and any<br />

loose, bulky outer layers of clothing. Wear shoes that cover the toes.<br />

4. Gather protective clothing (goggles, apron, gloves) at the beginning of the investigation.<br />

5. Emphasize teamwork. Help each other. Watch out for one another’s safety.<br />

6. Clean up spills immediately. Clean up all materials and supplies after an investigation.<br />

Know what to do when...<br />

7. working with heat:<br />

a. Always handle hot items with a hot pad. Never use your bare hands.<br />

b. Move carefully when you are near hot items. Sudden movements could cause burns if you touch<br />

or spill something hot.<br />

c. Inform others if they are near hot items or liquids.<br />

8. working with electricity:<br />

Materials<br />

• Poster board<br />

• Felt-tip markers<br />

a. Always keep electric cords away from water.<br />

b. Extension cords must not be placed where they may cause someone to trip or fall.<br />

c. If an electrical appliance isn’t working, feels hot, or smells hot, tell a teacher right away.<br />

9. disposing of materials and supplies:<br />

a. Generally, liquid household chemicals can be poured into a sink. Completely wash the chemical<br />

down the drain with plenty of water.<br />

b. Generally, solid household chemicals can be placed in a trash can.


Page 2 of 6<br />

c. Any liquids or solids that should not be poured down the sink or placed in the trash have<br />

special disposal guidelines. Follow your teacher’s instructions.<br />

d. If glass breaks, do not use your bare hands to pick up the pieces. Use a dustpan and a brush to<br />

clean up. “Sharps” trash (trash that has pieces of glass) should be well labeled. The best way to<br />

throw away broken glass is to seal it in a labeled cardboard box.<br />

10. you are concerned about your safety or the safety of others:<br />

a. Talk to your teacher immediately. Here are some examples:<br />

• You smell chemical or gas fumes. This might indicate a chemical or gas leak.<br />

• You smell something burning.<br />

• You injure yourself or see someone else who is injured.<br />

• You are having trouble using your equipment.<br />

• You do not understand the instructions for the investigation.<br />

b. Listen carefully to your teacher’s instructions.<br />

c. Follow your teacher’s instructions exactly.


Page 3 of 6<br />

Safety quiz<br />

1. Draw a diagram of your science lab in the space below. Include in your diagram the following items. Include<br />

notes that explain how to use these important safety items.<br />

• Exit/entrance ways • Eye wash and shower • Sink<br />

• Fire extinguisher(s) • First aid kit • Trash cans<br />

• Fire blanket • Location of eye goggles • Location of<br />

and lab aprons<br />

special safety instructions<br />

2. How many fire extinguishers are in your science lab? Explain how to use them.<br />

3. List the steps that your teacher and your class would take to safely exit the science lab and the building in<br />

case of a fire or other emergency.


Page 4 of 6<br />

4. Before beginning certain investigations, why should you first put on protective goggles and<br />

clothing?<br />

5. Why is teamwork important when you are working in a science lab?<br />

6. Why should you clean up after every investigation?<br />

7. List at least three things you should you do if you sense danger or see an emergency in your classroom or<br />

lab.<br />

8. Five lab situations are described below. What would you do in each situation?<br />

a. You accidentally knock over a glass container and it breaks on the floor.<br />

b. You accidentally spill a large amount of water on the floor.


Page 5 of 6<br />

c. You suddenly you begin to smell a “chemical” odor that gives you a headache.<br />

d. You hear the fire alarm while you are working in the lab. You are wearing your goggles and lab apron.<br />

e. While your lab partner has her lab goggles off, she gets some liquid from the experiment in her eye.<br />

f. A fire starts in the lab.<br />

Safety in the science lab is everyone’s responsibility!


Page 6 of 6<br />

Safety contract<br />

Keep this contract in your note<strong>book</strong> at all times.<br />

By signing it, you agree to follow all the steps necessary to be safe in your science class and lab.<br />

I, ____________________, (Your name)<br />

• Have learned about the use and location of the following:<br />

• Aprons and gloves<br />

• Eye protection<br />

• Eyewash fountain<br />

• Fire extinguisher and fire blanket<br />

• First aid kit<br />

• Heat sources (burners, hot plate, etc) and how to use them safely<br />

• Waste-disposal containers for glass, chemicals, matches, paper, and wood<br />

• Understand the safety information presented.<br />

• Will ask questions when I do not understand safety instructions.<br />

• Pledge to follow all of the safety guidelines that are presented on the Safety <strong>Skill</strong> Sheet at all times.<br />

• Pledge to follow all of the safety guidelines that are presented on investigation sheets.<br />

• Will always follow the safety instructions that my teacher provides.<br />

Additionally, I pledge to be careful about my own safety and to help others be safe. I understand that I am<br />

responsible for helping to create a safe environment in the classroom and lab.<br />

Signed and dated,<br />

______________________________<br />

Parent’s or Guardian’s statement:<br />

I have read the Safety <strong>Skill</strong>s sheet and give my consent for the student who has signed the preceding statement to<br />

engage in laboratory activities using a variety of equipment and materials, including those described. I pledge my<br />

cooperation in urging that she or he observe the safety regulations prescribed.<br />

____________________________________________________ _________________________________<br />

Signature of Parent or Guardian Date


Name: Date:<br />

1.1 Using Your Text<strong>book</strong><br />

1.1<br />

Your text<strong>book</strong> is a tool to help you understand and enjoy science. Colors, shapes, and symbols are used<br />

in the <strong>book</strong> to help you find information quickly. Take a few minutes to get familiar with these features—it will<br />

help you get the most out of your <strong>book</strong> all year long.<br />

Part 1: Organizing features of the student text<br />

Spend a few minutes answering the questions below. You will learn to recognize visual clues that organize the<br />

reading and help you find information quickly.<br />

1. What color is used to identify Unit four?<br />

2. List four important vocabulary words for section 3.2.<br />

3. What color are the boxes in which you found these vocabulary words?<br />

4. What is the main idea of the last paragraph on page 37?<br />

5. Where do you find section review questions?<br />

6. What is the first key question for Chapter 9?<br />

7. What are the four numbered parts (or steps) shown in each sample problem in the text?<br />

8. List the four sections of questions in each Chapter Assessment.<br />

Part 2: The Table of Contents<br />

The Table of Contents is found after the introduction pages. Use it to answer the following questions.<br />

1. How many units are in the text<strong>book</strong>? List their titles.<br />

2. Which unit will be the most interesting to you? Why?<br />

3. Where do you find the glossary and index? How are they different?<br />

Part 3: Glossary and Index<br />

The glossary and index can help you quickly find information in your text<strong>book</strong>. Use these tools to answer the<br />

following questions.<br />

1. What is the definition of velocity?<br />

2. On what pages will you find information about the layer of Earth’s atmosphere known as the troposphere?<br />

3. On what page will you find a short biography of agricultural scientist George Washington Carver?


Name: Date:<br />

1.1 SI Units<br />

In the late 1700's, as scientists began to develop the ideas of physics and chemistry, they needed better units of<br />

measurement to communicate scientific data. Scientists needed to prove their ideas with data based on<br />

measurements that other scientists could reproduce. A decimal system of units based on the meter as a standard<br />

length, the kilogram as a standard mass, and the liter as a standard volume was developed by the French. Today<br />

this system is known as the SI system, or metric system.<br />

The equations below show how the meter is related to other units in this system of measurements.<br />

1 meter = 100 centimeters<br />

1 cubic centimeter = 1 cm 3 = 1 milliliter<br />

1000 milliliters = 1 liter<br />

The SI system is easy to use because all the units are based on factors of 10. In<br />

the English system, there are 12 inches in a foot, 3 feet in a yard, and 5,280 feet<br />

in a mile. In the SI system, there are 10 millimeters in a centimeter,<br />

100 centimeters in a meter, and 1,000 meters in a kilometer.<br />

Question: Using the graphic at right, state how many kilometers it is from the<br />

North Pole to the equator.<br />

Answer: You need to convert 10,000,000 meters to kilometers.<br />

Since 1 meter = 0.001 kilometers, 0.001 is the multiplication factor. To solve,<br />

multiply 10,000,000 0.001 km = 10,000 km. So, it is 10,000 kilometers from<br />

the North Pole to the equator.<br />

These are the standard units of measurement that you will use in your scientific studies. The prefixes on the<br />

following page are used with the base units when measuring very large or very small quantities.<br />

When you are measuring: Use this standard unit: Symbol of unit<br />

mass kilogram kg<br />

length meter m<br />

volume liter l<br />

force newton N<br />

temperature degree Celsius °C<br />

time second s<br />

You may wonder why the kilogram, rather than the gram, is called the standard unit for mass. This is because the<br />

mass of an object is based on how much matter it contains as compared to the standard kilogram made from<br />

platinum and iridium and kept in Paris. The gram is such a small amount of matter that if it had been used as a<br />

standard, small errors in reproducing that standard would be multiplied into very large errors when large<br />

quantities of mass were measured.<br />

1.1


Page 2 of 3<br />

The following prefixes in the SI system indicate the multiplication factor to be used with the basic unit.<br />

For example, the prefix kilo- is a factor of 1,000. A kilometer is equal to 1,000 meters, and a kilogram is<br />

equal to 1,000 grams.<br />

Prefix kilo- hecto- deka- Basic unit<br />

(no prefix)<br />

deci- centi- milli-<br />

Symbol k h da m, l, g d c m<br />

Multiplication Factor<br />

or Place-Value<br />

1. How many centigrams are there in 24 grams?<br />

a. Restate the question: 24 grams = __________centigrams<br />

b. Use the place value chart to determine the multiplication factor, and solve:<br />

Since we want to convert grams (ones place) to centigrams<br />

(hundredths place), count the number of places on the chart<br />

it takes to move from the ones place to get to the hundredths<br />

place. Since it takes 2 moves to the right, the multiplication<br />

factor is 100.<br />

Solution: multiply 24 × 100 = 2,400.<br />

Answer: There are 2,400 centigrams in 24 grams.<br />

2. How many liters are there in 5,000 deciliters?<br />

a. Restate the question: 5,000 deciliters (dl) = __________ liters (l)?<br />

b. Use the place value chart to determine the multiplication factor, and solve:<br />

Since we want to convert deciliters (tenths place) to liters (ones<br />

place), count the number of places on the chart it takes to move<br />

from the ones place to get to the hundredths place. Since it<br />

takes 1 move to the left, the multiplication factor is 0.1.<br />

Solution: multiply 5,000 × 0.1 = 500.<br />

Answer: There are 500 liters in 5,000 deciliters.<br />

3. How many decimeters are in a dekameter?<br />

1,000 100 10 1 0.1 0.01 0.001<br />

kilo hecto deka meter, liter, or gram deci centi milli<br />

thousands hundreds tens ones tenths hundredths thousandths<br />

a. Restate the question: 1 dam =__________dm.<br />

b. Use the place value chart to determine the multiplication factor, and solve:<br />

1.1


Page 3 of 3<br />

Since we want to convert dekameters to decimeters, count<br />

the number of places on the chart it takes to move from the<br />

tens place (deka) to the tenths place (deci). It takes 2<br />

moves to the right, so the multiplication factor is 100.<br />

Solution: multiply 1 × 100 = 100.<br />

Answer: There are 100 decimeters in one dekameter.<br />

4. How many kilograms are equivalent to 520,000 centigrams?<br />

(1) Restate the question: 520,000 centigrams = __________ kilograms.<br />

(2) Determine the multiplication factor, and solve:<br />

Moving from the hundredths place (centi) to<br />

the thousands place (kilo) requires moving 5<br />

places to the left, so the multiplication factor is<br />

0.00001.<br />

Solution: Multiply 520,000 × 0.00001 = 5.2<br />

Answer: 5.2 kilograms are equivalent to 520,000 centigrams.<br />

1. How many grams are in a dekagram?<br />

2. How many millimeters are there in one meter?<br />

3. How many millimeters are in 6 decimeters?<br />

4. Convert 4,200 decigrams to grams.<br />

5. How many liters are equivalent to 500 centiliters?<br />

6. Convert 100 millimeters to meters.<br />

7. How many milligrams are equivalent to 150 dekagrams?<br />

8. How many liters are equivalent to 0.3 kiloliters?<br />

9. How many centimeters are in 65 kilometers?<br />

10. Twelve dekagrams are equivalent to how many milligrams?<br />

11. Seven hundred twenty centiliters is how many liters?<br />

12. A fountain can hold 53,000 deciliters of water. How many kiloliters is this?<br />

13. What is the name of a length that is 100 times larger than a millimeter?<br />

14. How many times larger than a centigram is a dekagram?<br />

15. Name the distance that is 10 times smaller than a centimeter.<br />

1.1


Name: Date:<br />

1.1 Scientific Notation<br />

1.1<br />

A number like 43,200,000,000,000,000,000 (43 quintillion, 200 quadrillion) can take a long time to<br />

write, and an even longer time to read. Because scientists frequently encounter very large numbers like this one<br />

(and also very small numbers, such as 0.000000012, or twelve trillionths), they developed a shorthand method<br />

for writing these types of numbers. This method is called scientific notation. A number is written in scientific<br />

notation when it is written as the product of two factors, where the first factor is a number that is greater than or<br />

equal to 1, but less than 10, and the second factor is an integer power of 10. Some examples of numbers written<br />

in scientific notation are given in the table below:<br />

Scientific Notation Standard Form<br />

4.32 × 10 19 43,200,000,000,000,000,000<br />

1.2 × 10 –8 0.000000012<br />

5.2777 × 10 7 52,777,000<br />

6.99 10 -5 0.0000699<br />

Rewrite numbers given in scientific notation in standard form.<br />

• Express 4.25 × 10 6 in standard form: 4.25 × 10 6 = 4,250,000<br />

Move the decimal point (in 4.25) six places to the right. The exponent of the “10” is 6, giving us the number<br />

of places to move the decimal. We know to move it to the right since the exponent is a positive number.<br />

• Express 4.033 × 10 –3 in standard form: 4.033 × 10 –3 = 0.004033<br />

Move the decimal point (in 4.033) three places to the left. The exponent of the “10” is negative 3, giving the<br />

number of places to move the decimal. We know to move it to the left since the exponent is negative.<br />

Rewrite numbers given in standard form in scientific notation.<br />

• Express 26,040,000,000 in scientific notation: 26,040,000,000 = 2.604 × 10 10<br />

Place the decimal point in 2 6 0 4 so that the number is greater than or equal to one (but less than ten). This<br />

gives the first factor (2.604). To get from 2.604 to 26,040,000,000 the decimal point has to move 10 places to<br />

the right, so the power of ten is positive 10.<br />

• Express 0.0001009 in scientific notation: 0.0001009 = 1.009 × 10 –4<br />

Place the decimal point in 1 0 0 9 so that the number is greater than or equal to one (but less than ten). This<br />

gives the first factor (1.009). To get from 1.009 to 0.0001009 the decimal point has to move four places to<br />

the left, so the power of ten is negative 4.


Page 2 of 3<br />

1. Fill in the missing numbers. Some will require converting scientific notation to standard form,<br />

while others will require converting standard form to scientific notation.<br />

a. 6.03 × 10 –2<br />

b. 9.11 × 10 5<br />

c. 5.570 × 10 –7<br />

Scientific Notation Standard Form<br />

d. 999.0<br />

e. 264,000<br />

f. 761,000,000<br />

g. 7.13 × 10 7<br />

h. 0.00320<br />

i. 0.000040<br />

j. 1.2 × 10 –12<br />

k. 42,000,000,000,000<br />

l. 12,004,000,000<br />

m. 9.906 × 10 –2<br />

2. Explain why the numbers below are not written in scientific notation, then give the correct way to write the<br />

number in scientific notation.<br />

Example: 0.06 × 10 5 is not written in scientific notation because the first factor (0.06) is not greater than or<br />

equal to 1. The correct way to write this number in scientific notation is 6.0×103 .<br />

a. 2.004 × 111 b. 56 × 10 –4<br />

c. 2 × 100 2<br />

d. 10 × 10 –6<br />

1.1


Page 3 of 3<br />

3. Write the numbers in the following statements in scientific notation:<br />

a. The national debt in 2005 was about $7,935,000,000,000.<br />

b. In 2005, the U.S. population was about 297,000,000<br />

c. Earth's crust contains approximately 120 trillion (120,000,000,000,000) metric tons of gold.<br />

d. The mass of an electron is 0.000 000 000 000 000 000 000 000 000 000 91 kilograms.<br />

e. The usual growth of hair is 0.00033 meters per day.<br />

f. The population of Iraq in 2005 was approximately 26,000,000.<br />

g. The population of California in 2005 was approximately 33,900,000.<br />

h. The approximate area of California is 164,000 square miles.<br />

i. The approximate area of Iraq in 2005 was 169,000 square miles.<br />

j. In 2005, one right-fielder made a salary of $12,500,000 playing professional baseball.<br />

1.1


Name: Date:<br />

1.2 Measuring Length<br />

How do you find the length of an object?<br />

Size matters! When you describe the length of an<br />

object, or the distance between two objects, you<br />

are describing something very important about the<br />

object. Is it as small as a bacteria (2 micrometers)?<br />

Is it a light year away (9.46 × 10 15 meters)? By<br />

using the metric system you can quickly see the<br />

difference in size between objects.<br />

Reading the meter scale correctly<br />

Look at the ruler in the picture above. Each small line on the top of the ruler represents one millimeter. Larger<br />

lines stand for 5 millimeter and 10 millimeter intervals. When the object you are measuring falls between the<br />

lines, read the number to the nearest 0.5 millimeter. Practice measuring several objects with your own metric<br />

ruler. Compare your results with a lab partner.<br />

Stop and think<br />

Materials<br />

• Metric ruler<br />

• Pencil<br />

• Paper<br />

• Small objects<br />

• Calculator<br />

a. You may have seen a ruler like this marked in centimeter units. How many millimeters are in one<br />

centimeter?<br />

b. Notice that the ruler also has markings for reading the English system. Give an example of when it would<br />

be better to measure with the English system than the metric system. Give a different example of when it<br />

would be better to use the metric system.


Page 2 of 4<br />

Example 1: Measuring objects correctly<br />

Look at the picture above. How long is the building block?<br />

1. Report the length of the building block to the nearest 0.5 millimeters.<br />

2. Convert your answer to centimeters.<br />

3. Convert your answer to meters.<br />

Example 2: Measuring objects correctly<br />

Look at the picture above. How long is the pencil?<br />

1. Report the length of the pencil to the nearest 0.5 millimeters.<br />

2. Challenge: How many building blocks in example 1 will it take to equal the length of the pencil?<br />

3. Challenge: Convert the length of the pencil to inches by dividing your answer by 25.4 millimeters per inch.


Page 3 of 4<br />

Example 3: Measuring objects correctly<br />

Look at the picture above. How long is the domino?<br />

1. Report the length of the domino to the nearest 0.5 millimeters.<br />

2. Challenge: How many dominoes will fit end to end on the 30 cm ruler?<br />

Practice converting units for length<br />

By completing the examples above you show that you are familiar with some of the prefixes used in the metric<br />

system like milli- and centi-. The table on the following page gives other prefixes you may be less familiar with.<br />

Try converting the length of the domino from millimeters into all the other units given in the table.<br />

Refer to the multiplication factor this way:<br />

• 1 kilometer equals 1000 meters.<br />

• 1000 millimeters equals 1 meter.<br />

1. How many millimeters are in a kilometer?<br />

1000 millimeters per meter × 1000 meters per kilometer = 1,000,000 millimeters per kilometer<br />

2. Fill in the table with your multiplication factor by converting millimeters to the unit given. The first one is<br />

done for you.<br />

1000 millimeters per meter × 10 –12 meters per picometer = 10 –9 millimeters per picometer<br />

3. Divide the domino’s length in millimeters by the number in your multiplication factor column. This is the<br />

answer you will put in the last column.<br />

42.5 millimeters ÷ 10 –9 millimeters per picometer = 42.5 ×10 9 picometers


Page 4 of 4<br />

4. .<br />

Prefix Symbol Multiplication<br />

factor<br />

Scientific notation<br />

in meters<br />

Your multiplication<br />

factor<br />

Your domino<br />

length in:<br />

pico- p 0.000000000001 10 –12 10 –9 42.5 ×10 9 pm<br />

nano- n 0.000000001 10 –9 nm<br />

micro- µ 0.000001 10 –6 μm<br />

milli m 0.001 10 –3<br />

centi- c 0.01 10 –2<br />

deci- d 0.1 10 –1<br />

deka- da 10 10 1<br />

hecto- h 100 10 2<br />

kilo- k 1000 10 3<br />

mm<br />

cm<br />

dm<br />

dam<br />

hm<br />

km


Name: Date:<br />

1.2 Averaging<br />

The most common type of average is called the mean. Usually when someone (who’s not your math teacher) asks<br />

you to find the average of something, it is the mean that they want. To find the mean, just sum (add) all the data,<br />

then divide the total by the number of items in the data set. This type of average is used daily by many people.<br />

Teachers and students use it to average grades. Meteorologists use it to average normal high and low<br />

temperatures for a certain date. Sports statisticians use it to calculate batting averages and many other things.<br />

• William has had three tests so far in his English class. His grades are 80%, 75%, and 90%. What is his<br />

average test grade?<br />

Solution:<br />

a. Find the sum of the data: 80 + 75 + 90 = 245<br />

b. Divide the sum (245) by the number of items in the data set (3): 245 ≥ 3 ≈ 82%<br />

William’s average (mean) test grade in English (so far) is about 82%<br />

1. The families on Carvel Street were cleaning out their basements and garages to prepare for their annual<br />

garage sale. At 202 Carvel Street, they found seven old baseball gloves. At 208, they found two baseball<br />

gloves. At 214, they found four gloves, and at 221 they found two gloves. If these are the only houses on the<br />

street, what is the average number of old baseball gloves found at a house on Carvel Street?<br />

2. During a holiday gift exchange, the members of the winter play cast set a limit of $10 per gift. The actual<br />

prices of each gift purchased were: $8.50, $10.29, $4.45, $12.79, $6.99, $9.29, $5.97, and $8.33. What was<br />

the average price of the gifts?<br />

3. During weekend baby sitting jobs, each sitter charged a different hourly rate. Rachel charged $4.00, Juanita<br />

charged $3.50, Michael charged $4.25, Rosa charged $5.00, and Smith charged $3.00.<br />

a. What was the average hourly rate charged among these baby sitters?<br />

b. If they each worked a total of eight hours, what was their average pay for the weekend?<br />

4. The boys on the ninth grade basketball team at Fillmore <strong>High</strong> <strong>School</strong> scored 22 points, 12 points, 8 points, 4<br />

points, 4 points, 3 points, 2 points, 2 points, and 1 point in Thursday’s game. What was the average number<br />

of points scored by each player in the game?<br />

5. Jerry and his friends were eating pizza together on a Friday night. Jerry ate a whole pizza (12 slices) by<br />

himself! Pat ate three slices, Jack ate seven slices, Don and Dave ate four slices each, and Teri ate just two<br />

slices. What was the average number of slices of pizza eaten by one of these friends that night?<br />

1.2


Name: Date:<br />

1.2 Reading Strategies (SQ3R)<br />

Students often read a science text<strong>book</strong> as if they were watching a movie—they just sit there and expect to take it<br />

all in. Actually, reading a science <strong>book</strong> is more like playing a video game. You have to interact with it! This skill<br />

builder will teach you active strategies that will improve your reading and study skills. Remember—just like in<br />

video game playing—the more you practice these strategies, the more skilled you will become.<br />

The SQ3R active reading method was developed in 1941 by Francis Robinson to help his students get the most<br />

out of their text<strong>book</strong>s. Using the SQ3R method will help you interact with your text, so that you understand and<br />

remember what you read. “SQ3R” stands for:<br />

Survey<br />

Question<br />

Read<br />

Recite<br />

Review<br />

Your student text has many features to help you organize your reading. These features are highlighted in<br />

Chapter 1: Measurement, found on pages 3–32 of your student text. Open your text to those pages so that you<br />

can see the features for yourself.<br />

Survey the chapter first.<br />

• Skim the introduction on the first page of every chapter. Notice the key questions. The key questions are<br />

thought-provoking and designed to spark your interest in the chapter. See if you can answer these questions<br />

after you have read the entire chapter.<br />

• You will find vocabulary words with their definitions in blue boxes on the right side of each page.<br />

Vocabulary words will be scattered throughout the chapter. Write down any vocabulary words that are<br />

unfamiliar to you to help you recognize them later.<br />

• Next, skim the chapter to get an overview. Notice the section numbers and titles. These divide the chapter<br />

into major topics. The subheadings in each section outline important points. Vocabulary words are<br />

highlighted in bold blue type. Solving Problems pages provide step-by-step examples to help you learn to<br />

use mathematical formulas. Tables, charts, and figures summarize important information.<br />

• Read the section review questions at the end of each section. The questions help you identify what you are<br />

expected to know when you finish your reading. You will also find Challenge, Solve It, Study <strong>Skill</strong>s,<br />

Journal, Science Fact, and/or Technology boxes scattered throughout each section. These boxes provide you<br />

with an interesting way to learn more about information in the section.<br />

• Carefully read the Chapter Assessment at the end of the chapter to see what kinds of questions you will need<br />

to be able to answer. Notice that it is divided into four subtitles: Vocabulary, Concepts, Problems, and<br />

Applying Your Knowledge. Each set is listed by chapter section.<br />

1.2


Page 2 of 2<br />

Question what you see. Turn headings into questions.<br />

• Look at each of the section headings and subheadings, found at the tops of pages in your text. 1.2<br />

Change each heading to a question by using words such as who, what, when, where, why, and how.<br />

For example, Section 1.1: Measurements could become What measurements will I need to make in physical<br />

science? The subheading Two common measurement systems could become What are two common<br />

measurement systems? Write down each question and try to answer it. Doing this will help you pinpoint<br />

what you already know and what you need to learn as you read.<br />

Read and look for answers to the questions you wrote.<br />

• Pay special attention to the sidenotes in the left margin of each page. For example, under the Section 1.3<br />

subheading Converting between English and SI units, the sidenotes are: The problem of multiple units<br />

and Comparing English and SI units. These phrases and short sentences are designed to guide you to the<br />

main idea of each paragraph. Also, note the sidebars and illustrations on the right side of the page with<br />

additional explanations and concepts. For example, the target diagrams in Figure 1.4 will help you<br />

understand the terms accuracy, precision, and resolution.<br />

• Slow your reading pace when you come to a difficult paragraph. Read difficult paragraphs out loud. Copy a<br />

confusing sentence onto paper. These methods force you to slow down and allow you time to think about<br />

what the author is saying.<br />

Recite concepts out loud.<br />

• This step may seem strange at first, because you are asked to talk to yourself! But studies show that saying<br />

concepts out loud can actually help you to record them in your long-term memory.<br />

• At the end of each section, stop reading. Ask yourself each of the questions you wrote in step two on the<br />

previous page. Answer each question out loud, in your own words. Imagine that you are explaining the<br />

concept to someone who hasn’t read the text.<br />

• You may find it helpful to write down your answers. By using your senses of seeing, hearing, and touch<br />

(when you write), you create more memory paths in your brain.<br />

Review it all.<br />

• Once you have finished the entire chapter, go back and answer all of the questions that you wrote for each<br />

section. If you can’t remember the answer, go back and reread that portion of the text. Recite and write the<br />

answer again.<br />

• Next, reread the key questions at the beginning of the chapter. Can you answer these?<br />

• Complete the section reviews and the chapter assessment. Use the glossary and index at the back of the <strong>book</strong><br />

to help you locate specific definitions.<br />

The SQ3R method may seem time-consuming, but it works! With practice, you will learn to recognize the<br />

important concepts quickly.<br />

Active reading helps you learn and remember what you have read, so you will have less to re-learn as you study<br />

for quizzes and tests.


Name: Date:<br />

1.2 Stopwatch Math<br />

What do horse racing, competitive swimming, stock car racing, speed skating, many track and field events, and<br />

some scientific experiments have in common? The need for some sort of stopwatch, and people to interpret the<br />

data. For competitive athletes in speed-related sports, finishing times (and split times taken at various intervals of<br />

a race) are important to help the athletes gauge progress and identify weaknesses so they can adjust their training<br />

and improve their performance.<br />

• Three girls ran the following times for one mile in their gym class: Julie ran 9:33.2 (9 minutes,<br />

33.2 seconds), Maggie ran 9:44.24 (9 minutes, 44.24 seconds), and Mel ran 9:33.27 (9 minutes,<br />

33.27 seconds). In what order did they finish?<br />

Solution:<br />

The girl who came in first is the one with the fastest (smallest) time. Compare each time digit by digit,<br />

starting with the largest place-value. Here, that would be the minutes place:<br />

There is a “9” in the minutes’ place of each time, so next, compare the seconds’ place. Since Maggie’s time<br />

has larger numbers in the seconds’ place (44) than Julie or Mel (33), her time is larger (slower) than the<br />

other two. We know Maggie finished third out of the three girls. Now, comparing Julie’s time (9:33.2) to<br />

Mel’s (9:33.27), it is helpful to rewrite Julie’s time (9:33.2) so that it has the same number of places as<br />

Mel’s. Julie’s time needs one more digit, so adding a zero onto the end of her time, it becomes 9:33.20.<br />

Notice that Mel’s time is larger (slower) than Julie’s (27 > 20). This means that Julie’s time was fastest<br />

(smallest), so she finished first, followed by Mel, and Maggie’s time was the slowest (largest).<br />

1. Put each set of times in order from fastest to slowest.<br />

a. 5.5 5.05 5 5.2 5.15<br />

Fastest Slowest<br />

b. 6:06.04 6:06 6:06.4 6:06.004<br />

Fastest Slowest<br />

1.2


Page 2 of 2<br />

2. The table below gives the winners and their times from eight USA track and field championship<br />

races in the men’s 100 meter run. Rewrite the table so that the times are in order from fastest to<br />

slowest. Include the times and the years. Please note that the “w” that occurs next to some times<br />

indicates that the time was wind aided.<br />

Year 2005 2004 2003 2002 2001 2000 1999 1998<br />

Time 10.08 9.91 10.11 9.88w 9.95w 10.01 9.97w 9.88w<br />

Name Justin<br />

Gatlin<br />

Time<br />

Year<br />

Maurice<br />

Greene<br />

Bernard<br />

Williams<br />

Maurice<br />

Greene<br />

Tim<br />

Montgomery<br />

Maurice<br />

Greene<br />

3. The following times were recorded during an experiment with battery-powered cars. Put them in order from<br />

fastest to slowest.<br />

a. 1:22.4 1:24.007 1:25 1:22.04 1:23.117 1:23.2 1:24 1:33<br />

b. 1:18.3 1:20.22 1:21.003 1:20 1:17.99 1:21.2 1:18.22<br />

c. 1:25 1:24.99 1:24.099 1:25.001 1:24.9901 1:24.9899<br />

Dennis<br />

Mitchell<br />

Tim<br />

Harden<br />

Fastest Slowest<br />

Fastest Slowest<br />

Fastest Slowest<br />

4. Write a set of five times (in order from fastest to slowest) that are all between 26:15.2 and 26:15.24. Do<br />

not include the given numbers in your set.<br />

1.2


Name: Date:<br />

1.2 Understanding Light Years<br />

1.2<br />

How far is it from Los Angeles to New York? Pretty far, but it can still be measured in miles or<br />

kilometers. How far is it from Earth to the Sun? It’s about one hundred forty-nine million, six hundred thousand<br />

kilometers (149,600,000, or 1.496 × 108 km). Because this number is so large, and many other distances in space<br />

are even larger, scientists developed bigger units in order to measure them. An Astronomical Unit (AU) is<br />

1.496 × 108 km (the distance from Earth to the sun). This unit is usually used to measure distances within our<br />

solar system. To measure longer distances (like the distance between Earth and stars and other galaxies), the light<br />

year (ly) is used. A light year is the distance that light travels through space in one year, or 9.468 × 1012 km.<br />

1. Converting light years (ly) to kilometers (km)<br />

Earth’s closest star (Proxima Centauri) is about 4.22 light years away. How far is this in kilometers?<br />

Explanation/Answer: Multiply the number of kilometers in one light year (9.468 × 1012 km/ly) by the<br />

number of light years given (in this case, 4.22 ly).<br />

2. Converting kilometers to light years<br />

Polaris (the North Star) is about 4.07124 × 10 15 km from the earth. How far is this in light years?<br />

Explanation/Answer: Divide the number of kilometers (in this case, 4.07124 × 10 15 km) by the number of<br />

kilometers in one light year (9.468 × 1012 km/ly).<br />

Convert each number of light years to kilometers.<br />

1. 6 light years<br />

2. 4.5 × 10 6 light years<br />

3. 4 × 10 –3 light years<br />

Convert each number of kilometers to light years.<br />

4. 5.06 × 10 16 km<br />

5. 11 km<br />

4.07124 10 15 × km<br />

6. 11,003,000,000,000 km<br />

9.468 10 12<br />

(<br />

--------------------------------------------<br />

× ) km<br />

× 4.22 ly 3.995 10<br />

1 ly<br />

13 ≈ × km<br />

9.468 10 12 × km 4.07124 10<br />

÷ ---------------------------------------<br />

1 ly<br />

15 × km 1 ly<br />

---------------------------------------------<br />

1 9.468 10 12 = × --------------------------------------- ≈<br />

430 light years<br />

× km


Page 2 of 2<br />

Solve each problem using what you have learned.<br />

7. The second brightest star in the sky (after Sirius) is Canopus. This yellow-white supergiant<br />

is about 1.13616 × 10 16 kilometers away. How far away is it in light years?<br />

8. Regulus (one of the stars in the constellation Leo the Lion) is about 350 times brighter than the sun. It is<br />

85 light years away from the earth. How far is this in kilometers?<br />

9. The distance from earth to Pluto is about 28.61 AU from the earth. Remember that an AU = 1.496 × 108 km.<br />

How many kilometers is it from Pluto to the earth?<br />

10. If you were to travel in a straight line from Los Angeles to New York City, you would travel<br />

3,940 kilometers. How far is this in AU’s?<br />

11. Challenge: How many AU’s are equivalent to one light year?<br />

1.2


Name: Date:<br />

1.2 Indirect Measurement<br />

Have you ever wondered how scientists and engineers measure large quantities like the mass of an iceberg, the<br />

volume of a lake, or the distance across a river? Obviously, balances, graduated cylinders, and measuring tapes<br />

could not do the job! Very large (or very small) quantities are calculated through a process called indirect<br />

measurement. This skill sheet will give you an opportunity to try indirect measurement for yourself.<br />

• The length of a tree’s shadow is 4.25 meters and the length of a meter stick’s shadow is 1.25 meters. Using<br />

these two values and the length of the meter stick, how tall is the tree?<br />

Looking for<br />

The height of a tree.<br />

Given<br />

Tree’s shadow = 4.25 m<br />

Meter’s stick’s shadow = 1.25 m<br />

Height of meter stick = 1 m<br />

Relationships<br />

There is a direct relationship between the height of<br />

objects and the length of their shadows.<br />

height of meter stick height of object<br />

=<br />

length of meter stick shadow length of object shadow<br />

Solution<br />

The height of the tree is 3.40 meters.<br />

• At the science museum, 12 first graders stand on a giant scale to measure their mass. The combined mass of<br />

the 12 first graders is 262 kilograms. What is the average mass of a first grader in this group?<br />

Looking for<br />

The average mass of a first grader.<br />

Given<br />

Total mass of 12 first graders = 262 kilograms<br />

Relationships<br />

To get the average mass of one first grader, divide the<br />

total mass by 12.<br />

262 kilograms<br />

12<br />

1.00 m height of tree<br />

=<br />

1.25 m 4.25 m<br />

1.00 m height of tree<br />

4.25 m× = × 4.25 m<br />

1.25 m 4.25 m<br />

3.40 m = height of tree<br />

Solution<br />

262 kilograms<br />

= 21.8 kilograms<br />

12<br />

The average mass of a first grader in this<br />

group is 21.8 kilograms.<br />

1. You go to another forest and measure the shadow of a tree to be 6 meters long. The shadow of your meter<br />

stick is 2 meters long. How tall is the tree?<br />

2. The height of a flagpole is 8.50 meters. If the length of the meter stick shadow is 1.50 meters, what is the<br />

length of the flagpole’s shadow?<br />

1.2


Page 2 of 2<br />

3. While touring a city, you see a skyscraper and wonder how tall it is. You see that it is clearly<br />

divided into floors. You estimate that each floor is 20. feet high. You count that the skyscraper has<br />

112 floors.<br />

a. Approximately how tall is this skyscraper in feet?<br />

b. Approximately how tall is the skyscraper in meters? (One foot is about 0.3 meter.)<br />

4. There are 10 apples in a one-kilogram bag of apples. What is the average mass of each apple in the bag?<br />

Give your answer in units of kilograms and grams.<br />

5. If you place one staple on an electronic balance, the balance still reads 0.00 grams. However, if you place<br />

210 staples on the balance, it reads 6.80 grams. What is the mass of one staple?<br />

6. A stack of 55 business cards is 1.85 cm tall. Use this information to determine the thickness of one business<br />

card.<br />

7. A stack of eight compact disks is 1.0 centimeter high. What is the thickness of one compact disk (CD) in<br />

centimeters? What is the thickness of one CD in millimeters?<br />

8. A quarter is 2.4 millimeters thick. How tall are the following stacks of quarters?<br />

a. A stack worth 50 cents<br />

b. A stack worth $1<br />

c. A stack worth $5<br />

d. A stack worth $1,000<br />

(Give your answer in millimeters and meters.)<br />

9. Yvonne has gained a reputation for her delicious cheesecakes. She takes orders for 10 cheesecakes and<br />

spends 8.5 hours on one Saturday baking them all. She earns $120 from selling these cheesecakes.<br />

a. What is the average length of time to make one cheesecake? Give your answer in minutes.<br />

b. What does Yvonne charge for each cheesecake?<br />

c. How much money is Yvonnne earning per hour for her work?<br />

10. A sculptor wants to create a statue. She goes to a quarry to buy a block of marble. She finds a chip of marble<br />

on the ground. The volume of the chip is 15.3 cm 3 . The mass of the chip is 41.3 grams. The sculptor<br />

purchases a block of marble 30.0-by-40.0-by-100.0 cm. Use a proportion to find the mass of her block of<br />

marble.<br />

11. The instructions on a bottle of eye drops say to place three drops in each eye, using the dropper. How could<br />

you find the volume of one of these drops? Write a procedure for finding the volume of a drop that includes<br />

using a glass of water, a 10.0-mL graduated cylinder, and the dropper.<br />

12. A student wants to use indirect measurement to find the thickness of a<br />

sheet of newspaper. In a 50-centimeter tall recycling bin, she finds 50<br />

sheets of newspaper. Each sheet in the bin is folded in fourths. Design a<br />

procedure for the student to use that would allow her to measure the<br />

thickness of one sheet of newspaper with little experimental error. The<br />

student has a meter stick and a calculator.<br />

1.2


Name: Date:<br />

1.3 Dimensional Analysis<br />

Dimensional analysis is a way to find the correct label (also called units or dimensions) for the solution to a<br />

problem. In dimensional analysis, we treat the units the same way that we treat the numbers. For example, this<br />

problem shows how can you can “cancel” the sevens and then perform the multiplication:<br />

3 7<br />

-- • --<br />

7 8<br />

In some problems, there are no numerical cancellations to make, but you need to pay close attention to the units<br />

(or dimensions):<br />

• If there are 16 ounces in one pound, how many ounces are in four pounds?<br />

16 oz<br />

------------ • 4 lb<br />

1 lb<br />

=<br />

16 • 4 oz • lb<br />

-------------------------------<br />

1 lb<br />

=<br />

64 oz<br />

------------<br />

1<br />

= 64 oz<br />

The “lbs” may be cancelled either before or after the multiplication.<br />

The goal of dimensional analysis is to simplify a problem by focusing on the units of measurement (dimensions).<br />

Dimensional analysis is very useful when converting between units (like converting inches to yards, or<br />

converting between the English and SI systems of measurement).<br />

• The ninth grade class is having a reward lunch for collecting the most food for a canned food drive. They<br />

have decided to order pizza. They are figuring two slices of pizza per student. Each pizza that will be ordered<br />

will have 12 slices. There are 220 students total in the ninth grade. How many pizzas should they order?<br />

Solution:<br />

1. Determine what we want to find out: here, it is the number or whole pizzas needed to feed 220 ninth graders.<br />

It’s important to remember that if the solution is to have the label “pizzas,” “pizzas” should be kept in the<br />

numerator as the problem is set up.<br />

2. Determine what we know. We know that they’re planning 2 slices of pizza per student, that there are<br />

12 slices in each pizza, and that there are 220 ninth graders.<br />

2<br />

3. Write what you know as fractions with units. Here, we have: ----------------slices<br />

12<br />

, -------------------slices<br />

and 220 students.<br />

student pizza<br />

=<br />

12<br />

Notice that in the fraction, -------------------slices<br />

, “pizza” is in the denominator.<br />

pizza<br />

Recall that (from step #1, above) “pizza” should be kept in the numerator, as it will be the label of the final<br />

solution. To correct this problem, just switch the numerator and denominator: --------------------<br />

1 pizza<br />

.<br />

12 slices<br />

3<br />

--<br />

8<br />

1.3


Page 2 of 2<br />

4. Set up the problem by focusing on the units. Just writing the information as a multiplication<br />

problem, we have:<br />

5. Calculate:<br />

Therefore, 37 pizzas will need to be ordered.<br />

Notice that canceling the units can be done either before or after the multiplication.<br />

6. Check your solution for reasonableness: Since there are 12 slices in each pizza, and we’re figuring that each<br />

student will eat 2 slices, one pizza will feed 6 students. It is expected that a little less than 40 pizzas would be<br />

needed. It does seem reasonable that 37 pizzas would feed 220 students.<br />

1. Multiply. Be sure to label your answers.<br />

a.<br />

b.<br />

$12.00<br />

---------------<br />

6 hr<br />

• -------------<br />

1 hr 1 day<br />

-------------------<br />

2 lbs<br />

1 person<br />

--------------------<br />

1 pizza<br />

12 slices<br />

7 days<br />

• ----------------<br />

1 week<br />

•<br />

--------------------<br />

1 pizza<br />

12 slices<br />

2<br />

----------------slices<br />

220 students<br />

• ----------------------------student<br />

1<br />

15 people<br />

•<br />

----------------------<br />

1 day<br />

•<br />

2<br />

----------------slices<br />

220 students<br />

• ----------------------------student<br />

1<br />

110<br />

-----------------------------------------------------------------pizza<br />

• slices • students<br />

36<br />

3 slices • students<br />

2<br />

= = -- pizzas<br />

3<br />

2. Use dimensional analysis to convert each. You may need to use a reference to find some conversion factors.<br />

Show all of your work.<br />

a. 11.0 quarts to some number of gallons<br />

b. 220. centimeters to some number of meters<br />

c. 6000. inches to some number of miles<br />

d. How many cups are there in 4.0 gallons?<br />

3. Use dimensional analysis to find each solution. You may need to use a reference to find some conversion<br />

factors. Show all of your work.<br />

a. Frank just graduated from eighth grade. Assuming exactly four years from now he will graduate from<br />

high school, how many seconds does he have until his high school graduation?<br />

b. In 2005, Christian Cantwell won the US outdoor track and field championship shot put competition with<br />

a throw of 21.64 meters. How far is this in feet?<br />

c. A recipe for caramel oatmeal cookies calls for 1.5 cups of milk. Sam is helping to make the cookies for<br />

the soccer and football teams plus the cheerleaders and marching band, and needs to multiply the recipe<br />

by twelve. How much milk (in quarts) will he need altogether?<br />

d. How many football fields (including the 10 yards in each end zone) would it take to make a mile?<br />

e. Corey’s sister’s car gets 30. miles on each gallon of gas. How many kilometers per gallon is this?<br />

f. Convert your answer from (e) to kilometers per liter.<br />

g. A car is traveling at a rate of 65 miles per hour. How many feet per second is this?<br />

1.3


Name: Date:<br />

<strong>Skill</strong> Builder Fractions Review<br />

1.3 Fractions Review<br />

In physical science classes, you will solve problems that involve fractions. Understanding the rules for addition,<br />

subtraction, multiplication, and division of fractions helps you solve these problems. The diagram below shows<br />

the parts of a fraction. This skill sheet guides you through a review of the rules for working with fractions. You<br />

will see how the rules are used in both simple and complex fractions.<br />

Addition and subtraction of fractions<br />

To add or subtract fractions you must first have a common denominator. For example, if you wanted to add or<br />

subtract 5 / 8 and 6 / 4 , you must first convert both denominators to the same number.<br />

Addition:<br />

Subtraction:<br />

As you can see, the rules for adding and subtracting positive and negative numbers also apply to fractions.<br />

Multiplication of fractions<br />

5 6 5 ⎛26⎞ 5 12 17<br />

+ = + ⎜ × ⎟=<br />

+ =<br />

8 4 8 ⎝2 4⎠ 8 8 8<br />

5 6 5 ⎛2 6⎞ 5 12 −7<br />

− = − ⎜ × ⎟=<br />

− =<br />

8 4 8 ⎝2 4⎠ 8 8 8<br />

To multiply fractions, first multiply the numerators and then multiply the denominators. For example:<br />

5 6 30<br />

× =<br />

8 4 32<br />

Fractions are commonly expressed in their lowest terms so that they are easier to recognize. To find a fraction’s<br />

lowest terms, you need to divide the numerator and the denominator by any common factors. The fraction in the<br />

example above can be rewritten like this:<br />

30 3× 2× 5<br />

=<br />

32 2× 2× 2× 2<br />

1.3


Page 2 of 4<br />

This form is called the prime factorization because all of the factors are prime numbers (this means they<br />

can’t be divided by any whole number except 1 to get a whole number answer). Notice that there’s a 2<br />

in both the numerator and the denominator. Cross out any factor that appears in both places. Multiply<br />

out the remaining factors. The simplified fraction is:<br />

At other times you may be asked to change the fraction to a decimal. This is very easy! Simply divide the<br />

numerator by the denominator. Remember that the divisor line between the numerator and the denominator in a<br />

fraction means “divide by” and is the same as a division sign (÷).<br />

Division of fractions<br />

To divide fractions you first invert (turn upside down) the second fraction and then multiply. Follow the rules for<br />

multiplying fractions. When necessary, reduce the fraction to its lowest terms. For example:<br />

Division of complex fractions<br />

An example of a complex fraction is shown below. A complex fraction is a fraction of fractions. You can divide<br />

complex fractions using the rules you already know for dividing fractions. For example:<br />

Reduce:<br />

3× 5 15<br />

=<br />

2× 2× 2× 2 32<br />

30<br />

= 30 ÷ 32 = 0.9375<br />

32<br />

5 6 5 4 20<br />

÷ = × =<br />

8 4 8 6 48<br />

20 2× 2× 5 5 5<br />

= = =<br />

48 2× 2× 2× 2× 3 2× 2× 3 12<br />

5<br />

8 ⎛5 6⎞ 5 4 20<br />

=<br />

6 ⎜ ÷ ⎟=<br />

× =<br />

⎝8 4⎠ 8 6 48<br />

4<br />

20 2× 2× 5 5<br />

= =<br />

48 2× 2× 2× 2× 3 12<br />

As you can see, the last two examples yielded the same answer. Can you see why? The two examples are the<br />

same, but written differently. The line between the two fractions, 5 / 8 and 6 / 4, acts the same as a division (≥) sign.<br />

1.3


Page 3 of 4<br />

Solving fraction problems<br />

Now it’s your turn to solve some problems. Be sure to show your work. Reduce your answers to lowest terms.<br />

Addition of fractions:<br />

1.<br />

2.<br />

3.<br />

4. Express the above fractions in decimal form.<br />

Subtraction of fractions:<br />

4<br />

1. -----<br />

12<br />

3<br />

– --<br />

4<br />

2.<br />

3.<br />

4. Express the above fractions in decimal form.<br />

Multiplication of fractions:<br />

1. -----<br />

4<br />

12<br />

3<br />

× --<br />

4<br />

2.<br />

3.<br />

-----<br />

4<br />

+<br />

3<br />

--<br />

12 4<br />

7 5<br />

-- + --<br />

8 7<br />

3 6 5<br />

-- + -- + --<br />

4 8 3<br />

7<br />

--<br />

8<br />

5<br />

– --<br />

7<br />

3<br />

--<br />

4<br />

6 5<br />

– -- – --<br />

8 3<br />

7<br />

--<br />

8<br />

5<br />

× --<br />

7<br />

3<br />

--<br />

4<br />

6 5<br />

× -- ×<br />

--<br />

8 3<br />

4. Express the above fractions in decimal form.<br />

1.3


Page 4 of 4<br />

Division of fractions:<br />

1.<br />

2.<br />

3.<br />

4.<br />

5. Express the above fractions in decimal form.<br />

Division of complex fractions:<br />

1.<br />

2.<br />

3.<br />

4.<br />

-----<br />

4<br />

12<br />

3<br />

÷ --<br />

4<br />

7<br />

--<br />

8<br />

5<br />

÷ --<br />

7<br />

10 1<br />

----- ÷ --<br />

15 3<br />

8 2<br />

----- ÷ --<br />

18 3<br />

3<br />

--<br />

4<br />

6<br />

--<br />

8<br />

3<br />

--<br />

4<br />

5<br />

--<br />

3<br />

7<br />

× --<br />

6<br />

5<br />

--<br />

--------------<br />

3<br />

10<br />

-----<br />

2<br />

× --<br />

4 3<br />

25<br />

-----<br />

--------------<br />

10<br />

15<br />

-----<br />

5<br />

÷ --<br />

3 3<br />

5. Express the above fractions in decimal form.<br />

1.3


Name: Date:<br />

1.3 Significant Digits<br />

Francisco is training for a 10-kilometer run. Each morning, he runs a loop around his neighborhood. To find out<br />

exactly how far he’s running, he asks his older sister to drive the loop in her car. Using the car’s trip odometer,<br />

they find that the route is 7.2 miles long.<br />

To find the distance in kilometers, Francisco looks in the reference<br />

section of his science text and finds that 1.000 mile =<br />

1.609 kilometers. He multiplies 7.2 miles by 1.609 km/mile. The<br />

answer, according to his calculator, is 11.5848 kilometers.<br />

Francisco wonders what all those numbers after the decimal point<br />

really mean. Can a car odometer measure distances as small as 0.0008<br />

kilometer? That’s a distance less than one meter!<br />

This skill sheet will help you answer Francisco’s question. It will also<br />

help you figure out which digits in your own calculations are<br />

significant.<br />

What are significant digits?<br />

Significant digits are the meaningful digits in a measured quantity. Scientists have agreed upon a number of rules<br />

to determine which numbers in a measurement are significant. The rules are:<br />

1. Non-zero digits in a measurement are always significant. This means that the distance measured by the<br />

car odometer, 7.2 miles, has two significant digits.<br />

2. Zeros between two significant digits in a measurement are significant. This means that the measurement<br />

of kilometers per mile, 1.609 kilometers, has four significant digits.<br />

3. All final zeros to the right of a decimal point in a measurement are significant. This means that the<br />

measurement 1.000 miles has four significant digits.<br />

4. If there is no decimal point, final zeros in a measurement are NOT significant. This means that the<br />

number 20 in the phrase “20-liter water cooler” has one significant digit. The water cooler isn’t marked off<br />

in 1-liter increments, so no measurement decision was made regarding the ones place.<br />

5. A decimal point is used after a whole number ending in zero to indicate that a final zero IS significant.<br />

If you measure 100 grams of lemonade powder to the nearest whole gram, write the number as 100. grams.<br />

This shows that your measurement has three significant digits.<br />

6. In a measurement, zeros that exist only to put the decimal point in the right place are NOT significant.<br />

This means that the number 0.0008 in the phrase “0.0008 kilometer” has one significant digit.<br />

7. A number that is found by counting rather than measuring is said to have an infinite number of<br />

significant digits. For example, the race officials count 386 runners at the starting line. The number 386, in<br />

this case, has an infinite number of significant digits.<br />

1.3


Page 2 of 4<br />

Find the number of significant digits<br />

Table 1: Number of Significant Digits<br />

Value How many significant digits does each value have?<br />

a. 36.33 minutes<br />

b. 100 miles<br />

c. 120.2 milliliters<br />

d. 0.0074 kilometers<br />

e. 0.010 kilograms<br />

f. 300. grams<br />

g. 42 students<br />

Using significant digits in calculations<br />

Taking measurements and recording data are often a part of science classes. When you use the data in<br />

calculations, keep in mind this important principle:<br />

When using data in a calculation, your answer can’t be more precise than the least precise measurement.<br />

You are using a ruler to measure the length of each side of a rectangle.The<br />

ruler is marked in tenths of a centimeter. This means that you can estimate the<br />

distance between two 0.1 cm marks and make measurements that are to two<br />

places after the decimal.<br />

You measure the two short sides of the triangle and find that they each have a<br />

length of 12.25 cm. The long sides each have a length of 20.75 cm.<br />

The rectangle’s perimeter (distance around) is 12.25 cm + 20.75 cm + 12.25 cm + 20.75 cm, or 66.00 cm. The<br />

two zeros to the right of the decimal point show that you measured with a precision of 0.01 cm.<br />

The area of the rectangle is found by multiplying the length of the short side by the length of the long side.<br />

2<br />

12.25 cm× 20.75 cm =<br />

254.1875 cm<br />

The answer you get from you calculator has seven significant digits. This incorrectly implies that your ruler can<br />

measure to one ten-thousandth of a centimeter. Your ruler can’t measure distances that small!<br />

1.3


Page 3 of 4<br />

Follow these steps for determining the right answer for your calculation:<br />

• When multiplying or dividing measurements, find the measurement in the calculation with the least<br />

number of significant digits. After doing your calculation, round the answer to that number of<br />

significant digits.<br />

In the rectangle example on the previous page, each measurement has 4 significant digits. When you<br />

1.3<br />

multiply the measurements to find the area, your answer should be rounded to four significant digits. The<br />

area should be reported as 254.2 cm2 .<br />

• When adding or subtracting measurements, the answer must not contain more decimal places than your least<br />

accurate measurement.<br />

In the rectangle example, the perimeter is reported to two decimal places to show that your ruler measures<br />

length to the nearest 0.01 centimeter.<br />

It is important to note that when adding or subtracting, you are not concerned with the number of significant<br />

digits to the left of the decimal point. When adding 1.25 cm + 1,000.50 cm + 2,000,000.75 cm, the answer is<br />

2,001,002.50 cm. It is okay to have an answer with nine significant digits, because only TWO of them are to<br />

the right of the decimal point.<br />

• When you are finding the average of several measurements, remember that numbers found by counting have<br />

an infinite number of significant digits.<br />

For example, a student measures the distance between two magnets when their attractive force is first felt.<br />

He repeats the experiment three times. His results are: 23.25 cm, 23.30 cm, 23.20 cm.<br />

To find the average distance, He adds the three times and divides the sum by three. “Three” is the number of<br />

times the experiment is repeated.<br />

23.25 cm + 23.30 cm + 23.20 cm<br />

=<br />

23.25 cm<br />

3<br />

In this equation, the number 3 is found by counting the number of times the experiment is repeated, not by<br />

measuring something. Therefore it is said to have an infinite number of significant digits. That’s why the<br />

answer has four significant digits, not just one.<br />

Report your answers with significant digits<br />

Have you ever participated in a road race? The following problems are all related to a road race event. Can you<br />

come up with some other problems that you might have to solve if you were running in or volunteering for a road<br />

race?<br />

1. The banner over the finish line of a running race is 400. centimeters long and 85 centimeters high. What is<br />

the area of the banner?


Page 4 of 4<br />

2. Heidi stops at three water stations during the running race. She drinks 0.25 liters of water at the first<br />

stop, 0.3 liters at the second stop, and 0.37 liters at the third stop. How much water does she<br />

consume throughout the race?<br />

3. The race officials want to set up portable bleachers near the finish line. Each set of bleachers is 4.50 meters<br />

long and 2.85 meters wide. How many square meters of open ground space do they need for each set?<br />

4. The race has been held annually for ten years. The high temperatures for the race dates (in degrees Celsius)<br />

are listed in the table below. What is the average high temperature for the race day based on the temperatures<br />

for the past ten years? Write your answer in the bottom row in the table.<br />

Table 2: Race Day Temperatures for Each Year<br />

Year Number of Race Race Day Temperature (°C)<br />

1 27.2<br />

2 18.3<br />

3 28.9<br />

4 22.2<br />

5 20.6<br />

6 25.5<br />

7 21.1<br />

8 23.9<br />

9 26.7<br />

10 27.8<br />

Average Temperature<br />

5. Challenge! Ji-Sun has participated in the race for the past four years. His times, reported in<br />

minutes:seconds, were<br />

40:30<br />

43:40<br />

39:06<br />

38:52<br />

What is his average time to complete the race? (Hint: Convert all times to seconds before averaging!)<br />

6. Come up with one more problem that uses information that is related to a road race. Write your problem in<br />

the space below and come up with the answer. Be sure to write your answer with the correct number of<br />

significant digits.<br />

1.3


Name: Date:<br />

1.3 Study Notes<br />

This skill builder will help you take notes while you read. Each paragraph in the text has a sidenote. Fill in the<br />

table as you read each section of your text<strong>book</strong>. Use the information to study for tests!<br />

• First, write in the number of the section that you are reading. For example, the first section of the text is 1.1.<br />

This is the first section in chapter 1 of the text.<br />

• For each paragraph that you read, write the sidenote. Then write a question based on this sidenote. As you<br />

read the paragraph, answer your own question!<br />

• When you study, fold this paper so that the answers are hidden. Use separate paper to write answers to each<br />

of your questions. Then unfold this paper and check your work.<br />

Section number: 1.1<br />

Page<br />

number<br />

Section number: __________<br />

Sidenote text Question based on<br />

sidenote<br />

8 Accuracy What is the science<br />

definition of accuracy?<br />

Page<br />

number<br />

Sidenote text Question based on<br />

sidenote<br />

Answer to question<br />

1.3<br />

In science, accuracy means<br />

how close a measurement is<br />

to an accepted or true value.<br />

Answer to question


Page 2 of 2<br />

Section number: _________<br />

Page<br />

number<br />

Sidenote text Question based on<br />

sidenote<br />

Answer to question<br />

1.3


Name: Date:<br />

1.3 Science Vocabulary<br />

One stumbling block for many science students is the number of new vocabulary words they encounter. Each<br />

field of science has its own body of terms, and there are many additional terms that are used in all the fields of<br />

science. However, few people are likely to run across these terms in their daily lives until they enter science<br />

classes in school. How do students master this new language? The same way they master any vocabulary— by<br />

looking to the roots!<br />

Prefixes and suffixes play an important role in word structure. Prefixes are word parts that begin a word, and<br />

suffixes are word parts that end a word. These parts, also called roots, often have special meanings due to their<br />

use in other languages.<br />

Prefixes and suffixes of words (and entire words themselves) in the English language are derived from other<br />

languages. Some of these languages like French are used in the world culture today and some languages belong<br />

to cultures long past. Latin and Greek are the two most common languages from which we derive pieces and<br />

parts of our words.<br />

The study of languages provides tremendous benefit to understanding the meanings of words. Other languages<br />

provide us with greater understanding of our own language since the roots of many of the words come from these<br />

languages. For example, English, French, Italian, and Spanish all have Latin as a common ancestral language.<br />

Therefore, studying French, Italian, or Spanish increases the size of your vocabulary toolbox. Studying Latin (or<br />

Greek) is also a tremendous aid for mastery and comprehension of the English vocabulary.<br />

When you encounter a new word and are unsure of its meaning, find and isolate the prefix and suffix of the word.<br />

It may help to write the word on a separate sheet of paper and circle or underline these parts. The remaining,<br />

uncircled parts of the word may also have a special meaning. Now, study each part of the word and work towards<br />

understanding its meaning.<br />

Consider the word blueberry. There are two pieces to this word—the prefix blue and the suffix berry. Each of<br />

these word parts has its own meaning which, when combined with the other word part, gives the whole word its<br />

own, unique meaning. Blue denotes a color with which you are familiar. A berry is a small fruit that birds (and<br />

humans!) like to eat. Put them together, and you understand that blueberry probably means a small fruit that is<br />

colored blue. From your past experiences, you realize that this is a pretty good description of blueberries. Science<br />

words can be broken apart and analyzed in the same way to get an understanding of their meanings.<br />

Below are some words you may encounter in a science class. For each word, circle the prefix and put a box<br />

around the suffix:<br />

thermometer electrolyte monoatomic<br />

volumetric endothermic spectroscope<br />

prototype convex supersaturated<br />

1.3


Page 2 of 3<br />

The table below lists some prefixes and suffixes that are found in scientific vocabulary along with their<br />

respective meanings. Use this table to write a definition for the following terms.<br />

Prefixes Suffixes<br />

homo – same, equal -escence – to exist<br />

poly – many -meter – measure<br />

hydro – water -ology – the study of<br />

lumen – light -mer – unit<br />

spectro – a continuous range or<br />

full extent<br />

hetero – different<br />

-geneous – kind or type<br />

1. hydrology _____________________________________<br />

2. polymer _____________________________________<br />

3. homogeneous _____________________________________<br />

4. heterogeneous _____________________________________<br />

5. luminescence _____________________________________<br />

6. spectrometer _____________________________________<br />

Now, using a dictionary, look up the words for which you provided your own definition, and write the formal<br />

definitions in the spaces below:<br />

1. hydrology _____________________________________<br />

2. polymer _____________________________________<br />

3. homogeneous _____________________________________<br />

4. heterogeneous _____________________________________<br />

5. luminescence _____________________________________<br />

6. spectrometer _____________________________________<br />

1.3


Page 3 of 3<br />

Using the table of prefixes and suffixes provided below, write a word that corresponds to each of the<br />

following definitions:<br />

Prefixes Suffixes<br />

thermo – heat -scope – to view<br />

mono – one -meter – measure<br />

tele – far -atomic – indivisible unit<br />

sono – sound, tone -graph, -gram – something written<br />

1. A device to measure heat or temperature: _____________________________________<br />

2. A graph showing the loudness and frequencies of sounds: ________________________________<br />

3. Having only one type of “indivisible” unit: _________________________________<br />

4. A device used to view distant objects: _____________________________________<br />

Look up the words you created in the dictionary. Write your words and the accepted definitions in the space<br />

below:<br />

Word Dictionary Definition<br />

How closely did your definitions match the accepted ones found in the dictionary? Your definitions based on<br />

your understanding of the roots for the prefixes and suffixes likely provided you with good results. A thorough<br />

knowledge of prefixes and suffixes will be a tremendous help to you as you proceed through your science<br />

education and will enable you to better understand the written and spoken language you encounter in your daily<br />

life.<br />

1.3


Name: Date:<br />

1.3 SI Unit Conversion—Extra Practice<br />

Science is about exploring and understanding the universe, from the tiniest subatomic particles to the mindboggling<br />

expanses of interstellar space. We gain understanding by asking questions, observing, measuring, and<br />

sharing results with others. You will use SI (metric) measuring tools for this purpose throughout the year. This<br />

skill sheet will help you become familiar with the SI prefixes you will use to measure length, mass, and volume.<br />

Here are the prefixes you will use to report measurements:<br />

Prefix kilo- hecto- deka- Basic unit<br />

(no prefix)<br />

1. How many centimeters are there in 24 meters?<br />

a. Restate the question: 24 meters = __________centimeters.<br />

b. Use the place value chart above to determine the<br />

multiplication factor, and solve.<br />

Count the number of places on the chart it takes to move<br />

from meters (the ones place) to centimeters (the<br />

hundredths place). Since it takes 2 moves to the right,<br />

the multiplication factor is 100. Move the decimal two places to the right.<br />

Solution: multiply 24 × 100 = 2,400.<br />

Answer: There are 2,400 centimeters in 24 meters.<br />

Use the chart above to help you answer the following questions.<br />

Part A: Distance conversions<br />

1. Earth’s diameter is 12,756 kilometers. How many meters is this?<br />

2. The diameter of Earth’s moon is 3,476 kilometers. Express this distance in centimeters.<br />

deci- centi- milli-<br />

Symbol k h da m, g, l d c m<br />

Multiplication Factor<br />

or Place-Value<br />

1,000 100 10 1 0.1 0.01 0.001<br />

3. The average distance between Earth and its moon is 384,000,000 meters. Express this distance in kilometers.<br />

4. A billion years ago, Earth and its moon were just 200,000 kilometers apart. Express this distance in meters.<br />

1.3


Page 2 of 2<br />

5. Earth’s moon is covered with impact craters. These craters form when an asteroid, comet, or<br />

meteorite crashes into the moon’s surface. The largest impact crater, called Clavius, is 160<br />

kilometers across. How many centimeters is this?<br />

6. A crater known as Aristarchus is 3.6 kilometers deep. What is this distance in millimeters?<br />

7. Another feature of the moon’s surface is long, narrow valleys called Rilles. The Hadley Rille is<br />

125 kilometers long, 0.4 kilometers deep, and 1.5 kilometers wide at its widest point. Express these<br />

distances in meters.<br />

8. To escape Earth’s gravitational pull, an object must reach a speed of 11,180 meters per second. How fast is<br />

this in kilometers per second?<br />

Part B: Mass conversions<br />

9. The largest land mammal is the African Elephant. The average adult has a mass of 5,400 kilograms. Express<br />

this mass in grams.<br />

10. One of the smallest mammals is the bumblebee bat, with a mass of about 0.002 kilograms. Express this mass<br />

in grams.<br />

11. A pygmy shrew is another tiny mammal, with a mass ranging from 1.2 grams to 2.7 grams. Express these<br />

measurements in milligrams.<br />

12. The largest mammal ever found on Earth was a female blue whale with a mass of more than<br />

158,000,000 grams. Express this mass in kilograms.<br />

13. A blue whale’s heart is so large that a person could crawl through its largest blood vessel (the aorta). The<br />

heart has a mass of about 450 kg. Express this mass in milligrams.<br />

14. A baby blue whale will drink between 23 and 90 kilograms of milk each day. Express these masses in grams.<br />

Part C: Volume conversions<br />

15. A mature sugar maple tree produces about 40 liters of sap each season. How many milliliters is this?<br />

16. After the sap is collected, extra water in it must be boiled off. Forty liters of sap is boiled down to produce<br />

one liter of maple syrup. How many milliliters of syrup are in one liter?<br />

17. Canada is the world’s largest commercial producer of maple syrup. In 2005, Canada produced about<br />

26,600,000 liters of maple syrup. How many kiloliters is this?<br />

18. Vermont is the United States’ largest commercial producer of maple syrup. Vermont produced about<br />

1,558 kiloliters of maple syrup in 2005. How many liters is this?<br />

19. One serving of maple syrup is 1/4 cup, or 0.06 liters. How many milliliters of syrup is this?<br />

20. A jug of maple syrup contains 947 milliliters. Express this volume in liters.<br />

1.3


Name: Date:<br />

1.3 SI-English Conversions<br />

Even though the United States adopted the SI system in the 1800’s, most Americans still use the English system<br />

(feet, pounds, gallons, etc.) in their daily lives. Because almost all other countries in the world, and many<br />

professions (medicine, science, photography, and auto mechanics among them) use the SI system, it is often<br />

necessary to convert between the two systems.<br />

It is useful to be familiar with examples of measurements in both systems. Most people in the United States are<br />

very familiar with English system units because they are used in everyday tasks. Some examples of SI system<br />

measurements are:<br />

One kilometer (1 km) is about two and a half<br />

times around a standard running track.<br />

One centimeter (1 cm) is about the width of<br />

your little finger.<br />

One kilogram (1 kg) is about the mass of a full<br />

one-liter bottle of drinking water.<br />

One gram (1 g) is about the mass of a paper<br />

clip.<br />

One liter (1 l) is a common size of a bottle of<br />

drinking water.<br />

One milliliter (1 mL) is about one droplet of<br />

liquid.<br />

1.3


Page 2 of 3<br />

When precise conversions between SI and English systems are necessary, you will need to know the<br />

conversion factors given in the table below.<br />

Table 1: English - SI measurement equivalents<br />

Measurement Equivalents<br />

Length: 1 inch = 2.54 centimeters<br />

Volume:<br />

Mass: Weight<br />

(on Earth)<br />

1. If we need to know the mass of a 50.-pound bag of dog food in kilograms, we take the following steps:<br />

(1) Restate the question: 50. lb ≈ __________ kg<br />

(2) Find the conversion factor from the table: 1 kg ≈ 2.2 lb<br />

(3) Multiply the ratios making sure that the unwanted units cancel, leaving only the desired units (kilograms)<br />

in the answer:<br />

2. How many inches are equivalent to 99 centimeters?<br />

1kilometer ≈ 0.62 mile<br />

1 liter ≈ 1.06 quart<br />

1 kilogram ≈ 2.2 pounds<br />

1 ounce ≈ 28 grams<br />

50<br />

----------lb<br />

-------------<br />

1 kg 50 kg<br />

• ≈ ------------- ≈22.7272 kg ≈23<br />

kg<br />

1 2.2 lb 2.2<br />

(1) Restate the question: 99 centimeters = __________ inches<br />

(2) Find the conversion factor from the table: 1 inch = 2.54 centimeters.<br />

(3) Multiply the ratios. Make sure the units cancel correctly to produce the desired unit in the answer.<br />

99 cm 1 in<br />

-------------- • ------------------<br />

1 2.54 cm<br />

99 in<br />

= ----------- = 38.97638 in ≈ 39 inches<br />

2.54<br />

3. An eighth grader is 5 feet 10. inches tall. We want to know how many centimeters that is without measuring.<br />

(1) Restate the question: 5 feet 10. inches = __________ centimeters<br />

(2) Convert units within one system if necessary: 5 feet 10. inches needs to be rewritten as either feet or<br />

inches. Since our conversion factor (from the table) is given as 1 inch = 2.54 centimeters, it makes sense to<br />

rewrite the quantity 5 feet 10. inches as some number of inches. First convert the number of feet (5) to<br />

inches, then add 10. inches. Since there are 12 inches in 1 foot, use that as your conversion factor to<br />

calculate:<br />

5<br />

------ft<br />

12 in<br />

• -----------<br />

1 1 ft<br />

60 in ·<br />

= ----------- =<br />

60 inches<br />

1<br />

1.3


Page 3 of 3<br />

1.<br />

2.<br />

3.<br />

Now add: 60. inches + 10. inches = 70. inches. 5 feet 10. inches = 70. inches. We now need to 1.3<br />

convert 70. inches to centimeters.<br />

(3) Restate the question: 70. inches = __________ centimeters.<br />

(4) Choose the correct conversion factor from the table. Here, we want to convert inches to centimeters, so<br />

use 1 inch = 2.54 centimeters.<br />

(5) Multiply the ratios. Make sure the units cancel correctly to produce the desired type of unit in the answer.<br />

7.0 km ≈ __________ mi<br />

115 g ≈ __________oz<br />

2,000. lb. ≈<br />

__________kg<br />

70<br />

----------in<br />

2.54 cm<br />

• ------------------<br />

1 1 in<br />

4. A 2.0-liter bottle of soda is about how many quarts?<br />

=<br />

177.8<br />

--------------------cm<br />

= 180 cm<br />

1<br />

5. A pumpkin weighs 5.4 pounds. What is its mass in grams? (Hint: There are 16 ounces in one pound).<br />

6. Felipe biked 54 kilometers on Sunday. How many miles is this?<br />

7. How many inches are in 72.0 meters? How many yards is this?<br />

8. In a track meet, Julian runs the 800. meter dash, the 1600. meter run, and the opening leg of the<br />

4 × 400. meter relay. How many miles is this altogether?<br />

9. How many liters are equivalent to 1.0 gallon? (There are exactly four quarts in a gallon.)<br />

10. The mass of a large order of french fries is about 170. grams. What is its approximate weight in pounds?


Name: Date:<br />

1.4 Creating Scatterplots<br />

Scatterplots allow you to present data in a form that shows a cause and effect relationship between two variables.<br />

The parts of a scatterplot include:<br />

1. Data pairs: Scatterplots are made using pairs of numbers. Each pair of numbers represents one data point on<br />

a graph. The first number in the pair represents the independent variable and is plotted on the x-axis. The<br />

second number represents the dependent variable and is plotted on the y-axis.<br />

2. Axis labels: The label on the x-axis is the name of the independent variable. The label on the y-axis is the<br />

name of the dependent variable. Be sure to write the units of each variable in parentheses after its label.<br />

3. Scale: The scale is the quantity represented per line on the graph. The scale of the graph depends on the<br />

number of lines available on your graph paper and the range of the data. Divide the range by the number of<br />

lines. To make the calculated scale easy-to-use, round the value to a whole number.<br />

4. Title: The format for the title of a graph is: “Dependent variable name versus independent variable name.”<br />

1. For each data pair in the table, identify the independent and dependent variable. Then, rewrite the data pair<br />

according to the headings in the next two columns of the table The first two data pairs are done for you.<br />

Data pair<br />

(not necessarily in order)<br />

Independent<br />

(x-axis)<br />

Dependent<br />

(y-axis)<br />

1 Temperature Hours of heating Hours of heating Temperature<br />

2 Stopping distance Speed of a car Speed of a car Stopping distance<br />

3 Number of people in a family Cost per week for groceries<br />

4 Stream flow rate Amount of rainfall<br />

5 Tree age Average tree height<br />

6 Test score Number of hours studying for a test<br />

7 Population of a city Number of schools needed<br />

2. Using the variable range and number of lines, calculate the scale for an axis. The first two are done for you.<br />

Variable<br />

range<br />

Number of lines Range ≥ Number of lines Calculated scale Adjusted scale<br />

13 24 13 ≥ 24 = 0.54 1<br />

83 43 83 ≥ 43 = 1.9 2<br />

31 35<br />

100 33<br />

300 20<br />

900 15<br />

1.4


Page 2 of 3<br />

3. Here is a data set to use to create a scatterplot. Follow these steps to make the graph.<br />

a. Place this data set in the table below. Each data point is given in the format of (x, y).<br />

The x- values represent time in minutes. The y-values represent distance in kilometers.<br />

(0, 5.0), (10, 9.5), (20, 14.0), (30, 18.5), (40, 23.0), (50, 27.5), (60, 32.0).<br />

Independent variable<br />

(x-axis)<br />

Dependent variable<br />

(y-axis)<br />

b. What is the range for the independent variable?<br />

c. What is the range for the dependent variable?<br />

d. Make your graph using the blank graph below. Each axis has twenty lines (boxes). Use this information<br />

to determine the adjusted scale for the x-axis and the y-axis.<br />

e. Label your scatterplot. Add a label for the x-axis, y-axis, and provide a title.<br />

f. Draw a smooth line through the data points.<br />

1.4


Page 3 of 3<br />

g. What is the position value after 45 minutes? Use your scatterplot to answer this question.<br />

1.4


Name: Date:<br />

1.4 What’s the Scale?<br />

Graphs allow you to present data in a form that is easy to understand. With a graph, you can see whether your<br />

data shows a pattern and you can picture the relationship between your variables.<br />

The scale on a graph is the quantity represented per line on the graph. Your graph’s scales will depend on the type<br />

of data you are plotting. Each of your graph’s axes has its own separate scale. You need to be consistent with your<br />

scales. If one line on a graph represents 1 cm on the x-axis, it has to stay that way for the entire x-axis.<br />

When figuring out the scale for your graph, you first need to know the range. When you want your axis to start at<br />

zero, your range is equal to your highest data value. Once you have the range, you can calculate the scale. Count<br />

the number of lines you have available on your graph paper. Now, divide the range by the number of lines. This<br />

number is your scale. Then you adjust your scale by rounding up to a whole number.<br />

Calculate the scales for the data set listed in the table below. Your graph paper is 20 boxes by 20 boxes.<br />

Identify the variables.<br />

1. Which is the independent variable? Time is your independent variable; it goes on the x-axis.<br />

Which is the dependent variable? Amount of rainfall is your dependent variable; it goes on the y-axis.<br />

Find the ranges.<br />

2. What is the range of data for the x-axis? 30 hours<br />

What is the range of data for the y-axis? 59 mL<br />

Calculate the scales.<br />

Time (hours) Amount of rainfall (mL)<br />

5 5<br />

10 11<br />

15 21<br />

20 28<br />

25 37<br />

30 59<br />

3. What is the scale for your x-axis?<br />

30 hrs divided by 20 boxes = 1.5 hrs/box rounded up to 2 hrs/box<br />

Each line on the graph is equal to 2 hours.<br />

The x-axis will start at zero and go up to 40 hours, with each line counting as 2 hours.<br />

What is the scale for your y-axis?<br />

59 mL divided by 20 boxes = 2.95 mL/box rounded up to 3 mL/box<br />

Each line on the graph is equal to 3 mL.<br />

The y-axis will start at zero and go up to 60 mL, with each line counting as 3 mL.<br />

1.4


Page 2 of 2<br />

1.4<br />

1. Given the variable range and the number of lines, calculate the scale for an axis. Often the<br />

calculated scale is not an easy-to-use value. To make the calculated scale easy-to-use, round the value and<br />

write this number in the column with the heading “Adjusted scale.” The first two are done for you.<br />

Range<br />

from 0<br />

Number<br />

of Lines<br />

Range ≥ Number of Lines Calculated scale Adjusted scale<br />

(whole number)<br />

14 10 14 ≥ 10 = ≥ 1.4 2<br />

8 5 8 ≥ 5 =≥ 1.6 2<br />

1000 20 1000 ≥ 20 = ≥<br />

547 15 547 ≥ 15 = ≥<br />

99 30 99 ≥ 30 = ≥<br />

35 12 35 ≥ 12 = ≥<br />

2. Calculate the range and the scale for the x-axis starting at zero, given the following data pairs and a 30 box<br />

by 30 box piece of graph paper. Each data point is given in the format of (x, y): (1, 27), (30, 32), (20, 19),<br />

(6, 80), (15, 21).<br />

3. Calculate the range and the scale for the y-axis starting at zero, given the following data pairs and a 10 box<br />

by 10 box piece of graph paper. Each data point is given in the format of (x, y): (1, 5), (2, 10), (3, 15), (4, 20),<br />

(5, 25).<br />

4. Calculate the scale for both the x-axis and the y-axis of a graph using the data set in the table below. Your<br />

graph paper is 20 boxes by 20 boxes. Start both the x- and y-axis at zero.<br />

a. Which is the independent variable? Which is the dependent variable?<br />

b. What is the range of data for the x-axis? What is the range of data for the y-axis?<br />

c. What is the scale for your x-axis? What is the scale for your y-axis?<br />

Day Average Daily<br />

Temperature (°F)<br />

1 67<br />

3 68<br />

5 73<br />

7 66<br />

9 70<br />

11 64


Name: Date:<br />

1.4 Interpreting Graphs<br />

The four main kinds of graphs are scatterplots, bar graphs, pie graphs, and line graphs.<br />

To learn how to interpret graphs, we will start with an example of a scatterplot. The data presented on the<br />

scatterplot below is the amount of money in a cash box during a car wash that lasted for five hours. Use this<br />

graph to follow the steps and answer the questions below.<br />

Step 1: Read the labels on the graph.<br />

1. What is the title of the graph?<br />

2. Read the labels for the x-axis and the y-axis. What two variables are represented on the graph?<br />

Step 2: Read the units used for the variable on the x-axis and the variable<br />

on the y-axis.<br />

3. What unit is used for the variable on the x-axis?<br />

4. What unit is used for the variable on the y-axis?<br />

1.4


Page 2 of 3<br />

Step 3: Look at the range of values on the x- and y-axes. Do the range of values make<br />

sense? What would the data look like if the range of values on the axes was<br />

spread out more or less?<br />

5. What is the range of values for the x-axis?<br />

6. The range of values for the y-axis is 0 to $120. What would the graph look like if the range of values was<br />

0 to $500? Where would the data appear on the graph if this were the case?<br />

Step 4: After looking at the parts of the graph, pay attention to the data that is plotted. Is there<br />

a relationship between the two variables?<br />

7. Is there a relationship between the variables that are represented on the graph?<br />

Step 5: Write a sentence that describes the information presented on the graph. For example,<br />

you may say, “As the values for the variable on the x-axis increase, the values for the<br />

variable on the y-axis decrease.”<br />

8. Write your own description of the graph.<br />

9. The theater club at your school needs to raise $1000 for a trip that they want to take. They will be taking the<br />

trip next fall. It is now April. Based on the graph, would you recommend that the group wash cars to raise<br />

money? Write out a detailed response to this question. Be sure to provide evidence to support your reasons<br />

for your recommendation.<br />

Now, apply what you know about interpreting graphs to a<br />

bar graph. Use the steps from part one to help you answer<br />

the questions.<br />

1. What is the title of this graph?<br />

2. What variables are represented on the graph? (Hint: there<br />

are three variables.) Describe each variable in terms of the<br />

categories or the range of values used.<br />

3. Write a sentence that describes how the percentage of<br />

teenagers employed compares from city to city. Do not<br />

state any conclusions about the data in your sentence.<br />

4. Write a sentence that describes how the percentage of boys<br />

employed compares to the percentage of girls employed.<br />

Do not state any conclusions about the data in your<br />

sentence.<br />

5. Based on the data represented in the graph, come up with a hypothesis for why the percentage of teenagers<br />

employed differs from city to city.<br />

6. Based on the data represented in the graph, come up with a hypothesis to explain the employment<br />

differences between boys and girls in these cities.<br />

1.4


Page 3 of 3<br />

Now, apply what you know about interpreting<br />

graphs to a pie graph. Use the steps from part one<br />

to help you answer the questions.<br />

1. What is the title of this graph?<br />

2. What variables are represented on the graph?<br />

(Hint: there are two variables.)<br />

3. Are any units used in this graph? Explain your<br />

answer.<br />

4. If you were going to report on this data, what<br />

would you say? Write two to three sentences that<br />

describe this graph. Do not state any conclusions<br />

about the data in your sentence.<br />

5. Come up with a hypothesis based on the data in<br />

this graph. Briefly describe how you would test your hypothesis.<br />

6. Do you have a job? If so, in which category does your job fit? Do you think this pie graph accurately<br />

represents the working teenager population in your area? Explain your response.<br />

Finally, apply what you know about interpreting graphs to a<br />

line graph. Use the steps from part one to help you answer<br />

the questions.<br />

1. What is the title of this graph?<br />

2. What variables are represented on the graph? (Hint: there<br />

are two variables.)<br />

3. What is the range of values for each variable?<br />

4. Write a sentence that describes the change in student<br />

population at Springfield <strong>High</strong> <strong>School</strong> from 1970 to 1985.<br />

Do not state any conclusions about the data in your<br />

sentence.<br />

5. a. Come up with a possible reason for the sudden drop in<br />

population between 1980 and 1985.<br />

b. If this were your high school, how could you find out if your explanation is correct?<br />

6. Explain why this graph is a line graph, not a scatterplot.<br />

1.4


Name: Date:<br />

1.4 Recognizing Patterns on Graphs<br />

In physical science class, you will do laboratory experiments to answer questions such as: If I change this, what<br />

will happen to that? For example, you might ask: If I change the mass of a toy car by adding some cargo, what<br />

will happen to its acceleration down a ramp? Or, you might ask: If I change the temperature of some water in a<br />

beaker by heating it on a burner, what will happen to the amount of sugar that I can dissolve in it?<br />

Making a scatterplot graph of your results can help you recognize patterns in your data. In order to share your<br />

results with others, it is helpful to understand the vocabulary that scientists use to describe patterns seen on<br />

scatterplot graphs. In this skill sheet, you will practice describing some of these patterns.<br />

Take a look at these two graphs:<br />

In each case, the line or curve slopes up from left to right. This tells you there is a direct relationship between<br />

the x- and y-variables. If you increase the x-value, the y-value will also increase.<br />

Here are two more graphs:<br />

Direct relationship between variables<br />

Inverse relationship between variables<br />

1.4


Page 2 of 3<br />

In each case, the line or curve slopes down from left to right. This tells you there is an inverse<br />

relationship between the x- and y-variables. If you increase the x-value, the y-value will decrease.<br />

Sometimes your graphs will be a straight line. This tells you there is a linear relationship between<br />

variables.<br />

If the graph is a curve, we say that the relationship is non-linear.<br />

Scatterplots can also help us describe the strength of the relationship between two variables. The following<br />

graphs show the number of grams of three different substances that will dissolve in 100 ml of water at different<br />

temperatures.<br />

No relationship Weak relationship Strong relationship<br />

Substance A: The amount that will dissolve is not related to temperature. No relationship.<br />

Substance B: The amount that will dissolve increases slightly with temperature. This is a weak relationship.<br />

Substance C: The amount that will dissolve increases a lot with temperature. This is a strong relationship.<br />

1.4


Page 3 of 3<br />

Answer the following questions about graphs A–F, below.<br />

1. Name three graphs which show a direct relationship between variables.<br />

2. Name two graphs which show an inverse relationship between variables.<br />

3. Name three graphs which show a linear relationship between variables.<br />

4. Name two graphs which show non-linear relationships.<br />

5. In which graph does a change in the x-variable cause NO CHANGE in the y-variable?<br />

6. Which graph shows a stronger relationship between variables, graph A or graph F?<br />

1.4


Name: Date:<br />

2.1 Scientific Processes<br />

The scientific method is a process that helps you find answers to your questions about the world. The process<br />

starts with a question and your answer to the question based on experience and knowledge. This “answer” is<br />

called your hypothesis. The next step in the process is to test your hypothesis by creating experiments that can be<br />

repeated by other people in other places. If your experiment is repeated many times with the same results and<br />

conclusions, these findings become part of the body of scientific knowledge we have about the world.<br />

1. Ask a question.<br />

2. Formulate a hypothesis.<br />

Read the following story. You will use this story to practice using the scientific method.<br />

Maria and Elena are supposed to help their mom chill some soda by putting the cans into a bucket filled with<br />

ice cubes, but Maria forgot to fill the ice cube trays. Elena says that she remembers reading somewhere that<br />

hot water freezes faster than cold water. Maria is skeptical. She learned in science class that the hotter the<br />

liquid, the faster the molecules are moving. Since hot water molecules have to slow down more than cold<br />

water molecules to become ice, Maria thinks that it will take hot water longer to freeze than cold water.<br />

The girls decide to conduct a scientific experiment to determine whether it is faster to make ice cubes with<br />

hot water or cold water.<br />

Now, answer the following questions about the process they used to reach their conclusion.<br />

Asking a question<br />

1. What is the question that Maria and Elena want to answer by performing an experiment?<br />

Formulate a hypothesis<br />

2. What is Maria’s hypothesis for the experiment? State why Maria thinks this is a good hypothesis.<br />

Design and conduct an experiment<br />

Steps to the Scientific Method<br />

3. Design an experiment to test your hypothesis.<br />

4. Conduct an experiment to test your hypothesis and collect the data.<br />

5. Analyze data.<br />

6. Make a tentative conclusion.<br />

7. Test your conclusion, or refine the question, and go through each step again.<br />

3. Variables: There are many variables that Maria and Elena must control so that their results will be valid.<br />

Name at least four of these variables.<br />

2.1


Page 2 of 2<br />

4. Measurements: List at least two types of measurements that Maria and Elena must make during<br />

their experiment.<br />

5. Procedure: If Maria and Elena want their friends to believe the results of their experiment, they<br />

need to conduct the experiment so that others could repeat it. Write a procedure that the girls could follow to<br />

answer their question.<br />

Collect and analyze data<br />

The girls conducted a carefully controlled experiment and found that after 3 hours and 15 minutes, the hot water<br />

had frozen solid, while the trays filled with cold water still contained a mixture of ice and water. They repeated<br />

the experiment two more times. Each time the hot water froze first. The second time they found that the the hot<br />

water froze in 3 hours and 30 minutes. The third time, the hot water froze in 3 hours and 0 minutes.<br />

6. What is the average time that it took for hot water in ice cube trays to freeze?<br />

7. Why is it a good idea to repeat your experiments?<br />

Make a tentative conclusion<br />

8. Which of the following statements is a valid conclusion to this experiment? Explain your reasoning for<br />

choosing a certain statement.<br />

a. Hot water molecules don’t move faster than cold water molecules.<br />

b. Hot water often contains more dissolved minerals than cold water, so dissolved minerals must help<br />

water freeze faster.<br />

c. Cold water can hold more dissolved oxygen than hot water, so dissolved oxygen must slow down the<br />

rate at which water freezes.<br />

d. The temperature of water affects the rate at which it freezes.<br />

e. The faster the water molecules are moving, the faster they can arrange themselves into the nice, neat<br />

patterns that are found in solid ice cubes.<br />

Test your conclusion or refine your question<br />

Maria and Elena are pleased with their experiment. They ask their teacher if they can share their findings with<br />

their science class. The teacher says that they can present their findings as long as they are sure their conclusion<br />

is correct.<br />

The last step of the scientific method is important. At the end of any set of experiments and before you present<br />

your findings, you want to make sure that you are confident about your work.<br />

9. Let’s say that there is a small chance that the results of the experiment that Maria and Elena performed were<br />

affected by the kind of freezer they used in the experiment. What could the girls do to make sure that their<br />

results were not affected by the kind of freezer they used?<br />

10. Statement 8(b) suggests a possible reason why temperature affects the speed at which water freezes. Refine<br />

your original question for this experiment. In other words, create a question for an experiment that would<br />

prove or disprove statement 8(b).<br />

2.1


Name: Date:<br />

2.1 What’s Your Hypothesis?<br />

After making observations, a scientist forms a question based on observations and then attempts to answer that<br />

question. A guess or a possible answer to a scientific question based on observations is called a hypothesis. It is<br />

important to remember that a hypothesis is not always correct. A hypothesis must be testable so that you can<br />

determine whether or not it is correct.<br />

In science class your teacher has told you that the ability of a river to transport material depends on how fast the<br />

river is flowing. Imagine the river has three speeds—slow, medium, and fast. Now, imagine the river bottom has<br />

sand, marble-sized pebbles, and baseball-sized rocks. Come up with a hypothesis for the answer to the following<br />

question. Then, justify your reasoning:<br />

Research question: At which flow rate—slow, medium, or fast—would a river be able to transport baseballsized<br />

rock?<br />

Example hypothesis and justification: The river would have to be flowing at a fast flow rate to be able to<br />

transport baseball-sized rocks. It takes more force to move larger rocks than small pebbles and sand. Fast flowing<br />

water has more pushing force than medium or slow flowing water. I know this from an experience I had wading<br />

in a river one time. As I waded from still water to areas where the river was flowing faster, I could feel the water<br />

pushing against my legs more and more.<br />

1. You left a glass full of water by a window in your house in the morning. Three hours later you walk by the<br />

glass, and the water level is noticeably lower than it was in the morning. You have made the observation that<br />

the water level in the cup is lower. Then, you ask the following question: “Why is the water level in the cup<br />

lower?”<br />

What is a possible hypothesis you could make?<br />

2.1


Page 2 of 2<br />

2. Your teacher shows you a demonstration in which there is a box with two chimneys. Under one<br />

chimney is a lit candle, and under the other chimney is smoke from burning incense. You observe<br />

2.1<br />

that the smoke always goes towards the candle and then exits the box from the chimney above the<br />

candle. You ask the following question: “Why does the smoke go toward the candle and leave the chimney<br />

above the candle?”<br />

What is a possible hypothesis you could make?<br />

3. You have learned in science class that evaporation is a process that describes when a liquid turns to a gas at<br />

a temperature below the boiling point. You are now about to investigate evaporation and factors that may<br />

increase the rate at which it occurs. You ask the question, “What causes the evaporation rate of water to<br />

increase?”<br />

What is a possible hypothesis you could make?<br />

4. Rivers and streams flow at various speeds. You ask, “What factors increase the flow rate of a river?”<br />

What is a possible hypothesis you could make?<br />

5. It is late fall and you notice that flower bulbs in your yard have been dug up and some have been eaten. You<br />

ask, “What has happened?”<br />

What is a possible hypothesis you could make?<br />

6. You know that sea otters eat sea urchins and that sea urchins eat kelp. You ask, “What would happen to this<br />

ecosystem if all the kelp died?”<br />

What is a possible hypothesis you could make?<br />

7. In Alaska, lynx (wild cats) are predators of the snowshoe hare. In the wintertime, the coat of the snowshoe<br />

hare turns from brown to white. You ask, “Why does the snoeshoe hare change color in the winter?”<br />

What is a possible hypothesis you could make?<br />

8. In the deserts of the southwestern United States, coyotes are dog-like animals that eat many different things<br />

such as small animals and cactus fruit. They are also scavengers, which means they eat dead and decaying<br />

animals. You ask, “Are there coyote-like animals that serve as predator-scavengers in other deserts on other<br />

continents?”<br />

What is a possible hypothesis you could make?


Name: Date:<br />

2.2 Recording Observations in the Lab<br />

How do you record valid observations for an experiment in the lab?<br />

When you perform an experiment you will be<br />

making important observations. You and others<br />

will use these observations to test a hypothesis.<br />

In order for an experiment to be valid, the<br />

evidence you collect must be objective and<br />

repeatable. This investigation will give you<br />

practice making and recording good<br />

observations.<br />

Making valid observations<br />

Valid scientific observations are objective and repeatable. Scientific observations are limited to one's senses and<br />

the equipment used to make these observations. An objective observation means that the observer describes only<br />

what happened. The observer uses data, words, and pictures to describe the observations as exactly as possible.<br />

An experiment is repeatable if other scientists can see or repeat the same result. The following exercise gives you<br />

practice identifying good scientific observations.<br />

Exercise 1<br />

Materials<br />

• Paper<br />

• Pencil<br />

• Calculator<br />

• Ruler<br />

1. Which observation is the most objective? Circle the correct letter.<br />

a. My frog died after 3 days in the aquarium. I miss him.<br />

b. The frog died after 3 days in the aquarium. We will test the temperature and water conditions to<br />

find out why.<br />

c. Frogs tend to die in captivity. Ours did after three days.<br />

2. Which observation is the most descriptive? Circle the correct letter.<br />

a. After weighing 3.000 grams of sodium bicarbonate into an Erlenmeyer flask, we slowly added<br />

50.0 milliliters of vinegar. The contents of the flask began to bubble.<br />

b. We weighed the powder into a glass container. We added acid. It bubbled a lot.<br />

c. We saw a fizzy reaction.<br />

3. Which experiment has enough detail to repeat? Circle the correct letter.<br />

a. Each student took a swab culture from his or her teeth. The swab was streaked onto nutrient agar<br />

plates and incubated at 37 C.<br />

b. Each student received a nutrient agar plate and a swab. Each student performed a swab culture of<br />

his or her teeth. The swab was streaked onto the agar plate. The plates were stored face down in<br />

the 37 C incubator and checked daily for growth. After 48 hours the plates were removed from the<br />

incubator and each student recorded his or her results.<br />

c. Each student received a nutrient agar plate and a swab. Each student performed a swab culture of<br />

his or her teeth. The swab was streaked onto the agar plate. The plates were stored face down in<br />

the 37 C incubator and checked daily for growth. After 48 hours the plates were removed from the<br />

incubator and each student counted the number of colonies present on the surface of the agar.


Page 2 of 4<br />

Recording valid observations<br />

As a part of your investigations you will be asked to record observations on a skill sheet or in the results<br />

section of a lab report. There are different ways to show your observations. Here are some examples:<br />

1. Short description: Use descriptive words to explain what you did or saw. Write complete sentences. Give as<br />

much detail as possible about the experiment. Try to answer the following questions: What? Where? When?<br />

Why? and How?<br />

2. Tables: Tables are a good way to display the data you have collected. Later, the data can be plotted on a<br />

graph. Be sure to include a title for the table, labels for the sets of data, and units for the values. Check values<br />

to make sure you have the correct number of significant figures.<br />

Year<br />

manufactured<br />

Mass<br />

(grams)<br />

U.S. penny mass by year<br />

1977 1978 1979 1980 1981 1982 1983 1984 1985<br />

3.0845 3.0921 3.0689 2.9915 3.0023 2.5188 2.5042 2.4883 2.5230<br />

3. Graphs and charts: A graph or chart is a picture of your data. There are different kinds of graphs and<br />

charts: line graphs, trend charts, bar graphs, and pie graphs, for example. A line graph is shown below.<br />

Label the important parts of your graph. Give your graph a title. The x-axis and y-axis should have labels for<br />

the data, the unit values, and the number range on the graph.<br />

The line graph in the example has a straight line through the data. Sometimes data does not fit a straight line.<br />

Often scientists will plot data first in a trend chart to see how the data looks. Check with your instructor if<br />

you are unsure how to display your data.


Page 3 of 4<br />

4. Drawings: Sometimes you will record observations by drawing a sketch of what you see. The<br />

example below was observed under a microscope.<br />

Give the name of the specimen. Draw enough detail to make the sketch look realistic. Use color, when<br />

possible. Identify parts of the object you were asked to observe. Provide the magnification or size of the<br />

image.<br />

Exercise 2: Practice recording valid observations<br />

A lab report form has been given to you by your instructor. This exercise gives you a chance to read through an<br />

experiment and fill in information in the appropriate sections of the lab report form. Use this opportunity to<br />

practice writing and graphing scientific observations. Then answer the following questions about the experiment.<br />

A student notices that when he presses several pennies in a pressed penny machine, his brand new penny has<br />

some copper color missing and he can see silver-like material underneath. He wonders, “Are some pennies made<br />

differently than others?” The student has a theory that not all U.S. pennies are made the same. He thinks that if<br />

pennies are made differently now he might be able to find out when the change occurred. He decides to collect a<br />

U.S. penny for each year from 1977 to the present, record the date, and take its mass. The student records the data<br />

in a table and creates a graph plotting U.S. penny mass vs. year. Below is a table of some of his data:<br />

Year<br />

manufactured<br />

Mass<br />

(grams)<br />

U.S. penny mass by year<br />

1977 1978 1979 1980 1981 1982 1983 1984 1985<br />

3.0845 3.0921 3.0689 2.9915 3.0023 2.5188 2.5042 2.4883 2.5230<br />

Stop and think<br />

a. What observation did the student make first before he began his experiment?


Page 4 of 4<br />

b. How did the student display his observations?<br />

c. In what section of the lab report did you show observations?<br />

d. What method did you use to display the observations? Explain why you chose this one.


Name: Date:<br />

2.2 Writing a Lab Report<br />

How do you share the results of an experiment?<br />

A lab report is like a story about an experiment. The details in the story help others learn from what you<br />

did. A good lab report makes it possible for someone else to repeat your experiment. If their results and<br />

conclusions are similar to yours, you have support for your ideas. Through this process we come to<br />

understand more about how the world works.<br />

The parts of a lab report<br />

A lab report follows the steps of the scientific method. Use the checklist below to create your own lab reports:<br />

� Title: The title makes it easy for readers to quickly identify the topic of your experiment.<br />

�<br />

�<br />

Research question: The research question tells the reader exactly what you want to find out<br />

through your experiment.<br />

Introduction: This paragraph describes what you already know about the topic, and shows how this<br />

information relates to your experiment.<br />

� Hypothesis: The hypothesis states the prediction you plan to test in your experiment.<br />

� Materials: List all the materials you need to do the experiment.<br />

�<br />

�<br />

�<br />

�<br />

Procedure: Describe the steps involved in your experiment. Make sure that you provide enough<br />

detail so readers can repeat what you did. You may want to provide sketches of the lab setup.<br />

Be sure to name the experimental variable and tell which variables you controlled.<br />

Data/Observations: This is where you record what happened, using descriptive words, data tables,<br />

and graphs.<br />

Analysis: In this section, describe your data in words. Here’s a good way to start: My data shows<br />

that...<br />

Conclusion: This paragraph states whether your hypothesis was correct or incorrect. It may suggest<br />

a new research question or a new hypothesis.<br />

A sample lab report<br />

Use the sample lab report on the next two pages as a guide for writing your own lab reports. Remember that you<br />

are telling a story about something you did so that others can repeat your experiment.


Page 2 of 3<br />

Name: Lucy O. Date: January 24, 2007<br />

Title: Pressure and Speed<br />

Research question: How does pressure affect the speed of the CPO air rocket?<br />

Introduction:<br />

Air pressure is a term used to describe how tightly air molecules are packed into a certain space. When air<br />

pressure increases, more air molecules are packed into the same amount of space. These molecules are moving<br />

around and colliding with each other and the walls of the container. As the number of molecules in the container<br />

increases, the number of molecular collisions in the container increases. A pressure gauge measures the force of<br />

these molecules as they strike a surface.<br />

In this lab, I will measure the speed of the CPO air rocket when it is launched with different amounts of initial<br />

pressure inside the plastic bottle. I want to know if a greater amount of initial air pressure will cause the air rocket<br />

to travel at a greater speed.<br />

Hypothesis: When I increase the pressure of the air rocket, the speed will increase.<br />

Materials:<br />

CPO air rocket CPO photogates<br />

CPO timer Goggles<br />

Procedure:<br />

1. I put on goggles and made sure the area was clear.<br />

2. The air rocket is attached to an arm so that it travels in a circular path.<br />

After it travels about 330°, the air rocket hits a stopper and its flight ends.<br />

I set up the photogate at 90°.<br />

3. My control variables were the mass of the rocket and launch technique, so<br />

I kept these constant throughout the experiment.<br />

4. My experimental variable was the initial pressure applied to the rocket. I<br />

tested the air rocket at three different initial pressures. The pressures that<br />

work effectively with this equipment range from 15 psi to 90 psi. I tested<br />

the air rocket at 20 psi, 50 psi, and 80 psi. I did three trials at each pressure.<br />

5. The length of the rocket wing is 5 cm. The wing breaks the photogate’s light beam. The photogate reports the<br />

amount of time that the wing took to pass through the beam. Therefore, I used wing length as distance and<br />

divide by time to calculate speed of the air rocket.<br />

6. I found the average speed in centimeters per second for each pressure.


Page 3 of 3<br />

Data/Observations:<br />

Analysis:<br />

My graph shows that the plots of the data for photogates A and B are linear. As the values for pressure increased,<br />

the speed increased also.<br />

Conclusion:<br />

Air pressure and speed of rocket<br />

Initial air pressure Time (sec) at 90° Speed (m/sec) at 90° Average speed<br />

cm/sec<br />

0.0227 2.20<br />

20 psi<br />

0.0231 2.16<br />

216<br />

0.0237 2.11<br />

0.0097 5.15<br />

50 psi<br />

0.0099 5.05<br />

510<br />

0.0098 5.10<br />

0.0060 8.33<br />

80 psi<br />

0.0064 7.81<br />

794<br />

0.0065 7.69<br />

The data shows that pressure does have an effect on speed. The graph shows that my hypothesis is correct. As the<br />

initial pressure of the rocket increased, the speed of the rocket increased as well. There is a direct relationship<br />

between pressure and speed of the rocket.


Name: Date:<br />

2.2 Using Computer Spreadsheets<br />

Computer spreadsheets provide an easy way to organize and evaluate data that you collect from an experiment.<br />

Numbers are typed into boxes called “cells.” The cells are organized in rows and columns. You can find the<br />

average of a lot of numbers or do more complicated calculations by writing formulas into the cells. Each cell has<br />

a name based on its column letter and row number. For example, the first cell in most spreadsheets is “A1.”<br />

This skill sheet will show you how to:<br />

1. Record data in a computer spreadsheet program.<br />

2. Do simple calculations for many data values at once using the<br />

spreadsheet.<br />

3. Make a graph with the data set.<br />

To complete this skill sheet, you will need:<br />

• Simple calculator<br />

• Access to a computer with a spreadsheet program<br />

1. Adding data: Open the spreadsheet program on your computer. You will see a window open that has rows<br />

and columns. The rows are numbered. The columns are identified by a letter.<br />

a. As shown in the graphic above, add headings for columns A, B, and C:<br />

cell A1, type “Time (sec)”<br />

cell B1, type “Temp (deg C)”<br />

cell C1, type “Slope”<br />

NOTE: You can change the width of the columns on your spreadsheet by clicking on the right-hand<br />

border and dragging the border to the left or right.<br />

b. <strong>High</strong>light column B. Then, go to the Format menu item and click on Cells. Make the format of these<br />

cells Number with one decimal place. <strong>High</strong>light column C and make the format of these cells Number<br />

with two decimal places.<br />

c. Type in the data for Time and Temperature as shown in the graphic above.<br />

2. Making a graph: Now, you will use the data you have added to the skill sheet to make a graph.<br />

a. Use your mouse to highlight the titles and data in columns A and B.<br />

b. Then, go to Insert and click on Chart.<br />

c. In step 1 of the chart wizard, choose the XY (Scatter) format for your chart and click “Next.”<br />

d. In step 2 of the chart wizard, you will see a graph of your data. Click “Next” again to get to step 3. Here<br />

you can change the appearance of the graph.<br />

e. In step 3 of the chart wizard, add titles and uncheck the show legend-option. In the box for the chart title<br />

write “Temperature vs. Time.” In the box for the value x-axis, write “Time (seconds).” In the box for the<br />

value for y-axis, write “Temperature (deg Celsius).”<br />

2.2


Page 2 of 3<br />

f. In step 4 of the chart wizard, click the option to show the graph as an object in Sheet 2. At this<br />

point you will finish your work with the chart wizard.<br />

g. Setting the scale on the x-axis: Place the cursor on the x-axis and double click. Set the<br />

minimum of the scale to be 0, the maximum to be 310. Set the major unit to be 100 and the minor unit to<br />

be 20. Then, click OK. Note: Make sure the boxes to the left of the changed values are NOT checked.<br />

h. Setting the scale on the y-axis.: Place the cursor on the y-axis and double click. Set the minimum of the<br />

scale to be 20, the maximum to be 41. Set the major unit to be 10 and the minor unit to be 2. Then, click<br />

OK. Note: Make sure the boxes to the left of the changed values are UNchecked.<br />

i. You are now finished with your graph. It is located on Sheet 2 of your spreadsheet.<br />

3. Performing calculations:<br />

a. Return to Sheet 1 of your spreadsheet.<br />

b. The third column of data, “Slope,” will be filled by performing a calculation using data in the other two<br />

columns.<br />

c. <strong>High</strong>light the second cell from the top in the Slope column (cell C2). Type the following and hit enter:<br />

= (B3-B2)/(A3-A2)<br />

Explanation of the formula: The equal sign (=) indicates that the information you type into the cell is a<br />

formula. The formula for the slope of a line is as follows. Do you see why the formula for cell C2 is<br />

written the way it is?<br />

y2 – y1 slope =<br />

--------------x2<br />

– x1 d. Adding the formula to all the cells: <strong>High</strong>light cell C2, then drag the mouse down the column until the<br />

cells (C2 to C11) are highlighted. Then click Edit, then Fill, then Down. The formula will copy into<br />

each cell in column C. However, the formula pattern will be appropriate for each cell. For example, the<br />

formula for C2 reads: = (B3-B2)/(A3-A2). The formula for C3 reads: = (B4-B3)/(A4-A3). Note: The<br />

“=” sign is important. Do not forget to add it to the formula.<br />

e. In column C, you will see the slope for pairs of data points. Now, answer the questions below.<br />

1. Which is the independent variable—time or temperature? Which is the dependent variable?<br />

2. When setting up the data in a spreadsheet, which data set goes in the first column, the independent variable<br />

or the dependent variable?<br />

3. Use the graph you created in step 2 of the example to describe the relationship between temperature and the<br />

time it takes to heat up a volume of water.<br />

4. Look at the values for slope. How do these values change for the graph of temperature versus time?<br />

2.2


Page 3 of 3<br />

5. The following data is from an experiment in which the temperature of a substance was taken as it<br />

was heated. Transfer this data into a spreadsheet file and make an XY(Scatter) graph.<br />

Time (seconds)<br />

Independent data<br />

Temperature (°C)<br />

Dependent data<br />

10 7.5<br />

20 10.8<br />

30 11.6<br />

40 11.9<br />

50 13.3<br />

60 21.9<br />

70 26.3<br />

80 26.6<br />

90 29.1<br />

100 31.1<br />

6. Use the following data set to make a graph in a spreadsheet program. Find the slope for pairs of data points<br />

along the plot of the graph. Is the slope the same for every pair of points?<br />

Independent<br />

data<br />

Dependent<br />

data<br />

1 5<br />

2 7<br />

2.5 8<br />

3.2 9.4<br />

1.5 6<br />

0.5 4<br />

4 11<br />

2.8 8.6<br />

4.2 11.4<br />

5 13<br />

2.2


Name: Date:<br />

2.2 Identifying Control and Experimental Variables<br />

An experiment is a situation set up to investigate relationships between variables. In a simple ideal experiment<br />

only one variable is changed at a time. You can assume that changes you see in other variables were caused by<br />

the one variable you changed. The variable you change is called the experimental variable. This is usually the<br />

variable that you can freely manipulate. For example, if you want to know if the mass of a toy car affects its<br />

speed down a ramp, the experimental variable is the car’s mass. You can add “cargo” to change the mass of car.<br />

The variables that you keep the same are called control variables. In the toy car experiment, control variables<br />

include the angle of the ramp, photogate positions, and release technique.<br />

Use this skill sheet to practice identifying control and experimental variables.<br />

• Alex is studying the effect of sunlight on plant growth. His hypothesis is that plants that are exposed to<br />

sunlight will grow better than plants that are not exposed to sunlight. In order to test his hypothesis, he<br />

follows the following procedures. He obtains two of the same type of plant, puts them in identical pots with<br />

potting soil from the same bag. Then he puts one plant in the sunlight and the other in a dark room. He<br />

waters the plants with 200 mL of water every other day for two weeks.<br />

Solution:<br />

The experimental variable in the experiment is the light exposure of the plant. One plant is put in sunlight<br />

and the other is put in darkness. The control variables are the type of plant, the pot, the soil, amount of water,<br />

and the time of the experiment.<br />

1. Julie sees commercials for antibacterial products that claim to kill almost all the bacteria in the area that has<br />

been treated with the product. Julie asks, “How effective is antibacterial cleaner in preventing the growth of<br />

bacteria?” She sets up an experiment in order to study the effectiveness of antibacterial products. Julie<br />

hypothesizes that the antibacterial soap will prevent bacterial growth. In her experiment, she follows the<br />

following procedure.<br />

a. Obtain two Petri dishes with nutrient agar.<br />

b. Rub a cotton swab along the surface of a desk at school. Then, carefully rub the nutrient agar with the<br />

cotton swab without breaking the gel.<br />

c. Repeat the same process with the other Petri dish.<br />

d. Spray one of the Petri dishes with an antibacterial kitchen spray.<br />

e. Carefully tape shut both of the Petri dishes and place them in an incubator.<br />

f. Check the Petri dishes and record the results once a day for one week.<br />

Identify the experimental variable and three control variables in the experiment.<br />

2.2


Page 2 of 2<br />

2. John notices that his mom waters the plants in their house every other day. He asks, “Will plants<br />

grow if they are not watered regularly?” He hypothesizes that plants that are not watered regularly<br />

will not grow as large as plants that are watered regularly. In order to test his hypothesis, he<br />

conducts the following experiment.<br />

a. Obtain two healthy plants of the same variety and size.<br />

b. Plant each plant in the same type of pot and the same brand of potting mix.<br />

c. Place both plants in the same window of the house.<br />

d. Water one of the plants every other day with 250 mL of water.<br />

e. Water the other plant once a week with 250 mL of water.<br />

f. Measure the height of the plants once a day for one month.<br />

Identify the experimental variable and three control variables in the experiment.<br />

3. Mike’s dad always buys bread with preservatives because he says it lasts longer. Mike asks, “Will bread with<br />

preservatives stay fresh longer than bread without preservatives?” He hypothesizes that bread with<br />

preservatives will not grow mold as quickly as bread without preservatives. In order to test his hypothesis, he<br />

conducts the following experiment.<br />

a. Obtain one slice of bread containing preservatives and one slice of bread without any preservatives.<br />

b. Dampen two paper towels. Fold the paper towels so that they will lay flat inside a zipper-top bag.<br />

c. Lay each paper towel inside a separate zipper-top bag.<br />

d. Place one slice of bread in each bag and seal the bags.<br />

e. Place bags with bread and paper towels in a dark environment for one week.<br />

f. Record mold growth once a day for one week.<br />

Identify the experimental variable and three control variables in the experiment.<br />

4. In science class, Kathy has been studying protists. She has been learning specifically about protists called<br />

algae that live in ponds. She knows that algae thrive when there are plenty of nutrients available for them.<br />

Kathy asks, “Will water that has been treated with fertilizer have more algae than water that has not been<br />

treated with fertilizer?” In order to test her hypothesis, Kathy does the following experiment.<br />

a. Obtain a sample of algae from the teacher.<br />

b. Obtain two beakers with 500 mL of water in each beaker.<br />

c. Put one teaspoon of plant fertilizer in one of the beakers.<br />

d. Put an equal amount of algae sample in each of the beakers.<br />

e. Place the beakers in a sunny window for two weeks.<br />

f. Using a microscope, examine algae growth in each of the beakers every other day for the two weeks and<br />

record your results.<br />

Identify the experimental variable and three control variables in the experiment.<br />

2.2


Name: Date:<br />

3.1 Position on the Coordinate Plane<br />

To describe any location in two dimensions, we use a grid called the coordinate plane. You can describe any<br />

position on the coordinate plane using two numbers called coordinates, which are shown in the form of (x, y).<br />

These coordinates are compared to a fixed reference point called the origin. The table below describes the x and<br />

y coordinates:<br />

Your home is at the origin, and a park is located 2 miles north and 1 mile east of your home.<br />

• Show your home and the park on a coordinate plane, and give the coordinates for each.<br />

• After you go to the park, you drive 2 miles east and 1 mile north to the grocery store. What are the<br />

coordinates of the grocery store?<br />

Solution<br />

If your home is at the origin, it is given the coordinates<br />

(0, 0). By counting over 1 box from the origin in the<br />

positive x-direction and up 2 boxes in the positive<br />

y-direction, you can place the park on the coordinate<br />

plane. The park’s coordinates are (+1 mile, +2 miles).<br />

From the park, count over 2 more boxes in the positive<br />

x-direction and up one more 1 box in the positive<br />

y-direction to place the grocery store. That makes the<br />

grocery store’s coordinates (+3 miles, +3 miles).<br />

_<br />

Coordinate Which axis is it on? Which is the positive<br />

direction?<br />

Which is the negative<br />

direction?<br />

x horizontal, called the x-axis right or east left or west<br />

y vertical, called the y-axis up or north down or south<br />

1. You are given directions to a friend’s house from your school. They read: “Go east one block, turn north and<br />

go 4 blocks, turn west and go 1 block, then go south for 2 blocks.” Using your school as the origin, draw a<br />

map of these directions on a coordinate plane. What are the coordinates of your friend’s house?<br />

2. A dog starts chasing a squirrel at the origin of a coordinate plane. He runs 20 meters east, then 10 meters<br />

north and stops to scratch. Then he runs 10 meters west and 10 meters north, where the squirrel climbs a tree<br />

and gets away.<br />

a. Draw the coordinate plane and trace the path the dog took in chasing the squirrel.<br />

b. Show where the dog scratched and where the squirrel escaped, and give coordinates for each.<br />

3. Does the order of the coordinates matter? Is the coordinate (2, 3) the same as the coordinate (3, 2)? Explain<br />

and draw your answer on a coordinate plane.<br />

3.1


Name: Date:<br />

3.1 Latitude and Longitude<br />

History: Latitude and longitude are part of a grid system that describes the<br />

location of any place on Earth. When formalized in the mid-18th century, the<br />

idea of a grid system was not a new one. More than 2000 years ago, ancient<br />

Greeks drew maps with grids that looked much like our maps today. Using<br />

mathematics and logic, they postulated that Earth could be mapped in degrees<br />

north and south of the Equator and east and west of a line of reference. From<br />

the ancient times, geographers and navigators used devices such as the crossstaff,<br />

astrolabe, sextant, and astronomical tables to determine latitude. But<br />

determining longitude required accurate timepieces, and they were not reliably<br />

designed until the 1700’s.<br />

Latitude: Think of Earth as a transparent sphere, just as the ancient Greeks<br />

did. Now imagine yourself standing so that your eyes are at the center of that<br />

sphere. If you tip your head back and look straight up, you will see the North<br />

Pole above you. If you look straight down, you will see the South Pole below<br />

you. If you turn around while looking straight out at the middle of the sphere, your eyes will follow the Equator,<br />

the line around the middle of Earth. The ancient Greeks realized that they could describe the location of any place<br />

by using its angle from the Equator as measured from that imaginary place at the center of Earth.<br />

All latitude lines run parallel to the Equator, creating circles that get smaller and smaller until they encircle the<br />

Poles. Because latitude lines never intersect, latitude lines are sometimes referred to as parallels.<br />

At first, you might be confused because when latitude lines are placed on a map. They<br />

appear to run from the left side of the page to the right. You might think they measure<br />

east and west, but they don't. The graphic at the right shows latitude lines. If you think<br />

of them as steps on a ladder, then you will see the lines are taking you “up” toward the<br />

north or “down” toward the south. (Of course, there is no real “up” or “down” on a<br />

map or globe, but the association of LAdder and LAtitude may help you.)<br />

The Equator is designated as 0º. The North Latitude lines measure from the Equator<br />

(0º) to the North Pole (90ºN). The South Latitude lines measure from the Equator (0º)<br />

to the South Pole (90ºS). There are other special latitude lines to note. The Tropic of<br />

Cancer is at 23.5ºN latitude, and at 23.5ºS latitude is the Tropic of Capricorn. These lines represent the farthest<br />

north and farthest south where the sun can shine directly overhead at noon. Latitudes of 66.5º N and 66.5º S mark<br />

the Arctic and Antarctic Circles, respectively. Because of the tilt of the Earth, there are winter days when the Sun<br />

does not rise and summer days when the sun does not set at these locations.<br />

Longitude: Now imagine yourself back in the transparent sphere. Look up at the<br />

North Pole and begin to draw a continuous line with your eyes along the outside of the<br />

sphere to the South Pole. Turn to face the opposite side of the sphere and draw a line<br />

from the South Pole to the North Pole. These lines, and all other longitude lines, are<br />

the same length because they start and end at the poles. Look at the graphic below and<br />

see that although longitude lines are drawn from north to south, they measure distance<br />

from east or west.<br />

3.1


Page 2 of 4<br />

There are no special longitude lines, so geographers had to choose one<br />

from which to measure east and west. Longitude lines are also called<br />

3.1<br />

meridians, so this special line is called the Prime Meridian and is labeled<br />

0º. The ancient Greeks chose a Prime Meridian that passed through the<br />

Greek Island of Rhodes. In the 1700's, the French chose one that passed<br />

through Ferro, an island in the Canary Islands. There are maps that show<br />

that America even used Philadelphia as their special reference location.<br />

But in 1884, the International Meridian Conference met in Washington,<br />

DC. They chose to adopt a Prime Meridian that passes through an<br />

observatory in Greenwich, England. At the same conference, they also<br />

determined a point exactly opposite of the Prime Meridian. This second<br />

important longitude line is the 180º meridian. Longitude lines measure eastward and westward from the Prime<br />

Meridian (0º) to the180º meridian. Superimposed on the 180º meridian is the International Dateline. This special<br />

line does not follow the 180º meridian exactly. It zigzags a bit to stay in the ocean, which is an unpopulated area.<br />

International agreements dictate that the date changes on either side of the Dateline.<br />

GPS and decimal notations. In the past, latitude and longitude lines always had measurement labels of degrees<br />

(º), minutes ( ' ), and seconds ( "). The labels of “minutes” and “seconds” did not denote time in these cases.<br />

Instead they described places between whole degrees of longitude or latitude more exactly. For example,<br />

consider Sacramento, CA. Traditionally, its location was said to be at 38º 34' 54"N (38 degrees, 34 minutes,<br />

54 seconds North) and 121º 29' 36"W (121 degrees 29 minutes, 36 seconds West). Now GPS (Global Positioning<br />

System), in decimal notation would say Sacramento is located at 38.58ºN and 121.49ºW. Note: As a matter of<br />

custom when giving locations, latitude is listed first and longitude second.<br />

Finding a Location on a Globe<br />

You can find any location by using latitude and longitude on a globe. See the<br />

example in the diagram. The position is 70ºN and 40ºW. First on the globe, you<br />

would find the latitude line 70ºN, seventy degrees north of the Equator. Next you<br />

would find the longitude line 40ºW, forty degrees west of the Prime Meridian. Trace<br />

the lines with your fingers. Where they intersect, you will find the location. In this<br />

case, you have located Greenland.<br />

Finding a Location on a Map<br />

You use the same procedure to find any location on a map. Look at the graphic<br />

below. The position is 10ºS and 160ºE. First you would find the latitude line<br />

10ºS, ten degrees south of the Equator. Next you would find the longitude<br />

160ºE, one hundred-sixty degrees east of the Prime Meridian. You have<br />

located the Solomon Islands.


Page 3 of 4<br />

Use an atlas or globe to answer these practice questions.<br />

1. What country will you find at the following latitude and longitude?<br />

a. 65ºN 20ºW<br />

b. 35ºN 5º E<br />

c. 50º S 70ºW<br />

d. 20ºS 140ºE<br />

e. 40ºS 175ºE<br />

2. What body of water will you find at the following latitude and longitude?<br />

a. 20ºN 90ºW<br />

b. 40ºN 25ºE<br />

c. 20ºN 38º E<br />

d. 25ºN 95ºW<br />

e. 0ºN 60ºW<br />

Converting Traditional Notation To Decimal Notation<br />

Sometimes you need to convert the traditional notation of degrees, minutes, and seconds into decimal notation.<br />

First you must understand this traditional notation, which was a base-60 system.<br />

Let's look at 34º 15' (thirty-four degrees 15 minutes).<br />

One degree = 60 minutes<br />

One minute = 60 seconds<br />

One degree = 3,600 seconds (60 × 60)<br />

Regardless of the system, the notation will begin with 34 degrees. To change the minutes into a decimal, you<br />

must divide 15 by 60, the number of minutes in one degree (15/60). The answer is 0.25. Therefore, the decimal<br />

notation would be 34.25º or thirty-four and twenty-five hundredths degrees.<br />

Let's look at 12º 20' 38" (twelve degrees, twenty minutes, thirty-eight seconds). We know the notation will begin<br />

with 12 degrees. Next we have to convert the 20 minutes into seconds (20 × 60 = 1,200 seconds. Then we add the<br />

38 seconds for a total of 1,238 seconds. There are 3,600 seconds in one degree, so you must divide 1,238 by<br />

3,600. (1,238 / 3,600). The answer is 0.34. Therefore the decimal notation would be 12.34º or twelve and thirtyfour<br />

hundredths degrees.<br />

3.1


Page 4 of 4<br />

3. Convert the following latitudes in traditional notation to decimal notation. (Round your answer to<br />

the nearest hundredth.)<br />

a. 30º 20' N<br />

b. 45º 45' N<br />

c. 20º 36' 40" S<br />

d. 60º 19' 38" S<br />

4. Convert the following longitudes in traditional notation to decimal notation. (Round your answer to the<br />

nearest hundredth.)<br />

a. 25º 55' E<br />

b. 145º 15' E<br />

c. 130º 37' 10" W<br />

d. 85º 26' 8" W<br />

3.1


Name: Date:<br />

3.1 Map Scales<br />

Mapmakers have developed a tool that allows them to accurately draw the entire world on a single piece of paper.<br />

It’s not magic—it’s drawing to scale. To be useful, maps must be accurately drawn to scale. The mapmaker must<br />

also reveal the scale that was used so that a map-reader can appreciate the larger real-life distances. The scale is<br />

usually written in or near the map’s legend or key.<br />

There are three kinds of map scales: fractional, verbal, and bar scales. A fractional scale shows the relationship of<br />

the map to actual distance in the form of a fraction. A scale of 1/100,000 means that one centimeter on the map<br />

represents 100,000 centimeters (or 1 kilometer) of real life distance.<br />

A verbal scale expresses the relationship using words. For example, “1 centimeter equals 500 kilometers.” This is<br />

a more usable scale, especially with large real-life distances. With a scale of 1 cm = 500 km, you could make a<br />

scale drawing of North America on one piece of paper.<br />

A bar scale is the most user-friendly scale tool of all. It is simply a bar drawn on the map with the size of the bar<br />

equal to a distance in real life. Even if you do not have a ruler, you can measure distances on the map with a bar<br />

scale. Just line up the edge of an index card under the bar scale and transfer the vertical marks to the card. Label<br />

the distances each mark represents. You can then move the card around your map to determine distances. You<br />

might wonder what you should do if a location falls between the vertical interval lines. You must use estimation<br />

to determine that distance. Be careful to look at the scale before estimating. For example, if the distance falls<br />

half-way between the 10 and the 20 kilometer scale marks, estimate 15 kilometers. If the scale is different and the<br />

distance falls half-way between 30 and 60 kilometer marks, you must estimate 45 kilometers.<br />

Let’s explore the different kinds of scales. You will need centimeter graph paper, plain paper, an index card, and<br />

a centimeter ruler or measuring tape for these activities.<br />

Fractional scale<br />

Materials: Graph paper<br />

Trace a simple object on a piece of graph paper. (A large paper clip, pen, small scissors, or paperback <strong>book</strong> works<br />

well.) Now make a scale drawing of the object using the fractional scale of 1/4. (Remember this means that for<br />

every 4 blocks occupied in the original tracing, you will have only one block on the scale drawing.) Be sure to<br />

label the scale on the finished scale drawing.<br />

3.1


Page 2 of 3<br />

Verbal scale<br />

Materials: Centimeter graph paper and centimeter measuring tape or ruler.<br />

You are going to make a scale drawing of a person. Ask a classmate to stand against a wall with his/her arms<br />

outstretched to the side. Take 4 measurements in centimeters: 1) Distance from top of head to floor. 2) Distance<br />

from right finger tip to left finger. 3) Distance from top of head to shoulders. 4) Distance from shoulders to waist.<br />

Write the verbal scale of “1 cm equals 10 cm” on the bottom of a piece of centimeter graph paper. On that paper<br />

translate the measurements that your took into a simple figure drawing of your classmate. (Remember if a<br />

measurement is 25 centimeters in real life, you will have to make a drawing that is within 2 ½ centimeter blocks<br />

of the graph paper.)<br />

Bar scale<br />

Materials: Plain paper and centimeter ruler or tape.<br />

Position the paper so that it is wider than it is long.<br />

Draw a bar scale that is a total of 6 centimeters long at bottom of the paper. Make four vertical marks at 0 cm,<br />

2 cm, 4 cm, and 6 cm. Write “0”over the first mark, “50” over the second, “100” over the third and “150” of the<br />

last mark. Underneath the bar write “Kilometers.” Mark “N” for north at the top of your paper. Now use your<br />

imagination to draw an island (of any shape) that is 450 miles long and 200 miles wide at its widest point. Draw<br />

a star to mark the capital city, which is located on the northern coast 100 miles from the west end of the island.<br />

1. Answer these questions about Andora and Calypso:<br />

a. Which island appears bigger?<br />

b. Can you tell whether you can ride a bike in one day<br />

from point A to point B on either map? If no, why not?<br />

c. Measure the distance from the center of the dot to the<br />

right of A to the center of the dot to the left of B on<br />

both maps. Are the measurements the same?<br />

d. If the measurements are the same on both maps, does<br />

that mean the distance from point A to point B is the<br />

same on both maps? Explain your answer.<br />

e. Let’s write in the scale for these two islands. Write the scale of 1cm = 5 km on Andora. Write the scale<br />

of 1 cm = 1000 km on Calypso. Now answer the question, which island is bigger?<br />

2. On the next page is a map of Monitor Island. Use the bar scale to find distances on this island. Assume that<br />

you are measuring “as the crow flies.” That means from point to point by air because there are no roads to<br />

follow. Always measure from dot to dot, and be sure to label your answers in kilometers.<br />

a. Point L to Point M<br />

b. Point Y to Point Z<br />

c. Point Z to Point P<br />

d. Point P to Point M<br />

e. Point Q to Point T<br />

3.1


Page 3 of 3<br />

f. Point Q to Point M<br />

g. Point T to Point M<br />

h. Point Z to Point P to Point X<br />

i. Point X to Point L to Point M<br />

j. Point X to Point P to Point Z to Point Y<br />

Bonus: Measure around the coast of Monitor Island. It’s hard to be exact, but write your best estimate.<br />

3.1


Name: Date:<br />

3.1 Navigation Project<br />

Nautical charts have long been used by ship captains to navigate the oceans. As land has been increasingly<br />

developed and harbors built, more and more information is needed to safely navigate near shore. Additionally,<br />

offshore shallow banks, reefs, islands, seamounts, and other obstructions needed to be identified so that they<br />

don’t hinder the passage of boats.<br />

In this project, you and two other captains will navigate through the waters around Puerto Rico and some of the<br />

Virgin Islands using three real nautical maps. Your journey includes a stop at Isla de Vieques, which was a US<br />

Navy testing ground for bombs, missiles, and other weapons. It was vacated in May 2003 and now is used by<br />

locals and tourists. Bon Voyage.<br />

Materials:<br />

• NOAA map #25640<br />

(laminated)<br />

• NOAA map #25641<br />

(laminated)<br />

• NOAA map #25647<br />

(laminated)<br />

Getting started:<br />

1. Have all three maps accessible.<br />

2. Before beginning your imaginary journey, spend some time studying the maps. Look at any legends<br />

(example: note on pipelines and cables), abbreviation lists, and Notes (such as Note E on map 25640). Look<br />

at the map scale. Note whether the soundings are in fathoms or feet.<br />

3. Note that the maps are laminated, so you can use an erasable marker to outline your path.<br />

Making predictions:<br />

Note:<br />

Laminated maps are<br />

available from NOAA<br />

(www.noaa.gov) or<br />

boating/marine supply<br />

stores, as well as some<br />

Coast Guard Stations.<br />

• Internet access<br />

• Erasable overhead<br />

projector marker<br />

a. What kind of ecosystem do you expect to find in these warm, sunlit waters?<br />

b. What does this mean about navigating this area?<br />

3.1


Page 2 of 5<br />

It’s time to go!<br />

3.1<br />

1. You and your two partners are tri-captains on a boat that is 12 feet deep. On board, you have a<br />

small row boat. Besides your clothes and toiletries for the trip, you will bring along wading boots, a solar<br />

still, a radio, your three maps, water, and food.<br />

2. You will be traveling from the west coast of Puerto Rico, eventually ending your trip on the island of St.<br />

John. As captains, you will be making decisions about the course the boat will be taking based on directions<br />

given below. You will need to look out for (among other things) shallow water, pipelines, and other<br />

obstructions. Listen to what the map is telling you.<br />

3. Let’s start with map 25640. What is the scale of this map?<br />

4. What does that mean?<br />

5. How many feet are there in a fathom? Hint: The answer is outside the border of the map.<br />

6. Find Punta Higuero on the west coast of Puerto Rico. What is located here? Use your abbreviations. You<br />

will probably have to look it up.<br />

7. You will now be moving south along the west coast and then the south coast of Puerto Rico. Notice the light<br />

blue area around the coast. At the seaward edge of this area is a line. Every few inches along this line, you<br />

will see a number 10. What this means is that anywhere along this line the depth of the water is 10 fathoms.<br />

Remember, your boat is 12 feet deep. How many fathoms is this?<br />

8. So your boat is fine anywhere along the line. However, as you head toward the coast from this 10-fathom<br />

line, the depth decreases, but since the depth is not marked again, you do not know how quickly it decreases<br />

and thus can't take your boat any closer to the shore. Remember this as you travel. So start traveling south.<br />

What do you encounter near the Bahia de Mayaguez?<br />

9. What does this mean?<br />

10. Is the depth of the water still suitable for traveling?<br />

11. Travel around the marine conservation district. Should anyone be fishing here?<br />

12. Can you pass between Bajas Gallardo and the Marine Conservation District? If so, trace the path through<br />

and if not, find another way around towards the south shore.<br />

13. Stay near to shore so you can have great views of the beautiful shallow blue waters. Find Punta Cayito and<br />

Punta Barrancas on the south shore. In the area offshore, there is a section between the 10 fathoms line and<br />

the next depth line of 100 fathoms were there are several abbreviated notations. Name three by noting the<br />

abbreviation and what it means.<br />

14. Find the lighthouse near Cayos de Ratones. What type of lighthouse is it and why is that different than<br />

occulting?<br />

15. 5M means that it can be seen for 5 nautical miles, which is 1.852 kilometers or 1.15 miles.<br />

16. As you travel towards the southeastern coast of Puerto Rico, what area in a square dashed purple box do you<br />

see?<br />

17. Do you think it would be a good or bad idea to drop anchor there?


Page 3 of 5<br />

18. Head to Isla de Vieques. There are supposed to be two beautiful bays that are filled with organisms<br />

that are bioluminescent. These one-celled organisms give off a blue-green glow when disturbed.<br />

You'll have to wait here until night-time in order to see this natural wonder. Can you take your<br />

ship right up to the shore? If not, what can you do to get there?<br />

19. How many lighthouses are there on the Island?<br />

20. Two lighthouses are flashing. One is occulting. What is the fourth, what does the symbol mean, and what<br />

two colors are associated with it?<br />

21. How far out can you see the flashing and occulting lighthouse lights on the Island?<br />

22. Your next stop is Savana Isle, a small island just west of St. Thomas. As you travel in that direction, what do<br />

you notice there are many of in the area of the Virgin Passage?<br />

23. What does that mean you should NOT do in this area?<br />

24. Can you bring your boat in directly to Savana Isle?<br />

25. What does the (269) mean?<br />

26. Now you are going to move to map 25641. The soundings are done in what units?<br />

27. Orient yourselves for a minute. You are currently at Savana Isle. Find it on the map.<br />

28. What is the scale on this map?<br />

29. You can see that the scales on the maps are quite different. What do you notice when you look at the maps<br />

themselves. How are they different?<br />

30. When you are sailing in this area, where do you call to report spills of oil and hazardous substances? There<br />

are two choices.<br />

31. For weather information, to what station do you tune?<br />

32. From Savana Isle, head toward Cricket Rock using Salt Cay or Dutchcap Passage. How many fathoms deep<br />

is the coastline?<br />

33. Should you anchor and row in or go right up to the shore?<br />

34. How much rock is covered and uncovered?<br />

35. What are the local bottom characteristics?<br />

36. By the way, what is a cay?<br />

3.1


Page 4 of 5<br />

37. Now you will head to White Horseface Reef at<br />

Hans Lollik Isle. Watch your depths as you<br />

travel in that direction. What is submerged en<br />

route to the Isle?<br />

38. Anchor where you can and spend some time<br />

snorkeling. Once you have completed your<br />

swim and returned to the ship, start heading<br />

through the Leeward Passage. Move to map<br />

25647 at this point. In what units are the<br />

soundings measured?<br />

39. What is the scale?<br />

40. Once again, what do you notice about the scale and amount of detail in the map?<br />

41. On this map, what do the green solid and dashed green lines represent?<br />

42. How does that affect your boat?<br />

43. Heading through Leeward Passage and south of Thatch Cay, there is a rectangular box with a blue tint in the<br />

waterway. It is there to let you know, as captains, that there is an obstruction, a fish haven, which is an<br />

artificial reef. These are usually made of rock, concrete, car bodies, and other debris. If you'll notice, inside<br />

the box, it states an authorized minimum depth of 60 feet. If you look at the depth of the water on the map<br />

around the box, the values are deeper than 60 feet. Because of the artificial reef, the map is telling you that<br />

you can be assured to not have any obstruction down to 60 feet, but it is hazardous after that depth. The<br />

minimum depth is checked by sweeping the area with a length of horizontal wire. If there is an obstruction,<br />

the wire would get snagged. Is your boat okay to travel through this area?<br />

44. Continue to Cabrita Point and through to St. James Bay. You are headed towards Jersey Bay, but you are<br />

going to have to be very careful navigating the Jersey Bay area as you will then head into Banner Bay<br />

Channel. It is recommended by the map that you seek local knowledge about some broken piles (wooden<br />

columns driven into the harbor sand beds on which structures can be built in the water) which may be below<br />

the waterline and are not marked on the map. As you look at the channel, make note of the depth of the<br />

water. Will you be able to take your boat in or row in? How can you tell?<br />

45. Bring your wading boots just in case you need them. The symbols that look like ties (colored in green and<br />

purple) will help you navigate your way. These are buoys. The first letter of each buoy is either an R for ‘red’<br />

or G for ‘green.’ The rule of thumb is to keep red buoys to the right (starboard) when returning to a harbor<br />

and green buoys to the left (port). Using that rule, get yourself to the coast and have some lunch in town,<br />

especially after all that rowing.<br />

3.1


Page 5 of 5<br />

46. Take a taxi ride west of the harbor to the<br />

mangrove lagoon. Can you wade in there with<br />

your boots?<br />

47. Mangroves are trees and shrubs that grown in<br />

saline marine areas. The mangrove roots impede<br />

the water flow. Since the water is carrying<br />

sediment, the slowed water deposits the sediment<br />

over time and actually builds coastline. These are<br />

very special ecosystems.<br />

48. Spend some time here, take the taxi back to the<br />

row boat, and get back to your ship.<br />

49. Find a path to St. John and choose a landing site. Describe three more nautical notations that you encounter<br />

and how they influenced your route.<br />

50. Congratulations! Your voyage has ended. Hope you learned how important map reading is for nautical<br />

navigation. Show your teacher your route.<br />

3.1


Name: Date:<br />

4.1 Vectors on a Map<br />

You have learned that velocity is a vector quantity—this means that when you talk about velocity, you must<br />

mention both speed and direction. You can use velocity vectors on a coordinate plane to help you figure out the<br />

position of a moving object at a certain point in time.<br />

Your home is at the origin. From there you ride your bicycle to the movie theater. You ride 30. km/hr north for<br />

0.50 hour, and then 20. km/hr east for 0.25 hours.<br />

Show your home and the movie theater on a coordinate plane, and give the coordinates for each.<br />

Solution:<br />

If your home is at the origin, it is given the coordinates<br />

(0, 0). To find the position of the movie theater, you need<br />

to find the change in position. Use the relationship:<br />

change in position = velocity × change in time<br />

First change in position: +30. km/hr × 0.50 hr = 15 km<br />

NORTH<br />

Second change in position: +20. km/hr × 0.25 = 5 km<br />

EAST<br />

From home, travel north 15 km. Then turn and go east 5 km.<br />

The coordinates of the movie theater are ( +5 km, +15 km).<br />

Note: Be careful to report the x-coordinate first. It does not matter which direction you traveled first. When<br />

reporting position, you always give the x- (east-west) coordinate first, then the y- (north-south) coordinate.<br />

1. Augustin and Edson are going to a baseball game. To get to the stadium, they travel east on the highway at<br />

120. km/hr for 30. minutes. Then they turn onto the stadium parkway and travel south at 60. km/hr for<br />

10. minutes. Assume their starting point is at the origin. What is the position of the stadium?<br />

2. Destiny and Franijza are at the swimming pool. They decide to walk to the ice cream shop. They walk north<br />

at a pace of 6 km/hr for 20. minutes, and then east at the same pace for 10. minutes. If the swimming pool is<br />

at the origin (0,0) what is the position of the ice cream shop?<br />

3. After finishing their ice cream, the girls decide to go to Destiny’s house. From the ice cream shop, they walk<br />

south at a pace of 4.0 km/hr for 15 minutes. What is the position of Destiny’s house?<br />

4. Draw a map showing the swimming pool at the origin (0,0). Show the coordinates of the ice cream shop and<br />

Destiny’s house.<br />

5. Challenge! Make up your own velocity question. Your object (or traveler) should make at least one turn.<br />

Use at least two different speeds in your problem. Trade questions with a partner. Use a coordinate plane to<br />

help you solve the new question.<br />

4.1


Name: Date:<br />

3.2 Topographic Maps<br />

Flat maps can easily show landmasses and political boundaries. However, mapmakers need to draw special<br />

maps, called topographic maps, to show hills, valleys, and mountains. Mapmakers use contour lines to show the<br />

elevation of land features. The 0 contour line refers to sea level. The height above sea level is measured in equal<br />

intervals. Always look at the legend to see the elevation of each contour line interval. Sometimes these contour<br />

lines describe an increase of 20 feet. On other maps, especially those showing mountains, the contour lines may<br />

show elevation intervals of 100 to 1000 feet.<br />

• Contour lines and elevations<br />

Look at Figure 1. On this map, the contour interval is 100 feet. Let’s look<br />

at the letters marked on the map. Point A is at sea level. That means it is on<br />

the 0 contour line all the way around the island. Point B is on the next<br />

contour line. That means that B is 100 feet above sea level. What is the<br />

elevation at Point C? If you said 200 feet, you would be correct. At what<br />

elevation is Point D? The correct answer is somewhere between 0 and 100<br />

feet. You can’t be exact because D is not on a contour line. Where is Point<br />

E? Yes, it’s at sea level, the same level as A.<br />

Take a minute to color the contour key and the map. Color green between 0 and 100 feet, color yellow<br />

between 100 and 200 feet, color red between 200 and 300 feet. Mapmakers generally use blue for water<br />

only, so do not use it in a contour key.<br />

• Profile Maps<br />

You can also translate contour lines into a profile map. In this way you can<br />

actually draw what you would see if you were approaching by sea. Look at<br />

the following map as you read the steps to create a profile map. First, you<br />

draw a graph above the map that shows the intervals. The contour is 20<br />

feet so you would label the elevation in 20-foot intervals on the left. Your<br />

task is to make dots on the lines of the graph directly above the island<br />

contour lines. Put a ruler against the left hand edge of the island, and make<br />

a dot on the 0' line of the graph. Slide your ruler to the right hand edge of<br />

the island and make a second dot on the 0' line. Next go to the second<br />

contour line and mark two dots on the 20-foot interval line in the graph<br />

above the map. Continue marking two dots at the widest dimensions for<br />

contour lines 40 feet and 60 feet. Now connect the dots. This shows you<br />

the profile of the island. Note, the island’s elevation is probably a little more than 60 feet, so you could draw<br />

a peak on the top of the hill taller than 60 but less than 80 feet. Even if the island were 79 feet tall, there<br />

would not be an 80-foot contour line.<br />

3.2


Page 2 of 3<br />

1. Draw a profile map of the island in Figure 3. (Hint: You will have four dots on the 20' line.)<br />

2. Reverse the process to make a topographic map from the profile map. For each dot on the graph, you will<br />

make a small dot on the map showing where the contour line begins or ends. Draw a free form contour line<br />

that runs through the two dots. The 0' contour (sea level) has been done for you.<br />

3.2


Page 3 of 3<br />

3. Color the following map and contour key, and answer the questions.<br />

a. What is the lowest elevation on this map? _______<br />

b. What is the highest elevation on this map? _______<br />

c. What is the elevation at X? _______<br />

d. What is the elevation at Y? _______<br />

e. What is the elevation at Z? _______<br />

4. Today scientists worry that global warming may cause the ice caps to melt, causing the sea levels to rise.<br />

Look at the map below. It has a contour of 15 feet.<br />

a. Let’s pretend that the sea level has risen 15 feet. Color the first contour (0–15') of the map dark blue, so<br />

that the new sea level is revealed. How has this island changed?<br />

b. Now let’s pretend that the sea level rises another 15 feet. Color the next contour light blue and describe<br />

the changes in the original island.<br />

c. What if the sea level rose by 30 feet due to global warming, and a hurricane hit the island? Could the<br />

people find dry land if there were a 35-foot storm wave?<br />

3.2


Name: Date:<br />

3.3 Bathymetric Maps<br />

Imagine that all the water in the oceans disappeared. If this happened, you would be able to see what the bottom<br />

of the ocean looks like. Fortunately, we don’t have to drain water from the ocean to get a picture of the ocean<br />

floor. Instead, scientists use echo sounding and other techniques to “see” the ocean floor. The result is a<br />

bathymetric map. This skill sheet will provide you with the opportunity to practice reading a bathymetric map.<br />

Main features on a bathymetric map<br />

1. Main features on a bathymetric map are mid-ocean ridges, rises, deep ocean trenches, plateaus, and fracture<br />

zones. Find one example of each of these on a bathymetric map.<br />

a. Mid-ocean ridge:____________________<br />

b. Rise: ____________________<br />

c. Deep ocean trench: ____________________<br />

d. Plateau: ____________________<br />

e. Fracture zones: ____________________<br />

2. All the ridges you see on the bathymetric map behave in the same way even though they may not be in the<br />

middle of an ocean. What happens at mid-ocean ridges?<br />

3. Find the Rio Grande Rise on the bathymetric map. Then, find the East Pacific Rise.<br />

a. Which of these features is an example of a mid-ocean ridge?<br />

b. Find another example of a rise that is a mid-ocean ridge. Justify your answer.<br />

c. Find another example of a rise that is not a mid-ocean ridge. Justify your answer.<br />

4. There are a number of deep ocean trenches on the western side of the North Pacific Ocean. What process is<br />

going on at these trenches?<br />

5. What plate tectonic process probably caused the fracture zones in the North Pacific Ocean? Justify your<br />

answer.<br />

How is the East Pacific Rise different from the Mid-Atlantic Ridge?<br />

6. Look carefully at the Mid-Atlantic Ridge. Describe what this ridge looks like. Be detailed in your<br />

description.<br />

7. Now, look carefully at the East Pacific Rise. Describe what this ridge looks like. Be detailed in your<br />

description.<br />

8. Which of these features has a noticeable dark line running along the middle of the feature? Look at the<br />

legend at the bottom of the map. What does this dark line indicate?<br />

3.3


Page 2 of 2<br />

9. Based on your observations of these two features, draw a cross-section of each in the boxes below.<br />

Mid-Atlantic Ridge cross-section East Pacific Rise cross-section<br />

10. One of these mid-ocean ridges has a very fast spreading rate. The other has a very slow spreading rate.<br />

Which one is which? Justify your answer based on your answer to questions 8 and 9.<br />

3.3


Page 1 of 2<br />

3.3 Tanya Atwater<br />

Tanya Atwater is a professor of Earth Science at the University of California, Santa Barbara. She<br />

has studied sea floor spreading and propagating rifts. She is currently researching the plate tectonic history<br />

of western North America. One of her main goals as a geologist is to educate people about our Earth.<br />

Artist and adventurer<br />

While growing up, Tanya<br />

Atwater wanted to be an<br />

artist. She loved figuring<br />

out how to record on paper<br />

the things she could see in<br />

three dimensions.<br />

Atwater and her family<br />

went on many vacations,<br />

where, she says, “I always<br />

hogged the maps, taking<br />

great pleasure in translating<br />

between the paper map and the passing countryside.”<br />

Whether it was camping, hiking, or river rafting, all of<br />

the trips had one thing in common—adventure. As a<br />

result, Atwater developed a deep love for the<br />

outdoors.<br />

Geology in the mountains and at sea<br />

Atwater started her college career at the<br />

Massachusetts Institute of Technology (MIT). She<br />

tried a variety of majors, including physics, chemistry,<br />

and engineering. Atwater then attended the Indiana<br />

University geology summer field camp in Montana.<br />

There, she learned about geological mapping and how<br />

land structures translate into lines and symbols.<br />

Atwater was hooked on geology!<br />

Atwater transferred to the University of California at<br />

Berkley. She had already completed many math and<br />

physics courses at MIT, so she decided to major in<br />

geophysics.<br />

After graduation, Atwater held an internship at Woods<br />

Hole Oceanographic Institute in Massachusetts.<br />

There, she combined the adventures of ocean sailing<br />

with geophysics.<br />

A close look in a tiny submarine<br />

In 1967, Atwater began graduate school at the Scripps<br />

Oceanographic Institution in La Jolla, California.<br />

During this time, many exciting geological discoveries<br />

were being made. The concept of sea floor spreading<br />

was emerging, leading to the current theory of plate<br />

tectonics.<br />

3.3<br />

While at Scripps, Atwater joined a research group that<br />

used sophisticated equipment on ships to study the sea<br />

floor near California.<br />

Part of Atwater’s later research on sea floor spreading<br />

involved twelve trips down to the ocean floor in the<br />

tiny submarine Alvin. Only Atwater and two other<br />

people could fit in it. Using mechanical arms, they<br />

collected samples on the ocean floor nearly two miles<br />

underwater! Atwater’s firsthand view through Alvin’s<br />

portholes gave her a better understanding of the<br />

pictures and sonar records she had studied.<br />

She was also amazed to see hot springs gushing out of<br />

the ocean floor near volcanoes. She adds, “A whole<br />

bunch of brand new kinds of animals were living<br />

there. We saw giant white tubes with bright red worms<br />

living in them, giant clams, octopuses, crabs, giant<br />

anemones, and lots of slimy things. Weird!”<br />

Propagating rifts<br />

In the 1980s, Atwater was part of a team that<br />

researched propagating rifts near the Galapagos<br />

Islands off the coast of Ecuador. Propagating rifts are<br />

created when sea floor spreading centers realign<br />

themselves in response to changes in plate motion or<br />

uneven magma supplies.<br />

Atwater also discovered many propagating rifts on the<br />

sea floor in the northeast Pacific Ocean and in ancient<br />

sea floor records worldwide.<br />

An Earth educator<br />

Atwater has been a professor at the University of<br />

California, Santa Barbara for over 25 years. She has<br />

received many awards for her work in geophysics. She<br />

currently studies the plate tectonic history of western<br />

North America. This includes how the San Andreas<br />

Fault and Rocky Mountains were formed.<br />

Atwater also works with media, museums, and<br />

teachers and she creates educational animations to<br />

educate people about Earth. She explains, “My job as<br />

a geoscience educator is to help as many students as<br />

possible to know and understand and respect our<br />

planet—to help them really care about it and act on<br />

their caring.”


Page 2 of 2<br />

Reading reflection<br />

1. How did Atwater’s family contribute to her passion for planet Earth?<br />

2. Why was it an exciting time to study geology while Atwater was in graduate school?<br />

3. Describe how Atwater has gotten close-up views of the ocean floor.<br />

4. What are propagating rifts and where has Atwater observed them?<br />

5. How does Atwater educate people about Earth?<br />

6. Research: The Woods Hole Oceanographic Institution—Marine Operations has used the submarine Alvin<br />

for many research endeavors for over 40 years. Describe some of Alvin’s noteworthy trips.<br />

3.3


Name: Date:<br />

4.1 Solving Equations with One Variable<br />

It is useful to know formulas for calculating different quantities. Often, the formulas are very straightforward. It’s<br />

easy to calculate the volume of a rectangular solid when you know the formula:<br />

Volume = V = length × width × height (V = l × w × h)<br />

and the length, width, and height of the solid. It’s a little more challenging when you know the volume, length,<br />

and width, but need to find the height. It then becomes necessary to solve an equation in order to determine the<br />

unknown (in this case, the height).<br />

1. The volume of a rectangular solid, with a length of 1.5 cm, is 10.98 cm 3 . The width of the same solid is<br />

1.2 cm. Find its height.<br />

Explanation/Answer: use the formula V = l × w × h, and then plug in what is known, leaving the variable<br />

(h) for the unknown. Solve the equation for h to find the height.<br />

The Work: What’s happening:<br />

V = l × w × h Formula<br />

10.98 cm 3 = 1.5 cm × 1.2 cm × h Plug in known values.<br />

10.98 cm 3 = 1.8 cm 2 × h Complete arithmetic, multiply 1.5 × 1.2<br />

Check the Work:<br />

Divide both sides by 1.8 cm 2 , to get h alone.<br />

6.1 cm = 1 × h Do the division; 10.98 cm 3 ÷ 1.8 cm 2 = 6.1 cm,<br />

6.1 cm = h<br />

V = l × w × h<br />

1.8 cm 2 ÷ 1.8 cm 2 = 1<br />

V = 1.5 cm × 1.2 cm × 6.1 cm Multiply 1.5 cm × 1.2 cm × 6.1 cm.<br />

If the answer is 10.98 cm 3 , the solution, h = 6.1 cm, is correct.<br />

1.5 cm × 1.2 cm × 6.1 cm = 10.98 cm 3 The product does equal 10.98 cm 3 , the solution is correct.<br />

In summary:<br />

10.98 cm 3<br />

1.8 cm 2<br />

------------------------<br />

1.8 cm 2<br />

× h<br />

1.8 cm 2<br />

=<br />

---------------------------<br />

The height (h) of a rectangular solid whose volume is 10.98 cm 3 , whose length is 1.5 cm, and whose width is 1.2<br />

cm, is 6.1 cm.<br />

4.1


Page 2 of 4<br />

2. The density of titanium is 4.5 g/cm3 . A titanium pendant’s mass is 2.25 grams. Use the formula<br />

Density = ----------------mass<br />

, or D =<br />

m<br />

--- , or to find its volume.<br />

volume v<br />

The Work: What’s happening:<br />

4.5 g × V = (2.25 g) × (1 cm 3 )<br />

Formula<br />

Plug in known values.<br />

Rewrite 4.5 g/cm 3 as<br />

Think of 4.5 g<br />

1 cm<br />

as a proportion.<br />

Then set the cross products equal<br />

3<br />

------------- =<br />

2.25 g<br />

--------------<br />

V<br />

4.5 g × V = 2.25 g × 1 cm 3 Do arithmetic: 2.25 g × 1cm 3 = 2.25 g × 1 cm 3<br />

Check the Work:<br />

In summary:<br />

D<br />

=<br />

m<br />

--v<br />

3<br />

4.5 g/cm =<br />

2.25<br />

-------------g<br />

V<br />

4.5g<br />

1 cm 3<br />

-------------<br />

=<br />

2.25<br />

-------------g<br />

V<br />

4.5<br />

--------------------g<br />

× V 2.25 g 1cm<br />

4.5 g<br />

3<br />

= ----------------------------------<br />

×<br />

4.5 g<br />

Divide both sides of the equation by 4.5 g to get V alone.<br />

V = 0.5 cm 3 Do the division on each side. Remember to cancel units as well as<br />

divide the numbers.<br />

D<br />

=<br />

m<br />

--v<br />

3<br />

4.5 g/cm<br />

2.25 g<br />

0.5 cm 3<br />

=<br />

------------------<br />

Divide 2.25 ÷ 0.5. If the answer is 4.5 g/cm 3 , the solution,<br />

V = 0.5 cm 3 , is correct.<br />

2.25 ÷ 0.5 = 4.5 g/cm 3 The quotient does equal 4.5 g/cm 3 ; therefore, the solution is<br />

correct.<br />

The volume of a titanium pendant whose mass is 2.25 grams is 4.5 g/cm 3 .<br />

4.5 g<br />

1 cm 3<br />

-------------<br />

4.1


Page 3 of 4<br />

Use the formula V = l × w × h to set up and solve for the unknown in each.<br />

1. Find the width (w) of a rectangular solid whose length is 12 mm, and whose height is 15 mm, if the volume<br />

of the solid is 720 mm 3 .<br />

2. Find the length of this rectangular solid whose volume is 0.12 m 3 .<br />

3. The length and width of a rectangular solid are 2.15 cm. Its volume is 36.98 cm 3 . Find the height of this<br />

rectangular solid.<br />

Use the formula Speed =<br />

distance<br />

-------------------- , or S =<br />

d<br />

-- to set up and solve for the unknown in each.<br />

time t<br />

Here, speed is measured in meters/second (m/s), distance is measured in meters (m), and time is measured<br />

in seconds (s).<br />

0.25 m<br />

0.6 m<br />

4. How far will a marble rolling at a speed of 0.25 m/s travel in 30. seconds?<br />

5. Nate throws a paper wad to Ali who is sitting exactly 1.8 meters away. The paper wad was only in the air for<br />

0.45 seconds. How fast was it traveling?<br />

6. How long does it take a battery operated toy car to travel 3 meters at a speed of 0.1 m/s?<br />

7. A dog is running 3.20 m/s. How long will it take him to go 100. meters?<br />

mass m<br />

Use the formula Density = ----------------- , or D =<br />

--- , to set up and solve for the unknown in each.<br />

volume v<br />

Here, density (D) is measured in grams per cubic centimeter (g/cm3), mass (m) is measured in grams (g),<br />

and volume (V) is measured in cubic centimeters (cm3).<br />

8. What is the density of a steel nail whose volume is 3.2 cm 3 and whose mass is 25 g?<br />

9. Find the mass of a cork whose density is 0.12 g/cm 3 and whose volume is 9.0 cm 3 ?<br />

10. An ice cube’s volume is 4.9 cm 3 . Find its mass if its density is 0.92 g/cm 3 .<br />

11. A solid plastic ball’s mass is 225 g. The density of the plastic is 2.00 g/cm 3 . What is the volume of the ball?<br />

12. Find the volume of an ice cube whose mass is 2.08 g. See question #10 for the density of ice.<br />

Use the formula: Force = pressure × area to set up and solve for each unknown.<br />

?<br />

4.1


Page 4 of 4<br />

Here, force is measured in Newtons (N), pressure is measured in Pascals (Pa), and area is<br />

measured in square meters (m2). Hint: 1 Pa = 1 N / m 2 .<br />

• A drinking glass is sitting on the kitchen table. The glass has a weight of 2 N. Its base has an area of<br />

0.005 m2 . How much pressure does the drinking glass exert on the table?<br />

Explanation/Answer:<br />

The Work: What’s happening:<br />

Force = pressure × area Formula<br />

2 N = p × 0.005m 2 Plug in known values.<br />

2 N<br />

0.005 m 2<br />

---------------------<br />

In summary:<br />

×<br />

0.005 m 2<br />

=<br />

------------------------------<br />

p 0.005 m 2<br />

400 N/m 2 = p × 1<br />

Divide both sides by 0.005 m 2 to get p alone.<br />

Do the division:<br />

2 N ÷ 0.005 m2 = 400 N / m2 , 0.005 m2 ÷ 0.005 m2 = 1<br />

400 Pa = p Rewrite 400 N/m 2 as 400 Pa, multiply; p × 1 = p.<br />

A drinking glass with a weight of 2 N and whose base has area 0.005 m 2 exerts 400 Pa of pressure on the table<br />

it sits on.<br />

1. A tea kettle’s base has an area of 0.008 m 2 . It is puts 1,000 Pascals of pressure on the stove where it sits.<br />

What is the weight of the kettle?<br />

2. A block of wood whose base has an area of 4 m 2 has a weight of 80 N. How much pressure does the block<br />

place on the floor on which it sits?<br />

3. A sculpture’s base has an area of 2.50 m 2 . How much pressure does the sculpture place on the wooden<br />

display case where it sits, if it has a weight of 540. N?<br />

4. A student is breaking class rules by standing on a chair. If her feet have a total area of 0.04 m 2 , and her<br />

weight is 600. N, how much pressure is she putting on the chair?<br />

4.1


Name: Date:<br />

4.1 Problem Solving Boxes<br />

Looking for Solution<br />

Given<br />

Relationships<br />

Looking for Solution<br />

Given<br />

Relationships<br />

Looking for Solution<br />

Given<br />

Relationships<br />

4.1


Name: Date:<br />

4.1 Problem Solving with Rates<br />

Solving mathematical problems often involves using rates. An upside down rate is called a reciprocal rate.<br />

A rate may be written as its reciprocal because no matter how you write it the rate gives you the same amount of<br />

one thing per amount of the other thing. For example, you can write 5 cookies/ $1.00 or $1.00/5 cookies. For<br />

$1.00, you know you will get 5 cookies no matter how you write the rate. In this activity, you will choose how to<br />

write each rate in order to solve the problem the easiest way.<br />

Steps for solving problems with rates are listed below. Remember, after you have set up your problem, analyze<br />

and cancel the units by crossing them out, then do the arithmetic, and provide the answer. Remember that the<br />

answer always consists of a number and a unit.<br />

In the space provided, write the reciprocal rate of each given rate. The first one is done for you.<br />

1.<br />

2.<br />

3.<br />

4.<br />

5.<br />

Step 1 What quantity or rate are you asked for in the problem? Write it down.<br />

Step 2 What do you know from reading the problem? List all known rates and quantities.<br />

Step 3 Arrange the known quantities and rates to get an answer that has the right units. This arrangement<br />

might include a formula.<br />

Step 4 Plug in the values you know.<br />

Step 5 Solve the problem and write the answer with a number and a unit.<br />

--------------------<br />

1 year<br />

365 days<br />

=<br />

----------------------<br />

12 inches<br />

=<br />

foot<br />

3<br />

--------------------------------small<br />

pizzas<br />

=<br />

$10.00<br />

36<br />

----------------------pencils<br />

=<br />

3 boxes<br />

365<br />

-------------------days<br />

1 year<br />

18<br />

---------------------------------------------------gallons<br />

of gasoline<br />

=<br />

360 miles<br />

4.1


Page 2 of 3<br />

In problems 6 and 7, you will be shown how to set up steps 1–4. For step 5, you will need to solve<br />

the problem and write the answer as a number and unit.<br />

6. Downhill skiing burns about 600. calories per hour. How many calories will you burn if you<br />

downhill ski for 3.5 hours?<br />

Step 1 Looking for calories.<br />

Step 2 600. calories/hour; 3.5 hours<br />

Step 3<br />

Step 4<br />

Step 5 Answer:<br />

7. How many cans of soda will John drink in a year if he drinks 3 sodas per day? (Remember that there are 365<br />

days in a year.)<br />

Step 1 Looking for cans of soda per year.<br />

Step 2 3 sodas/day; 365 days/year<br />

Step 3<br />

Step 4<br />

Step 5 Answer:<br />

8. How many heartbeats will a person have in a week if he has an average heart rate of 72 beats per minute?<br />

(Remember the days/week, hours/day, and minutes/hour.<br />

Step 1 Looking for number of heartbeats per week.<br />

Step 2 72 heartbeats/minute, 7 days/week, 24 hours/day, 60 minutes/hour<br />

Step 3<br />

Step 4<br />

Step 5 Answer:<br />

calories<br />

----------------- × hours = calorie<br />

hour<br />

600<br />

----------------------------calories<br />

× 3.5 hours = calories<br />

hour<br />

soda<br />

--------day<br />

days<br />

× ---------- =<br />

year<br />

sodas<br />

-----------year<br />

3 sodas 365 days<br />

----------------- × --------------------day<br />

year<br />

=<br />

sodas<br />

-----------year<br />

heartbeats<br />

----------------------minute<br />

minutes<br />

-----------------hours<br />

× ------------days<br />

× × ----------hour<br />

day week<br />

---------------------------------<br />

72 heartbeats<br />

minute<br />

60 minutes<br />

× -------------------------hour<br />

=<br />

24 hours<br />

× -------------------day<br />

heartbeats<br />

----------------------week<br />

7 days<br />

× --------------week<br />

=<br />

heartbeats<br />

----------------------week<br />

4.1


Page 3 of 3<br />

Using the five problem-solving steps, solve the following problems on your own. Be sure to read<br />

the problem carefully. Show your work in the blank provided.<br />

Step 1 What quantity or rate are you asked for in the problem? Write it down.<br />

Step 2 What do you know from reading the problem? List all known rates and quantities.<br />

Step 3 Arrange the known quantities and rates to get an answer that has the right units. This<br />

arrangement might include a formula.<br />

Step 4 Plug in the values you know.<br />

Step 5 Solve the problem and write the answer with a number and a unit.<br />

9. How much will you pay for 5 pounds of shrimp if the cost is 2 pounds for $10.99?<br />

10. How many miles can you get on one tank of gas if your tank holds 18 gallons and you get 23 miles per<br />

gallon?<br />

11. What is your rate in miles/hour if you run at a speed of 2.2 miles in 20. minutes?<br />

12. Suppose for your cookout you need to make 100 hamburgers. You know that 2.00 pounds will make<br />

9 hamburgers. How many pounds will you need?<br />

13. What is your mass in kilograms if you weigh 120. pounds? (There are approximately 2.2 pounds in one<br />

kilogram.)<br />

14. Mt. Everest is 29,028 feet high. How many miles is this? (There are exactly 5,280 feet in one mile.)<br />

15. Susan works 8 hours a day and makes $7.00 per hour. How much money does Susan earn in one week if she<br />

works 5 days per week?<br />

16. How many years will it take a major hamburger fast food chain to sell 45,000,000 burgers if it sells<br />

approximately 12,350 burgers per day?<br />

17. Your science teacher needs to make more of a salt-water mixture. The concentration of the mixture that is<br />

needed is 35 grams of salt in 1,000. milliliters of water. How many grams of salt will be needed to make<br />

1,500. milliliters of the salt-water?<br />

18. A cart travels down a ramp at an average speed of 5.00 centimeters/second. What is the speed of the cart in<br />

miles/hour? (Remember there are 100 centimeters per meter, 1000 meters/kilometer, and 1.6 kilometer per<br />

mile.)<br />

19. A person goes to the doctor and will need a 3-month prescription of medicine. The person will be required to<br />

take 3 pills per day. How many pills will the doctor write the prescription for assuming there are 30 days in a<br />

month?<br />

20. If you are traveling at 65 miles per hour, how many feet will you be traveling in one second?<br />

4.1


Name: Date:<br />

4.1 Percent Error<br />

When you do scientific experiments that involve measurements, your results may fit the trend that is<br />

expected. However, it is unlikely that the numbers will turn out exactly as expected.<br />

In an experiment, you often make a prediction about an event’s outcome, but find that your actual measured<br />

outcome is slightly different. The percent error (% Error) gives you a means to evaluate how far apart your<br />

prediction and measured values are.<br />

Percent error is calculated as the absolute value of the difference between the predicted and measured values<br />

divided by the true value multiplied by 100, or:<br />

measured value – predicted value<br />

% Error =<br />

---------------------------------------------------------------------------------- × 100<br />

true value<br />

Which value is the true value? That depends on your experiment design. If you want to evaluate how well a graph<br />

is able to predict an actual event (like how far a marble will travel or how long a car will take to travel down a<br />

ramp) then you use the measured value as the true value.<br />

On the other hand, if you have carefully calculated how much product you should get in a chemical reaction, and<br />

you want to evaluate how carefully you made your measurements and followed the procedure, then you would<br />

use the predicted value as the true value.<br />

Remember that with percent error, smaller is better. A perfect outcome would have zero percent error.<br />

Some students are conducting an experiment using a toy car with a track, timer, and photogates.<br />

Their task is to determine how quickly the car will travel a given distance, and then to predict and test the last trip<br />

that the car takes. The table below shows the distances and times traveled by the car so far.<br />

Distance from A to B<br />

(cm)<br />

Time from A to B (sec)<br />

10. 0.3305<br />

20. 0.3380<br />

40. 0.3535<br />

50. 0.3610<br />

60. ?<br />

Based on an estimation made by extending their graph, the students predict that it will take the car 0.3685<br />

seconds to travel 60 centimeters. When the experiment was conducted three times, it took the car 0.3669, 0.3680,<br />

and 0.3694 seconds to make the trip. Calculate the percent of error based on the predicted and actual outcomes.<br />

4.1


Page 2 of 3<br />

Solution:<br />

The process:<br />

1. Average the times recorded in the three 60-centimeter trials to use as the measured value in the formula.<br />

2. Calculate percent error using the formula given above, using the average from (1) as the measured value.<br />

The work:<br />

1. Find the average:<br />

2. Calculate:<br />

0.3669<br />

----------------------------------------------------------<br />

+ 0.368 + 0.3694<br />

=<br />

1.1043<br />

--------------- = 0.3681<br />

3<br />

3<br />

% Error<br />

measured value – predicted value<br />

= ------------------------------------------------------------------------------- × 100<br />

true value<br />

0.3681 – 0.3685<br />

% Error --------------------------------------<br />

0.0004<br />

= × 100 = --------------- × 100 ≈<br />

0.11%<br />

0.3681<br />

0.3981<br />

The Answer: The percent error in this particular experiment is 0.11%. This means that the student’s predicted<br />

time was 0.11% off the actual, measured time.<br />

Use the method shown in the example to calculate the percent error in each of the following problems.<br />

Part I: This table was constructed by a group of students conducting an experiment similar to the one in the<br />

example above, but using a different incline. Complete the table using the average time calculated at each<br />

distance from the information provided in the problems below.<br />

Distance from A to B<br />

(cm)<br />

Time from A to B (sec)<br />

10. 1.0050<br />

20. 1.8877<br />

30. 2.8000<br />

40. 3.7850<br />

50. ?<br />

60. ?<br />

70. ?<br />

80. ?<br />

90. ?<br />

4.1


Page 3 of 3<br />

1. The lab group conducting this experiment decided to call themselves “the Science Sleuths.” They<br />

graphed the data shown in the table and based on their graph, predicted that it would take the car<br />

4.7500 seconds to travel 50 centimeters. The three trials they conducted resulted in 4.8020, 4.8100,<br />

and 4.7000 seconds. What is the percent error? Remember to update the table.<br />

2. The Sleuths predict that the car will travel 60 centimeters in 5.7950 seconds. Their trials gave times of<br />

5.7702, 5.8000, and 5.2600 seconds. What is the percent error here?<br />

3. For 70 centimeters, the trial runs resulted in 6.9150, 6.8080, and 7.0003 seconds. The Sleuths had predicted<br />

that it would take the car 6.8150 seconds to cover the distance. Calculate the percent error.<br />

4. The Sleuths’ car took 7.9903, 7.9995, and 7.9047 seconds to travel 80 centimeters. They had predicted a<br />

time of 7.9520 seconds. What is the percent error?<br />

5. This time, the Sleuths predicted that it would take the car 9.0000 seconds flat to cover the 90 centimeters it<br />

needed to travel. It actually took the car 8.9907, 9.0006, and 9.0507 seconds in each of three trials. Find the<br />

percent error.<br />

6. Lisa was trying out for the track team at her middle school. The coach asked her to make predictions about<br />

how fast she could run each of the sprint events, then timed her in each event on three different days. All the<br />

information is shown in the table below. Calculate Lisa’s percent error for her prediction in each event.<br />

Event Predicted time (s) Actual times (s)<br />

17.55<br />

100 m 18.05<br />

18.94<br />

15.06<br />

41.05<br />

200 m 34.70<br />

38.22<br />

35.90<br />

72.75<br />

400 m 67.45<br />

65.10<br />

65.88<br />

Calculate the percent error for each event:<br />

a. 100 m<br />

b. 200 m<br />

c. 400 m<br />

4.1


Name: Date:<br />

4.1 Speed<br />

To determine the speed of an object, you need to know the distance traveled and the time taken to travel that<br />

distance. If you know the speed, you can determine the distance traveled or the time it took—you just rearrange<br />

the formula for speed, v = d/t. For example,<br />

Equation... Gives you... If you know...<br />

v = d/t speed distance and time<br />

d = v × t distance speed and time<br />

t = d/v time distance and speed<br />

Use the SI system to solve the practice problems unless you are asked to write the answer using the English<br />

system of measurement. As you solve the problems, include all units and cancel appropriately.<br />

Example 1: What is the speed of a cheetah that travels 112.0 meters in 4.0 seconds?<br />

Looking for<br />

Given<br />

Speed of the cheetah.<br />

Distance = 112.0 meters<br />

Time = 4.0 seconds<br />

Relationship<br />

Solution<br />

The speed of the cheetah is 28 meters per second.<br />

Example 2: There are 1,609 meters in one mile. What is this cheetah’s speed in miles/hour?<br />

Looking for<br />

Given<br />

Speed of the cheetah in miles per hour.<br />

Speed = 28 m/s (from solution to Example 1)<br />

Relationships<br />

speed<br />

=<br />

speed =<br />

d<br />

--<br />

t<br />

and 1, 609 meters = 1 mile<br />

d<br />

--<br />

t<br />

Solution<br />

speed<br />

28<br />

----------m<br />

s<br />

×<br />

=<br />

d<br />

-- =<br />

112.0<br />

-----------------m<br />

=<br />

t 4.0 s<br />

------------------<br />

1 mile<br />

1,609 m<br />

3, 600 s<br />

× -----------------<br />

1 hour<br />

28<br />

----------m<br />

s<br />

63<br />

------------------miles<br />

hour<br />

The speed of the cheetah in miles per hour is<br />

63 mph.<br />

=<br />

4.1


Page 2 of 3<br />

1. A bicyclist travels 60.0 kilometers in 3.5 hours. What is the cyclist’s average speed?<br />

Looking for Solution<br />

Given<br />

Relationships<br />

2. What is the average speed of a car that traveled 300.0 miles in 5.5 hours?<br />

3. How much time would it take for the sound of thunder to travel 1,500 meters if sound travels at a speed of<br />

330 m/s?<br />

4. How much time would it take for an airplane to reach its destination if it traveled at an average speed of<br />

790 kilometers/hour for a distance of 4,700 kilometers? What is the airplane’s speed in miles/ hour?<br />

5. How far can a person run in 15 minutes if he or she runs at an average speed of 16 km/hr?<br />

(HINT: Remember to convert minutes to hours.)<br />

6. In problem 5, what is the runner’s distance traveled in miles?<br />

7. A snail can move approximately 0.30 meters per minute. How many meters can the snail cover in<br />

15 minutes?<br />

8. You know that there are 1,609 meters in a mile. The number of feet in a mile is 5,280. Use these equalities to<br />

answer the following problems:<br />

a. How many centimeters equals one inch?<br />

b. What is the speed of the snail in problem 7 in inches per minute?<br />

9. Calculate the average speed (in km/h) of a car stuck in traffic that drives 12 kilometers in 2 hours.<br />

10. How long would it take you to swim across a lake that is 900 meters across if you swim at 1.5 m/s?<br />

a. What is the answer in seconds?<br />

b. What is the answer in minutes?<br />

11. How far will a you travel if you run for 10. minutes at 2.0 m/s?<br />

12. You have trained all year for a marathon. In your first attempt to run a marathon, you decide that you want to<br />

complete this 26.2-mile race in 4.5 hours.<br />

a. What is the length of a marathon in kilometers (1 mile = 1.6 kilometers)?<br />

b. What would your average speed have to be to complete the race in 4.5 hours? Give your answer in<br />

kilometers per hour.<br />

4.1


Page 3 of 3<br />

13. Suppose you are walking home after school. The distance from school to your home is five<br />

kilometers. On foot, you can get home in 25 minutes. However, if you rode a bicycle, you could get<br />

home in 10 minutes.<br />

a. What is your average speed while walking?<br />

b. What is your average speed while bicycling?<br />

c. How much faster you travel on your bicycle?<br />

14. Suppose you ride your bicycle to the library traveling at 0.50 km/min. It takes you 25 minutes to get to the<br />

library. How far did you travel?<br />

15. You ride your bike for a distance of 30 km. You travel at a speed of 0.75 km/ minute. How many minutes<br />

does this take?<br />

16. A train travels 225 kilometers in 2.5 hours. What is the train’s average speed?<br />

17. An airplane travels 3,280 kilometers in 4.0 hours. What is the airplane’s average speed?<br />

18. A person in a kayak paddles down river at an average speed of 10. km/h. After 3.25 hours, how far has she<br />

traveled?<br />

19. The same person in question 18 paddles upstream at an average speed of 4 km/h. How long would it take her<br />

to get back to her starting point?<br />

20. An airplane travels from St. Louis, Missouri to Portland, Oregon in 4.33 hours. If the distance traveled is<br />

2,742 kilometers, what is the airplane’s average speed?<br />

21. The airplane returns to St. Louis by the same route. Because the prevailing winds push the airplane along,<br />

the return trip takes only 3.75 hours. What is the average speed for this trip?<br />

22. The airplane refuels in St. Louis and continues on to Boston. It travels at an average speed of 610 km/h. If<br />

the trip takes 2.75 hours, what is the flight distance between St. Louis and Boston?<br />

Challenge Problems:<br />

23. The speed of light is about 3.00 × 10 5 km/s. It takes approximately 1.28 seconds for light reflected from the<br />

moon to reach Earth. What is the average distance from Earth to the moon?<br />

24. The average distance from the sun to Pluto is approximately 6.10 × 10 9 km. How long does it take light from<br />

the sun to reach Pluto? Use the speed of light from the previous question to help you.<br />

25. Now, make up three speed problems of your own. Give the problems to a friend to solve and check their<br />

work.<br />

a. Make up a problem that involves solving for average speed.<br />

b. Make up a problem that involves solving for distance.<br />

c. Make up a problem that involves solving for time.<br />

4.1


Name: Date:<br />

4.1 Velocity<br />

Speed and velocity do not have the same meaning to scientists. Speed is a scalar quantity, which means it can be<br />

completely described by its magnitude (or size). The magnitude is given by a number and a unit. For example, an<br />

object’s speed may be measured as 15 meters per second.<br />

Velocity is a vector quantity. In order to measure a vector quantity, you must know the both its magnitude and<br />

direction. The velocity of an object is determined by measuring both the speed and direction in which an object is<br />

traveling.<br />

• If the speed of an object changes, then its velocity also changes.<br />

• If the direction in which an object is traveling changes, then its velocity changes.<br />

• A change in either speed, direction, or both causes a change in velocity.<br />

You can rearrange v = d/t to solve velocity problems the same way you solved speed problems earlier in this<br />

course. The boldfaced v is used to represent velocity as a vector quantity. The variables d and t are used for<br />

distance and time. The velocity of an object in motion is equal to the distance it travels per unit of time in a<br />

given direction.<br />

Example 1: What is the velocity of a car that travels 100.0 meters, northeast in 4.65 seconds?<br />

Looking for<br />

Velocity of the car.<br />

Given<br />

Distance = 100.0 meters<br />

Time = 4.65 seconds<br />

Relationship<br />

Solution<br />

The velocity of the car is 21.5 meters per second,<br />

northeast.<br />

Example 2: A boat travels with a velocity equal to 14.0 meters per second, east in 5.15 seconds. What distance<br />

in meters does the boat travel?<br />

Looking for<br />

Given<br />

velocity<br />

Distance the boat travels.<br />

Velocity = 14.0 meters per second, east<br />

Time = 5.15 seconds<br />

Relationship<br />

=<br />

d<br />

--<br />

t<br />

distance =<br />

v × t<br />

Solution<br />

velocity<br />

distance = v× t =<br />

=<br />

d<br />

-- =<br />

100.0<br />

-----------------m<br />

=<br />

t 4.65 s<br />

The boat travels 72.1 meters.<br />

21.5<br />

--------------m<br />

s<br />

14.0<br />

--------------m<br />

× 5.15 s=<br />

72.1 m<br />

s<br />

4.1


Page 2 of 2<br />

1. An airplane flies 525 kilometers north in 1.25 hours. What is the airplane’s velocity?<br />

Looking for Solution<br />

Given<br />

Relationship<br />

2. A soccer player kicks a ball 6.5 meters. How much time is needed for the ball to travel this distance if its<br />

velocity is 22 meters per second, south?<br />

3. A cruise ship travels east across a river at 19.0 meters per minute. If the river is 4,250 meters wide, how long<br />

does it take for the ship to reach the other side?<br />

4. Joaquin mows the lawn at his grandmother’s home during the summer months. Joaquin measured the<br />

distance across his grandmother’s lawn as 11.5 meters.<br />

a. If Joaquin mows one length across the lawn from east to west in 7.10 seconds, then what is the velocity<br />

of the lawnmower?<br />

b. Once he reaches the edge of the lawn, Joaquin turns the lawnmower around. He mows in the opposite<br />

direction but maintains the same speed. What is the velocity of the lawnmower?<br />

5. A family drives 881 miles from Houston, Texas to Santa Fe, New Mexico for vacation. How long will it take<br />

the family to reach their destination if they travel at a velocity of 55.0 miles per hour, northwest?<br />

6. A shopping cart is pushed 15.6 meters west across a parking lot in 5.2 seconds. What is the velocity of the<br />

shopping cart?<br />

7. Katie and her best friend Liam play tennis every Saturday morning. When Katie serves the ball to Liam, it<br />

travels 9.5 meters south in 2.1 seconds.<br />

a. What is the velocity of the tennis ball?<br />

b. If the tennis ball travels at constant speed, what is its velocity when Liam returns Katie’s serve?<br />

8. A driver realizes that she is traveling in the wrong direction on a one-way street. She has already driven<br />

350 meters at a velocity of 16 meters per second, east before deciding to make a U-turn. How long did it take<br />

for the driver to realize her error?<br />

9. Juan’s mother drives 7.25 miles southwest to her favorite shopping mall. What is the average velocity of her<br />

automobile if she arrives at the mall in 20. minutes?<br />

10. A bus is traveling at 79.7 kilometers per hour east. How far does the bus travel 1.45 hours?<br />

11. A girl scout troop hiked 5.8 kilometers southeast in 1.5 hours.What was the troop’s velocity?<br />

12. A volcanologist noted that a lahar rushed down a mountain at 32.2 kilometers per hour, south. How far did<br />

the mud flow in 17.5 minutes?<br />

4.1


Name: Date:<br />

4.2 Calculating Slope from a Graph<br />

To determine the slope of a line in a graph, first choose two points on the line. Then count how many steps up or<br />

down you must move to be on the same horizontal line as your second point. Multiply this number by the scale of<br />

your horizontal axis. For example, if your x-axis has a scale of 1 box = 20 cm, then multiply the number of boxes<br />

you counted by 20 cm.<br />

Put the result along with the positive or negative sign as the top (numerator) of your slope ratio. Then count how<br />

many steps you must move right or left to land on your second point. Multiply the number of steps by the scale of<br />

your vertical axis. Place the results as the bottom (denominator) of your slope ratio. Then reduce the fraction of<br />

your ratio. This number is the slope of the line. Note: The letter m is used to represent slope in an equation.<br />

A The chosen points for Example A are (0, 0) and (3, 9). There are many<br />

choices for this graph, but only one slope. If you have the point (0, 0),<br />

you should choose it as one of your points.<br />

It takes 9 vertical steps to move from (0, 0) to (0, 9). Put a 9 in the<br />

numerator of your slope ratio (or subtract 9 – 0). Then count the<br />

number of steps to move from (0, 9) to (3, 9). This is your denominator<br />

of your slope ratio. Again, you can do this by subtraction (3 – 0).<br />

m =<br />

9<br />

3<br />

=<br />

3<br />

1<br />

B The two points that have been chosen for Example B are (0, 24) and<br />

(6, 15). Be careful of the scales on each of the axes.<br />

It takes 3 vertical steps to go from (0, 24) to (0, 15). But each of these<br />

steps has a scale of 3. So you put a –9 into the numerator of the slope<br />

ratio. It is negative because you are moving down from one point to the<br />

other. Then count the steps over to (6, 15). There are 3 steps but each<br />

counts for 2 so you put a 6 into the denominator of the slope ratio.<br />

m =<br />

−9 6<br />

=<br />

−3<br />

2<br />

Find the slope of the line in each of the following graphs:<br />

Graph #1: Graph #2:<br />

4.2


Page 2 of 2<br />

Graph #3: Graph #4:<br />

Graph #5: Graph #6:<br />

Graph #7: Graph #8:<br />

Graph #9: Graph #10:<br />

4.2


Name: Date:<br />

4.2 Analyzing Graphs of Motion With Numbers<br />

Speed can be calculated from position-time graphs and distance can be calculated from speed-time graphs. Both<br />

calculations rely on the familiar speed equation: v = d/t.<br />

This graph shows position and time for a sailboat starting from its<br />

home port as it sailed to a distant island. By studying the line, you can<br />

see that the sailboat traveled 10 miles in 2 hours.<br />

• Calculating speed from a position-time graph<br />

The speed equation allows us to calculate that the boat’s speed during<br />

this time was 5 miles per hour.<br />

v = d/t<br />

v = 10 miles ⁄ 2 hours<br />

v =<br />

5 miles/hour, read as 5 miles per hour<br />

This result can now be transferred to a speed-time graph. Remember<br />

that this speed was measured during the first two hours.<br />

The line showing the boat’s speed is horizontal because the speed was<br />

constant during the two-hour period.<br />

• Calculating distance from a speed-time graph<br />

Here is the speed-time graph of the same sailboat later in the voyage.<br />

Between the second and third hours, the wind freshened and the<br />

sailboat gradually increased its speed to 7 miles per hour. The speed<br />

remained 7 miles per hour to the end of the voyage.<br />

How far did the sailboat go during the six-hour trip? We will first<br />

calculate the distance traveled during the fourth, fifth, and sixth hours.<br />

4.2


Page 2 of 4<br />

On a speed-time graph, distance is equal to the area between the baseline and the plotted line. You know<br />

that the area of a rectangle is found with the equation: A = L × W. Similarly, multiplying the speed from<br />

the y-axis by the time on the x-axis produces distance. Notice how the labels cancel to produce miles:<br />

speed × time = distance<br />

Now that we have seen how distance is calculated, we can consider the distance<br />

covered between hours 2 and 3.<br />

The easiest way to visualize this problem is to think in geometric terms. Find the<br />

area of the triangle (Area A), then find the area of the rectangle (Area B), and add<br />

the two areas.<br />

Area of triangle A<br />

Geometry formula<br />

Area of rectangle B<br />

Geometry formula<br />

Add the two areas<br />

7 miles/hour × ( 6 hours – 3 hours)<br />

= distance<br />

7 miles/hour × 3 hours = distance = 21 miles<br />

The area of a triangle is one-half the area of a rectangle.<br />

time<br />

speed × --------- = distance<br />

2<br />

( 7 miles/hour – 5 miles/hour)<br />

speed × time = distance<br />

The Position vs. Time graph on page 1 tells us that the boat traveled<br />

10 miles in the first two hours. According to our calculations, the boat<br />

traveled 6 miles during the third hour and 21 miles in hours four through<br />

six.<br />

Therefore the total distance traveled is 10 + 6 + 21 = 37 miles.<br />

We can now use this information about distance to complete our positiontime<br />

graph:<br />

( 3 hours – 2 hours)<br />

× ---------------------------------------------- = distance = 1 mile<br />

2<br />

5 miles/hour × ( 3 hours – 2 hours)<br />

= distance = 5 miles<br />

Area A + Area B = distance<br />

1 miles+ 5 mile = distance =<br />

6 miles<br />

4.2


Page 3 of 4<br />

1. For each position-time graph, calculate and plot speed on the speed-time graph to the right.<br />

a. The bicycle trip through hilly country<br />

a. A walk in the park<br />

b. Strolling up and down the supermarket aisles<br />

4.2


Page 4 of 4<br />

2. For each speed-time graph, calculate and plot the distance on the position-time graph to the right.<br />

For this practice, assume that movement is always away from the starting position.<br />

a. The honey bee among the flowers<br />

b. Rover runs the street<br />

c. The amoeba<br />

4.2


Name: Date:<br />

4.2 Analyzing Graphs of Motion Without Numbers<br />

Position-time graphs<br />

The graph at right represents the story of “The Three Little Pigs.” The<br />

parts of the story are listed below.<br />

• The wolf started from his house. The graph starts at the origin.<br />

• Traveled to the straw house. The line moves upward.<br />

• Stayed to blow it down and eat dinner. The line is flat because position<br />

is not changing.<br />

• Traveled to the stick house. The line moves upward again.<br />

• Again stayed, blew it down, and ate seconds. The line is flat.<br />

• Traveled to the brick house. The line moves upward.<br />

• Died in the stew pot at the brick house. The line is flat.<br />

The graph illustrates that the pigs’ houses are generally in a line away from the wolf’s house and that the brick<br />

house was the farthest away.<br />

Speed-time graphs<br />

A speed-time graph displays the speed of an object over time and is based on<br />

position-time data. Speed is the relationship between distance (position) and<br />

time, v = d/t. For the first part of the wolf’s trip in the position versus time<br />

graph, the line rises steadily. This means the speed for this first leg is constant.<br />

If the wolf traveled this first leg faster, the slope of the line would be steeper.<br />

The wolf moved at the same speed toward his first two “visits.” His third trip<br />

was slightly slower. Except for this slight difference, the wolf was either at one<br />

speed or stopped (shown by a flat line in the speed versus time graph).<br />

Read the steps for each story. Sketch a position-time graph and a speed-time graph for each story.<br />

1. Graph Red Riding Hood’s movements<br />

according to the following events listed in<br />

the order they occurred:<br />

• Little Red Riding Hood set out for<br />

Grandmother’s cottage at a good walking pace.<br />

• She stopped briefly to talk to the wolf.<br />

• She walked a bit slower because they were<br />

talking as they walked to the wild flowers.<br />

• She stopped to pick flowers for quite a<br />

while.<br />

• Realizing she was late, Red Riding Hood ran the rest of the way to Grandmother’s cottage.<br />

4.2


Page 2 of 2<br />

2. Graph the movements of the Tortoise and the Hare. Use two lines to show the movements of each<br />

animal on each graph. The movements of each animal is listed in the order they occurred.<br />

• The tortoise and the hare began<br />

their race from the combined<br />

start-finish line. By the end of<br />

the race, the two will be at the<br />

same position at which they<br />

started.<br />

• Quickly outdistancing the<br />

tortoise, the hare ran off at a<br />

moderate speed.<br />

• The tortoise took off at a slow<br />

but steady speed.<br />

• The hare, with an enormous lead, stopped for a short nap.<br />

• With a startle, the hare awoke and realized that he had been sleeping for a long time.<br />

• The hare raced off toward the finish at top speed.<br />

• Before the hare could catch up, the tortoise’s steady pace won the race with an hour to spare.<br />

3. Graph the altitude of the sky rocket on its flight according to the following sequence of events listed in order.<br />

• The sky rocket was placed on the<br />

launcher.<br />

• As the rocket motor burned, the rocket<br />

flew faster and faster into the sky.<br />

• The motor burned out; although the<br />

rocket began to slow, it continued to<br />

coast even higher.<br />

• Eventually, the rocket stopped for a split<br />

second before it began to fall back to<br />

Earth.<br />

• Gravity pulled the rocket faster and faster toward Earth until a parachute popped out, slowing its<br />

descent.<br />

• The descent ended as the rocket landed gently on the ground.<br />

4. A story told from a graph: Tim, a student at Cumberland <strong>School</strong>, was determined to ask Caroline for a movie<br />

date. Use these graphs of his movements from his house to Caroline’s to write the story.<br />

4.2


Name: Date:<br />

4.3 Acceleration<br />

Acceleration is the rate of change in the speed of an object. To determine the rate of acceleration, you use the<br />

formula below. The units for acceleration are meters per second per second or m/s 2 .<br />

A positive value for acceleration shows speeding up, and negative value for acceleration shows slowing down.<br />

Slowing down is also called deceleration.<br />

The acceleration formula can be rearranged to solve for other variables such as final speed (v2) and time (t).<br />

v2 = v1 + ( a× t)<br />

t =<br />

v2 – v1 --------------a<br />

1. A skater increases her velocity from 2.0 m/s to 10.0 m/s in 3.0 seconds. What is the skater’s acceleration?<br />

Looking for<br />

Acceleration of the skater<br />

Given<br />

Beginning speed = 2.0 m/s<br />

Final speed = 10.0 m/s<br />

Change in time = 3.0 seconds<br />

Relationship<br />

v2 – v1 a = --------------t<br />

Solution<br />

The acceleration of the skater is 2.7 meters per<br />

second per second.<br />

2. A car accelerates at a rate of 3.0 m/s 2 . If its original speed is 8.0 m/s, how many seconds will it take the car<br />

to reach a final speed of 25.0 m/s?<br />

Looking for<br />

Acceleration<br />

The time to reach the final speed.<br />

Given<br />

Beginning speed = 8.0 m/s; Final speed = 25.0 m/s<br />

Acceleration = 3.0 m/s2 Relationship<br />

v2 – v1 t =<br />

--------------a<br />

=<br />

Final<br />

-----------------------------------------------------------------------speed<br />

– Beginning speed<br />

Time<br />

a =<br />

v2 – v1 --------------t<br />

Acceleration<br />

Solution<br />

Time<br />

10.0<br />

-------------------------------------------m/s<br />

– 2.0 m/s<br />

2.7 m/s<br />

3.0 s<br />

2<br />

= =<br />

25.0 m/s – 8.0 m/s<br />

3.0 m/s 2<br />

= ------------------------------------------- = 5.7 s<br />

The time for the car to reach its final speed is<br />

5.7 seconds.<br />

4.3


Page 2 of 2<br />

1. While traveling along a highway, a driver slows from 24 m/s to 15 m/s in 12 seconds. What is the<br />

automobile’s acceleration? (Remember that a negative value indicates a slowing down or deceleration.)<br />

2. A parachute on a racing dragster opens and changes the speed of the car from 85 m/s to 45 m/s in a period of<br />

4.5 seconds. What is the acceleration of the dragster?<br />

3. The table below contains data for a ball rolling down a hill. Fill in the missing data values in the table and<br />

determine the acceleration of the rolling ball.<br />

Time (seconds) Speed (km/h)<br />

0 (start) 0 (start)<br />

2 3<br />

6<br />

9<br />

8<br />

10 15<br />

4. A car traveling at a speed of 30.0 m/s encounters an emergency and comes to a complete stop. How much<br />

time will it take for the car to stop if it decelerates at –4.0 m/s 2 ?<br />

5. If a car can go from 0 to 60. mph in 8.0 seconds, what would be its final speed after 5.0 seconds if its starting<br />

speed were 50. mph?<br />

6. A cart rolling down an incline for 5.0 seconds has an acceleration of 4.0 m/s 2 . If the cart has a beginning<br />

speed of 2.0 m/s, what is its final speed?<br />

7. A helicopter’s speed increases from 25 m/s to 60 m/s in 5 seconds. What is the acceleration of this<br />

helicopter?<br />

8. As she climbs a hill, a cyclist slows down from 25 mph to 6 mph in 10 seconds. What is her deceleration?<br />

9. A motorcycle traveling at 25 m/s accelerates at a rate of 7.0 m/s 2 for 6.0 seconds. What is the final speed of<br />

the motorcycle?<br />

10. A car starting from rest accelerates at a rate of 8.0 m/s. What is its final speed at the end of 4.0 seconds?<br />

11. After traveling for 6.0 seconds, a runner reaches a speed of 10. m/s. What is the runner’s acceleration?<br />

12. A cyclist accelerates at a rate of 7.0 m/s 2 . How long will it take the cyclist to reach a speed of 18 m/s?<br />

13. A skateboarder traveling at 7.0 meters per second rolls to a stop at the top of a ramp in 3.0 seconds. What is<br />

the skateboarder’s acceleration?<br />

4.3


Name: Date:<br />

4.3 Acceleration and Speed-Time Graphs<br />

Acceleration is the rate of change in the speed of an object. The graph below shows that object A accelerated<br />

from rest to 10 miles per hour in two hours. The graph also shows that object B took four hours to accelerate<br />

from rest to the same speed. Therefore, object A accelerated twice as fast as object B.<br />

Calculating acceleration from a speed-time graph<br />

The steepness of the line in a speed-time graph is related to<br />

acceleration. This angle is the slope of the line and is found by<br />

dividing the change in the y-axis value by the change in the xaxis<br />

value.<br />

Acceleration<br />

Δy<br />

-----<br />

Δx<br />

In everyday terms, we can say that the speed of object A<br />

“increased 10 miles per hour in two hours.” Using the slope<br />

formula:<br />

Acceleration<br />

=<br />

Δy<br />

----- = --------------------------------------<br />

10 mph – 0 mph<br />

=<br />

Δx 2 hours – 0 hour<br />

• Acceleration = Δy/Δx (the symbol Δ means “change in”)<br />

• Acceleration = (10 mph – 0 mph)/(2 hours – 0 hours)<br />

• Acceleration = 5 mph/hour (read as 5 miles per hour per hour)<br />

The double per time label attached to all accelerations may seem confusing at first. It is not so alien a concept if<br />

you break it down into its parts:<br />

The speed changes. . . . . . during this amount of time:<br />

5 miles per hour each hour<br />

Accelerations can be negative. If the line slopes downward, Δy will be a negative number because a larger value<br />

of y will be subtracted from a smaller value of y.<br />

Calculating distance from a speed-time graph<br />

=<br />

5<br />

-------------mph<br />

hour<br />

The area between the line on a speed-time graph and the baseline is equal to the distance that an object travels.<br />

This follows from the rate formula:<br />

Rate or Speed =<br />

Distance<br />

--------------------<br />

Time<br />

v =<br />

d<br />

--<br />

t<br />

4.3


Page 2 of 3<br />

Or, rewritten:<br />

Notice how the labels cancel to produce a new label that fits the result.<br />

Here is a speed-time graph of a boat starting from one place and<br />

sailing to another:<br />

The graph shows that the sailboat accelerated between the second and<br />

third hour. We can find the total distance by finding the area between<br />

the line and the baseline. The easiest way to do that is to break the<br />

area into sections that are easy to solve and then add them together.<br />

• Use the formula for the area of a rectangle, A = L × W, to find<br />

areas A, B, and D.<br />

• Use the formula for finding the area of a triangle, A = l × w/2, to find area C.<br />

Calculate acceleration from each of these graphs.<br />

1. Graph 1:<br />

2. Graph 2:<br />

A+ B + C + D = distance<br />

vt = d<br />

miles/hour × 3 hours = 3 miles<br />

A+ B + C + D = distance<br />

10 miles + 5 miles + 1 mile + 21 miles =<br />

37 miles<br />

4.3


Page 3 of 3<br />

3. Graph 3:<br />

4. Find acceleration for segment 1 and segment 2 in this graph:<br />

5. Calculate total distance for this graph:<br />

6. Calculate total distance for this graph:<br />

7. Calculate total distance for this graph:<br />

4.3


Name: Date:<br />

4.3 Acceleration Due to Gravity<br />

Acceleration due to gravity is known to be 9.8 meters/second/second or<br />

9.8 m/s 2 and is represented by g. Three conditions must be met before we can<br />

use this value:<br />

(1) the object must be in free fall<br />

(2) the object must have negligible air resistance, and<br />

(3) the object must be close to the surface of Earth.<br />

In all of the examples and problems, we will assume that these conditions have<br />

been met. Remember that speed refers to “how fast” in any direction, but<br />

velocity refers to “how fast” in a specific direction. The sign of numbers in<br />

these calculations is important. Velocities upward shall be positive, and<br />

velocities downward shall be negative. Because the y-axis of a graph is<br />

vertical, change in height shall be indicated by y.<br />

Here is the equation for solving for velocity:<br />

final velocity = initial velocity + ( the acceleration due to the force of gravity × time)<br />

v = v0+ gt<br />

Imagine that an object falls for one second. We know that at the end of the<br />

second it will be traveling at 9.8 meters/second. However, it began its fall at<br />

zero meters/second. Therefore, its average velocity is half of<br />

9.8 meters/second. We can find distance by multiplying this average velocity<br />

by time.<br />

Here is the equation for solving for distance. See if you can find these concepts in the equation:<br />

distance<br />

OR<br />

the acceleration due to the force of gravity × time<br />

= ---------------------------------------------------------------------------------------------------------------------- × time<br />

2<br />

OR<br />

y =<br />

1<br />

--gt<br />

2<br />

2<br />

4.3


Page 2 of 3<br />

Example 1: How fast will a pebble be traveling 3.0 seconds after being dropped?<br />

v = v0+ gt<br />

+ ( – × 3.0 s)<br />

v 0 9.8 m/s 2<br />

(Note that gt is negative because the direction is downward.)<br />

Example 2: A pebble dropped from a bridge strikes the water in 4.0 seconds. How high is the bridge?<br />

y<br />

y<br />

=<br />

=<br />

=<br />

v = – 29 m/s<br />

y<br />

1<br />

-- × 9.8 m/s × 4.0 s × 4.0 s<br />

2<br />

Note that the seconds cancel. The answer written with the correct number of significant figures is 78 meters. The<br />

bridge is 78 meters high.<br />

1. A penny dropped into a wishing well reaches the bottom in 1.50 seconds. What was the velocity at impact?<br />

2. A pitcher threw a baseball straight up at 35.8 meters per second. What was the ball’s velocity after<br />

2.50 seconds? (Note that, although the baseball is still climbing, gravity is accelerating it downward.)<br />

3. In a bizarre but harmless accident, a watermelon fell from the top of the Eiffel Tower. How fast was the<br />

watermelon traveling when it hit the ground 7.80 seconds after falling?<br />

4. A water balloon was dropped from a high window and struck its target 1.1 seconds later. If the balloon left<br />

the person’s hand at –5.0 meters per second, what was its velocity on impact?<br />

5. A stone tumbles into a mine shaft and strikes bottom after falling for 4.2 seconds. How deep is the mine<br />

shaft?<br />

6. A boy threw a small bundle toward his girlfriend on a balcony 10. meters above him. The bundle stopped<br />

rising in 1.5 seconds. How high did the bundle travel? Was that high enough for her to catch it?<br />

=<br />

1<br />

--gt<br />

2<br />

2<br />

1<br />

-- 9.8 m/s<br />

2<br />

2<br />

× × 4.0 s × 4.0 s<br />

y =<br />

78.4 meters<br />

The equations demonstrated so far can be used to find time of flight from speed or distance,<br />

respectively. Remember that an object thrown into the air represents two mirror-image flights, one up<br />

and the other down.<br />

4.3


Page 3 of 3<br />

Time from velocity<br />

Time from distance<br />

Original equation Rearranged equation to solve for time<br />

v = v0+ gt<br />

t =<br />

v – v0 ------------g<br />

y<br />

1<br />

--gt<br />

2<br />

2<br />

= t =<br />

2y<br />

----g<br />

7. At about 55 meters per second, a falling parachuter (before the parachute opens) no longer accelerates. Air<br />

friction opposes acceleration. Although the effect of air friction begins gradually, imagine that the parachuter<br />

is free falling until terminal speed (the constant falling speed) is reached. How long would that take?<br />

8. The climber dropped her compass at the end of her 240-meter climb. How long did it take to strike bottom?<br />

4.3


Name: Date:<br />

5.1 Ratios and Proportions<br />

Professional chefs use ratios and proportions daily to figure out how much of various ingredients they will need<br />

to make a particular dish. They may serve the same dessert one day to a wedding party with 300 guests, and<br />

another day to a dinner party for eight people. Ratios and proportions help them figure out the right quantities of<br />

ingredients to buy for each meal. In this skill sheet, you will practice converting a recipe for different group sizes.<br />

A recipe for Double Fudge Brownies<br />

Ingredients:<br />

3 /4 c. sugar 2 eggs<br />

6 tablespoons unsalted butter 1 teaspoon vanilla extract<br />

2 tablespoons milk<br />

2 cups semi-sweet chocolate chips<br />

3 /4 cup all-purpose flour<br />

1 /3 teaspoon baking soda<br />

1 /4 teaspoon salt 2 tablespoons confectioner’s sugar<br />

Makes 16 brownies.<br />

1. What is the ratio of milk to chocolate chips in the recipe above?<br />

2<br />

-------------------------------tablespoons<br />

2 cups<br />

2. When we know the ratios, we can make proportions by setting two ratios equal to one another. This will help<br />

us to find missing answers.<br />

Suppose Patricia needs only 8 brownies and doesn’t want any leftovers. Find out how much of each<br />

ingredient she needs. The original recipe will make 16 brownies. You will use the ratio of 8 / 16 = 1 / 2 to find<br />

the amount for each of the ingredients. Use cross-multiplication to solve the proportions.<br />

For flour:<br />

Step 1<br />

Step 2<br />

Step 3<br />

Step 4<br />

Step 5 Patricia needs 3 -----<br />

8<br />

16<br />

= --------x<br />

3 ⁄ 4<br />

3<br />

8 × --<br />

4<br />

= 16x<br />

6 = 16x<br />

-----<br />

6<br />

16<br />

=<br />

16x<br />

--------<br />

16<br />

3<br />

--<br />

8<br />

=<br />

x<br />

/ 8 cup of flour to<br />

make 8 brownies.<br />

5.1


Page 2 of 2<br />

1. What is the ratio of unsalted butter to eggs?<br />

For every __________ tablespoons of butter, you will need __________ eggs.<br />

2. What is the ratio of flour to baking soda?<br />

For every __________ cups of flour, you will need __________ teaspoon of baking soda.<br />

3. What is the ratio of salt to flour?<br />

For every __________ teaspoon(s) of salt, you will need __________ cups of flour<br />

4. Find the correct amount of each ingredient to make 8 brownies ( 1 / 2 of the recipe).<br />

Flour<br />

Sugar<br />

Butter<br />

Milk<br />

Chocolate chips<br />

Eggs<br />

Vanilla extract<br />

Baking soda<br />

Salt<br />

Ingredient Amount<br />

Confectioner’s sugar<br />

3 /8 cup<br />

5. Why are the eggs and confectioner’s sugar amounts easy to work with to make 8 brownies?<br />

6. Patricia has a little extra of all the ingredients. How many brownies can be made using 3 cups of chocolate<br />

chips?<br />

7. How much vanilla will she need when she makes the batch of brownies using 3 cups of chocolate chips?<br />

5.1


Name: Date:<br />

5.1 Internet Research <strong>Skill</strong>s<br />

The Internet is a valuable tool for finding answers to your questions about the world. A search engine is like an<br />

on-line index to information on the World Wide Web. There are many different search engines to choose from.<br />

Search engines differ in how often they are updated, how many documents they contain in their index, and how<br />

they search for information. Your teacher may suggest several search engines for you to try.<br />

Search engines ask you to type a word or phrase into a box known as a field. Knowing how search engines work<br />

can help you pinpoint the information you need. However, if your phrase is too vague, you may end up with a lot<br />

of unhelpful information.<br />

How could you find out who was the first woman to participate in a space shuttle flight?<br />

First, put key phrases in quotation marks. You want to know about the “first woman” on a “space shuttle.”<br />

Quotation marks tell the engine to search for those words together.<br />

Second, if you only want websites that contain both phrases, use a + sign between them. Typing “first woman”<br />

+ “space shuttle” into a search engine will limit your search to websites that contain both phrases.<br />

If you want to broaden your search, use the word or between two terms. For example, if you type “first female”<br />

or “first woman” + “space shuttle” the search engine will list any website that contains either of the first two<br />

phrases, as long as it also contains the phrase “space shuttle.”<br />

You can narrow a search by using the word not. For example, if you wanted to know about marine mammals<br />

other than whales, you could type “marine mammals” not “whales” into the field. Please note that some search<br />

engines use the minus sign (-) rather than the word not.<br />

1. If you wanted to find out about science museums in your state that are not in your own city or town, what<br />

would you type into the search engine?<br />

2. If you wanted to find out which dog breeds are inexpensive, what would you type into the search engine?<br />

3. How could you research alternatives to producing electricity through the combustion of coal or natural gas?<br />

5.1


Page 2 of 2<br />

The quality of information found on the Internet varies widely. This section will give you some things<br />

to think about as you decide which sources to use in your research.<br />

1. Authority: How well does the author know the subject matter? If you search for “Newton’s laws” on the<br />

Internet, you may find a science report written by a fifth grade student, and a study guide written by a college<br />

professor. Which website is the most authoritative source?<br />

Museums, national libraries, government sites, and major, well-known “encyclopedia sources” are good<br />

places to look for authoritative information.<br />

2. Bias: Think about the author’s purpose. Is it to inform, or to persuade? Is it to get you to buy something?<br />

Comparing several authoritative sources will help you get a more complete understanding of your subject.<br />

3. Target audience: For whom was this website written? Avoid using sites designed for students well below<br />

your grade level. You need to have an understanding of your subject matter at or above your own grade<br />

level. Even authoritative sites for younger students (children’s encyclopedias, for example) may leave out<br />

details and simplify concepts in ways that would leave gaps in your understanding of your subject.<br />

4. Is the site up-to-date, clear, and easy to use? Try to find out when the website was created, and when it<br />

was last updated. If the site contains links to other sites, but those links don’t work, you may have found a<br />

site that is infrequently or no longer maintained. It may not contain the most current information about your<br />

subject. Is the site cluttered with distracting advertisements? You may wish to look elsewhere for the<br />

information you need.<br />

1. What is your favorite sport or activity? Search for information about this sport or activity. List two sites that<br />

are authoritative and two sites that are not authoritative. Explain your reasoning. Finally, write down the best<br />

site for finding out information about your favorite sport.<br />

2. Search for information about a physical science topic of your choice on the Internet (i.e., “simple machines,”<br />

“Newton’s Laws,” “Galileo”). Find one source that you would NOT consider authoritative. Write the key<br />

words you used in your search, the web address of the source, and a sentence explaining why this source is<br />

not authoritative.<br />

3. Find a different source that is authoritative, but intended for a much younger audience. Write the web<br />

address and a sentence describing who you think the intended audience is.<br />

4. Find three sources that you would consider to be good choices for your research here. Write two to three<br />

sentence description of each. Describe the author, the intended audience, the purpose of the site, and any<br />

special features not found in other sites.<br />

5.1


Name: Date:<br />

5.1 Preparing a Bibliography<br />

When you write a research paper or prepare a presentation for your class, it is important to document your<br />

sources. A bibliography serves two purposes. First, a bibliography gives credit to the authors who wrote the<br />

material you used to learn about your subject. Second, a bibliography provides your audience with sources they<br />

can use if they would like to learn more about your subject.<br />

This skill sheet provides bibliography formats and examples for various types of research materials you may use<br />

when preparing science papers and presentations.<br />

Books:<br />

Author last name, First name. (Year published). Title of <strong>book</strong>. Place of publication: Name of publisher.<br />

Vermeij, Geerat. (1997). Privileged Hands: A Scientific Life. New York: W.H. Freeman and Company.<br />

Newspaper and Magazine Articles:<br />

Author listed:<br />

Author last name, First name. (Date of publication). Title of Article. Title of Newspaper or Magazine, page<br />

# or #’s.<br />

Searcy, Dionne. (2006, March 20). Wireless Internet TV Is Launched in Oklahoma. The Wall Street Journal,<br />

p. B4.<br />

Brody, Jane. (2006, February/March). 10 Kids’ Nutrition Myth Busters. Nick Jr Family Magazine, pp. 72–73.<br />

No author listed:<br />

Title of article. (Date of publication). Title of Newspaper or Magazine, page # or #’s.<br />

Chew on this: Gum may speed recovery. (2006, March 20). St. Louis Post-Dispatch, p.H2.<br />

Adventures in Turning Trash into Treasure: (2006, April). Reader’s Digest, p. 24.<br />

5.1


Page 2 of 2<br />

Online Newspaper or Magazine:<br />

Author listed:<br />

Author last name, First name. (Date of publication). Title of Article. Title of Newspaper or Magazine,<br />

Retrieved date, from web address.<br />

Dybas, Cheryl Lyn. (2006, March 20). Early Spring Disturbing Life on Northern Rivers. The Washington Post,<br />

Retrieved March 22, 2006, from www.washingtonpost.com.<br />

No author listed:<br />

Title of Article. (Date of publication). Title of Newspaper or Magazine. Retrieved date, from web address.<br />

Comet mystery turns from hot to cold. (2006, March 20). The Boston Globe, retrieved March 22, 2006, from<br />

Boston.com.<br />

Online document:<br />

Author listed:<br />

Author last name, author first name. (Date of publication). Title of document. Retrieved date, from web<br />

address.<br />

Martinez, Carolina. (2006, March 9). NASA’s Cassini Discovers Potential Liquid Water on Enceladus. Retrieved<br />

March 22, 2006, from http://www.nasa.gov/mission_pages/cassini/media/cassini-20060309.html<br />

No Author listed:<br />

Organization responsible for website. (Date of publication). Title of document. Retrieved date, from web<br />

address.<br />

National Science Foundation. (2005, December 15). A fish of a different color. Retrieved March 22, 2206 from<br />

http://www.nsf.gov/news/news_summ.jsp?cntn_id=105661&org=NSF&from=news.<br />

5.1


Name: Date:<br />

5.1 Mass vs. Weight<br />

What is the difference between mass and weight?<br />

Weightlessness: When a diver dives off of a 10-meter diving board, she is in free-fall. If she jumped off the board<br />

with a scale attached to her feet, the scale would read zero even though she is under the influence of gravity. She<br />

is “weightless” because her feet have nothing to push against. Similarly, astronauts and everything inside a space<br />

shuttle seem to be weightless because they are in constant free fall. The space shuttle moves at high speed,<br />

therefore its constant fall toward Earth results in an orbit around the planet.<br />

• On Earth’s surface, the force of gravity acting on one kilogram is 2.2 pounds. So, if an object has a mass of<br />

2.0 kilograms, the force of gravity acting on that mass on Earth will be:<br />

• On the moon’s surface, the force of gravity is about 0.37 pounds per kilogram. The same object, if it were<br />

carried to the moon, would have a mass of 2.0 kilograms, but its weight would be just 0.74 pounds.<br />

1. What is the weight (in pounds) of a 7.0-kilogram bowling ball on Earth’s surface?<br />

2. What is the weight (in pounds) of a 7.0-kilogram bowling ball on the surface of the moon?<br />

3. What is the mass of a 7.0-kilogram bowling ball on the surface of the moon?<br />

4. Would a balance function correctly on the moon? Why or why not?<br />

Challenge Question<br />

mass weight<br />

• Mass is a measure of the amount of matter in an<br />

object. Mass is not related to gravity.<br />

• The mass of an object does not change when it<br />

is moved from one place to another.<br />

• Mass is commonly measured in grams or<br />

kilograms.<br />

5. Take a bathroom scale into an elevator. Step on the scale.<br />

• Weight is a measure of the gravitational force<br />

between two objects.<br />

• The weight of an object does change when the<br />

amount of gravitational force changes, as when an<br />

object is moved from Earth to the moon.<br />

• Weight is commonly measured in newtons or pounds.<br />

0.37 pounds<br />

2.0 kg × = 4.4 pounds<br />

kg<br />

0.37 pounds<br />

2.0 kg × =<br />

0.74 pounds<br />

kg<br />

a. What happens to the reading on the scale as the elevator begins to move upward? to move downward?<br />

b. What happens to the reading on the scale when the elevator stops moving?<br />

c. Why does your weight appear to change, even though you never left Earth’s gravity?<br />

5.1


Name: Date:<br />

5.1 Mass, Weight, and Gravity<br />

How do we define mass, weight, and gravity?<br />

How are mass, weight, and gravity related?<br />

The weight equation W = mg shows that an object’s weight (in newtons) is equal to its mass (in kilograms)<br />

multiplied by the strength of gravity (in newtons per kilogram) where the object is located. The weight equation<br />

can be rearranged to find weight, mass, or the strength of gravity if you know any two of the three.<br />

• Calculate the weight (in newtons) of a 5.0-kilogram backpack on Earth (g = 9.8 N/kilogram).<br />

Solution:<br />

• The same backpack weighs 8.2 newtons on Earth’s moon. What is the strength of gravity on the moon?<br />

Solution:<br />

mass weight gravity<br />

• Mass is a measure of the<br />

amount of matter in an<br />

object. Mass is not related<br />

to gravity.<br />

• The mass of an object does<br />

not change when it is<br />

moved from one place to<br />

another.<br />

• Mass is commonly<br />

measured in grams or<br />

kilograms.<br />

• Weight is a measure of the<br />

gravitational force between two<br />

objects.<br />

• The weight of an object does<br />

change when the amount of<br />

gravitational force changes, as<br />

when an object is moved from<br />

Earth to the moon.<br />

• Weight is commonly measured<br />

in newtons or pounds.<br />

5.1<br />

• The force that causes all masses<br />

to attract one another. The<br />

strength of the force depends on<br />

the size of the masses and their<br />

distance apart.<br />

Use... if you want to find... and you know...<br />

W = mg weight (W) mass (m) and strength of gravity (g)<br />

m = W/g mass (m) weight (W) and strength of gravity<br />

(g)<br />

g = W/m strength of gravity (g) weight (W) and mass (m)<br />

W = m( g)<br />

W = (5.0 kg)(9.8 N/kg)<br />

W = 49N<br />

g = W/ m<br />

g = 8.2 N/5.0 kg<br />

g =<br />

1.6 N/kg


Page 2 of 2<br />

1. A physical science text<strong>book</strong> has a mass of 2.2 kilograms.<br />

a. What is its weight on Earth?<br />

b. What is its weight on Mars? (g = 3.7 N/kg)<br />

c. If the text<strong>book</strong> weighs 19.6 newtons on Venus, what is the strength of gravity on that planet?<br />

2. An astronaut weighs 104 newtons on the moon, where the strength of gravity is 1.6 newtons per kilogram.<br />

a. What is her mass?<br />

b. What is her weight on Earth?<br />

c. What would she weigh on Mars?<br />

3. Of all the planets in our solar system, Jupiter has the greatest gravitational strength.<br />

a. If a 0.500-kilogram pair of running shoes would weigh 11.55 newtons on Jupiter, what is the strength of<br />

gravity there?<br />

b. If the same pair of shoes weighs 0.3 newtons on Pluto (a dwarf planet), what is the strength of gravity<br />

there?<br />

c. What does the pair of shoes weigh on Earth?<br />

4. A tractor-trailer truck carrying boxes of toy rubber ducks stops at a weigh station on the highway. The driver<br />

is told that the truck weighs 44,000. pounds.<br />

a. If there are 4.448 newtons in a pound, what is the weight of the toy-filled truck in newtons?<br />

b. What is the mass of the toy-filled truck?<br />

c. The truck drops off its load of toys, then stops at a second weigh station. Now the truck weighs<br />

33,000. pounds. What is its weight in newtons?<br />

d. Challenge! Find the total mass of the rubber duck-filled boxes that were carried by the truck.<br />

5.1


Name: Date:<br />

5.1 Gravity Problems<br />

In this skill sheet, you will practice using proportions as you learn more about the strength of gravity on different<br />

planets.<br />

Comparing the strength of gravity on the planets<br />

Table 1 lists the strength of gravity on each planet in our solar system. We can see more clearly how these values<br />

compare to each other using proportions. First, we assume that Earth’s gravitational strength is equal to “1.”<br />

Next, we set up the proportion where x equals the strength of gravity on another planet (in this case, Mercury) as<br />

compared to Earth.<br />

1<br />

x<br />

=<br />

Earth gravitational strength Mercury gravitational strength<br />

1 x<br />

=<br />

9.8 N/kg 3.7 N/kg<br />

(1× 3.7 N/kg) = (9.8 N/kg × x)<br />

3.7 N/kg<br />

= x<br />

9.8 N/kg<br />

0.38 =<br />

x<br />

Note that the units cancel. The result tells us that Mercury’s gravitational strength is a little more than a third of<br />

Earth’s. Or, we could say that Mercury’s gravitational strength is 38% as strong as Earth’s.<br />

Now, calculate the proportions for the remaining planets.<br />

Table 1: The strength of gravity on planets in our solar system<br />

Planet Strength of gravity<br />

(N/kg)<br />

Value compared to Earth’s<br />

gravitational strength<br />

Mercury 3.7 0.38<br />

Venus 8.9<br />

Earth 9.8 1<br />

Mars 3.7<br />

Jupiter 23.1<br />

Saturn 9.0<br />

Uranus 8.7<br />

Neptune 11.0<br />

Pluto 0.6<br />

5.1


Page 2 of 2<br />

How much does it weigh on another planet?<br />

Use your completed Table 1 to solve the following problems.<br />

Example:<br />

• A bowling ball weighs 15 pounds on Earth. How much would this bowling ball weigh on Mercury?<br />

Weight on Earth 1<br />

=<br />

Weight on Mercury 0.38<br />

1 15 pounds<br />

=<br />

0.38 x<br />

0.38× 15 pounds = x<br />

x = 5.7 pounds<br />

1. A cat weighs 8.5 pounds on Earth. How much would this cat weigh on Neptune?<br />

2. A baby elephant weighs 250 pounds on Earth. How much would this elephant weigh on Saturn? Give your<br />

answer in newtons (4.48 newtons = 1 pound).<br />

3. On Pluto, a baby would weigh 2.7 newtons. How much does this baby weigh on Earth? Give your answer in<br />

newtons and pounds.<br />

4. Imagine that it is possible to travel to each planet in our solar system. After a space “cruise,” a tourist returns<br />

to Earth. One of the ways he recorded his travels was to weigh himself on each planet he visited. Use the list<br />

of these weights on each planet to figure out the order of the planets he visited. On Earth he weighs<br />

720 newtons. List of weights: 655 N; 1,699 N; 806 N; 43 N; and 662 N.<br />

Challenge: Using the Universal Law of Gravitation<br />

Here is an example problem that is solved using<br />

the equation for Universal Gravitation.<br />

Example<br />

What is the force of gravity between Pluto and<br />

Earth? The mass of Earth is 6.0 × 10 24 kg. The<br />

mass of Pluto is 1.3 × 10 22 kg. The distance<br />

between these two planets is 5.76 × 10 12 meters.<br />

Force of gravity between Earth and Pluto<br />

Now use the equation for Universal Gravitation to solve this problem:<br />

6.67 10 -11<br />

× N-m 2<br />

kg 2<br />

⎛ ⎞<br />

⎜------------------------------------------ 6.0 10<br />

⎟<br />

⎝ ⎠<br />

24<br />

( × kg)<br />

1.3 10 22<br />

× ( × kg)<br />

5.76 10 12<br />

( × m)<br />

2<br />

=<br />

-----------------------------------------------------------------------------<br />

Force of gravity<br />

52.0 10 35<br />

×<br />

33.2 10 24<br />

-------------------------- 1.57 10<br />

×<br />

11 = =<br />

× N<br />

5. What is the force of gravity between Jupiter and Saturn? The mass of Jupiter is 6.4 × 10 24 kg. The mass of<br />

Saturn is 5.7 × 10 26 kg. The distance between Jupiter and Saturn is 6.52 × 10 11 m.<br />

5.1


Name: Date:<br />

5.1 Universal Gravitation<br />

The law of universal gravitation allows you to calculate the gravitational force between two objects from their<br />

masses and the distance between them. The law includes a value called the gravitational constant, or “G.” This<br />

value is the same everywhere in the universe. Calculating the force between small objects like grapefruits or huge<br />

objects like planets, moons, and stars is possible using this law.<br />

What is the law of universal gravitation?<br />

The force between two masses m 1 and m 2 that are separated by a distance r is given by:<br />

So, when the masses m 1 and m 2 are given in kilograms and the distance r is given in meters, the force has the<br />

unit of newtons. Remember that the distance r corresponds to the distance between the center of gravity of<br />

the two objects.<br />

For example, the gravitational force between two spheres that are touching each other, each with a radius of<br />

0.300 meter and a mass of 1,000. kilograms, is given by:<br />

=<br />

× 1,000. kg 1,000. kg<br />

F 6.67 10 11<br />

– N-m 2 kg 2<br />

⁄<br />

×<br />

( 0.300 m + 0.300 m)<br />

2<br />

---------------------------------------------------- = 0.000185 N<br />

Note: A small car has a mass of approximately 1,000. kilograms. Try to visualize this much mass<br />

compressed into a sphere with a diameter of 0.300 meters (30.0 centimeters). If two such spheres were<br />

touching one another, the gravitational force between them would be only 0.000185 newtons. On Earth, this<br />

corresponds to the weight of a mass equal to only 18.9 milligrams. The gravitational force is not very strong!<br />

5.1


Page 2 of 2<br />

Answer the following problems. Write your answers using scientific notation.<br />

1. Calculate the force between two objects that have masses of 70. kilograms and 2,000. kilograms. Their<br />

centers of gravity are separated by a distance of 1.00 meter.<br />

2. Calculate the force between two touching grapefruits each with a radius of 0.080 meters and a mass of<br />

0.45 kilograms.<br />

3. Calculate the force between one grapefruit as described above and Earth. Earth has a mass of<br />

5.9742 × 10 24 kilograms and a radius of 6.3710 × 10 6 meters. Assume the grapefruit is resting on Earth’s<br />

surface.<br />

4. A man on the moon with a mass of 90. kilograms weighs 146 newtons. The radius of the moon is<br />

1.74 × 106 meters. Find the mass of the moon.<br />

5. For m = 5.9742 × 1024 kilograms and r = 6.3710 × 106 meters, what is the value given by: G ?<br />

a. Write down your answer and simplify the units.<br />

b. What does this number remind you of?<br />

c. What real-life values do m and r correspond to?<br />

m<br />

r 2<br />

----<br />

6. The distance between the centers of Earth and its moon is 3.84 × 10 8 meters. Earth’s mass is<br />

5.9742 × 10 24 kilograms and the mass of the moon is 7.36 × 10 22 kilograms. What is the force between Earth<br />

and the moon?<br />

7. A satellite is orbiting Earth at a distance of 35.0 kilometers. The satellite has a mass of 500. kilograms. What<br />

is the force between the planet and the satellite? Hint: Recall Earth’s mass and radius from earlier problems.<br />

8. The mass of the sun is 1.99 × 10 30 kilograms and its distance from Earth is 150. million kilometers<br />

(150. × 10 9 meters). What is the gravitational force between the sun and Earth?<br />

5.1


Name: Date:<br />

5.2 Friction<br />

Just about every move we make involves friction of some sort. This skill sheet will provide you with the<br />

opportunity to practice identifying the friction force(s) involved in real-world situations.<br />

Marco and his dad are unloading cinder blocks from the back of their pickup truck. They need to haul the blocks<br />

across the grass to their backyard, where they are going to make a sandbox for Marco’s younger sister. Marco<br />

would like to haul a bunch of blocks at once. In the garage, he finds a plastic sled and his sister’s red wagon.<br />

• Which type of friction would resist Marco’s motion if he pulled the blocks in the sled?<br />

Solution: Sliding friction.<br />

1. Answer these additional questions about Marco’s sandbox building project.<br />

a. Which type of friction would resist Marco’s motion if he pulled the blocks in the wagon?<br />

b. Do you think it would take more force to transport five blocks in the sled or in the wagon? Why?<br />

c. Would the friction force increase, decrease, or stay the same if Marco added two more blocks to the sled<br />

or wagon? Explain your answer.<br />

d. Marco tries piling twelve cinder blocks into the wagon. He pulls and pulls but the wagon doesn’t move.<br />

What type of force is resisting motion now?<br />

2. Brianna is rowing a small boat across a pond. The air is calm; there is no wind blowing.<br />

a. What type of friction is resisting her motion?<br />

b. If two friends join her in the boat, will the friction force change? Why or why not?<br />

3. A freight train speeds along the railroad tracks at 150 km/hr.<br />

a. Name two types of friction resisting this motion.<br />

b. If this train were replaced with a mag-lev train, which type of friction would be eliminated?<br />

4. Research: Some sports cars are designed with rear spoiler to make the car more stable when turning,<br />

accelerating, and braking.<br />

a. Use the Internet or your local library to find an illustration of a spoiler to share with your class.<br />

b. Does the spoiler increase or decrease friction between the rear tires and the road?<br />

c. Some small hybrid cars and sport utility vehicles also have spoilers. What is their purpose? Is it the<br />

same or different from the spoiler on a sports car?<br />

5.2


Name: Date:<br />

5.3 Equilibrium<br />

When all forces acting on a body are balanced, the forces are in equilibrium. This skill<br />

sheet provides free-body diagrams for you to use for practice in working with<br />

equilibrium.<br />

Remember that an unbalanced force results in acceleration. Therefore, the forces<br />

acting on an object that is not accelerating must be balanced. These objects may be at<br />

rest, or they could be moving at a constant velocity. Either way, we say that the forces<br />

acting on these objects are in equilibrium.<br />

What force is necessary in the free-body diagram at right to achieve equilibrium?<br />

Looking for<br />

The unknown force: ? N<br />

Given<br />

600 N is pressing down on the box.<br />

400 N is pressing up on the box.<br />

Relationship<br />

You can solve equilibrium problems using simple equations:<br />

600 N = 400 N + ?N<br />

1. Supply the missing force<br />

necessary to achieve<br />

equilibrium.<br />

Solution<br />

600 N = 400 N + ?N<br />

600 N – 400 N = 400 N – 400 N + ? N<br />

200 N = ?N<br />

5.3


Page 2 of 2<br />

2. Supply the missing forces necessary to achieve equilibrium.<br />

3. In the picture, a girl with a weight of 540 N is balancing on her bike in equilibrium, not moving at all. If the<br />

force exerted by the ground on her front wheel is 200 N, how much force is exerted by the ground on her<br />

back wheel?<br />

Challenge Question:<br />

4. Helium balloons stay the same size as you hold them, but swell and burst<br />

as they rise to high altitudes when you let them go. Draw and label force<br />

arrows inside and/or outside the balloons on the graphic at right to show<br />

why the near Earth balloon does not burst, but the high altitude balloon<br />

does eventually burst. Hint: What are the forces on the inside of the<br />

balloon? What are the forces on the outside of the balloons?<br />

5.3


Name: Date:<br />

6.1 Net Force and Newton’s First Law<br />

Newton’s first law tells us that when the net force is zero, objects at rest stay at rest and objects in motion keep<br />

moving with the same speed and direction. Changes in motion come from unbalanced forces.<br />

In this skill sheet, you will practice identifying balanced and unbalanced forces in everyday situations.<br />

• An empty shopping cart is pushed along a grocery store aisle at constant velocity. Find the cart’s weight and<br />

the friction force if the shopper produces a force of 40.0 newtons between the wheels and the floor, and the<br />

normal force on the cart is 105 newtons.<br />

1. Looking for: You are asked for the cart’s weight and the friction force.<br />

2. Given: You are given the normal force and the force produced by the shopper pushing the<br />

cart.<br />

3. Relationships: Newton’s first law states that if the shopping cart is moving at a constant velocity, the<br />

net force must be zero.<br />

4. Solution: The weight of the cart balances the normal force. Therefore, the weight of the cart is a<br />

downward force: -105 N. The forward force produced by the shopper balances the<br />

friction force, so the friction force is -40.0 N.<br />

1. Identify the forces on the same cart at rest.<br />

2. While the cart is moving along an aisle, it comes in contact with a smear of margarine that had recently been<br />

dropped on the floor. Suddenly the friction force is reduced from -40.0 newtons to -20.0 newtons. What is<br />

the net force on the cart if the “pushing force” remains at 40.0 newtons? Does the grocery cart move at<br />

constant velocity over the spilled margarine?<br />

3. Identify the normal force on the shopping cart after 75 newtons of groceries are added to the cart.<br />

4. The shopper pays for his groceries and pushes the shopping cart out of the store, where he encounters a ramp<br />

that helps him move the cart from the sidewalk down to the parking lot. What force accelerates the cart down<br />

the ramp?<br />

5. Compare the friction force on the cart when it is rolling along the blacktop parking lot to the friction force on<br />

the cart when it is inside the grocery store (assume the flooring is smooth vinyl tile).<br />

6. Why is it easy to get one empty cart moving but difficult to get a line of 20 empty carts moving?<br />

6.1


Name: Date:<br />

6.1 Isaac Newton<br />

6.1<br />

Isaac Newton is one of the most brilliant figures in scientific history. His three laws of motion are<br />

probably the most important natural laws in all of science. He also made vital contributions to the fields of optics,<br />

calculus, and astronomy.<br />

Plague provides opportunity for genius<br />

Isaac Newton was born in<br />

1642 in Lincolnshire,<br />

England. His childhood years<br />

were difficult. His father died<br />

just before he was born. When<br />

he was three, his mother<br />

remarried and left her son to<br />

live with his grandparents.<br />

Newton bitterly resented his<br />

stepfather throughout his life.<br />

An uncle helped Newton remain in school and in<br />

1661, he entered Trinity College at Cambridge<br />

University. He earned his bachelor’s degree in 1665.<br />

Ironically, it was the closing of the university due to<br />

the bubonic plague in 1665 that helped develop<br />

Newton’s genius. He returned to Lincolnshire and<br />

spent the next two years in solitary academic pursuit.<br />

During this period, he made significant advances in<br />

calculus, worked on a revolutionary theory of the<br />

nature of light and color, developed early versions of<br />

his three laws of motion, and gained new insights into<br />

the nature of planetary motion.<br />

Fear of criticism stifles scientist<br />

When Cambridge reopened in 1667, Newton was<br />

given a minor position at Trinity and began his<br />

academic career. His studies in optics led to his<br />

invention of the reflecting telescope in the early<br />

1670s. In 1672, his first public paper was presented,<br />

on the nature of light and color.<br />

Newton longed for public recognition of his work but<br />

dreaded criticism. When another bright young<br />

scientist, Robert Hooke, challenged some of his<br />

points, Newton was furious. An angry exchange of<br />

words left Newton reluctant to make public more of<br />

his work.<br />

Revolutionary law of universal gravitation<br />

In the 1680s, Newton turned his attention to forces and<br />

motion. He worked on applying his three laws of<br />

motion to orbiting bodies, projectiles, pendulums, and<br />

free-fall situations. This work led him to formulate his<br />

famous law of universal gravitation.<br />

According to legend, Newton thought of the idea<br />

while sitting in his Lincolnshire garden. He watched<br />

an apple fall from a tree. He wondered if the same<br />

force that caused the apple to fall toward the center of<br />

Earth (gravity) might be responsible for keeping the<br />

moon in orbit around Earth, and the planets in orbit<br />

around the sun.<br />

This concept was truly revolutionary. Less than<br />

50 years earlier, it was commonly believed that some<br />

sort of invisible shield held the planets in orbit.<br />

Important contributor in spite of conflict<br />

In 1687, Newton published his ideas in a famous work<br />

known as the Principia. He jealously guarded the<br />

work as entirely his. He bitterly resented the<br />

suggestion that he should acknowledge the exchange<br />

of ideas with other scientists (especially Hooke) as he<br />

worked on his treatise.<br />

Newton left Cambridge to take a government position<br />

in London in 1696. His years of active scientific<br />

research were over. However, almost three centuries<br />

after his death in 1727, Newton remains one of the<br />

most important contributors to our understanding of<br />

how the universe works.


Page 2 of 2<br />

Reading reflection<br />

1. Important phases of Newton’s education and scientific work occurred in isolation. Why might this have been<br />

helpful to him? On the other hand, why is working in isolation problematic for developing scientific ideas?<br />

2. Newton began his academic career in 1667. For how long was he a working scientist? Was he a very<br />

productive scientist? Justify your answer.<br />

3. Briefly state one of Newton’s three laws of motion in your own words. Give an explanation of how this law<br />

works.<br />

4. Define the law of universal gravitation in your own words.<br />

5. The orbit of a space shuttle is surprisingly like an apple falling from a tree to Earth. The space shuttle is<br />

simply moving so fast that the path of its fall is an orbit around our planet. Which of Newton’s laws helps<br />

explain the orbit of a space shuttle around Earth and the orbit of Earth around the sun?<br />

6. Research: Newton was outraged when, in 1684, German mathematician Wilhelm Leibniz published a<br />

calculus <strong>book</strong>. Find out why, and describe how the issue is generally resolved today.<br />

6.1


Name: Date:<br />

6.2 Newton's Second Law<br />

• Newton’s second law states that the acceleration of an object is directly related to the force on it, and<br />

inversely related to the mass of the object. You need more force to move or stop an object with a lot of mass<br />

(or inertia) than you need for an object with less mass.<br />

• The formula for the second law of motion (first row below) can be rearranged to solve for mass and force.<br />

What do you want to know? What do you know? The formula you will use<br />

acceleration (a) force (F) and mass (m)<br />

mass (m) acceleration (a) and force (F)<br />

force (F) acceleration (a) and mass (m)<br />

• How much force is needed to accelerate a truck with a mass of 2,000 kilograms at a rate of 3 m/s 2 ?<br />

• What is the mass of an object that requires 15 N to accelerate it at a rate of 1.5 m/s 2 ?<br />

1. What is the acceleration of a 2,000.-kilogram truck if a force of 4,200. N is used to make it start moving<br />

forward?<br />

2. What is the acceleration of a 0.30-kilogram ball that is hit with a force of 25 N?<br />

3. How much force is needed to accelerate a 68-kilogram skier at 1.2 m/s 2 ?<br />

4. What is the mass of an object that requires a force of 30 N to accelerate at 5 m/s 2 ?<br />

acceleration<br />

5. What is the force on a 1,000.-kilogram elevator that is falling freely under the acceleration of gravity only?<br />

mass<br />

F m × a 2,000 kg 3 m/s2 × 6,000 kg-m/s2 = = = = 6,000 N<br />

m --<br />

F 15 N<br />

a 1.5 m/s 2<br />

--------------------<br />

15 kg-m/s2 1.5 m/s2 = = = -------------------------- =<br />

10 kg<br />

6. What is the mass of an object that needs a force of 4,500 N to accelerate it at a rate of 5 m/s 2 ?<br />

7. What is the acceleration of a 6.4-kilogram bowling ball if a force of 12 N is applied to it?<br />

=<br />

=<br />

force<br />

----------mass<br />

---------------------------force<br />

acceleration<br />

force = acceleration × mass<br />

6.2


Name: Date:<br />

6.3 Applying Newton’s Laws of Motion<br />

In the second column of the table below, write each of Newton’s three laws of motion. Use your own wording. In<br />

the third column of the table, describe an example of each law. To find examples of Newton’s laws, think about<br />

all the activities you do in one day.<br />

Newton’s laws of<br />

motion<br />

The first law<br />

The second law<br />

The third law<br />

Write the law here in your own<br />

words<br />

1. When Jane drives to work, she always places her pocket<strong>book</strong> on the passenger’s seat. By the time she gets to<br />

work, her pocket<strong>book</strong> has fallen on the floor in front of the passenger seat. One day, she asks you to explain<br />

why this happens in terms of physical science. What do you say?<br />

2. You are waiting in line to use the diving board at your local pool. While watching people dive into the pool<br />

from the board, you realize that using a diving board to spring into the air before a dive is a good example of<br />

Newton’s third law of motion. Explain how a diving board illustrates Newton’s third law of motion.<br />

3. You know the mass of an object and the force applied to the object to make it move. Which of Newton’s laws<br />

of motion will help you calculate the acceleration of the object?<br />

4. How many newtons of force are represented by the following amount: 3 kg · m/s 2 ?<br />

Select the correct answer (a, b, or c) and justify your answer.<br />

Example of the law<br />

a. 6 newtons b. 3 newtons c. 1 newton<br />

5. Your shopping cart has a mass of 65 kilograms. In order to accelerate the shopping cart down an aisle at<br />

0.30 m/s 2 , what force would you need to use or apply to the cart?<br />

6. A small child has a wagon with a mass of 10 kilograms. The child pulls on the wagon with a force of<br />

2 newtons. What is the acceleration of the wagon?<br />

7. You dribble a basketball while walking on a basketball court. List and describe the pairs of action-reaction<br />

forces in this situation.<br />

8. Pretend that there is no friction at all between a pair of ice skates and an ice rink. If a hockey player using<br />

this special pair of ice skates was gliding along on the ice at a constant speed and direction, what would be<br />

required for him to stop?<br />

6.3


Name: Date:<br />

6.3 Momentum<br />

Which is more difficult to stop: A tractor-trailer truck barreling down the<br />

highway at 35 meters per second, or a small two-seater sports car<br />

traveling the same speed?<br />

You probably guessed that it takes more force to stop a large truck than a<br />

small car. In physics terms, we say that the truck has greater momentum.<br />

We can find momentum using this equation:<br />

momentum = mass of object × velocity of object<br />

Velocity is a term that refers to both speed and direction. For our purposes we will assume that the vehicles are<br />

traveling in a straight line. In that case, velocity and speed are the same.<br />

The equation for momentum is abbreviated like this: P =<br />

m × v.<br />

Momentum, symbolized with a P, is expressed in units of kg · m/s; m is the mass of the object, in kilograms; and<br />

v is the velocity of the object in m/s.<br />

Use your knowledge about solving equations to work out the following problems:<br />

1. If the truck has a mass of 4,000. kilograms, what is its momentum? Express your answer in kg · m/s.<br />

2. If the car has a mass of 1,000. kilograms, what is its momentum?<br />

3. An 8-kilogram bowling ball is rolling in a straight line toward you. If its momentum is 16 kg · m/s, how fast<br />

is it traveling?<br />

4. A beach ball is rolling in a straight line toward you at a speed of 0.5 m/s. Its momentum is 0.25 kg · m/s.<br />

What is the mass of the beach ball?<br />

5. A 4,500.-kilogram truck travels in a straight line at 10. m/s. What is its momentum?<br />

6. A 1,500.-kilogram car is also traveling in a straight line. Its momentum is equal to that of the truck in the<br />

previous question. What is the velocity of the car?<br />

7. Which would take more force to stop in 10. seconds: an 8.0-kilogram ball rolling in a straight line at a speed<br />

of 0.2 m/s or a 4.0-kilogram ball rolling along the same path at a speed of 1.0 m/s?<br />

8. The momentum of a car traveling in a straight line at 25 m/s is 24,500 kg·m/s. What is the car’s mass?<br />

9. A 0.14-kilogram baseball is thrown in a straight line at a velocity of 30. m/s. What is the momentum of the<br />

baseball?<br />

10. Another pitcher throws the same baseball in a straight line. Its momentum is 2.1 kg · m/s. What is the<br />

velocity of the ball?<br />

11. A 1-kilogram turtle crawls in a straight line at a speed of 0.01 m/s. What is the turtle’s momentum?<br />

6.3


Name: Date:<br />

6.3 Momentum Conservation<br />

Just as forces are equal and opposite (according to Newton’s third law), changes in momentum are also equal and<br />

opposite. This is because when objects exert forces on each other, their motion is affected.<br />

The law of momentum conservation states that if interacting objects in a system are not acted on by outside<br />

forces, the total amount of momentum in the system cannot change.<br />

The formula below can be used to find the new velocities of objects if both keep moving after the collision.<br />

total momentum of a system before = total momentum of a system after<br />

m1v1 ( initial)<br />

m2v + 2 (initial)<br />

m1v3 (final) m2v = + 4 (final)<br />

If two objects are initially at rest, the total momentum of the system is zero.<br />

For the final momentum to be zero, the objects must have equal momenta in opposite directions.<br />

Example 1: What is the momentum of a 0.2-kilogram steel ball that is rolling at a velocity of 3.0 m/s?<br />

3 m<br />

momentum m× v 0.2 kg × -------s<br />

0.6 kg m<br />

= = = ⋅ --s<br />

Example 2: You and a friend stand facing each other on ice skates. Your mass is 50. kilograms and your friend’s<br />

mass is 60. kilograms. As the two of you push off each other, you move with a velocity of 4.0 m/s to the right.<br />

What is your friend’s velocity?<br />

Looking for<br />

Solution<br />

Your friend’s velocity to the left.<br />

Given<br />

Your mass of 50. kg.<br />

Your friend’s mass of 60. kg.<br />

Your velocity of 4.0 m/s to the right.<br />

Relationship<br />

m1v3 =<br />

the momentum of a system before a collision = 0<br />

0 = the momentum of a system after a collision<br />

0 = m1v3 + m2v4 m1v3 = – ( m2v4 )<br />

– ( m2v4 )<br />

m1v3 = – ( m2v4 )<br />

( 50. kg)<br />

( 4.0 m/s)<br />

= – 60. kg ( )<br />

200 kg-m/s<br />

– ( 60 kg)<br />

-------------------------- = v4 – 3.3 m/s = v4 ( ) v 4<br />

Your friend’s velocity to the left is 3.3 m/s.<br />

6.3


Page 2 of 2<br />

1. If a ball is rolling at a velocity of 1.5 m/s and has a momentum of 10.0 kg·m/s, what is the mass of<br />

the ball?<br />

2. What is the velocity of an object that has a mass of 2.5 kg and a momentum of 1,000 kg · m/s?<br />

3. A pro golfer hits 45.0-gram golf ball, giving it a speed of 75.0 m/s. What momentum has the golfer given to<br />

the ball?<br />

4. A 400-kilogram cannon fires a 10-kilogram cannonball at 20 m/s. If the cannon is on wheels, at what<br />

velocity does it move backward? (This backward motion is called recoil velocity.)<br />

5. Eli stands on a skateboard at rest and throws a 0.5-kg rock at a velocity of 10.0 m/s. Eli moves back at<br />

0.05 m/s. What is the combined mass of Eli and the skateboard?<br />

6. As the boat in which he is riding approaches a dock at 3.0 m/s, Jasper stands up in the boat and jumps toward<br />

the dock. Jasper applies an average force of 800. newtons on the boat for 0.30 seconds as he jumps.<br />

a. How much momentum does Jasper’s 80.-kilogram body have as it lands on the dock?<br />

b. What is Jasper's speed on the dock?<br />

7. Daryl the delivery guy gets out of his pizza delivery truck but forgets to set the parking brake. The<br />

2,000.-kilogram truck rolls down hill reaching a speed of 30 m/s just before hitting a large oak tree. The<br />

vehicle stops 0.72 s after first making contact with the tree.<br />

a. How much momentum does the truck have just before hitting the tree?<br />

b. What is the average force applied by the tree?<br />

8. Two billion people jump up in the air at the same time with an average velocity of 7.0 m/s. If the mass of an<br />

average person is 60 kilograms and the mass of Earth is 5.98 × 10 24 kilograms:<br />

a. What is the total momentum of the two billion people?<br />

b. What is the effect of their action on Earth?<br />

9. Tammy, a lifeguard, spots a swimmer struggling in the surf and jumps from her lifeguard chair to the sand<br />

beach. She makes contact with the sand at a speed of 6.00 m/s, leaving an indentation in the sand 0.10 m<br />

deep.<br />

a. If Tammy's mass is 60. kilograms, what is the momentum as she first touches the sand?<br />

b. What is the average force applied on Tammy by the sand beach?<br />

10. When a gun is fired, the shooter describes the sensation of the gun kicking. Explain this in terms of<br />

momentum conservation.<br />

11. What does it mean to say that momentum is conserved?<br />

6.3


Name: Date:<br />

6.3 Collisions and Conservation of Momentum<br />

The law of conservation of momentum tells us that as long as colliding objects are not influenced by outside<br />

forces like friction, the total amount of momentum in the system before and after the collision is the same.<br />

We can use the law of conservation of momentum to predict how two objects will move after a collision. Use the<br />

problem solving steps and the examples below to help you solve collision problems.<br />

Problem Solving Steps<br />

1. Draw a diagram.<br />

2. Assign variables to represent the masses and velocities of the objects before and after the collision.<br />

3. Write an equation stating that the total momentum before the collision equals the total after.<br />

4. Plug in the information that you know.<br />

5. Solve your equation.<br />

A 2,000-kilogram railroad car moving at 5 m/s collides with a<br />

6,000-kilogram railroad car at rest. If the cars coupled together,<br />

what is their velocity after the inelastic collision?<br />

Looking for<br />

v 3 = the velocity of the combined<br />

railroad cars after an inelastic collision<br />

Given<br />

Initial speed and mass of both cars:<br />

m1 = 2,000 kg, v1 = 5 m/s<br />

m2 = 6,000 kg, v2 = 0 m/s<br />

Combined mass of the two cars:<br />

m1 + m2 = 8,000 kg<br />

Relationship<br />

m1v1 + m2v2 = (m1 + m2)v3 Solution<br />

( 2000 kg)<br />

( 5 m/s)<br />

+ ( 6000 kg)<br />

( 0 m/s)<br />

= ( 2000 kg + 6000 kg)v<br />

3<br />

10,000 kg-m/s = ( 8000 kg)v<br />

3<br />

10,000 kg-m/s<br />

----------------------------<br />

8000 kg<br />

= v3 10 m/s =<br />

v<br />

3<br />

The velocity of the two combined cars after the<br />

collision is 10 m/s.<br />

6.3


Page 2 of 2<br />

1. What is the momentum of a 100.-kilogram fullback carrying a football on a play at a velocity of<br />

3.5 m/s?<br />

2. What is the momentum of a 75.0-kilogram defensive back chasing the fullback at a velocity of 5.00 m/s?<br />

3. A 2,000-kilogram railroad car moving at 5 m/s to the east collides with a 6,000-kilogram railroad car moving<br />

at 3 m/s to the west. If the cars couple together, what is their velocity after the collision?<br />

4. A 4.0-kilogram ball moving at 8.0 m/s to the right collides with a 1.0-kilogram ball at rest. After the<br />

collision, the 4.0-kilogram ball moves at 4.8 m/s to the right. What is the velocity of the 1-kilogram ball?<br />

5. A 0.0010-kg pellet is fired at a speed of 50.0m/s at a motionless 0.35-kg piece of balsa wood. When the<br />

pellet hits the wood, it sticks in the wood and they slide off together. With what speed do they slide?<br />

6. Terry, a 70.-kilogram tailback, runs through his offensive line at a speed of 7.0 m/s. Jared, a 100-kilogram<br />

linebacker, running in the opposite direction at 6.0 m/s, meets Jared head-on and “wraps him up.” What is<br />

the result of this tackle?<br />

7. Snowboarding cautiously down a steep slope at a speed of 7.0 m/s, Sarah, whose mass is 50. kilograms, is<br />

afraid she won't have enough speed to travel up a slight uphill grade ahead of her. She extends her hand as<br />

her friend Trevor, who has a mass of 100. kilograms, is about to pass her traveling at 16 m/s. If Trevor grabs<br />

her hand, calculate the speed at which the friends will be sliding.<br />

8. Tex, an 85.0 kilogram rodeo bull rider is thrown from the bull after a short ride. The 520. kilogram bull<br />

chases after Tex at 13.0 m/s. While running away at 3.00 m/s, Tex jumps onto the back of the bull to avoid<br />

being trampled. How fast does the bull run with Tex aboard?<br />

9. Identical twins Kate and Karen each have a mass of 45 kg. They are rowing their boat on a hot summer<br />

afternoon when they decide to go for a swim. Kate jumps off the front of the boat at a speed of 3.00 m/s.<br />

Karen jumps off the back at a speed of 4.00 m/s. If the 70.-kilogram rowboat is moving at 1.00 m/s when the<br />

girls jump, what is the speed of the rowboat after the girls jump?<br />

10. A 0.10-kilogram piece of modeling clay is tossed at a motionless 0.10-kilogram block of wood and sticks.<br />

The block slides across a frictionless table at 15 m/s.<br />

a. At what speed was the clay tossed?<br />

b. The clay is replaced with a “bouncy” ball with the same mass. It is tossed with the same speed. The<br />

bouncy ball rebounds from the wooden block at a speed of 10 m/s. What effect does this have on the<br />

wooden block? Why?<br />

6.3


Name: Date:<br />

6.3 Rate of Change of Momentum<br />

Momentum is given by the expression p = mv where p is the momentum of an object of mass m moving with<br />

velocity v. The units of momentum are kg-m/s. Change of momentum (represented Δp) over a time interval<br />

(represented Δt) is also called the rate of change of momentum.<br />

Since, momentum is p = mv, if the mass remains constant during the time Δt, then:<br />

Δv<br />

The expression, ----- , represents change in velocity over change in time, also known as acceleration. From<br />

Δt<br />

Newton’s second law, we know that acceleration equals force divided by mass (a = F/m). Rearranging the<br />

equation, we see that force equals mass times acceleration (F = ma). Similarly, force (F) equals change in<br />

momentum over change in time.<br />

A mass, m, moving with velocity, v, has momentum mv. If this momentum becomes zero over some change in<br />

time (Δt), then there is a force, F =(mv –0)/Δt.<br />

• mv is the initial momentum.<br />

• 0 is the momentum after a change in time Δt.<br />

Δp<br />

------ m<br />

Δt<br />

Δv<br />

= -----<br />

Δt<br />

F ma m Δv<br />

= = ----- =<br />

Δt<br />

When a car accelerates or decelerates, we feel a force that pushes back during acceleration and pushes us forward<br />

during deceleration. When the car brakes slowly, the force is small. However, when the car brakes quickly, the<br />

force increases considerably.<br />

Example 1. An 80-kg woman is a passenger in a car going 90 km/h. The driver puts on the brakes and the car<br />

comes to a stop in 2 seconds. What is the average force felt by the passenger?<br />

First, convert the velocity to a value that is in meters per second: 90 km/h = 25 m/s. Next, use the equation that<br />

relates force and momentum:<br />

This is a large force, and for the passenger to stay in her seat, she must be strapped in with a seat belt.<br />

When the stopping time decreases from 2 seconds to 1 second, the force increases to 2,000 newtons. When the<br />

car is involved in a crash, the change in momentum happens over a much shorter period of time, thereby creating<br />

very large forces on the passenger. Air bags and seat belts help by slowing down the person’s momentum change,<br />

resulting in smaller forces and a reduced chance for injury. Let’s look at some numbers.<br />

Δp<br />

------<br />

Δt<br />

Δp<br />

Force ------ m<br />

Δt<br />

Δv<br />

-----<br />

25 0<br />

80 kg<br />

Δt<br />

– ( ) m/s<br />

= = = ----------------------------- =<br />

1,000 N<br />

2 s<br />

6.3


Page 2 of 3<br />

The car travels at 90 kilometers per hour, crashes, and comes to a stop in 0.1 seconds. The air bag<br />

inflates and cushions the person for 1.5 seconds. Let’s calculate the force experienced by the passenger<br />

in an automobile without air bags and in one case with air bags.<br />

• Without the air bag, the momentum change happens over 0.1 seconds. This results in a force:<br />

The human body is not likely to survive a force as large as this.<br />

• With the air bag, the force created is:<br />

The chances for survival are much higher.<br />

25 m/s<br />

Force = 80 kg--------------- = 2,000 N<br />

0.1 s<br />

25 m/s<br />

Force = 80 kg--------------- = 1,333 N<br />

1.5 s<br />

Example 2. A pile is driven into the ground by hitting it repeatedly. If the pile is hit by the driver mass at a rate of<br />

100 kg/s and with a speed of 10 m/s, calculate the resulting average force on the pile.<br />

We are told that the driver mass hits the pile at a rate of 100 kg/s. What does this mean exactly? We can have a<br />

100-kilogram mass hitting the pile every second, or a 50-kilogram mass hitting the pile every half-second, or a<br />

200-kilogram mass hitting the pile every 2 seconds. You get the idea.<br />

The speed (v) with which the mass hits the pile is 10 m/s. The mass (m) is 100 kilograms. Time changes occur at<br />

1-second intervals. The force on the pile is:<br />

Force m Δv<br />

----- 100<br />

Δt<br />

kg<br />

----- 10<br />

s<br />

m<br />

--kg<br />

m<br />

= = = 1,000 ----------- =<br />

1,000 N<br />

s<br />

1. A 1,000-kg wrecking ball hits a wall with a speed of 2 m/s and comes to a stop in 0.01 s. Calculate the force<br />

experienced by the wall.<br />

2. A 0.15-kg soccer ball is rolling with a speed of 10 m/s and is stopped by the frictional force between it and<br />

the grass. If the average friction force is 0.5 N, how long would this take?<br />

3. Water comes out of a fire hose at a rate of 5.0 kg/s and with a speed of 50 m/s. Calculate the force on the<br />

hose. (This is the force that the firefighter has to provide in order to hold the hose.)<br />

4. Water from a fire hose is hitting a wall straight on. The water comes out with a flow rate of 25 kg/s and hits<br />

the wall with a speed of 30. m/s. What is the resulting force exerted on the wall by the water?<br />

5. The water at Niagara Falls flows at a rate of 3.0 million kg/s. The water hits the bottom of the falls at a speed<br />

of 25 m/s. What is the force generated by the change in momentum of the falling water?<br />

s 2<br />

6.3


Page 3 of 3<br />

6. A 50.-g (0.050-kg) egg that is dropped from a height of 5.0 m will hit the floor with a speed of<br />

about 10. m/s. The hard floor forces the egg to stop very quickly. Let’s say that it will stop in<br />

0.0010 second.<br />

a. What is the force created on the egg?<br />

b. The egg will break at the force you calculated for 6(a). Imagine that a 50.-kilogram person fell down on<br />

the egg falling under the influence of gravity. What would the force of the person on the egg be?<br />

c. Do you think the egg will break if the person fell on it? Why or why not?<br />

d. If we now drop the egg onto a pillow, it will allow the egg to stop over a much longer time compared with<br />

the time it takes for it to stop on the hard surface. The weight and the velocity of the egg is still the same,<br />

but now the time it takes for the egg to come to rest is much longer, about 0.5 second or about 500 times<br />

longer than the time it took to stop on the floor. What would the force on the egg be under these<br />

circumstances?<br />

e. Do you think the egg will break when it drops on the pillow? Why or why not?<br />

6.3


Name: Date:<br />

7.1 Mechanical Advantage<br />

Mechanical advantage (MA) is the ratio of output force to input force for a machine.<br />

F<br />

MA =<br />

F<br />

or<br />

MA =<br />

O<br />

i<br />

output force (N)<br />

input force (N)<br />

Did you notice that the force unit involved in the calculation, the newton (N) is present in both the numerator and<br />

the denominator of the fraction? These units cancel each other, leaving the value for mechanical advantage<br />

unitless.<br />

newtons N<br />

= = 1<br />

newtons N<br />

Mechanical advantage tells you how many times a machine multiplies the force put into it. Some machines<br />

provide us with more output force than we applied to the machine—this means MA is greater than one. Some<br />

machines produce an output force smaller than our effort force, and MA is less than one. We choose the type of<br />

machine that will give us the appropriate MA for the work that needs to be performed.<br />

Example 1: A force of 200 newtons is applied to a machine in order to lift a 1,000-newton load. What is the<br />

mechanical advantage of the machine?a<br />

output force (N) 1000 N<br />

MA = = = 5<br />

input force (N) 200 N<br />

Machines make work easier. Work is force times distance (W = F × d). The unit for work is the newton-meter.<br />

Using the work equation, as shown in example 2 below, can help calculate the mechanical advantage.<br />

Example 2: A force of 30 newtons is applied to a machine through a distance of 10 meters. The machine is<br />

designed to lift an object to a height of 2 meters. If the total work output for the machine is 18 newton-meters<br />

(N-m), what is the mechanical advantage of the machine?<br />

input force = 30 N output force = (work ÷ distance) = (18 N=m ÷ 2 m) = 9N<br />

output force (N) 9 N<br />

MA = = =<br />

0.3<br />

input force (N) 30 N<br />

7.1


Page 2 of 2<br />

1. A machine uses an input force of 200 newtons to produce an output force of 800 newtons. What is<br />

the mechanical advantage of this machine?<br />

2. Another machine uses an input force of 200 newtons to produce an output force of 80 newtons. What is the<br />

mechanical advantage of this machine?<br />

3. A machine is required to produce an output force of 600 newtons. If the machine has a mechanical<br />

advantage of 6, what input force must be applied to the machine?<br />

4. A machine with a mechanical advantage of 10 is used to produce an output force of 250 newtons. What input<br />

force is applied to this machine?<br />

5. A machine with a mechanical advantage of 2.5 requires an input force of 120 newtons. What output force is<br />

produced by this machine?<br />

6. An input force of 35 newtons is applied to a machine with a mechanical advantage of 0.75. What is the size<br />

of the load this machine could lift (how large is the output force)?<br />

7. A machine is designed to lift an object with a weight of 12 newtons. If the input force for the machine is set<br />

at 4 newtons, what is the mechanical advantage of the machine?<br />

8. An input force of 50. newtons is applied through a distance of 10. meters to a machine with a mechanical<br />

advantage of 3. If the work output for the machine is 450 newton · meters and this work is applied through a<br />

distance of 3 meters, what is the output force of the machine?<br />

9. 200. newton·meters of work is put into a machine over a distance of 20. meters. The machine does<br />

150. newton·meters of work as it lifts a load 10. meters high. What is the mechanical advantage of<br />

the machine?<br />

10. A machine has a mechanical advantage of 5. If 300. newtons of input force is used to produce<br />

3,000. newton · meters of work,<br />

a. What is the output force?<br />

b. What is the distance over which the work is applied?<br />

7.1


Name: Date:<br />

7.1 Mechanical Advantage of Simple Machines<br />

We use simple machines to make tasks easier. While the output work of a simple machine can never be greater<br />

than the input work, a simple machine can multiply input forces OR multiply input distances (but never both at<br />

the same time). You can use this skill sheet to practice calculating mechanical advantage (MA) for two common<br />

simple machines: levers and ramps.<br />

The general formula for the mechanical advantage (MA) of levers:<br />

Or you can use the ratio of the input arm length to the output arm<br />

length:<br />

Most of the time, levers are used to multiply force to lift heavy objects.<br />

The general formula for the mechanical advantage (MA) of ramps:<br />

A ramp makes it possible to move a heavy load to a new height using less<br />

force (but over a longer distance).<br />

Example 1: A construction worker uses a board and log as a lever to lift a<br />

heavy rock. If the input arm is 3 meters long and the output arm is 0.75 meters<br />

long, what is the mechanical advantage of the lever?<br />

3 meters<br />

MA = 4<br />

0.75 meter =<br />

Example 2: Sometimes levers are used to multiply distance. For a broom, your<br />

upper hand is the fulcrum and your lower hand provides the input force: Notice<br />

the input arm is shorter than the output arm. The mechanical advantage of this<br />

broom is:<br />

3 meter<br />

MA = = 0.25<br />

0.75 meter<br />

A mechanical advantage less than one doesn’t mean a machine isn’t useful. It just<br />

means that instead of multiplying force, the machine multiplies distance. A broom<br />

doesn’t push the dust with as much force as you use to push the broom, but a small<br />

movement of your arm pushes the dust a large distance.<br />

o<br />

MA lever =<br />

F i (input force)<br />

7.1<br />

F (output force)<br />

L (length of input arm)<br />

i<br />

MA lever =<br />

L o (length of output arm)<br />

ramp length<br />

MA ramp =<br />

ramp height


Page 2 of 2<br />

Example 3: A 500-newton cart is lifted to a height of 1 meter<br />

using a 10-meter long ramp. You can see that the worker only<br />

has to use 50 newtons of force to pull the cart. You can figure<br />

the mechanical advantage in either of these two ways:<br />

Or using the standard formula for mechanical advantage:<br />

Lever problems<br />

1. A lever used to lift a heavy box has an input arm of 4 meters and an output arm of 0.8 meters. What is the<br />

mechanical advantage of the lever?<br />

2. What is the mechanical advantage of a lever that has an input arm of 3 meters and an output arm of 2 meters?<br />

3. A lever with an input arm of 2 meters has a mechanical advantage of 4. What is the output arm’s length?<br />

4. A lever with an output arm of 0.8 meter has a mechanical advantage of 6. What is the length of the input<br />

arm?<br />

5. A rake is held so that its input arm is 0.4 meters and its output arm is 1.0 meters. What is the mechanical<br />

advantage of the rake?<br />

6. A broom with an input arm length of 0.4 meters has a mechanical advantage of 0.5. What is the length of the<br />

output arm?<br />

7. A child’s toy rake is held so that its output arm is 0.75 meters. If the mechanical advantage is 0.33, what is<br />

the input arm length?<br />

Ramp problems<br />

ramp length 10 meters<br />

MA ramp = = = 10<br />

ramp height 10 meter<br />

output force 500 newtons<br />

MA = = =<br />

10<br />

input force 50 newtons<br />

8. A 5-meter ramp lifts objects to a height of 0.75 meters. What is the mechanical advantage of the ramp?<br />

9. A 10-meter long ramp has a mechanical advantage of 5. What is the height of the ramp?<br />

10. A ramp with a mechanical advantage of 8 lifts objects to a height of 1.5 meters. How long is the ramp?<br />

11. A child makes a ramp to push his toy dump truck up to his sandbox. If he uses 5 newtons of force to push the<br />

12-newton truck up the ramp, what is the mechanical advantage of his ramp?<br />

12. A ramp with a mechanical advantage of 6 is used to move a 36-newton load. What input force is needed to<br />

push the load up the ramp?<br />

13. Gina wheels her wheelchair up a ramp using a force of 80 newtons. If the ramp has a mechanical advantage<br />

of 7, what is the output force (in newtons)?<br />

14. Challenge! A mover uses a ramp to pull a 1000-newton cart up to the floor of his truck (0.8 meters high). If<br />

it takes a force of 200 newtons to pull the cart, what is the length of the ramp?<br />

7.1


Name: Date:<br />

7.1 Work<br />

In science, “work” is defined with an equation. Work is the amount of force applied to an object (in the same<br />

direction as the motion) over a distance. By measuring how much force you have used to move something over a<br />

certain distance, you can calculate how much work you have accomplished.<br />

The formula for work is:<br />

A joule of work is actually a newton·meter; both units represent the same thing: work! In fact, one joule of work<br />

is defined as the amount of work done by pushing with a force of one newton for a distance of one meter.<br />

1.0 joule = 1.0 newton × 1.0 meter = 1.0 newton ⋅ meter<br />

• How much work is done on a 10-N block that is lifted 5 m off the ground by a pulley?<br />

Solution: The force applied by the pulley to lift the block is equal to the block’s weight.We can use the<br />

formula W = F × d to solve the problem:<br />

1. In your own words, define work as a scientific term.<br />

2. How are work, force, and distance related?<br />

Work (joules) = Force (newtons) × distance (meters)<br />

W = F × d<br />

Work = 10 newtons × 5 meters = 50 newton ⋅<br />

meters<br />

3. What are two different units that represent work?<br />

4. For the following situations, determine whether work was done. Write “work done” or “no work done” for<br />

each situation.<br />

a. An ice skater glides for two meters across ice.<br />

b. The ice skater’s partner lifts her up a distance of 1 m.<br />

c. The ice skater’s partner carries her across the ice a distance of 3 m.<br />

d. After setting her down, the ice skater’s partner pulls her across the ice a distance of 10 m.<br />

e. After skating practice, the ice skater lifts her 20-N gym bag up 0.5 m.<br />

5. A woman lifts her 100-N child up one meter and carries her for a distance of 50 m to the child’s bedroom.<br />

How much work does the woman do?<br />

6. How much work does a mother do if she lifts each of her twin babies upward 1.0 m? Each baby weighs<br />

90. N.<br />

7.1


Page 2 of 2<br />

7. You pull your sled through the snow a distance of 500 m with a horizontal force of 200 N. How<br />

much work did you do?<br />

8. Because the snow suddenly gets too slushy, you decide to carry your 100-N sled the rest of the way<br />

home. How much work do you do when you pick up the sled, lifting it 0.5 m upward? How much work do<br />

you do to carry the sled if your house is 800 m away?<br />

9. An ant sits on the back of a mouse. The mouse carries the ant across the floor for a distance of 10 m. Was<br />

there work done by the mouse? Explain.<br />

10. You decide to add up all the work you did yesterday. If you accomplished 10,000 N · m of work yesterday,<br />

how much work did you do in units of joules?<br />

11. You did 150. J of work lifting a 120.-N backpack.<br />

a. How high did you lift the backpack?<br />

b. How much did the backpack weigh in pounds? (Hint: There are 4.448 N in one pound.)<br />

12. A crane does 62,500 J of work to lift a boulder a distance of 25.0 m. How much did the boulder weigh?<br />

(Hint: The weight of an object is considered to be a force in units of newtons.)<br />

13. A bulldozer does 30,000. J of work to push another boulder a distance of 20. m. How much force is applied<br />

to push the boulder?<br />

14. You lift a 45-N bag of mulch 1.2 m and carry it a distance of 10. m to the garden. How much work was<br />

done?<br />

15. A 450.-N gymnast jumps upward a distance of 0.50 m to reach the uneven parallel bars. How much work did<br />

she do before she even began her routine?<br />

16. It took a 500.-N ballerina a force of 250 J to lift herself upward through the air. How high did she jump?<br />

17. A people-moving conveyor-belt moves a 600-N person a distance of 100 m through the airport.<br />

a. How much work was done?<br />

b. The same 600-N person lifts his 100-N carry-on bag upward a distance of 1 m. They travel another 10 m<br />

by riding on the “people mover.” How much work was done in this situation?<br />

18. Which person did the most work?<br />

a. John walks 1,000. m to the store. He buys 4.448 N of candy and then carries it to his friend’s house<br />

which is 500. m away.<br />

b. Sally lifts her 22-N cat a distance of 0.50 m.<br />

c. Henry carries groceries from a car to his house. Each bag of groceries weighs 40 N. He has 10 bags. He<br />

lifts each bag up 1 m to carry it and then walks 10 m from his car to his house.<br />

7.1


Name: Date:<br />

7.1 Types of Levers<br />

A lever is a simple machine that can be used to multiply force, multiply distance, or change the direction of a<br />

force. All levers contain a stiff structure that rotates around a point called the fulcrum. The force applied to a<br />

lever is called the input force. The force applied to a load is called the output force.<br />

There are three types or classes of levers. The class of a lever depends on the location of the fulcrum and input<br />

and output forces. The picture below shows examples of the three classes of levers. Look at each lever carefully,<br />

noticing the location of the fulcrum, input force, and output force.<br />

1. In which class of lever is the output force between the fulcrum and input force?<br />

2. In which class of lever is the fulcrum between the input force and output force?<br />

3. In which class of lever is the fulcrum on one end and the output force on the other end?<br />

4. Do the following for each of the levers shown below and at the top of the next page:<br />

a. Label the fulcrum (F).<br />

b. Label the location of the input force (I) and output force (O).<br />

c. Classify the lever as first, second, or third class.<br />

7.1


Page 2 of 2<br />

The relationship between a lever’s input force and output force depends on the length of the input arm and<br />

output arm. The input arm is the distance between the fulcrum and input force. The output arm is the distance<br />

between the fulcrum and output force.<br />

If the input and output arms are the same length, the forces are equal. If the input arm is longer, the input force is<br />

less than the output force. If the input arm is shorter, the input force is greater than the output force.<br />

O<br />

1. Label the input arm (IA) and output arm (OA) on each of the levers you labeled above and on the previous<br />

page.<br />

2. In which of the levers is the input force greater than the output force?<br />

3. In which of the levers is the output force greater than the input force?<br />

4. In which of the levers are the input and output forces equal in strength?<br />

5. Find two other examples of levers. Draw each lever and label the fulcrum, input force, output force, input<br />

arm, and output arm. State whether the input or output force is stronger.<br />

7.1


Name: Date:<br />

7.1 Gear Ratios<br />

A gear ratio is used to figure out the number of turns each gear in<br />

a pair will make based on the number of teeth each gear has.<br />

To calculate the gear ratio for a pair of gears that are working<br />

together, you need to know the number of teeth on each gear. The<br />

formula below demonstrates how to calculate a gear ratio.<br />

Notice that knowing the number of teeth on each gear allows you<br />

to figure out how many turns each gear will take.<br />

Why would this be important in figuring out how to design a<br />

clock that has a minute and hour hand?<br />

A gear with 48 teeth is connected to a gear with 12 teeth. If the 48-tooth gear makes one complete turn, how<br />

many times will the 12-tooth gear turn?<br />

Turns of output gear? 48 input teeth<br />

=<br />

One turn for the input gear 12 output teeth<br />

48 teeth × 1 turn<br />

Turns of output gear? = =<br />

4 turns<br />

12 teeth<br />

1. A 36-tooth gear turns three times. It is connected to a 12-tooth gear. How many times does the 12-tooth gear<br />

turn?<br />

2. A 12-tooth gear is turned two times. How many times will the 24-tooth gear to which it is connected turn?<br />

3. A 60-tooth gear is connected to a 24-tooth gear. If the smaller gear turns ten times, how many turns does the<br />

larger gear make?<br />

4. A 60-tooth gear is connected to a 72-tooth gear. If the smaller gear turns twelve times, how many turns does<br />

the larger gear make?<br />

5. A 72-tooth gear is connected to a 12-tooth gear. If the large gear makes one complete turn, how many turns<br />

does the small gear make?<br />

7.1


Page 2 of 2<br />

6. Use the gear ratio formula to help you fill in the table below.<br />

Table 1: Using the gear ratio to calculate number of turns<br />

7.1<br />

Input Output<br />

Gear ratio How many turns does How many turns does<br />

Gear<br />

(# of teeth)<br />

Gear<br />

(# of teeth)<br />

(Input Gear:<br />

Output Gear)<br />

the output gear make<br />

if the input gear turns<br />

3 times?<br />

the input gear make if<br />

the output gear turns<br />

2 times?<br />

24 24<br />

36 12<br />

24 36<br />

48 36<br />

24 48<br />

7. The problems in this section involve three gears stacked on top of each other. Once you have filled in<br />

Table 2, answer the questions that follow. Use the gear ratio formula to help. Remember, knowing the gear<br />

ratios allows you to figure out the number of turns for a pair of gears.<br />

Table 2: Set up for three gears<br />

Setup Gears Number<br />

of teeth<br />

1 Top gear 12<br />

Middle gear 24<br />

Bottom gear 36<br />

2 Top gear 24<br />

Middle gear 36<br />

Bottom gear 12<br />

3 Top gear 12<br />

Middle gear 48<br />

Bottom gear 24<br />

4 Top gear 24<br />

Middle gear 48<br />

Bottom gear 36<br />

Ratio<br />

(top gear:<br />

middle gear)<br />

Ratio 2<br />

(middle gear:<br />

bottom gear)<br />

Total gear ratio<br />

(Ratio 1 x Ratio 2)<br />

8. As you turn the top gear to the right, what direction does the middle gear turn? What direction will the<br />

bottom gear turn?<br />

9. How many times will you need to turn the top gear (input) in setup 1 to get the bottom gear (output) to turn once?<br />

10. If you turn the top gear (input) in setup 2 two times, how many times will the bottom gear (output) turn?<br />

11. How many times will the middle gear (output) in setup 3 turn if you turn the top gear (input) two times?<br />

12. How many times will you need to turn the top gear (input) in setup 4 to get the bottom gear (output) to turn 4<br />

times?


Name: Date:<br />

7.1 Levers in the Human Body<br />

Your skeletal and muscular systems work together to move your body parts. Some of your body parts can be<br />

thought of as simple machines or levers.<br />

There are three parts to all levers:<br />

• Fulcrum - the point at which the lever rotates.<br />

• Input force (also called the effort) - the force applied to the lever.<br />

• Output force (also called the load) - the force applied by the lever to move the load.<br />

There are three types of levers: first class, second class and third class. In a first class lever, the fulcrum is located<br />

between the input force and output force. In a second class lever, the output force is between the fulcrum and the<br />

input force. In a third class lever, the input force is between the fulcrum and the output force. An example of each<br />

type of lever is shown below.<br />

7.1


Page 2 of 2<br />

The three classes of levers can be found in your body. Use<br />

diagrams A, B, and C to answer the questions below. Also<br />

label the effort (input force), fulcrum and load (output force)<br />

on each diagram.<br />

LEVER A<br />

1. Type of Lever: __________<br />

2. How is this lever used in the body?<br />

LEVER B<br />

3. Type of Lever: __________<br />

4. How is this lever used in the body?<br />

LEVER C<br />

5. Type of Lever: __________<br />

6. How is this lever used in the body?<br />

7.1


Name: Date:<br />

7.1 Bicycle Gear Ratios Project<br />

How many gears does your bicycle really have?<br />

Bicycle manufacturers describe any bicycle with two gears in the front and five in the back as a ten-speed. But do<br />

you really get ten different speeds? In this project, you will determine and record the gear ratio for each speed of<br />

your bicycle. You will then write up an explanation of the importance (or lack of, in some cases) of each speed.<br />

You will explain what the rider experiences due to the physics of the gear ratio, and in what situation the rider<br />

would take advantage of that particular speed.<br />

To complete this project, you will need:<br />

• Multi-speed bicycle<br />

• Simple calculator<br />

• Access to a library or the Internet for research<br />

• Access to a computer for work with a spreadsheet (optional)<br />

On a multi-speed bicycle, there are two groups of gears: the front group and the rear group. You may want to<br />

carefully place your bicycle upside down on the floor to better work with the gears. The seat and handlebars will<br />

keep the bicycle balanced.<br />

1. Draw a schematic diagram to show how the gears are set up on your bicycle.<br />

2. Now, count the number of teeth on each gear in each group. Record your data in a table on paper or in a<br />

computer spreadsheet. Use these questions to guide you.<br />

a. How many gears are in the front group?<br />

b. How many teeth on each gear in the front group?<br />

c. How many gears are in the rear group?<br />

d. How many teeth on each gear in the rear group?<br />

7.1


Page 2 of 2<br />

3. Now, calculate the gear ratio for each front/rear combination of gears.<br />

Use the formula: front gear ÷ rear gear.<br />

Organize the results of your calculations into a new table either on paper or in a computer<br />

spreadsheet.<br />

How many different gear ratios do you actually have?<br />

4. Use your library or the Internet to research the development of the multi-speed bicycle. Take careful notes<br />

while you do your research as you will use the information you find to write a report (see step 7). In your<br />

research, find the answers to the following questions.<br />

a. In what circumstances would a low gear ratio be helpful? Why?<br />

b. In what circumstances would a high gear ratio be helpful? Why?<br />

5. Write up your findings and results according to the guidelines below.<br />

Your final project should include:<br />

• A brief (one page) report that discusses the evolution of the bicycle. What was the first bicycle like? How<br />

did we end up with the modern bicycle? Why was the multi-speed bicycle an important invention?<br />

• A schematic diagram of your bicycle’s gears. Include labels.<br />

• An organized, professional data table showing the gear ratios of your bicycle.<br />

• A summary report (one page) in which you interpret your findings and explain the trade-off between force<br />

and distance when pedaling a bicycle in each of the different speeds. Include answers to questions 4(a) and<br />

4(b). In your research, you should make a surprising discovery about the speeds—what is it?<br />

• Reflection: Finish the report with one or two paragraphs that express your reflections on this project.<br />

7.1


Name: Date:<br />

7.2 Potential and Kinetic Energy<br />

This skill sheet reviews various forms of energy and introduces formulas for two kinds of mechanical energy—<br />

potential and kinetic. You will learn how to calculate the amount of kinetic or potential energy for an object.<br />

Forms of energy<br />

Forms of energy include radiant energy from the sun, chemical energy from the food you eat, and electrical<br />

energy from the outlets in your home. Mechanical energy refers to the energy an object has because of its motion.<br />

All these forms of energy may be used or stored. Energy that is stored is called potential energy. Energy that is<br />

being used for motion is called kinetic energy. All types of energy are measured in joules or newton-meters.<br />

Potential energy<br />

The word potential means that something is capable of becoming active. Potential energy sometimes is referred<br />

to as stored energy. This type of energy often comes from the position of an object relative to Earth. A diver on<br />

the high diving board has more energy than someone who dives into the pool from the low dive.<br />

The formula to calculate the potential energy of an object is the mass of the object times the acceleration due to<br />

gravity (9.8 m/s 2 ) times the height of the object.<br />

Did you notice that the mass of the object in kilograms times the acceleration of gravity (g) is the same as the<br />

weight of the object in newtons? Therefore you can think of an object’s potential energy as equal to the object’s<br />

weight multiplied by its height.<br />

So...<br />

Kinetic energy<br />

1 N = 1 kg i<br />

2<br />

m<br />

2<br />

s =<br />

1 joule= 1 kg i 1 N i m<br />

Kinetic energy is the energy of motion. Kinetic energy depends on the mass of the object as well as the speed of<br />

that object. Just think of a large object moving at a very high speed. You would say that the object has a lot of<br />

energy. Since the object is moving, it has kinetic energy. The formula for kinetic energy is:<br />

m<br />

2<br />

s<br />

Ep= mgh<br />

9.8 m<br />

mass of the object (kilograms) × = weight of the object (newtons)<br />

2<br />

s<br />

E p = weight of object × height of object<br />

1<br />

Ek= mv<br />

2<br />

2<br />

7.2


Page 2 of 2<br />

To do this calculation, square the velocity value. Next, multiply by the mass, and then divide by 2.<br />

How are these mechanical energy formulas used in everyday situations? Take a look at two example problems.<br />

• A 50 kg boy and his 100 kg father went jogging. Both ran at a rate of 5 m/s. Who had more kinetic energy?<br />

Show your work and explain.<br />

Solution: Although the boy and his father were running at the same speed, the father has more kinetic<br />

energy because he has more mass.<br />

The kinetic energy of the boy:<br />

The kinetic energy of the father:<br />

2 2<br />

1 ⎛5 m⎞<br />

m<br />

(50 kg) ⎜ ⎟ = 625 kg i = 625 joules<br />

2<br />

2 ⎝ s ⎠<br />

s<br />

2 2<br />

1 ⎛5 m⎞<br />

m<br />

(100 kg) ⎜ ⎟ = 1,250 kg i = 1,250 joules<br />

2<br />

2 ⎝ s ⎠<br />

s<br />

• What is the potential energy of a 10 N <strong>book</strong> that is placed on a shelf that is 2.5 m high?<br />

Solution: The <strong>book</strong>’s weight (10 N) is equal to its mass times the acceleration of gravity. Therefore, you can<br />

easily use this value in the potential energy formula:<br />

potential energy = mgh = (10 N)(2.5 m) = 25 N i<br />

m = 25 joules<br />

Now it is your turn to try calculating potential and kinetic energy. Don’t forget to keep track of the units!<br />

1. Determine the amount of potential energy of a 5.0-N <strong>book</strong> that is moved to three different shelves on a<br />

<strong>book</strong>case. The height of each shelf is 1.0 m, 1.5 m, and 2.0 m.<br />

2. You are on in-line skates at the top of a small hill. Your potential energy is equal to 1,000. J. The last time<br />

you checked, your mass was 60.0 kg.<br />

a. What is your weight in newtons?<br />

b. What is the height of the hill?<br />

c. If you start rolling down this hill, your potential energy will be converted to kinetic energy. At the bottom<br />

of the hill, your kinetic energy will be equal to your potential energy at the top. Calculate your speed at<br />

the bottom of the hill.<br />

3. A 1.0-kg ball is thrown into the air with an initial velocity of 30. m/s.<br />

a. How much kinetic energy does the ball have?<br />

b. How much potential energy does the ball have when it reaches the top of its ascent?<br />

c. How high into the air did the ball travel?<br />

4. What is the kinetic energy of a 2,000.-kg boat moving at 5.0 m/s?<br />

5. What is the velocity of an 500-kg elevator that has 4000 J of energy?<br />

6. What is the mass of an object traveling at 30. m/s if it has 33,750 J of energy?<br />

7.2


Name: Date:<br />

7.2 Identifying Energy Transformations<br />

Systems change when energy flows and changes from one part of a system to another. Parts of a system may<br />

speed up or slow down, get warmer or colder, or change in other measurable ways. Each change transfers energy<br />

or transforms energy from one form to another. In this skill sheet, you will practice identifying energy<br />

transformations in various systems.<br />

• At 5:30 a.m., Miranda’s electric alarm clock starts beeping (1). It’s still dark outside so she switches on the<br />

light (2). She stumbles sleepily down the hall to the kitchen (3), where she lights a gas burner on the stove<br />

(4) to warm some oatmeal for breakfast.<br />

Miranda has been awake for less than ten minutes, and she’s already participated in at least four energy<br />

transformations. Describe an energy transformation that took place in each of the numbered events above.<br />

Solution:<br />

1. Electrical energy to sound energy; 2. Electrical energy to radiant energy (light and heat); 3. Chemical<br />

energy from food to kinetic energy; 4. Chemical energy from natural gas to radiant energy (heat and light).<br />

1. There is a spring attached to the screen door on Elijah’s front porch. Elijah opens the door, stretching the<br />

spring (1). After walking through the doorway (2), Elijah lets go of the door, and the spring contracts,<br />

pulling the door shut (3). Describe an energy transformation that took place in each of the numbered events<br />

above.<br />

2. Name two energy transformations that occur as Gabriella heats a bowl of soup in the microwave.<br />

3. Dmitri uses a hand-operated air pump to inflate a small swimming pool for his younger siblings. Name two<br />

energy transformations that occurred.<br />

4. Simon puts new batteries in his radio-controlled car and its controller. He activates the controller, which<br />

sends a radio signal to the car. The car moves forward. Name at least three energy transformations that<br />

occurred.<br />

5. Name two energy transformations that occur as Adeline pedals her bicycle up a steep hill and then coasts<br />

down the other side.<br />

7.2


Name: Date:<br />

7.2 Energy Transformations—Extra Practice<br />

You have learned that the amount of energy in the universe is constant and that in any situation requiring energy,<br />

all of it must be accounted for. This is the basis for the law of conservation of energy. In this skill sheet you will<br />

analyze different scenarios in terms of what happens to energy. Based on your experience with the CPO energy<br />

car, you already know that potential energy can be changed into kinetic energy and vice versa.<br />

As you study the scenarios below, specify whether kinetic energy is being changed to potential energy, potential<br />

is being converted to kinetic, or neither. Explain your answers.<br />

For each scenario, see if you can also answer the following questions: Are other energy transformations<br />

occurring? In each scenario, where did all the energy go?<br />

• A roller coaster car travels from point A to point B.<br />

Solution:<br />

First, potential energy is changed into kinetic energy<br />

when the roller coaster car rolls down to the bottom of<br />

the first hill. But when the car goes up the second hill to<br />

point B, kinetic energy is changed to potential energy.<br />

Some energy is lost to friction. That is why point B is a little lower than point A.<br />

1. A bungee cord begins to exert an upward force on a falling<br />

bungee jumper.<br />

2. A football is spiraling downward toward a football player.<br />

3. A solar cell is charging a battery.<br />

7.2


Page 2 of 2<br />

Energy Scenarios<br />

7.2<br />

Read each scenario below. Then complete the following for each scenario:<br />

• Identify which of the following forms of energy are involved in the scenario:<br />

mechanical, radiant, electrical, chemical, and nuclear.<br />

• Make an energy flow chart that shows how the energy changes from one form to another, in the correct order.<br />

Use a separate paper and colored markers to make your flow charts more interesting.<br />

• In Western states, many homes generate electricity from<br />

windmills. In a particular home, a young boy is using the<br />

electricity to run a toy electric train.<br />

Solution:<br />

Mechanical energy of the windmill is changed to electrical energy which is changed to the mechanical<br />

energy of the toy train.<br />

1. A camper is using a wood fire to heat up a pot of water for<br />

tea. The pot has a whistle that lets the camper know when the<br />

water boils.<br />

2. The state of Illinois generates some of its electricity from<br />

nuclear power. A young woman in Chicago is watching a<br />

broadcast of a sports game on television.<br />

3. A bicyclist is riding at night. He switches on his bike’s<br />

generator so that his headlight comes on. The harder he<br />

pedals, the brighter his headlight glows.


Name: Date:<br />

7.2 Conservation of Energy<br />

The law of conservation of energy tells us that energy can never be created or destroyed—it is just transformed<br />

from one form to another. The total energy after a transformation (from potential to kinetic energy, for example)<br />

is equal to the total energy before the transformation. We can use this law to solve real-world problems, as shown<br />

in the example below.<br />

• A 0.50-kilogram ball is tossed upward with a kinetic energy of 100. joules. How high does the ball travel?<br />

1. Looking for: The maximum height of the ball.<br />

2. Given: The mass of the ball, 0.50 kg, and the kinetic energy at the start: 100. joules<br />

3. Relationships: E K = 1 / 2mv 2 ; E p = mgh<br />

4. Solution: The potential energy at the top of the ball’s flight is equal to its kinetic energy at the<br />

start. Therefore, E p = mgh = 100. joules.<br />

Substitute into the equation m = 0.50 kg and g = 9.8 m/s 2 .<br />

100. = mgh = (0.50)(9.8)h = 4.9h<br />

Solve for h.<br />

100. = 4.9h; 100.÷ 4.9 = h<br />

h = 20. m<br />

1. A 3.0-kilogram toy dump truck moving with a speed of 2.0 m/s starts up a ramp. How high does the truck<br />

roll before it stops?<br />

2. A 2.0-kilogram ball rolling along a flat surface starts up a hill. If the ball reaches a height of 0.63 meters,<br />

what was its initial speed?<br />

3. A 500.-kilogram roller coaster starts from rest at the top of an 80.0-meter hill. What is its speed at the bottom<br />

of this hill?<br />

4. Find the potential energy of this roller coaster when it is halfway down the hill.<br />

5. A 2.0-kilogram ball is tossed straight up with a kinetic energy of 196 joules. How high does it go?<br />

6. A 50.-kilogram rock rolls off the edge of a cliff. If it is traveling at a speed of 24.2 m/s when it hits the<br />

ground, what is the height of the cliff?<br />

7. Challenge! Make up your own energy conservation problem. Write the problem and the answer on separate<br />

index cards. Exchange problem cards with a partner. Solve the problems and then check each other’s work<br />

using the answer cards. If your answers don’t agree, work together to find the source of error.<br />

7.2


Name: Date:<br />

7.2 James Prescott Joule<br />

7.2<br />

James Joule was known for the accuracy and precision of his work in a time when exactness of<br />

measurements was not held in high regard. He demonstrated that heat is a form of energy. He studied the<br />

nature of heat and the relationship of heat to mechanical work. Joule has also been credited with finding the<br />

relationship between the flow of electricity through a resistance, such as a wire, and the heat given off from it.<br />

This is now known as Joule’s Law. He is remembered for his work that led to the First Law of Thermodynamics<br />

(Law of Conservation of Energy).<br />

The young student<br />

James Joule was born near Manchester, England on<br />

December 24, 1818. His father was a wealthy brewery<br />

owner. James injured his spine when he was young<br />

and as a result he spent a great deal of time indoors,<br />

reading and studying. When he became interested in<br />

science, his father built him a lab in the basement.<br />

When James was fifteen years old, his father hired<br />

John Dalton, a leading scientist at the time, to tutor<br />

James and his brother, Benjamin. Dalton believed that<br />

a scientist needed a strong math background. He spent<br />

four years teaching the boys Euclidian mathematics.<br />

He also taught them the importance of taking exact<br />

measurements, a skill that strongly influenced James<br />

in his scientific endeavors.<br />

Brewer first, scientist second<br />

After their father became ill, James and Benjamin ran<br />

the family brewery. James loved the brewery, but he<br />

also loved science. He continued to perform<br />

experiments as a serious hobby. In his lab, he tried to<br />

make a better electric motor using electromagnets.<br />

James wanted to replace the old steam engines in the<br />

brewery with these new motors.<br />

Though he learned a lot about magnets, heat, motion,<br />

and work, he was not able to change the steam engines<br />

in the brewery. The cost of the zinc needed to make<br />

the batteries for the electric motors was much too<br />

high. Steam engines fired by coal were more cost<br />

efficient.<br />

The young scientist<br />

In 1840, when he was only twenty-two years old,<br />

Joule wrote what would later be known as Joule’s<br />

Law. This law explained that electricity produces heat<br />

when it travels through a wire due to the resistance of<br />

the wire. Joule’s Law is still used today to calculate<br />

the amount of heat produced from electricity.<br />

By 1841, Joule focused<br />

most of his attention on the<br />

concept of heat. He<br />

disagreed with most of his<br />

peers who believed that<br />

heat was a fluid called<br />

caloric. Joule argued that<br />

heat was a state of vibration<br />

caused by the collision of<br />

molecules. He showed that<br />

no matter what kind of<br />

mechanical work was done,<br />

a given amount of mechanical work always produced<br />

the same amount of heat. Thus, he concluded, heat<br />

was a form of energy. He established this kinetic<br />

theory nearly 100 years before others truly accepted<br />

that molecules and atoms existed.<br />

On his honeymoon<br />

In 1847, Joule married Amelia Grimes, and the couple<br />

spent their honeymoon in the Alps. Joule had always<br />

been fascinated by waterfalls. He had observed that<br />

water was warmer at the bottom of a waterfall than at<br />

the top. He believed that the energy of the falling<br />

water was transformed into heat energy. While he and<br />

his new bride were in the Alps, he tried to prove his<br />

theory. His experiment failed because there was too<br />

much spray from the waterfall, and the water did not<br />

fall the correct distance for his calculations to work.<br />

From 1847–1854, Joule worked with a scientist named<br />

William Thomson. Together they studied<br />

thermodynamics and the expansion of gases. They<br />

learned how gases react under different conditions.<br />

Their law, named the Joule-Thomson effect, explains<br />

that compressed gases cool when they are allowed to<br />

expand under the right conditions. Their work later led<br />

to the invention of refrigeration.<br />

James Joule died on October 11, 1889. The international<br />

unit of energy is called the Joule in his honor.


Page 2 of 2<br />

Reading reflection<br />

1. Why do you think that Joule’s father built him a science lab when he was young?<br />

2. What evidence is there that Joule had an exceptional education?<br />

3. Why was Joule so interested in electromagnets?<br />

4. Why would you consider Joule’s early experiments with electric motors important even though he did not<br />

achieve his goal?<br />

5. Explain Joule’s Law in your own words.<br />

6. Describe something Joule believed that contradicted the beliefs of his peers.<br />

7. Describe the experiment that Joule tried to conduct on his honeymoon.<br />

8. Name one thing that we use today that was invented as a result of his research.<br />

9. What unit of measurement is named after him?<br />

10. Research: Find out more information about one of Joule’s more well-known experiments, and share your<br />

findings with the class. Try to find a picture of some of the apparatus that he used in his experiments.<br />

Suggested topics: galvanometer, heat energy, kinetic energy, mechanical work, conservation of energy,<br />

Kelvin scale of temperature, thermodynamics, Joule-Thomson Effect, electric welding, electromagnets,<br />

resistance in wires.<br />

7.2


Name: Date:<br />

7.3 Efficiency<br />

In a perfect machine, the work input would equal the work output. However, there aren’t any perfect machines in<br />

our everyday world. Bicycles, washing machines, and even pencil sharpeners lose some input work to friction.<br />

Efficiency is the ratio of work output to work input. It is expressed as a percent. A perfect machine would have an<br />

efficiency of 100 percent.<br />

An engineer designs a new can opener. For every twenty joules of work input, the can opener produces ten joules<br />

of work output. The engineer tries different designs and finds that her improved version produces thirteen joules<br />

of work output for the same amount of work input. How much more efficient is the new version?<br />

Efficiency of the first design Efficiency of the second design<br />

work output<br />

Efficiency =<br />

work input<br />

10 joules<br />

=<br />

20 joules<br />

= 50%<br />

The second design is 15% more efficient than the first.<br />

work output<br />

Efficiency =<br />

work input<br />

13 joules<br />

=<br />

20 joules<br />

= 65%<br />

1. A cell phone charger uses 4.83 joules per second when plugged into an outlet, but only 1.31 joules per<br />

second actually goes into the cell phone battery. The remaining joules are lost as heat. That’s why the battery<br />

feels warm after it has been charging for a while. How efficient is the charger?<br />

2. A professional cyclist rides a bicycle that is 92 percent efficient. For every 100 joules of energy he exerts as<br />

input work on the pedals, how many joules of output work are used to move the bicycle?<br />

3. An automobile engine is 15 percent efficient. How many joules of input work are required to produce<br />

15,000 joules of output work to move the car?<br />

4. It takes 56.5 kilojoules of energy to raise the temperature of 150 milliliters of water from 5 °C to 95 °C. If<br />

you use an electric water heater that is 60% efficient, how many kilojoules of electrical energy will the<br />

heater actually use by the time the water reaches its final temperature?<br />

5. A power station burns 75 kilograms of coal per second. Each kg of coal contains 27 million joules of energy.<br />

a. What is the total power of this power station in watts? (watts = joules/second)<br />

b. The power station’s output is 800 million watts. How efficient is this power station?<br />

6. A machine requires 2,000 joules to raise a 20. kilogram block a distance of 6.0 meters. How efficient is the<br />

machine? (Hint: Work done against gravity = mass × acceleration due to gravity × height.)<br />

7.3


Name: Date:<br />

7.3 Power<br />

In science, work is defined as the force needed to move an object a certain distance. The amount of work done<br />

per unit of time is called power.<br />

Suppose you and a friend are helping a neighbor to reshingle the roof of his home. You each carry 10 bundles of<br />

shingles weighing 300 newtons apiece up to the roof which is 7 meters from the ground. You are able to carry the<br />

shingles to the roof in 10 minutes, but your friend needs 20 minutes.<br />

Both of you did the same amount of work (force × distance) but you did the work in a shorter time.<br />

However, you had more power than your friend.<br />

Let’s do the math to see how this is possible.<br />

Step one: Convert minutes to seconds.<br />

Step two: Find power.<br />

W = F× d<br />

W = 10 bundles of shingles (300 N/bundle) × 7 m = 21,000 joules<br />

Work (joules)<br />

Power (watts) = Time (seconds)<br />

60 seconds<br />

10 minutes × = 600 seconds (You)<br />

minute<br />

60 seconds<br />

20 minutes × = 1,200 seconds (Friend)<br />

minute<br />

21,000 joules<br />

= 35 watts (You)<br />

600 seconds<br />

21,000 joules<br />

=<br />

17.5 watts (Friend)<br />

1,200 seconds<br />

As you can see, more power is produced when the same amount of work is done in a shorter time period. You<br />

have probably heard the word watt used to describe a light bulb. Is it now clear to you why a 100-watt bulb is<br />

more powerful than a 40-watt bulb?<br />

7.3


Page 2 of 2<br />

1. A motor does 5,000. joules of work in 20. seconds. What is the power of the motor?<br />

2. A machine does 1,500 joules of work in 30. seconds. What is the power of this machine?<br />

3. A hair dryer uses 72,000 joules of energy in 60. seconds. What is the power of this hair dryer?<br />

4. A toaster oven uses 67,500 joules of energy in 45 seconds to toast a piece of bread. What is the power of the<br />

oven?<br />

5. A horse moves a sleigh 1.00 kilometer by applying a horizontal 2,000.-newton force on its harness for<br />

45.0 minutes. What is the power of the horse? (Hint: Convert time to seconds.)<br />

6. A wagon is pulled at a speed of 0.40 m/s by a horse exerting an 1,800-newton horizontal force. What is the<br />

power of this horse?<br />

7. Suppose a force of 100. newtons is used to push an object a distance of 5.0 meters in 15 seconds. Find the<br />

work done and the power for this situation.<br />

8. Emily’s vacuum cleaner has a power rating of 200. watts. If the vacuum cleaner does 360,000 joules of<br />

work, how long did Emily spend vacuuming?<br />

9. Nicholas spends 20.0 minutes ironing shirts with his 1,800-watt iron. How many joules of energy were used<br />

by the iron? (Hint: convert time to seconds).<br />

10. It take a clothes dryer 45 minutes to dry a load of towels. If the dryer uses 6,750,000 joules of energy to dry<br />

the towels, what is the power rating of the machine?<br />

11. A 1000-watt microwave oven takes 90 seconds to heat a bowl of soup. How many joules of energy does it<br />

use?<br />

12. A force of 100. newtons is used to move an object a distance of 15 meters with a power of 25 watts. Find the<br />

work done and the time it takes to do the work.<br />

13. If a small machine does 2,500 joules of work on an object to move it a distance of 100. meters in<br />

10. seconds, what is the force needed to do the work? What is the power of the machine doing the work?<br />

14. A machine uses a force of 200 newtons to do 20,000 joules of work in 20 seconds. Find the distance the<br />

object moved and the power of the machine. (Hint: A joule is the same as a Newton-meter.)<br />

15. A machine that uses 200. watts of power moves an object a distance of 15 meters in 25 seconds. Find the<br />

force needed and the work done by this machine.<br />

7.3


Name: Date:<br />

7.3 Power in Flowing Energy<br />

Power is the rate of doing work. You do work if you lift a heavy box up a flight of stairs. You do the same<br />

amount of work whether you lift the box slowly or quickly. But your power is greater if you do the work in a<br />

shorter amount of time.<br />

Power can also be used to describe the rate at which energy is converted from one form into another. A light bulb<br />

converts electrical energy into heat (thermal energy) and light (radiant energy). The power of a light bulb is the<br />

rate at which the electrical energy is converted into these other forms.<br />

To calculate the power of a person, machine, or other device, you must know the work done or energy converted<br />

and the time. Work can be calculated using the following formula:<br />

Both work and energy are measured in joules. A joule is actually another name for a newton·meter. If you push<br />

an object along the floor with a force of 1 newton for a distance of 1 meter, you have done 1 joule of work. A<br />

motor could be used to do this same task by converting 1 joule of electrical energy into mechanical energy.<br />

Power is calculated by dividing the work or energy by the time. Power is measured in watts. One watt is equal to<br />

one joule of work or energy per second. In one second, a 60-watt light bulb converts 60 joules of electrical energy<br />

into heat and light. Power can also be measured in horsepower. One horsepower is equal to 746 watts.<br />

A cat who cat weighs 40 newtons climbs 15 meters up a tree in 10 seconds. Calculate the work done by the cat<br />

and the cat’s power.<br />

Looking for<br />

Solution<br />

The work and power of the cat.<br />

Given<br />

The force is 40 N.<br />

The distance is 15 m.<br />

The time is 10 s.<br />

Relationships<br />

Work = Force × distance<br />

Power = Work/time<br />

Work (joules) = Force (newtons) × distance (meters)<br />

W = F× d<br />

Work or Energy (joules)<br />

Power (watts) =<br />

Time (s)<br />

P = W / t<br />

Work = 40 N × 15 m = 600 J<br />

600 J<br />

Power = 60 W<br />

10 s =<br />

The work done by the cat is 600 joules.<br />

The power of the cat is 60 watts.<br />

In units of horsepower, the cat’s power is<br />

(60 watts)(1 hp / 746 watts) = 0.12 horsepower.<br />

7.3


Page 2 of 2<br />

1. Complete the table below:<br />

Force (N) Distance (m) Time (sec) Work (J) Power (W)<br />

100 2 5<br />

100 2 10<br />

100 4 10<br />

100 25 500<br />

20 20 1000<br />

30 10 60<br />

9 20 60<br />

3 75 5<br />

2. Oliver weighs 600. newtons. He climbs a flight of stairs that is 3.0 meters tall in 4.0 seconds.<br />

a. How much work did he do?<br />

b. What was Oliver’s power in watts?<br />

3. An elevator weighing 6,000. newtons moves up a distance of 10.0 meters in 30.0 seconds.<br />

a. How much work did the elevator’s motor do?<br />

b. What was the power of the elevator’s motor in watt and in horsepower?<br />

4. After a large snowstorm, you shovel 2,500. kilograms of snow off of your sidewalk in half an hour. You lift<br />

the shovel to an average height of 1.5 meters while you are piling the snow in your yard.<br />

a. How much work did you do? Hint: The force is the weight of the snow.<br />

b. What was your power in watts? Hint: You must always convert time to seconds when calculating power.<br />

5. A television converts 12,000 joules of electrical energy into light and sound every minute. What is the power<br />

of the television?<br />

6. The power of a typical adult’s body over the course of a day is 100. watts. This means that 100. joules of<br />

energy from food are needed each second.<br />

a. An average apple contains 500,000 joules of energy. For how many seconds would an apple power a<br />

person?<br />

b. How many joules are needed each day?<br />

c. How many apples would a person need to eat to get enough energy for one day?<br />

7. A mass of 1,000. kilograms of water drops 10.0 meters down a waterfall every second.<br />

a. How much potential energy is converted into kinetic energy every second?<br />

b. What is the power of the waterfall in watts and in horsepower<br />

8. An alkaline AA battery stores approximately 12,000 joules of energy. A small flashlight uses two AA<br />

batteries and will produce light for 2.0 hours. What is the power of the flashlight bulb? Assume all of the<br />

energy in the batteries is used.<br />

7.3


Name: Date:<br />

7.3 Efficiency and Energy<br />

Efficiency describes how well energy is converted from one form into another. A process is 100% efficient if no<br />

energy is “lost” due to friction, to create sound, or for other reasons. In reality, no process is 100% efficient.<br />

Efficiency is calculated by dividing the output energy by the input energy. If<br />

you multiply the result by 100, you will get efficiency as a percentage. For<br />

example, if the answer you get is 0.50, you can multiply by 100 and write<br />

your answer as 50%.<br />

You drop a 2-kilogram box from a height of 3 meters. Its speed is 7 m/s when it hits the ground. How efficiently<br />

did the potential energy turn into kinetic energy?<br />

Looking for<br />

You are asked to find the efficiency.<br />

Given<br />

The mass is 2 kilograms, the height is 3 meters,<br />

and the landing speed is 7 m/s.<br />

Relationships<br />

Kinetic energy = 1/2mv 2<br />

Potential energy = mgh<br />

Efficiency = (output energy)/(input energy)<br />

Solution<br />

The input energy is the potential energy, and the<br />

output energy is the kinetic energy.<br />

The efficiency is 0.83 or 83% (0.83 × 100).<br />

1. Engineers who design battery-operated devices such as cell phones and MP3 players try to make them as<br />

efficient as possible. An engineer tests a cell phone and finds that the batteries supply 10,000 J of energy to<br />

make 5500 J of output energy in the form of sound and light for the screen. How efficient is the phone?<br />

2. What’s the efficiency of a car that uses 400,000 J of energy from gasoline to make 48,000 J of kinetic<br />

energy?<br />

3. A 1000.-kilogram roller coaster goes down a hill that is 90. meters tall. Its speed at the bottom is 40. m/s.<br />

a. What is the efficiency of the roller coaster? Assume it starts from rest at the top of the hill.<br />

b. What do you think happens to the “lost” energy?<br />

c. Use the concepts of energy and efficiency to explain why the first hill on a roller coaster is the tallest.<br />

4. You see an advertisement for a new free fall ride at an amusement park. The ad says the ride is 50. meters tall<br />

and reaches a speed of 28 m/s at the bottom. How efficient is the ride? Hint: You can use any mass you wish<br />

because it cancels out.<br />

5. Imagine that you are working as a roller coaster designer. You want to build a record-breaking coaster that<br />

goes 70.0 m/s at the bottom of the first hill. You estimate that the efficiency of the tracks and cars you are<br />

using is 90.0%. How high must the first hill be?<br />

E<br />

E<br />

P<br />

K<br />

= =<br />

= =<br />

7.3<br />

Output energy (J)<br />

Efficiency = Input energy (J)<br />

2<br />

(2 kg)(9.8 m/s )(3 m) 58.8 J<br />

2<br />

(1/ 2)(2 kg)(7 m/s) 49 J<br />

Efficiency = (49 J)/(58.8 J) =<br />

0.83 or 83%


Name: Date:<br />

8.2 Measuring Temperature<br />

How do you find the temperature of a substance?<br />

There are many different kinds of thermometers<br />

used to measure temperature. Can you think of<br />

some you find at home? In your classroom you<br />

will use a glass immersion thermometer to find<br />

the temperature of a liquid. The thermometer<br />

contains alcohol with a red dye in it so you can<br />

see the alcohol level inside the thermometer.<br />

The alcohol level changes depending on the<br />

surrounding temperature. You will practice<br />

reading the scale on the thermometer and<br />

report your readings in degrees Celsius.<br />

Safety: Glass thermometers are breakable. Handle them carefully. Overheating the thermometer can cause the<br />

alcohol to separate and give incorrect readings. Glass thermometers should be stored horizontally or vertically<br />

(never upside down) to prevent alcohol from separating.<br />

Reading the temperature scale correctly<br />

Look at the picture at right. See the close-up of the line inside the thermometer on the<br />

scale. The tens scale numbers are given. The ones scale appears as lines. Each small line<br />

equals 1 degree Celsius. Practice reading the scale from the bottom to the top. One small<br />

line above 20 °C is read as 21 °C. When the level of the alcohol is between two small<br />

lines on the scale, report the number to the nearest 0.5 °C.<br />

Stop and think<br />

a. What number does the large line between 20 °C and 10 °C equal? Figure out by<br />

counting the number of small lines between 20 °C and 10 °C.<br />

b. Give the temperature of the thermometer in the picture above.<br />

Materials<br />

• Alcohol immersion thermometer<br />

• Beakers<br />

• Water at different temperatures<br />

• Ice<br />

c. Practice rounding the following temperature values to the nearest 0.5 °C:<br />

23.1 °C, 29.8 °C, 30.0 °C, 31.6 °C, 31.4 °C.<br />

d. Water at 0 °C and 100 °C has different properties. Describe what water looks<br />

like at these temperatures.<br />

e. What will happen to the level of the alcohol if you hold the thermometer by the<br />

bulb?


Page 2 of 2<br />

Reading the temperature of water in a beaker<br />

An immersion thermometer must be placed in liquid up to the solid line on the thermometer (at least 2<br />

and one half inches of liquid). Wait about 3 minutes for the temperature of the thermometer to equal the<br />

temperature of the liquid. Record the temperature to the nearest 0.5 °C when the level stops moving.<br />

1. Place the thermometer in the beaker. Check to make sure that the water level is above the solid line on the<br />

thermometer.<br />

2. Wait until the alcohol level stops moving (about three minutes). Record the temperature to the nearest<br />

0.5 °C.<br />

Reading the temperature of warm water in a beaker<br />

A warm liquid will cool to room temperature. For a warm liquid, record the warmest temperature you observe<br />

before the temperature begins to decrease.<br />

1. Repeat the procedure above with a beaker of warm (not boiling) water.<br />

2. Take temperature readings every 30 seconds. Record the warmest temperature you observe.<br />

Reading the temperature of ice water in a beaker<br />

When a large amount of ice is added to water, the temperature of the water will drop<br />

until the ice and water are the same temperature. After the ice has melted, the cold<br />

water will warm to room temperature.<br />

1. Repeat the procedure above with a beaker of ice and water.<br />

2. Take temperature readings every 30 seconds. Record the coldest temperature<br />

you observe.


Name: Date:<br />

8.2 Temperature Scales<br />

The Fahrenheit and Celsius temperature scales are commonly used scales for reporting temperature values.<br />

Scientists use the Celsius scale almost exclusively, as do many countries of the world. The United States relies on<br />

the Fahrenheit scale for reporting temperature information. You can convert information reported in degrees<br />

Celsius to degrees Fahrenheit or vice versa using conversion formulas.<br />

Fahrenheit (°F) to Celsius (°C) conversion formula:<br />

Celsius (°C) to Fahrenheit (°F) conversion formula:<br />

• What is the Celsius value for 65° Fahrenheit?<br />

Solution:<br />

• 200 °C is the same temperature as what value on the Fahrenheit scale?<br />

Solution:<br />

o<br />

⎛5⎞ C = ⎜ ⎟ ° F −32<br />

⎝9⎠ ( )<br />

⎛9⎞ ° F = ⎜ ×° C⎟+ 32<br />

⎝5⎠ ⎛5⎞ ° C = ⎜ ⎟(<br />

65° F−32) ⎝9⎠ ⎛5⎞ ° C = ⎜ ⎟(<br />

33 ) = ( 5 × 33 ) ÷ 9<br />

⎝9⎠ ° C = 165 ÷9<br />

° C = 18.3<br />

⎛9⎞ ° F = ⎜ ⎟(<br />

200 ° C) + 32<br />

⎝5⎠ ° F = [ ( 9 × 200 °C ) ÷ 5] + 32<br />

° F = [1800 ÷ 5] + 32<br />

° F = 360 + 32<br />

°<br />

F = 392<br />

8.2


Page 2 of 4<br />

1. For each of the problems below, show your calculations. Follow<br />

the steps from the examples on the previous page.<br />

a. What is the Celsius value for 212 °F?<br />

b. What is the Celsius value for 98.6 °F?<br />

c. What is the Celsius value for 40 °F?<br />

d. What is the Celsius value for 10 °F?<br />

e. What is the Fahrenheit value for 0 °C?<br />

f. What is the Fahrenheit value for 25 °C?<br />

g. What is the Fahrenheit value for 75 °C?<br />

2. The weatherman reports that today will reach a high of 45 °F. Your friend from Sweden asks what the<br />

temperature will be in degrees Celsius. What value would you report to your friend?<br />

3. Your parents order an oven from England. The temperature dial on the new oven is calibrated in degrees<br />

Celsius. If you need to bake a cake at 350 °F in the new oven, at what temperature should you set the dial?<br />

4. A German automobile’s engine temperature gauge reads in Celsius, not Fahrenheit. The engine temperature<br />

should not rise above about 225 °F. What is the corresponding Celsius temperature on this car’s gauge?<br />

5. Your grandmother in Ireland sends you her favorite cookie recipe. Her instructions say to bake the cookies at<br />

190 °C. To what Fahrenheit temperature would you set the oven to bake the cookies?<br />

6. A scientist wishes to generate a chemical reaction in his laboratory. The temperature values in his laboratory<br />

manual are given in degrees Celsius. However, his lab thermometers are calibrated in degrees Fahrenheit. If<br />

he needs to heat his reactants to 232 °C, what temperature will he need to monitor on his lab thermometers?<br />

7. You call a friend in Europe during the winter holidays and say that the temperature in Boston is 15 degrees.<br />

He replies that you must enjoy the warm weather. Explain his comment using your knowledge of the<br />

Fahrenheit and Celsius scales. To help you get started, fill in this table. What is 15 °F on the Celsius scale?<br />

What is 15°C on the Fahrenheit scale?<br />

°F f °C<br />

15 °F =<br />

= 15°C<br />

8. Challenge questions:<br />

a. A gas has a boiling point of –175 °C. At what Fahrenheit temperature would this gas boil?<br />

b. A chemist notices some silvery liquid on the floor in her lab. She wonders if someone accidentally<br />

broke a mercury thermometer, but did not thoroughly clean up the mess. She decides to find out if the<br />

silver stuff is really mercury. From her tests with the substance, she finds out that the melting point for<br />

the liquid is 35°F. A reference <strong>book</strong> says that the melting point for mercury is –38.87 °C. Is this<br />

substance mercury? Explain your answer and show all relevant calculations.<br />

8.2


Page 3 of 4<br />

Extension: the Kelvin temperature scale<br />

8.2<br />

For some scientific applications, a third temperature scale is used: the Kelvin scale. The Kelvin scale is<br />

calibrated so that raising the temperature one degree Kelvin raises it by the same amount as one degree Celsius.<br />

The difference between the scales is that 0 °C is the freezing point of water, while 0 K is much, much colder. On<br />

the Kelvin scale, 0K (degree symbols are not used for Kelvin values) represents absolute zero. Absolute zero is<br />

the temperature when the average kinetic energy of a perfect gas is zero—the molecules display no energy of<br />

motion. Absolute zero is equal to –273 °C, or –459 °F. When scientists are conducting research, they often obtain<br />

or report their temperature values in Celsius, and other scientists must convert these values into Kelvin for their<br />

own use, or vice versa. To convert Celsius values to their Kelvin equivalents, use the formula:<br />

K = °C + 273<br />

Water boils at a temperature of 100 °C. What would be the corresponding temperature for the Kelvin scale?<br />

K = °C + 273<br />

K = 100 °C + 273<br />

K = 373<br />

To convert Kelvin values to Celsius, you perform the opposite operation; subtract 273 from the Kelvin value to<br />

find the Celsius equivalent.<br />

A substance has a melting point of 625 K. At what Celsius temperature would this substance melt?<br />

°C = K + 273<br />

°C = 625 K −273<br />

°C = 352<br />

Although we rarely need to convert between Kelvin and Fahrenheit, use the following formulas to do so:<br />

⎛9⎞ °F = ⎜ × K ⎟−460<br />

⎝5⎠ 5<br />

K = (°F +<br />

460)<br />

9


Page 4 of 4<br />

1. Surface temperatures on the planet Mars range from –89 °C to –31 °C. Express this temperature<br />

range in Kelvin.<br />

2. The average surface temperature on Jupiter is about 165K. Express this temperature in degrees Celsius.<br />

3. The average surface temperature on Saturn is 134K. Express this temperature in degrees Celsius.<br />

4. The average surface temperature on the dwarf planet Pluto is 50K. Express this temperature in degrees<br />

Celsius.<br />

5. The Sun has several regions. The apparent surface that we can see from a distance is called the photosphere.<br />

Temperatures of the photosphere range from 5,000 °C to 8,000 °C. Express this temperature range in<br />

Kelvin.<br />

6. The chromosphere is a hot layer of plasma just above the photosphere. Chromosphere temperatures can<br />

reach 10,000 °C. Express this temperature in Kelvin.<br />

7. The outermost layer of the Sun’s atmosphere is called the corona. Its temperatures can reach over<br />

1,000,000 °C. Express this temperature in Kelvin.<br />

8. Nuclear fusion takes place in the center, or core, of the Sun. Temperatures there can reach 15,000,000 °C.<br />

Express this temperature in Kelvin.<br />

9. Challenge! Surface temperatures on Mercury can reach 660 °F. Express this temperature in Kelvin.<br />

10. Challenge! Surface temperatures on Venus, the hottest planet in our solar system, can reach 755K. Express<br />

this temperature in degrees Fahrenheit.<br />

8.2


Name: Date:<br />

8.3 Reading a Heating/Cooling Curve<br />

A heating curve shows how the temperature of a substance changes as<br />

heat is added at a constant rate. The heating curve at right shows what<br />

happens when heat is added at a constant rate to a beaker of ice. The flat<br />

spot on the graph, at zero degrees, shows that although heat was being<br />

added, the temperature did not rise while the solid ice was changing to<br />

liquid water. The heat energy was used to break the intermolecular<br />

forces between water molecules. Once all the ice changed to water, the<br />

temperature began to rise again. In this skill sheet, you will practice<br />

reading heating and cooling curves.<br />

The heating curve at right shows the temperature change in a<br />

sample of iron as heat is added at a constant rate. The sample<br />

starts out as a solid and ends as a gas.<br />

• Describe the phase change that occurred between points B<br />

and C on the graph.<br />

Solution:<br />

Between points B and C, the sample changed from solid to<br />

liquid.<br />

1. In the heating curve for iron, describe the phase change that<br />

occurred between points D and E on the graph.<br />

2. Explain why the temperature stayed constant between points D and E.<br />

3. What is the melting temperature of iron?<br />

4. What is the freezing temperature of iron? How do you know?<br />

5. What is the boiling temperature of iron?<br />

6. Compare the boiling temperatures of iron and water (water boils at 100°C). Which substance has stronger<br />

intermolecular forces? How do you know?<br />

8.3


Page 2 of 2<br />

The graph below shows a cooling curve for stearic acid. Stearic acid is a waxy solid at room<br />

temperature. It is derived from animal and vegetable fats and oils. It is used as an ingredient in soap,<br />

8.3<br />

candles, and cosmetics. In this activity, a sample of stearic acid was placed in a heat-resistant test tube<br />

and heated to 95 °C, at which point the stearic acid was completely liquefied. The test tube was placed in a<br />

beaker of ice water, and the temperature monitored until it reached 40 °C. Answer the following questions about<br />

the cooling curve:<br />

7. Between which two points on the graph did freezing occur?<br />

8. What is the freezing temperature of stearic acid? What is its melting temperature?<br />

9. Compare the melting temperature of stearic acid with the melting temperature of water. Which substance has<br />

stronger intermolecular forces? How do you know?<br />

10. Can a substance be cooled to a temperature below its freezing point? Use evidence from any of the graphs in<br />

this skill sheet to defend your answer.


Name: Date:<br />

9.1 Specific Heat<br />

Specific heat is the amount of thermal energy needed to raise the temperature of 1 gram of a substance 1 °C.<br />

The higher the specific heat, the more energy is required to cause a change in temperature. Substances with<br />

higher specific heats must lose more thermal energy to lower their temperature than substances with a low<br />

specific heat. Some sample specific heat values are presented in the table below:<br />

Material Specific Heat (J/kg °C)<br />

water (pure) 4,184<br />

aluminum 897<br />

silver 235<br />

oil 1,900<br />

concrete 880<br />

gold 129<br />

wood 1,700<br />

Water has the highest specific heat of the listed types of matter. This means that water is slower to heat but is<br />

also slower to lose heat.<br />

Using the table above, solve the following heat problems.<br />

1. If 100 joules of energy were applied to all of the substances listed in the table at the same time, which would<br />

have the greatest temperature change? Explain your answer.<br />

2. Which of the substances listed in the table would you choose as the best thermal insulator? A thermal<br />

insulator is a substance that requires a lot of heat energy to change its temperature. Explain your answer.<br />

3. Which substance—wood or silver—is the better thermal conductor? A thermal conductor is a material that<br />

requires very little heat energy to change its temperature. Explain your answer.<br />

4. Which has more thermal energy, 1 kg of aluminum at 20 °C or 1 kg of gold at 20 °C?<br />

5. How much heat in joules would you need to raise the temperature of 1 kg of water by 5 °C?<br />

6. How does the thermal energy of a large container of water compare to a small container of water at the same<br />

temperature?<br />

9.1


Name: Date:<br />

9.1 Using the Heat Equation<br />

You can solve real-world heat and temperature problems using the following equation:<br />

Below is a table that provides the specific heat of six everyday materials.<br />

Material Specific Heat (J/kg °C) Material Specific Heat (J/kg °C)<br />

water (pure) 4,184 concrete 880<br />

aluminum 897 gold 129<br />

silver 235 wood 1,700<br />

• How much heat does it take to raise the temperature of 10 kg of water by 10 °C?<br />

Solution:<br />

Find the specific heat of water from the table above: 4,184 J/kg °C. Plug the values into the equation.<br />

Thermal Energy (J) = 10 kg × 10 °C ×<br />

4,184 J/kg • °C<br />

= 418,400 joules<br />

Use the specific heat table to answer the following questions. Don’t forget to show your work.<br />

1. How much heat does it take to raise the temperature of 0.10 kg of gold by 25 °C?<br />

2. How much heat does it take to raise the temperature of 0.10 kg of silver by 25 °C?<br />

3. How much heat does it take to raise the temperature of 0.10 kg of aluminum by 25 °C?<br />

4. Which one of the three materials above would cool down fastest after the heat was applied? Explain.<br />

5. A coffee maker heats 2 kg of water from 15 °C to 100 °C. How much thermal energy was required?<br />

6. The Sun warms a 100-kg slab of concrete from 20 °C to 25 °C. How much thermal energy did it absorb?<br />

7. 5,000 joules of thermal energy were applied to 1-kg aluminum bar. What was the temperature increase?<br />

8. In the 1920’s, many American homes did not have hot running water from the tap. Bath water was heated on<br />

the stove and poured into a basin. How much thermal energy would it take to heat 30 kg of water from 15 °C<br />

to a comfortable bath temperature of 50 °C?<br />

9.1


Name: Date:<br />

9.2 Heat Transfer<br />

Thermal energy flows from higher temperature to lower temperature. This process is called heat transfer. Heat<br />

transfer can occur three different ways: heat conduction, convection, and thermal radiation. Using section 9.2 of<br />

your student text as a guide, define each method of heat transfer.<br />

Heat conduction:<br />

______________________________________________________________________________________<br />

______________________________________________________________________________________<br />

______________________________________________________________________________________<br />

Convection:<br />

______________________________________________________________________________________<br />

______________________________________________________________________________________<br />

______________________________________________________________________________________<br />

Thermal radiation:<br />

______________________________________________________________________________________<br />

______________________________________________________________________________________<br />

______________________________________________________________________________________<br />

Read each scenario below. Then explain which type of heat transfer is described. Some scenarios involve more<br />

than one type of heat transfer.<br />

1. Mia places some frozen shrimp in a strainer and pours hot water over it so the shrimp will thaw faster.<br />

2. On a hot summer day, Juan can walk comfortably in bare feet on the concrete sidewalk, but finds that the<br />

asphalt road will burn the soles of his feet.<br />

3. A hawk soars upward, riding a thermal.<br />

4. An electric space heater warms an office.<br />

5. A mother duck sits on her eggs to keep them warm.<br />

6. A car parked in the sun reaches an interior temperature of 140°F in 40 minutes.<br />

7. Nick always adds milk to his coffee so that it’s not too hot to drink.<br />

8. A strong sea breeze makes a regatta (sailboat race) more exciting.<br />

9. Warm water piped under a marble floor makes the floor feel warm to bare feet.<br />

10. The warm water pipes under the marble floor heat the entire room.<br />

9.2


Name: Date:<br />

10.1 Measuring Mass with a Triple Beam Balance<br />

How do you find the mass of an object?<br />

Why can’t you use a bathroom scale to<br />

measure the mass of a paperclip? You could if<br />

you were finding the mass of a lot of them at<br />

one time! To find the mass of objects less than<br />

a kilogram you will need to use the triple beam<br />

balance.<br />

Parts of the triple beam balance<br />

Setting up and zeroing the balance<br />

The triple beam balance works like a see-saw. When the mass of your object is perfectly balanced by the counter<br />

masses on the beam, the pointer will rest at 0. Add up the readings on the three beams to find the mass of your<br />

object. The unit of measure for this triple beam balance is grams.<br />

1. Place the balance on a level surface.<br />

2. Clean any objects or dust off the pan.<br />

Materials<br />

• Triple beam balance<br />

• Small objects<br />

• Mass set (optional)<br />

• Beaker<br />

3. Move all counter masses to 0. The pointer should rest at 0. Use the adjustment screw to adjust the pointer to<br />

0, if necessary. When the pointer rests at 0 with no objects on the pan, the balance is said to be zeroed.


Page 2 of 3<br />

Finding a known mass<br />

You can check that the triple beam balance is working correctly by using a mass set. Your teacher will<br />

provide the correct mass value for these objects.<br />

1. After zeroing the balance, place an object with a known mass on the pan.<br />

2. Move the counter masses to the right one at a time from largest to smallest. When the pointer is resting at 0<br />

the numbers under the three counter masses should add up to the known mass.<br />

3. If the pointer is above or below 0, recheck the balance set up. Recheck the position of the counter masses.<br />

Counter masses must be properly seated in a groove. Check with your teacher to make sure you are getting<br />

the correct mass before finding the mass an unknown object.<br />

Finding the mass of an unknown object<br />

1. After zeroing the balance, place an object with an unknown mass on the pan. Do not place hot objects or<br />

chemicals directly on the pan<br />

2. Move the largest counter mass first. Place it in the first notch after zero. Wait until the pointer stops moving.<br />

If the pointer is above 0, move the counter mass to the next notch. Continue to move the counter mass to the<br />

right, one notch at a time until the pointer is slightly above 0. Go to step 3. If the pointer is below 0, move the<br />

counter mass back one notch. When the pointer rests at 0, you do not need to move any more counter<br />

masses.<br />

3. Move the next largest counter mass from 0 to the first notch. Watch to see where the pointer rests. If it rests<br />

above 0, move the counter mass to the next notch. Repeat until the point rests at 0, or slightly above. If the<br />

pointer is slightly above 0, go to step 4.<br />

4. Move the smallest counter mass from 0 to the position on the beam where the pointer rests at 0.<br />

5. Add the masses from the three beams to get the mass of the unknown object. You should be able to record a<br />

number for the hundreds place, the tens place, the ones place, and the tenths place and the hundredths place.<br />

The hundredths place can be read to 0.00 or 0.05. You may have zeros in your answer.


Page 3 of 3<br />

Reading the balance correctly<br />

Look at the picture above. To find the mass of the object, locate the counter mass on each beam. Read the<br />

numbers directly below each counter mass. You can read the smallest mass to 0.05 grams. Write down the three<br />

numbers. Add them together. Report your answer in grams. Does your answer agree with others? If not, check<br />

your mass values from each beam to find your mistake.<br />

Finding the mass of an object in a container<br />

To measure the mass of a liquid or powder you will need an empty container on the pan to hold the sample. You<br />

must find the mass of the empty container first. After you place the object in the container and find the total mass,<br />

you can subtract the container’s mass from the total to find the object’s mass.<br />

1. After zeroing the balance, place a beaker on the pan.<br />

2. Follow directions for finding the mass of an unknown object. Record the mass of the beaker.<br />

3. Place a small object in the beaker.<br />

4. Move the counter masses to the right, largest to smallest, to find the total mass.<br />

5. Subtract the beaker’s mass from the total mass. This is the mass of your object in grams.


Name: Date:<br />

10.1 Measuring Volume<br />

How do you find the volume of an irregular object?<br />

It’s easy to find the volume of a shoebox or a<br />

basketball. You just take a few measurements,<br />

plug the numbers into a math formula, and you<br />

have figured it out. But what if you want to find<br />

the volume of a bumpy rock, or an acorn, or a<br />

house key? There aren’t any simple math<br />

formulas to help you out. However, there’s an<br />

easy way to find the volume of an irregular<br />

object, as long the object is waterproof!<br />

Setting up the displacement tank<br />

Materials<br />

• Displacement tank<br />

• Water source<br />

• Disposable cup<br />

• Beaker<br />

• Graduated cylinder<br />

• Sponges or paper towel<br />

• Object to be measured<br />

Set the displacement tank on a level surface. Place a disposable cup<br />

under the tank’s spout. Carefully fill the tank until the water begins to<br />

drip out of the spout. When the water stops flowing, discard the water<br />

collected in the disposable cup. Set the cup aside and place a beaker<br />

under the spout.<br />

Stop and think<br />

a. What do you think will happen when you place an object into the<br />

tank?<br />

b. Which object would cause more water to come out of the spout, an acorn or a fist-sized rock?<br />

c. Why are we interested in how much water comes out of the spout?


Page 2 of 2<br />

d. Explain how the displacement tank measures volume.<br />

Measuring volume with the displacement tank<br />

1. Gently place a waterproof object into the displacement tank. It is important to avoid splashing the water or<br />

creating a wave that causes extra water to flow out of the spout. It may take a little practice to master this<br />

step.<br />

2. When the water stops flowing out of the spout, it can be poured from the beaker into a graduated cylinder for<br />

precise measurement. The volume of the water displaced is equal to the object’s volume.<br />

Note: Occasionally, when a small object is placed in the tank, no water will flow out. This happens because<br />

an air bubble has formed in the spout. Simply tap the spout with a pencil to release the air bubble.<br />

3. If you wish to measure the volume of another object, don’t forget to refill the tank with water first!


Name: Date:<br />

10.1 Calculating Volume<br />

How do you find the volume of a three dimensional shape?<br />

Volume is the amount of space an object takes<br />

up. If you know the dimensions of a solid object,<br />

you can find the object's volume. A two<br />

dimensional shape has length and width. A<br />

three dimensional object has length, width, and<br />

height. This investigation will give you practice<br />

finding volume for different solid objects.<br />

Calculating volume of a cube<br />

A cube is a geometric solid that has length, width and height. If you<br />

measure the sides of a cube, you will find that all the edges have the<br />

same measurement. The volume of a cube is found by multiplying the<br />

length times width times height. In the picture each side is<br />

4 centimeters so the problem looks like this:<br />

Example:<br />

V= l × w × h<br />

Volume = 4 centimeters × 4 centimeters × 4 centimeters = 64 centimeters 3<br />

Stop and think<br />

a. What are the units for volume in the example above?<br />

b. In the example above, if the edge of the cube is 4 inches, what will the volume be? Give the units.<br />

c. How is finding volume different from finding area?<br />

Materials<br />

• Pencil<br />

• Calculator<br />

d. If you had cubes with a length of 1 centimeter, how many would you need to build the cube in the picture above?<br />

Calculating volume of a rectangular prism<br />

Rectangular prisms are like cubes, except not all of the sides are<br />

equal. A shoebox is a rectangular prism. You can find the<br />

volume of a rectangular prism using the same formula given<br />

above (V= l × w × h.)<br />

Another way to say it is to multiply the area of the base times the<br />

height.<br />

1. Find the area of the base for the rectangular prism pictured<br />

above.<br />

2. Multiply the area of the base times the height. Record the volume of the rectangular prism.<br />

3. PRACTICE: Find the volume for a rectangular prism with a height 6 cm, length 5 cm, and width 3 cm. Be<br />

sure to include the units in all of your answers.


Page 2 of 3<br />

Calculating volume of a triangular prism<br />

Triangular prisms have three sides and two triangular<br />

bases. The volume of the triangular prism is found by<br />

multiplying the area of the base times the height. The<br />

base is a triangle.<br />

1. Find the area of the base by solving for the area of<br />

the triangle: B = 1 / 2 × l × w.<br />

2. Find the volume by multiplying the area of the base<br />

times the height of the prism:<br />

V= B × h. Record the volume of the triangular prism<br />

shown above.<br />

3. PRACTICE: Find the volume of the triangular prism with a height 10 cm, triangular base width 4 cm, and<br />

triangular base length 5 cm.<br />

Calculating volume of a cylinder<br />

A soup can is a cylinder. A cylinder has two circular bases and a<br />

round surface. The volume of the cylinder is found by multiplying<br />

the area of the base times the height. The base is a circle.<br />

1. Find the area of the base by solving for the area of a<br />

circle: A = π×r 2 .<br />

2. Find the volume by multiplying the area of the base times the<br />

height of the cylinder: V = A × h. Record the volume of the<br />

cylinder shown above.<br />

3. PRACTICE: Find the volume of the cylinder with height 8 cm<br />

and radius 4 cm.<br />

Calculating volume of a cone<br />

An ice cream cone really is a cone! A cone has height and a circular<br />

base. The volume of the cone is found by multiplying 1 / 3 times the<br />

area of the base times the height.<br />

1. Find the area of the base by solving for the area of a circle:<br />

A = π×r2 .<br />

2. Find the volume by multiplying 1 / 3 times the area of the base<br />

times the height:<br />

V = 1 / 3 × A × h. Record the volume of the cone shown above.<br />

3. PRACTICE: Find the volume of the cone with height 8 cm and<br />

radius 4 cm. Contrast your answer with the volume you found for the cylinder with the same dimensions.<br />

What is the difference in volume? Does this make sense?


Page 3 of 3<br />

Calculating the volume of a rectangular pyramid<br />

A pyramid looks like a cone. It has height and a rectangular base. The<br />

volume of the rectangular pyramid is found by multiplying 1 / 3 times<br />

the area of the base times the height.<br />

1. Find the area of the base by multiplying the length times the<br />

width: A = l × w.<br />

2. Find the volume by multiplying 1 / 3 times the area of the base<br />

times the height:<br />

V = 1 / 3 × A × h. Record the volume of the rectangular pyramid<br />

shown above.<br />

3. PRACTICE: Find the volume of a rectangular pyramid with<br />

height 10 cm and width 4 cm and length 5 cm.<br />

4. EXTRA CHALLENGE: If a rectangular pyramid had a height of 8 cm and a width of 4 cm, what length<br />

would it need to have to give the same volume as the cone in practice question 3 above?<br />

Calculating volume of a triangular pyramid<br />

A triangular pyramid is like a rectangular pyramid, but its base is<br />

a triangle. Find the area of the base first. Then calculate the<br />

volume by multiplying 1 / 3 times the area of the base times the<br />

height.<br />

1. Find the area of the base by solving for the area of a triangle:<br />

B = 1 / 2 × l × w.<br />

2. Find the volume by multiplying 1 / 3 times the area of the<br />

base times the height:<br />

V= 1 / 3 × A × h. Find the volume of the triangular pyramid<br />

shown above.<br />

3. PRACTICE: Find the volume of the triangular pyramid with<br />

height 10 cm and whose base has width 6 cm and length<br />

5cm.<br />

Calculating volume of a sphere<br />

To find the volume of a sphere, you only need to know one dimension<br />

about the sphere, its radius.<br />

1. Find the volume of a sphere: V = 4 / 3πr 3 . Find the volume for the<br />

sphere shown above.<br />

2. PRACTICE: Find the volume for a sphere with radius 2 cm.<br />

3. EXTRA CHALLENGE: Find the volume for a sphere with<br />

diameter 10 cm.


Name: Date:<br />

10.1 Density<br />

The density of a substance does not depend on its size or shape. As<br />

long as a substance is homogeneous, the density will be the same.<br />

This means that a steel nail has the same density as a cube of steel or a<br />

steel girder used to build a bridge.<br />

The formula for density is:<br />

density<br />

------------------mass<br />

volume<br />

One milliliter takes up the same amount of space as one cubic<br />

centimeter. Therefore, density can be expressed in units of g/mL or<br />

g/cm 3 . Liquid volumes are most commonly expressed in milliliters,<br />

while volumes of solids are usually expressed in cubic centimeters.<br />

Density can also be expressed in units of kilograms per cubic meter (kg/m 3 ).<br />

If you know the density of a substance and the volume of a sample, you can calculate the mass of the sample. To<br />

do this, rearrange the equation above to find mass: volume × density = mass<br />

If you know the density of a substance and the mass of a sample, you can find the volume of the sample. This<br />

time, you will rearrange the density equation to find volume:<br />

mass<br />

volume = ---------------density<br />

Example 1: What is the density of a block of aluminum with a volume of 30.0 cm3 and a mass of 81.0 grams?<br />

density<br />

81.0 g<br />

30.0 cm3 ----------------------<br />

2.70 g<br />

cm3 = = --------------<br />

Answer: The density of aluminum is 2.70 g/cm 3 .<br />

Example 2: What is the mass of an iron horseshoe with a volume of 89.0 cm3 ? The density of iron is 7.90 g/cm3 .<br />

mass 89.0cm<br />

Answer: The mass of the horseshoe is 703 grams.<br />

3 7.90 g<br />

cm3 = × ---------- = 703 grams<br />

Example 3: What is the volume of a 525-gram block of lead? The density of lead is 11.3 g/cm3 .<br />

volume<br />

525 g<br />

11.3 g<br />

cm3 ---------------------<br />

----------<br />

46.5 cm3 = =<br />

Answer: The volume of the block is 46.5 cm 3 .<br />

=<br />

10.1


Page 2 of 2<br />

Answer the following density questions. Report your answers using the correct number of significant<br />

digits.<br />

1. A solid rubber stopper has a mass of 33.0 grams and a volume of 30.0 cm 3 . What is the density of rubber?<br />

2. A chunk of paraffin (wax) has a mass of 50.4 grams and a volume of 57.9 cm 3 . What is its density?<br />

3. A marble statue has a mass of 6,200 grams and a volume of 2,296 cm 3 . What is the density of marble?<br />

4. The density of ice is 0.92 g/cm 3 . An ice sculptor orders a 1.0-m 3 block of ice. What is the mass of the block?<br />

Hint: 1 m 3 = 1,000,000 cm 3 . Give your answer in grams and kilograms.<br />

5. What is the mass of a pure platinum disk with a volume of 113 cm 3 ? The density of platinum is 21.4 g/cm 3 .<br />

Give your answer in grams and kilograms.<br />

6. The density of seawater is 1.025 g/mL. What is the mass of 1.000 liter of seawater in grams and in<br />

kilograms? (Hint: 1 liter = 1,000 mL)<br />

7. The density of cork is 0.24 g/cm 3 . What is the volume of a 288-gram piece of cork?<br />

8. The density of gold is 19.3 g/cm 3 . What is the volume of a 575-gram bar of pure gold?<br />

9. The density of mercury is 13.6 g/mL. What is the volume of a 155-gram sample of mercury?<br />

10. Recycling centers use density to help sort and identify different types of plastics so that they can be properly<br />

recycled. The table below shows common types of plastics, their recycling code, and density. Use the table<br />

to solve problems 10a–d.<br />

Plastic name Common uses Recycling code Density (g/cm 3 ) Density (kg/m 3 )<br />

PETE plastic soda bottles 1 1.38–1.39 1,380–1,390<br />

HDPE milk cartons 2 0.95–0.97 950–970<br />

PVC plumbing pipe 3 1.15–1.35 1,150–1,350<br />

LDPE trash can liners 4 0.92–0.94 920–940<br />

PP yogurt containers 5 0.90–0.91 900–910<br />

PS cd “jewel cases” 6 1.05–1.07 1,050–1,070<br />

a. A recycling center has a 0.125 m 3 box filled with one type of plastic. When empty, the box had a mass<br />

of 0.755 kilograms. The full box has a mass of 120.8 kilograms. What is the density of the plastic? What<br />

type of plastic is in the box?<br />

b. A truckload of plastic soda bottles was finely shredded at a recycling center. The plastic shreds were<br />

placed into 55-liter drums. What is the mass of the plastic shreds inside one of the drums?<br />

Hint: 55 liters = 55,000 milliliters = 55,000 cm 3 .<br />

c. A recycling center has 100. kilograms of shredded plastic yogurt containers. What volume is needed to<br />

hold this amount of shredded plastic? How many 10.-liter (10,000 mL) containers do they need to hold<br />

all of this plastic? Hint: 1 m 3 = 1,000,000 mL.<br />

d. A solid will float in a liquid if it is less dense than the liquid, and sink if it is more dense than the liquid.<br />

If the density of seawater is 1.025 g/mL, which types of plastics would definitely float in seawater?<br />

10.1


Name: Date:<br />

10.3 Pressure in Fluids<br />

Have you ever wondered how a 1,500-kilogram car is raised off the ground in a mechanic’s shop? A hydraulic<br />

lift does the trick. All hydraulic machines operate on the basis of Pascal’s principle, which states that the pressure<br />

applied to an incompressible fluid in a closed container is transmitted equally in all parts of the fluid.<br />

In the diagram above, a piston at the top of the small tube pushes down on the fluid. This input force generates a<br />

certain amount of pressure, which can be calculated using the formula:<br />

That pressure stays the same throughout the fluid, so it remains the same in the large cylinder. Since the large<br />

cylinder has more area, the output force generated by the large cylinder is greater. The output force exerted by the<br />

piston at the top of the large cylinder can be calculated using the formula:<br />

You can see that the small input force created a large output force. But there’s a price: The small piston must be<br />

pushed a greater distance than the large piston moves. Work output (output force × output distance) can never be<br />

greater than work input (input force × input distance).<br />

• A 50.-newton force is applied to a small piston with an area of 0.0025 m 2 . What pressure, in pascals, will be<br />

transmitted in the hydraulic system?<br />

Solution:<br />

• The area of the large cylinder’s piston in this hydraulic system is 2.5 m 2 . What is the output force?<br />

Solution:<br />

Pressure<br />

Pressure<br />

=<br />

Force<br />

--------------<br />

Area<br />

Force = Pressure × Area<br />

=<br />

Force<br />

--------------<br />

Area<br />

=<br />

50. N<br />

0.0025 m 2<br />

------------------------- = 20000 Pa<br />

Force Pressure × Area = 20000 Pa 2.5 m 2<br />

=<br />

× = 50000 N<br />

10.3


Page 2 of 2<br />

1. In a hydraulic system, a 100.-newton force is applied to a small piston with an area of 0.0020 m 2 .<br />

What pressure, in pascals, will be transmitted in the hydraulic system?<br />

2. The area of the large cylinder’s piston in this hydraulic system is 3.14 m 2 . What is the output force?<br />

3. An engineer wishes to design a hydraulic system that will transmit a pressure of 10,000 pascals using a force<br />

of 15 newtons. How large an area should the input piston have?<br />

4. This hydraulic system should produce an output force of 50,000 newtons. How large an area should the<br />

output piston have?<br />

5. Another engineer is running a series of experiments with hydraulic systems. If she doubles the area of the<br />

input piston, what happens to the amount of pressure transmitted by the system?<br />

6. If all other variables remain unchanged, what happens to the output force when the area of the input piston is<br />

doubled?<br />

7. If the small piston in the hydraulic system described in problems 1 and 2 is moved a distance of 2 meters,<br />

will the large piston also move 2 meters? Explain why or why not.<br />

8. A 540-newton woman can make dents in a hardwood floor wearing high-heeled shoes, yet if she wears<br />

snowshoes, she can step effortlessly over soft snow without sinking in. Explain why, using what you know<br />

about pressure, force, and area.<br />

10.3


Name: Date:<br />

10.3 Boyle’s Law<br />

The relationship between the volume of a gas<br />

and the pressure of a gas, at a constant<br />

temperature, is known as Boyle’s law. The<br />

equation for Boyle’s law is shown at right.<br />

Units for pressure include: atmospheres<br />

(atm), pascals (Pa), or kilopascals (kPa).<br />

Units for volume include: cubic centimeters<br />

(cm 3 ), cubic meters (m 3 ), or liters.<br />

A kit used to fix flat tires consists of an aerosol can containing compressed air and a patch to seal the hole in the<br />

tire. Suppose 10.0 L of air at atmospheric pressure (101.3 kilopascals, or kPa) is compressed into a 1.0-L aerosol<br />

can. What is the pressure of the compressed air in the can?<br />

Looking for<br />

Pressure of compressed air in a can (P2) Given<br />

P1 = 101.3 kPa; V1 = 10.0 liters; V2 = 1.0 liters<br />

Relationship<br />

Use Boyle’s Law to solve for P2. Divide each side by V2 to isolate P2 on one side of the equation.<br />

P V<br />

P<br />

1 1<br />

2<br />

=<br />

-------------<br />

V<br />

2<br />

Solution<br />

The pressure inside the aerosol can is<br />

1,013 kPa.<br />

1. The air inside a tire pump occupies a volume of 130. cm 3 at a pressure of one atmosphere. If the volume<br />

decreases to 40.0 cm 3 , what is the pressure, in atmospheres, inside the pump?<br />

2. A gas occupies a volume of 20. m 3 at 9,000. Pa. If the pressure is lowered to 5,000. Pa, what volume will the<br />

gas occupy?<br />

3. You pump 25.0 L of air at atmospheric pressure (101.3 kPa) into a soccer ball that has a volume of 4.50 L.<br />

What is the pressure inside the soccer ball if the temperature does not change?<br />

4. Hyperbaric oxygen chambers (HBO) are used to treat divers with decompression sickness. As pressure<br />

increases inside the HBO, more oxygen is forced into the bloodstream of the patient inside the chamber. To<br />

work properly, the pressure inside the chamber should be three times greater than atmospheric pressure<br />

(101.3 kPa). What volume of oxygen, held at atmospheric pressure, will need to be pumped into a 190.-L<br />

HBO chamber to make the pressure inside three times greater than atmospheric pressure?<br />

5. A 12.5-liter scuba tank holds oxygen at a pressure of 202.6 kPa. What is the original volume of oxygen at<br />

101.3 kPa that is required to fill the scuba tank?<br />

P 2<br />

101.3 kPa × 10.0 L<br />

= --------------------------------------------- = 1,013 kPa<br />

1.0 L<br />

10.3


Name: Date:<br />

10.4 Buoyancy<br />

When an object is placed in a fluid (liquid or gas), the fluid exerts an upward force upon the object. This force is<br />

called a buoyant force.<br />

At the same time, there is an attractive force between the object and Earth—the force of gravity. It acts as a<br />

downward force. You can compare the two forces to determine whether the object floats or sinks in the fluid.<br />

Buoyant force > Gravitational force Buoyant force < Gravitational force<br />

Object floats Object sinks<br />

Example 1: A 13-N object is placed in a container of fluid. If the fluid exerts a 60-N buoyant (upward) force on<br />

the object, will the object float or sink?<br />

Answer: Float. The upward buoyant force (60 N) is greater than the weight of the object (13 N).<br />

Example 2: The rock weighs 2.25 N when suspended in air. In water, it<br />

appears to weigh only 1.8 N. Why?<br />

Answer: The water exerts a buoyant force on the rock. This buoyant force<br />

equals the difference between the rock’s weight in air and its apparent weight<br />

in water.<br />

2.25 N – 1.8 N =<br />

0.45 N<br />

The water exerts a buoyant force of 0.45 newtons on the rock.<br />

1. A 4.5-N object is placed in a tank of water. If the water exerts a force of 4.0 N on the object, will the object<br />

sink or float?<br />

2. The same 4.5-N object is placed in a tank of glycerin. If the glycerin exerts a force of 5.0 N on the object,<br />

will the object sink or float?<br />

3. You suspend a brass ring from a spring scale. Its weight is 0.83 N while it is suspended in air. Next, you<br />

immerse the ring in a container of light corn syrup. The ring appears to weigh 0.71 N. What is the buoyant<br />

force acting on the ring in the light corn syrup?<br />

4. You wash the brass ring (from question 3) and then suspend it in a container of vegetable oil. The ring<br />

appears to weigh 0.73 N. What is the buoyant force acting on the ring?<br />

5. Which has greater buoyant force, light corn syrup or the vegetable oil? Why do you think this is so?<br />

6. A cube of gold weighs 1.89 N when suspended in air from a spring scale. When suspended in molasses, it<br />

appears to weigh 1.76 N. What is the buoyant force acting on the cube?<br />

7. Do you think the buoyant force would be greater or smaller if the gold cube were suspended in water?<br />

Explain your answer.<br />

10.4


Name: Date:<br />

10.4 Charles’s Law<br />

Charless law shows a direct relationship between the volume of a gas and the temperature of a gas when the<br />

temperature is given in the Kelvin scale. The Charles’s law equation is below.<br />

Converting from degrees Celsius to Kelvin is easy—you add 273 to the Celsius temperature. To convert from<br />

Kelvins to degrees Celsius, you subtract 273 from the Kelvin temperature.<br />

A truck tire holds 25.0 liters of air at 25 °C. If the temperature drops to 0 °C, and the pressure remains constant,<br />

what will be the new volume of the tire?<br />

Looking for<br />

The new volume of the tire (V2) Given<br />

V1 = 25.0 liters; T1 = 25 °C; T2 = 0 °C<br />

Relationships<br />

Use Charles’ Law to solve for V2. Multiply each side<br />

by T2 to isolate V2 on one side of the equation.<br />

V T<br />

V =<br />

-------------<br />

1 2<br />

2 T<br />

1<br />

Convert temperature values in Celsius degrees to<br />

Kelvin: T Kelvin = T Celsius + 273<br />

Solution<br />

T 1 = 25 °C + 273 = 298<br />

T 2 = 0 °C + 273 = 273<br />

The new volume inside the tire is 23.0 liters.<br />

1. If a truck tire holds 25.0 liters of air at 25.0 °C, what is the new volume of air in the tire if the temperature<br />

increases to 30.0 °C?<br />

2. A balloon holds 20.0 liters of helium at 10.0 °C. If the temperature increases to 50.0 °C, and the pressure<br />

does not change, what is the new volume of the balloon?<br />

3. Use Charles’ Law to fill in the following table with the correct values. Pay attention to the temperature units.<br />

V1 T1 V2 T2 a. 840 K 1,070 mL 147 K<br />

b. 3250 mL 475 °C 50 °C<br />

c. 10 L 15 L 50 °C<br />

V 2<br />

25.0 L × 273<br />

= ------------------------------ = 23.0 L<br />

298<br />

10.4


Name: Date:<br />

10.4 Pressure-Temperature Relationship<br />

The pressure-temperature relationship shows a<br />

direct relationship between the pressure of a gas<br />

and its temperature when the temperature is given<br />

in the Kelvin scale. Another name for this<br />

relationship is the Gay-Lussac Law. The pressuretemperature<br />

equation is shown at right.<br />

Converting from degrees Celsius to Kelvin is easy<br />

—you add 273 to the Celsius temperature. To<br />

convert from Kelvins to degrees Celsius, you<br />

subtract 273 from the Kelvin temperature.<br />

A constant volume of gas is heated from 25.0°C to 100°C. If the gas pressure starts at 1.00 atmosphere, what is<br />

the final pressure of this gas?<br />

Looking for<br />

The new pressure of the gas (P2) Given<br />

T1 = 25 °C; P1 = 1 atm; T2 = 100 °C<br />

Relationships<br />

Use pressure-temperature relation to solve for P2. Multiply<br />

each side by T2 to isolate P2 on one side of the equation.<br />

P<br />

1<br />

T<br />

2<br />

P<br />

2<br />

=<br />

-------------<br />

T<br />

1<br />

Convert temperature values in Celsius degrees to Kelvin:<br />

T Kelvin = T Celsius + 273<br />

Solution<br />

T 1 = 25 °C + 273 = 298<br />

T 2 = 100 °C + 373 = 313<br />

The new pressure of the volume of gas<br />

is 1.25 atmospheres.<br />

1. At 400. K, a volume of gas has a pressure of 0.40 atmospheres. What is the pressure of this gas at 273 K?<br />

2. What is the temperature of the volume of gas (starting at 400. K with a pressure of 0.4 atmospheres), when<br />

the pressure increases to 1 atmosphere?<br />

3. Use the pressure-temperature relationship to fill in the following table with the correct values. Pay attention<br />

to the temperature units.<br />

P1 T1 P2 T2 a. 30.0 atm –100 °C 500 °C<br />

b. 15.0 atm 25.0 °C 18.0 atm<br />

c. 5.00 atm 3.00 atm 293 K<br />

P 2<br />

1 atm × 373<br />

= ---------------------------- = 1.25 atm<br />

298<br />

10.4


Name: Date:<br />

10.4 Archimedes<br />

10.4<br />

Archimedes was a Greek mathematician who specialized in geometry. He figured out the value of pi<br />

and the volume of a sphere, and has been called “the father of integral calculus.” During his lifetime, he was<br />

famous for using compound pulleys and levers to invent war machines that successfully held off an attack on his<br />

city for three years. Today he is best known for Archimedes’ principle, which was the first explanation of how<br />

buoyancy works.<br />

Archimedes’ screw<br />

Archimedes was born in<br />

Syracuse, on Sicily (then an<br />

independent Greek city-state),<br />

in 287 B.C. His letters suggest<br />

that he studied in Alexandria,<br />

Egypt, as a young man.<br />

Historians believe it was there<br />

that he invented a device for<br />

raising water by means of a<br />

rotating screw or spirally bent<br />

tube within an inclined hollow<br />

cylinder. The device known as Archimedes’ screw is<br />

still used in many parts of the world.<br />

“Eureka!”<br />

A famous Greek legend says that King Hieron II of<br />

Syracuse asked Archimedes to figure out if his new<br />

crown was pure gold or if the craftsman had mixed<br />

some less expensive silver into it. Archimedes had to<br />

determine the answer without destroying the crown.<br />

He thought about it for days and then, as he lowered<br />

himself into a bath, the method for figuring it out<br />

struck him. The legend says Archimedes ran through<br />

the streets, shouting “Eureka!”—meaning “I have<br />

found it.”<br />

A massive problem<br />

Archimedes realized that if<br />

he had equal masses of gold<br />

and silver, the denser gold<br />

would have a smaller<br />

volume. Therefore, the gold<br />

would displace less water<br />

than the silver when<br />

submerged.<br />

Archimedes found the mass<br />

of the crown and then made<br />

a bar of pure gold with the<br />

same mass. He submerged<br />

the gold bar and measured the volume of water it<br />

displaced. Next, he submerged the crown. He found<br />

the crown displaced more water than the gold bar had<br />

and, therefore, could not be pure gold. The gold had<br />

been mixed with a less dense material. Archimedes<br />

had confirmed the king’s doubts.<br />

Why do things float?<br />

Archimedes wrote a treatise titled On Floating Bodies,<br />

further exploring density and buoyancy. He explained<br />

that an object immersed in a fluid is pushed upward by<br />

a force equal to the weight of the fluid displaced by the<br />

object. Therefore, if an object weighs more than the<br />

fluid it displaces, it will sink. If it weighs less than the<br />

fluid it displaces, it will float. This statement is known<br />

as Archimedes’ principle. Although we commonly<br />

assume the fluid is water, the statement holds true for<br />

any fluid, whether liquid or gas. A helium balloon<br />

floats because the air it displaces weighs more than the<br />

balloon filled with lightweight gas.<br />

Cylinders, circles, and exponents<br />

Archimedes wrote several other treatises, including<br />

“On the Sphere and the Cylinder,” “On the<br />

Measurement of the Circle,” “On Spirals,” and “The<br />

Sand Reckoner.” In this last treatise, he devised a<br />

system of exponents that allowed him to represent<br />

large numbers on paper—up to 8 × 10 63 in modern<br />

scientific notation. This was large enough, he said, to<br />

count the grains of sand that would be needed to fill<br />

the universe. This paper is even more remarkable for<br />

its astronomical calculations than for its new<br />

mathematics. Archimedes first had to figure out the<br />

size of the universe in order to estimate the amount of<br />

sand needed to fill it. He based his size calculations on<br />

the writings of three astronomers (one of them was his<br />

father). While his estimate is considered too small by<br />

today’s standard, it was much, much larger than<br />

anyone had previously suggested. Archimedes was the<br />

first to think on an “astronomical scale.”<br />

Archimedes was killed by a Roman soldier during an<br />

invasion of Syracuse in 212 B.C.


Page 2 of 2<br />

Reading reflection<br />

1. The boldface words in the article are defined in the glossary of your text<strong>book</strong>. Look them up and<br />

then explain the meaning of each in your own words.<br />

2. Imagine you are Archimedes and have to write your resume for a job. Describe yourself in a brief paragraph.<br />

Be sure to include in the paragraph your skills and the jobs you are capable of doing.<br />

3. What was Archimedes’ treatise “The Sand Reckoner” about?<br />

4. Why does a balloon filled with helium float in air, but a balloon filled with air from your lungs sink?<br />

5. Research one of Archimedes’ inventions and create a poster that shows how the device worked.<br />

10.4


Name: Date:<br />

10.4 Narcís Monturiol<br />

10.4<br />

Monturiol, a visionary and peaceful revolutionary, wanted to improve the social and economic lives of<br />

his countrymen. Moved by the suffering of coral divers due to their extremely dangerous working conditions,<br />

Monturiol built a submarine to transport the divers to the reefs. He hoped that in time, his invention would also<br />

help people understand the ocean world.<br />

Birth of a Spanish inventor<br />

Narcís Monturiol was born<br />

on September 28, 1819, in<br />

Figueres, Catalonia, a<br />

region of northeastern<br />

Spain. Monturiol’s father<br />

was a cooper—which<br />

means that he handcrafted<br />

wooden barrels that held<br />

wine, oil, and milk. Narcís<br />

was one of five children. At<br />

an early age he showed an<br />

interest in design and<br />

invention. When he was ten, he created a realistic<br />

model of a wooden clock.<br />

His mother wanted Narcís to become a priest, but he<br />

earned a law degree instead. He never practiced law,<br />

however. Instead, he became a self-taught engineer.<br />

Monturiol was active politically. He supported<br />

socialism, communism, and the ideal of a utopia<br />

where everyone lived together in harmony. He turned<br />

to science with the hope of creating that utopia.<br />

The perils of coral diving<br />

Monturiol was concerned about the danger involved in<br />

the work of Spanish coral fishermen. A diver, holding<br />

his breath for several minutes, dove nearly 20 meters<br />

beneath the ocean surface to retrieve valuable pieces<br />

of coral. The divers risked drowning, injuries from<br />

rocks and coral, and possible shark attacks.<br />

In 1857, Monturiol formed a company to design and<br />

build a submarine. His goal was to develop a vessel to<br />

help coral divers with their physical work and to<br />

lessen the risk involved.<br />

Monturiol was not the first to build a submarine.<br />

Historical records show that Aristotle, Renaissance<br />

period inventors, and others had attempted to build<br />

submarines. These models were often created for<br />

warfare. Most early submarines were unsuccessful and<br />

dangerous.<br />

Ictineo I<br />

Monturiol’s first submarine, Ictineo I, made its first dive<br />

in 1859. The name Ictineo is derived from Greek, and is<br />

translated “fish ship.” During its initial dive, Ictineo I hit<br />

underwater pilings. Repairing what he could, Monturiol<br />

sent Ictineo I on its second dive within a few hours.<br />

Monturiol’s seven-meter submarine had a spherical inner<br />

hull built to withstand water pressure, and an elliptical<br />

(egg-shaped) outer hull for ease of movement. Between<br />

the two hulls were tanks that stored and released water to<br />

control the submarine’s depth. Oxygen tanks were also<br />

stored in this space. The submarine was powered by four<br />

men turning the propeller by hand.<br />

Ictineo I was equipped with a ventilator, two sets of<br />

propellers, and several portholes. The submarine had<br />

the ability to retrieve objects and was equipped with a<br />

back-up system to raise it to the surface in an<br />

emergency.<br />

Ictineo I made nearly 20 dives that first year, to a<br />

depth of 20 meters. The submarine eventually stayed<br />

underwater for nearly two hours.<br />

Ictineo II<br />

Monturiol created a second and improved model<br />

called Ictineo II. Rather than relying on manpower, the<br />

Ictineo II had a steam engine. These engines were<br />

traditionally powered by an open flame. Monturiol<br />

created a submarine-safe alternative to power the<br />

engine using a chemical reaction. The open flame<br />

would have removed oxygen, but the chemical<br />

reaction added oxygen to the cabin instead.<br />

The Ictineo II was 17 meters long, had two engines,<br />

dove to depths of nearly 30 meters, had many portholes,<br />

and remained underwater for almost seven hours.<br />

Unfortunately, Monturiol ran out of funds and was<br />

forced to sell his submarine for scrap. Although he<br />

didn’t receive much credit for his inventions during<br />

his lifetime, he is now recognized as an important<br />

contributor to submarine development. Monturiol died<br />

in 1885.


Page 2 of 2<br />

Reading reflection<br />

1. What moved Monturiol to create a submarine?<br />

2. Identify key features of the Ictineo I and II.<br />

3. Research: Where are model replicas of Ictineo I and II located?<br />

4. Research: What happed to Monturiol’s Ictineo I?<br />

5. Research: Name three things Monturiol invented in addition to the submarine.<br />

6. Research: How did Spain honor Monturiol in 1987?<br />

7. Research: What is the Narcís Monturiol medal?<br />

10.4


Name: Date:<br />

10.4 Archimedes’ Principle<br />

More than 2,000 years ago, Archimedes discovered the relationship between buoyant force and how much fluid<br />

is displaced by an object. Archimedes’ principle states:<br />

The buoyant force acting on an object in a fluid is equal to the weight of the fluid displaced by the object.<br />

We can practice figuring out the buoyant force using a beach ball and a big tub of water. Our beach ball has a<br />

volume of 14,130 cm 3 . The weight of the beach ball in air is 1.5 N.<br />

If you put the beach ball into the water and don’t push down on it, you’ll see that<br />

the beach ball floats on top of the water by itself. Only a small part of the beach<br />

ball is underwater. Measuring the volume of the beach ball that is under water, we<br />

find it is 153 cm 3 . Knowing that 1 cm 3 of water has a mass of 1 g, you can<br />

calculate the weight of the water displaced by the beach ball.<br />

153 cm 3 of water = 153 grams = 0.153 kg<br />

weight = mass × force of gravity per kg = (0.153 kg) × 9.8 N/kg = 1.5 N<br />

According to Archimedes principle, the buoyant force acting on the beach ball equals the weight of the water<br />

displaced by the beach ball. Since the beach ball is floating in equilibrium, the weight of the ball pushing down<br />

must equal the buoyant force pushing up on the ball. We just showed this to be true for our beach ball.<br />

Have you ever tried to hold a beach ball underwater? It takes a lot<br />

of effort! That is because as you submerge more of the beach ball,<br />

the more the buoyant force acting on the ball pushes it up. Let’s<br />

calculate the buoyant force on our beach ball if we push it all the<br />

way under the water. Completely submerged, the beach ball<br />

displaces 14,130 cm 3 of water. Archimedes principle tells us that<br />

the buoyant force on the ball is equal to the weight of that water:<br />

14,130 cm 3 of water = 14,130 grams = 14.13 kg<br />

weight = mass × force of gravity per kg = (14.13 kg) × 9.8 N/kg = 138 N<br />

If the buoyant force is pushing up with 138 N, and the weight of the ball is only 1.5 N, your hand pushing down<br />

on the ball supplies the rest of the force, 136.5 N.<br />

• A 10-cm3 block of lead weighs 1.1 N. The lead is placed in a tank of water. One cm3 of water weighs 0.0098<br />

N. What is the buoyant force on the block of lead?<br />

Solution:<br />

The lead displaces 10 cm3 of water.<br />

buoyant force = weight of water displaced<br />

10 cm 3 of water × 0.0098 N/cm3 = 0.098 N<br />

10.4


Page 2 of 2<br />

1. A block of gold and a block of wood both have the same volume. If they are both submerged in<br />

water, which has the greater buoyant force acting on it?<br />

2. A 100-cm3 block of lead that weighs 11 N is carefully submerged in water. One cm3 of water weighs<br />

0.0098 N.<br />

a. What volume of water does the lead displace?<br />

b. How much does that volume of water weigh?<br />

c. What is the buoyant force on the lead?<br />

d. Will the lead block sink or float in the water?<br />

3. The same 100-cm 3 lead block is carefully submerged in a container of mercury. One cm3 of mercury weighs<br />

0.13 N.<br />

a. What volume of mercury is displaced?<br />

b. How much does that volume of mercury weigh?<br />

c. What is the buoyant force on the lead?<br />

d. Will the lead block sink or float in the mercury?<br />

4. According to problems 2 and 3, does an object’s density have anything to do with whether or not it will float<br />

in a particular liquid? Justify your answer.<br />

5. Based on the table of densities, explain whether the object would float or sink in the following situations:<br />

a. A block of solid paraffin (wax) in molasses.<br />

b. A bar of gold in mercury.<br />

c. A piece of platinum in gasoline.<br />

d. A block of paraffin in gasoline.<br />

material density (g/ cm 3 )<br />

gasoline 0.7<br />

gold 19.3<br />

lead 11.3<br />

mercury 13.6<br />

molasses 1.37<br />

paraffin 0.87<br />

platinum 21.4<br />

10.4


Name: Date:<br />

11.1 Layers of the Atmosphere<br />

Use the table below to organize the information in Section 11.1 of your text. You can use the table as a study<br />

guide as you review for tests.<br />

Layer Distance from<br />

Earth’s Surface<br />

Troposphere<br />

Stratosphere<br />

Mesosphere<br />

Thermosphere<br />

Exosphere<br />

Thickness Facts<br />

11.1


Page 1 of 2<br />

11.2 Gaspard Gustave de Coriolis<br />

Gaspard Gustave de Coriolis was a French mechanical engineer and mathematician in the early<br />

1800’s. His name is famous today for his work on wind deflection by the Coriolis effect.<br />

From Paris to Nancy to Paris again<br />

Gaspard Gustave de Coriolis<br />

(Kor-e-olis) was born in 1792<br />

in Paris, France.<br />

Shortly after his birth, his<br />

family left Paris and settled in<br />

the town of Nancy,<br />

pronounced nasi in French. It<br />

was here in Nancy that<br />

Coriolis grew up and attended<br />

school.<br />

He was exceptionally gifted in<br />

the area of mathematics, and took the entrance exam for<br />

Ecole Polytechnique when he was 16 years old. Ecole is<br />

French for school. Ecole Polytechnique is one of the bestknown<br />

French Grandes ecoles (Great <strong>School</strong>s) for<br />

engineering. Coriolis ranked second out of all the students<br />

entering Ecole Polytechnique that year.<br />

After graduating from Ecole Polytechnique, he continued<br />

his studies at Ecole des Ponts et Chaussees (<strong>School</strong> of<br />

Bridges and Roads) in Paris.<br />

Then Coriolis’ dreams of becoming an engineer were put<br />

on hold. Faced with the responsibility of supporting his<br />

family after his father’s death in 1816, he accepted a<br />

position as a tutor in mathematical analysis and mechanics<br />

back at Ecole Polytechnique. At this time, Coriolis was<br />

only 24 years old.<br />

The tutor becomes a professor<br />

Coriolis earned great respect for his studies and research in<br />

mechanics, engineering, and mathematics. He published his<br />

first official work in 1829 titled On the Calculation of<br />

Mechanical Action. This same year he became professor of<br />

mechanics at Ecole Centrales des Artes et Manufactures.<br />

Coriolis became one of the leading scientific thinkers by<br />

introducing the terms work and kinetic energy.<br />

In 1830 he once again found himself back at Ecole<br />

Polytechnique after accepting the position of professor.<br />

Coriolis went on to be elected chair of the Academie des<br />

Sciences, and later appointed director of studies at Ecole<br />

Polytechnique.<br />

11.2<br />

The paper that made him famous<br />

In 1835, Coriolis published the paper that made his name<br />

famous: On the Equations of Relative Motion of Systems of<br />

Bodies. The paper discussed the transfer of energy in<br />

rotating systems. Coriolis’s research helped explain how<br />

the Earth’s rotation causes the motion of air to curve with<br />

respect to the surface of the Earth.<br />

His name did not become linked with meteorology until the<br />

beginning of the twentieth century. He is noted for the<br />

explanation of the bending of air currents known as the<br />

Coriolis effect.<br />

Bending of currents<br />

There are patterns of winds that naturally cover the Earth.<br />

The global surface wind patterns in the northern and<br />

southern hemisphere bend due to the Earth’s rotation.<br />

For example, the Coriolis effect bends the trade winds<br />

moving across the surface. They flow from northeast to<br />

southwest in the northern hemisphere, and from southwest<br />

to northeast in the southern hemisphere.<br />

The Coriolis effect has helped scientist explain many<br />

rotational patterns, yet it does not determine the direction of<br />

water draining in sinks, bathtubs, and toilets (as some have<br />

suggested). However, it does explain the rotation of<br />

cyclones.<br />

Gaspard Gustave de Coriolis died in 1843 in Paris.


Page 2 of 2<br />

Reading reflection<br />

1. How did Coriolis’s education influence his work?<br />

2. Explain the importance of Coriolis’s first <strong>book</strong> titled On the Calculation of Mechanical Action.<br />

3. To understand why Earth’s rotation affects the path of air currents, imagine the following situation: You are a pilot who<br />

wants to fly an airplane from St. Paul, Minnesota, 700 miles south to Little Rock, Arkansas. If you set your compass and<br />

try to fly straight south, you will probably end up in New Mexico! Why would you end up in New Mexico instead of Little<br />

Rock?<br />

4. Compare the Coriolis effect in the northern hemisphere with the Coriolis effect in the southern hemisphere.<br />

5. Research the following global surface wind patterns: trade winds, polar easterlies, prevailing westerlies, and explain<br />

the Coriolis effect on each wind pattern.<br />

6. Research why Coriolis’s work on Earth’s rotation was not accepted until long after his death in 1843.<br />

7. Research the other <strong>book</strong>s that Coriolis wrote, such as Mathematical Theory of the Game of Billiards and Treatise on the<br />

Mechanics of Solid Bodies, and explain their scientific impact.<br />

11.2


Name: Date:<br />

11.2 Degree Days<br />

Freezing winter weather or sweltering summer heat—in either condition, people use energy to keep their homes,<br />

schools, and businesses comfortable. You can use degree day values to help predict how much energy will be<br />

needed each month to heat or cool a building. In this activity, you will learn how degree day values are calculated<br />

and how to use them to evaluate energy needs.<br />

Understanding degree days<br />

Degree day values are calculated by comparing a day’s average<br />

temperature to 65 °Fahrenheit. The more extreme the temperature, the<br />

higher the degree day value. For example, if the average daily<br />

temperature were 72°F, the degree day value would be 72 minus 65, or<br />

7. On a day with an average temperature of 35°F, the degree day value<br />

would be 65 minus 35, or 30.<br />

When the average daily temperature is lower than 65°F, we use the term<br />

heating degree day value, because you need to add heat to a building<br />

to bring it to a comfortable temperature. When the average daily<br />

temperature is higher than 65°F, we talk about the cooling degree day<br />

value.<br />

We compare the daily average temperature to 65°F because 65°F is a<br />

temperature at which most people are comfortable without heating or air<br />

conditioning. If the average temperature is close to 65°F, you won’t need<br />

to spend much money heating or cooling your home that day. However, if<br />

the average temperature is well above or below 65°F, you’ll be spending<br />

a lot more money on electricity or fuel.<br />

1. On July 22, 2002, the average daily temperature in St. Louis, Missouri, was 88°F. Calculate the cooling<br />

degree day value.<br />

2. On January 22, 2003, the average daily temperature in St. Louis was 14°F. Calculate the cooling degree day<br />

value.<br />

3. On which day—July 22, 2002 or January 22, 2003—was the heating degree day value zero? On which day<br />

was the cooling degree day value zero?<br />

11.2


Page 2 of 3<br />

Using temperature data to calculate degree day values<br />

The table below shows temperature data recorded by the National Weather Service in May 2003.<br />

Day <strong>High</strong> temp<br />

(°F)<br />

Table 1: Temperature data for St. Louis, May 1-14, 2003<br />

Low temp<br />

(°F)<br />

Average temp<br />

(high +low)÷2<br />

Heating<br />

degree day value<br />

Cooling<br />

degree day value<br />

1 73 61 (73 + 61)¸2 = 67 0 2<br />

2 63 52<br />

3 70 44<br />

4 65 52<br />

5 83 58<br />

6 79 59<br />

7 74 60<br />

8 71 53<br />

9 90 70<br />

10 82 62<br />

11 65 52<br />

12 71 52<br />

13 74 56<br />

14 75 60<br />

Two week totals:<br />

1. Calculate the average temperature, the heating degree day value, and the cooling degree day value for each<br />

day. Record your answers in the Table 1. The first one is done for you.<br />

2. During the first two weeks of May, on how many days were St. Louis residents more likely to use their<br />

heating systems? On how many days were they more likely to cool their homes?<br />

Calculating monthly totals for degree day values<br />

3. Find the sum of the numbers in the fifth column of Table 1. This will give you the total heating degree day<br />

value for May 1–14, 2003. Record your answer in the table’s last row.<br />

4. Find the total cooling degree day value for same time period by finding the sum of the sixth column of<br />

Table 1. Record your answer in the table’s last row.<br />

5. The total heating degree day value for May 15-31, 2003 was 31. The total cooling degree day value was 32.<br />

Find the monthly total heating and cooling degree day values.<br />

6. In St. Louis, the average total heating degree day value for May is 79. The average total cooling degree day<br />

value for May is 114. How was May 2003 different from the average? Do you think residents used more<br />

energy than usual to keep their homes comfortable, or less?<br />

11.2


Page 3 of 3<br />

Using average monthly degree day values<br />

The National Weather Service provides average monthly degree day values to help citizens better<br />

evaluate their energy needs.<br />

Average monthly heating degree day (HDD) and cooling degree day (CDD) values for St. Louis<br />

1. On a separate piece of paper, make a bar graph showing the average monthly heating and cooling degree day<br />

values for St. Louis. Place months on the x-axis and monthly average degree day values on the y-axis. Use<br />

red bars for the heating degree day values and blue bars for the cooling degree day values. Use your graph to<br />

answer the following questions:<br />

2. In which month should a St. Louis resident budget the most money for heating costs?<br />

3. In which month should a St. Louis resident budget the most money for cooling costs?<br />

4.<br />

January February March April May June<br />

HDD CDD HDD CDD HDD CDD HDD CDD HDD CDD HDD CDD<br />

1097 0 844 0 613 7 294 32 79 114 6 316<br />

Average monthly heating degree day (HDD) and cooling degree day (CDD) values for St. Louis<br />

July August September October November December<br />

HDD CDD HDD CDD HDD CDD HDD CDD HDD CDD HDD CDD<br />

0 461 1 396 46 196 246 36 583 3 949 0<br />

a In which month do you think a St. Louis resident will spend the least amount of money to keep their home<br />

at a comfortable temperature? Explain.<br />

b Challenge! What additional information would you need to calculate the actual monthly heating and<br />

cooling costs for a particular building?<br />

All climate data courtesy of the National Weather Service St. Louis Weather Station.<br />

11.2


Page 1 of 2<br />

11.3 Joanne Simpson<br />

11.3<br />

Dr. Joanne Simpson was the first woman to serve as president of the American Meteorological Society.<br />

Her road to success was not easy. She chose to forge ahead in the field of meteorology for the sake of women<br />

who would enter the field after her.<br />

Early goals<br />

Joanne Simpson was born in<br />

1923 in Boston,<br />

Massachusetts. At a young<br />

age, Simpson was<br />

determined to have a career<br />

that would provide her with<br />

financial independence. Her<br />

mother, a journalist,<br />

remained in a difficult<br />

marriage because she could<br />

not afford to provide for her<br />

children on her own.<br />

Simpson knew at age ten that<br />

she wanted to be able to support herself and any future<br />

children.<br />

So Simpson’s journey began. As a child, Simpson loved<br />

clouds. She spent time gazing at clouds when she sailed<br />

off the Cape Cod coast. Simpson’s father, aviation editor<br />

for the Boston Herald newspaper, probably sparked<br />

Simpson’s interest in flight. Joanne loved to fly and<br />

earned her pilot’s license at 16. Her interest in weather<br />

took off.<br />

The sky’s the limit<br />

Simpson earned her degree from the University of<br />

Chicago in 1943. It was here that she developed a love<br />

for science. She planned to study astrophysics. However,<br />

as a student pilot she was required to take a meteorology<br />

course. Meteorology was fascinating. She wanted to take<br />

more courses. Carl-Gustaf Rossby, a great twentieth<br />

century meteorologist, had just started an institute of<br />

meteorology at the university. Simpson met with Rossby<br />

and enrolled in the World War II meteorology program<br />

as a teacher-in-training. She taught meteorology to<br />

aviation cadets.<br />

Women temporarily filled the roles of men away at war.<br />

At the end of the war, most women returned home, but<br />

not Simpson. She completed a master’s degree and<br />

wanted to earn a Ph.D. Her advisor said that women did<br />

not earn Ph.D.s in meteorology. The all-male faculty felt<br />

that women were unable to do the work which included<br />

night shifts and flying planes. She was even told that if<br />

she earned the degree no one would ever hire a woman.<br />

Determined even more, Simpson pursued her dream. She<br />

took a course with Herbert Riehl, a leader in the field of<br />

tropical meteorology. She asked Riehl if he would be her<br />

advisor and he agreed. Not surprisingly, Simpson chose<br />

to study clouds. Her new advisor thought it would be a<br />

perfect topic “for a little girl to study.” Throughout her<br />

Ph.D. program, she studied in an unsupportive academic<br />

environment. She persevered and became the first<br />

woman to earn a Ph.D. in meteorology.<br />

Working woman<br />

As a woman, Simpson did have difficulty finding a job.<br />

Eventually she became an assistant physics professor.<br />

Two years later, she took a job at Woods Hole<br />

Oceanographic Institute to study tropical clouds. People<br />

at the time believed clouds were produced by the<br />

weather and were not the cause for weather. Simpson,<br />

studying cumulus clouds in the tropics, proved that<br />

clouds do affect the weather. She found that very tall<br />

clouds near the equator created enough energy to<br />

circulate the atmosphere. Together, Simpson and Riehl<br />

developed the “Hot Tower Theory.” Tall cloud towers<br />

can carry moist ocean air as high as 50,000 feet into the<br />

air, create heat, and release energy.<br />

While studying hurricanes, Simpson discovered that hot<br />

towers release energy to the hurricane eye and act as the<br />

hurricane’s engine. Simpson’s work with clouds<br />

continued as she created the first cloud model. Using a<br />

slide rule, she created a model well before computers<br />

were invented. She later became the first person to create<br />

a computerized cloud model.<br />

A life of achievement<br />

Simpson’s career spans many decades, many<br />

institutions, and many positions. She has won numerous<br />

awards including the Carl-Gustaf Rossby Research<br />

Award. In 1979, she joined NASA’s Goddard Space<br />

Flight Center and enjoyed finally working with other<br />

female scientists. As a NASA chief scientist, Simpson<br />

does not plan to retire. Today, she continues to study<br />

rainfall, satellite images, and hurricanes.


Page 2 of 2<br />

Reading reflection<br />

1. Dr. Simpson achieved many “firsts” in the field of meteorology. Identify three of these first time<br />

achievements.<br />

2. Simpson’s road to success in the field of meteorology was not easy. What obstacles did she overcome on her<br />

journey to eventual success?<br />

3. What have you learned about working towards goals based on Simpson’s biography?<br />

4. Research: What is a slide rule? What caused the slide rule to fade from use?<br />

5. Research: What is the Carl-Gustaf Rossby Research Award?<br />

6. Research: Where is the Woods Hole Oceanographic Institute located and what does it do?<br />

7. Research: Use a library or the Internet to find a photo or sketch of hot tower clouds. Present the image to<br />

your class, citing your source.<br />

11.3


Name: Date:<br />

11.3 Weather Maps<br />

You have learned how the Sun heats Earth and how the heating of land is different than the heating of water. In<br />

this skill sheet, you will analyze the national weather forecast and make inferences as to what causes differences<br />

in weather across the nation. To complete this skill sheet, you will need a national weather forecast from a daily<br />

newspaper and a map of North America from an atlas.<br />

Analyzing temperature<br />

Study the national weather forecast from a daily newspaper. Locate the list of the temperature and sky cover in<br />

cities around the country. Also, locate the weather map showing sunny regions, the temperature, high- and lowpressure<br />

regions, and fronts. Record the high and low temperatures for cities in the table below. Then find the<br />

difference between the two temperature readings. Sky cover and pressure will be filled in later.<br />

City <strong>High</strong> Low Temp difference Sky cover Pressure<br />

Seattle, Washington<br />

Los Angeles, California<br />

Las Vegas, Nevada<br />

Phoenix, Arizona<br />

Atlanta, Georgia<br />

Tampa, Florida<br />

San Francisco, California<br />

Oklahoma City, Oklahoma<br />

New Orleans, Louisiana<br />

Kansas City, Kansas<br />

Tucson, Arizona<br />

Denver, Colorado<br />

Dallas, Texas<br />

Houston, Texas<br />

Minneapolis, Minnesota<br />

Memphis, Tennessee<br />

Chicago, Illinois<br />

Miami, Florida<br />

New York, New York<br />

Baltimore, Maryland<br />

11.3


Page 2 of 2<br />

What causes the wide variety of temperature conditions across the map?<br />

Use the table on the first page to respond to the following questions. It will also be helpful for you to<br />

study a map of the United States that includes the Pacific and Atlantic Oceans and details about major<br />

topographical features.<br />

1. Give examples of differences in the cities’ high temperatures due to latitude. For example, Dallas, Texas is<br />

in a lower latitude than Seattle, Washington. Explain why these differences exist.<br />

2. Give examples of differences in the cities’ high temperatures due to geographical features such as the Pacific<br />

Ocean, the Rocky Mountains, the Great Lakes, or the Atlantic Ocean. Explain why geography influences<br />

temperatures.<br />

3. Fill in the table for the sky cover for each city. How does the sky cover affect the temperatures of cities near<br />

the same latitude? Why do you think this is?<br />

What does atmospheric pressure tell us about the weather?<br />

4. On your weather map, over which states are areas of high pressure centered? Over which states are lowpressure<br />

areas centered?<br />

5. In the sixth column of the table (the heading is Pressure), record whether you think each city is in a region of<br />

high pressure, low pressure, or in-between.<br />

6. What kind of cloud cover or weather is associated with high-pressure regions? Look at the sky cover for the<br />

cities in the high-pressure regions. What do you think the humidity is like in these regions?<br />

7. What kind of cloud cover or weather is associated with low-pressure regions? Look at the sky cover for the<br />

cities in the low-pressure regions. What do you think the humidity is like in these regions?<br />

8. Locate the fronts shown on the weather map. The flags on the fronts tell us the direction of the wind. The<br />

cold fronts are symbolized by triangular flags, the warm fronts by semicircular flags. Are fronts associated<br />

with high- or low-pressure regions?<br />

9. What type of weather is associated with a warm front? What type of weather is associated with a cold front?<br />

10. Based on what you have learned so far about low- and high-pressure regions, let’s investigate the effect they<br />

have on the wind. <strong>High</strong>-pressure regions tend to push air toward low-pressure regions. Do you think the air<br />

in a low-pressure region tends to sink or rise? Does the air in a high-pressure region sink or rise?<br />

11. Based on those conclusions, how do you think low-pressure regions contribute to the formation of<br />

rainstorms?<br />

12. Precipitation occurs when warm, moist air is cooled to a certain temperature called the dew point. At the dew<br />

point temperature water in the air condenses into droplets of water called “dew” and soon these droplets fall<br />

out of the sky as precipitation. Why would a low-pressure region be a good place for a volume of air to reach<br />

the dew point temperature?<br />

11.3


Name: Date:<br />

11.3 Tracking a Hurricane<br />

Hurricane Andrew (August 1992) was one of the most devastating storms of the twentieth century. Originally<br />

labelled a Category 4 storm, it was recently upgraded to a Category 5, the most severe type of hurricane.<br />

Scientists use satellite data and weather instruments dropped by aircraft to measure the storm’s intensity. As<br />

research techniques improve, weather experts can more accurately analyze data collected by these instruments.<br />

NOAA scientists have now determined that Andrew’s sustained winds reached at least 165 miles per hour. In this<br />

activity, you will track Hurricane Andrew’s treacherous journey.<br />

The storm’s beginning<br />

Hurricane Andrew was born as a result of a tropical wave which moved off the west coast of Africa and passed<br />

south of the Cape Verde Islands. On August 17, 1992, it became a tropical storm. That means it had sustained<br />

winds of 39-73 miles per hour.<br />

1. At 1200 Greenwich Mean Time (GMT) on August 17, Tropical Storm Andrew was located at<br />

12.3°N latitude and 42.0°W longitude. The wind speed was 40 miles per hour. Plot the storm’s location on<br />

your map.<br />

2. For the next four days, Tropical Storm Andrew moved uneventfully west-northwest across the Atlantic. Plot<br />

the storm’s path as it traveled toward the Caribbean Islands.<br />

Table 1:Tropical Storm Andrew’s path<br />

Date Time (GMT) Latitude (°N) Longitude (°W) Wind speed (mph)<br />

8/18/1992 1200 14.6 49.9 52<br />

8/19/1992 1200 18.0 56.9 52<br />

8/20/1992 1200 21.7 60.7 46<br />

8/21/1992 1200 24.4 64.2 58<br />

11.3


Page 2 of 4<br />

The storm intensifies<br />

Late on August 21, a deep high pressure center developed over the southeastern United States and<br />

extended eastward to an area just north of Tropical Storm Andrew. In response to this more favorable<br />

environment, the storm strengthened rapidly and turned westward. At 1200 GMT on August 22, the storm<br />

reached hurricane status, meaning it had sustained winds of at least 74 miles per hour.<br />

1. Plot Hurricane Andrew’s path over the next two days.<br />

Table 2:Hurricane Andrew’s path<br />

Date Time (GMT) Latitude (°N) Longitude (°W) Wind speed (mph)<br />

8/22/1992 1200 25.8 68.3 81<br />

8/23/1992 1200 25.4 74.2 138<br />

2. Hurricane watches are issued when hurricane conditions are possible in the area, usually within 36 hours.<br />

Hurricane warnings are issued when hurricane conditions are expected in the area within 24 hours. Look at<br />

the distance the hurricane travelled in the last 24 hours and use that information to predict where it might be<br />

in 24 hours, and in 36 hours. Name one area that you would declare under a hurricane watch, and an area that<br />

you would declare under a hurricane warning.<br />

Landfall<br />

On the evening of August 23, Hurricane Andrew first made landfall. Landfall is defined as when the center of the<br />

hurricane’s eye is over land.<br />

1. Plot the point of Hurricane Andrew’s first landfall.<br />

Table 3:Hurricane Andrew’s first landfall<br />

Date Time (GMT) Latitude (°N) Longitude (°W) Wind speed (mph)<br />

8/23/1992 2100 25.4 76.6 150<br />

2. Where did this first landfall occur?<br />

11.3


Page 3 of 4<br />

Hurricane Andrew crosses the Gulf Stream and strikes the U.S.<br />

11.3<br />

During the night of August 23, Hurricane Andrew briefly weakened as it moved over land. However,<br />

once the storm moved back over open waters, it rapidly regained strength. The warm water of the Gulf Stream<br />

increased the intensity of the hurricane’s convection cycle. At 0905 GMT on August 24, Hurricane Andrew<br />

made landfall again.<br />

1. Plot the point of Hurricane Andrew’s next landfall.<br />

2. Where did this landfall occur?<br />

The final landfall<br />

Table 4:Hurricane Andrew’s next landfall<br />

Date Time (GMT) Latitude (°N) Longitude (°W) Wind speed (mph)<br />

8/24/1992 0905 25.5 80.3 144<br />

After making its first landfall in the United States (where it caused an estimated $25 billion in damage),<br />

Hurricane Andrew moved northwest across the Gulf of Mexico. On the morning of August 26, 1992, Hurricane<br />

Andrew made its final landfall. Afterward, Andrew weakened rapidly to tropical storm strength in about 10<br />

hours, and then began to dissipate.<br />

1. Plot Andrew’s course across the Gulf of Mexico and its final landfall.<br />

Table 5:Hurricane Andrew’s next landfall<br />

Date Time (GMT) Latitude (°N) Longitude (°W) Wind speed (mph)<br />

8/24/1992 1800 25.8 83.1 133<br />

8/25/1992 1800 27.8 89.6 138<br />

8/26/1992 0830 29.6 91.5 121<br />

2. In which state did Hurricane Andrew’s final landfall occur?<br />

Hurricane information provided by National Oceanographic and Atmospheric Administration’s National<br />

Hurricane Center.


Page 4 of 4<br />

11.3


Name: Date:<br />

12.1 Structure of the Atom<br />

Atoms are made of three tiny subatomic particles: protons, neutrons, and<br />

electrons. The protons and neutrons are grouped together in the nucleus,<br />

which is at the center of the atom. The chart below compares electrons,<br />

protons, and neutrons in terms of charge and mass.<br />

The atomic number of an element is the number of protons in the nucleus<br />

of every atom of that element.<br />

Isotopes are atoms of the same element that have different numbers of<br />

neutrons. The number of protons in isotopes of an element is the same.<br />

The mass number of an isotope tells you the number of protons plus the number of neutrons.<br />

Mass number = number of protons + number of neutrons<br />

• Carbon has three isotopes: carbon-12, carbon-13, and carbon-14. The atomic number of carbon is 6.<br />

a. How many protons are in the nucleus of a carbon atom?<br />

Solution:<br />

6 protons<br />

The atomic number indicates how many protons are in the nucleus of an atom. All atoms of carbon have<br />

6 protons, no matter which isotope they are.<br />

b. How many neutrons are in the nucleus of a carbon-12 atom?<br />

Solution:<br />

the mass number - the atomic number = the number of neutrons.<br />

12 – 6 = 6<br />

6 neutrons<br />

c. How many electrons are in a neutral atom of carbon-13?<br />

Solution:<br />

6 electrons. All neutral carbon atoms have 6 protons and 6 electrons.<br />

d. How many neutrons are in the nucleus of a carbon-14 atom?<br />

Solution:<br />

the mass number - the atomic number = the number of neutrons<br />

14 – 6 = 8<br />

8 neutrons<br />

12.1


Page 2 of 2<br />

Use a periodic table of the elements to answer these questions.<br />

1. The following graphics represent the nuclei of atoms. Using a periodic table of elements, fill in the table.<br />

What the<br />

nucleus looks<br />

like<br />

What is this<br />

element?<br />

How many electrons does<br />

the neutral atom have?<br />

2. How many protons and neutrons are in the nucleus of each isotope?<br />

a. hydrogen-2 (atomic number = 1)<br />

b. scandium-45 (atomic number = 21)<br />

c. aluminum-27 (atomic number = 13)<br />

d. uranium-235 (atomic number = 92)<br />

e. carbon-12 (atomic number = 6)<br />

What is the mass<br />

number?<br />

3. Although electrons have mass, they are not considered in determining the mass number of an atom. Why?<br />

4. A hydrogen atom has one proton, two neutrons, and no electrons. Is this atom an ion? Explain your answer.<br />

5. An atom of sodium-23 (atomic number = 11) has a positive charge of +1. Given this information, how many<br />

electrons does it have? How many protons and neutrons does this atom have?<br />

12.1


Name: Date:<br />

12.1 Atoms and Isotopes<br />

You have learned that atoms contain three smaller particles called protons, neutrons, and electrons, and that the<br />

number of protons determines the type of atom. How can you figure out how many neutrons an atom contains,<br />

and whether it is neutral or has a charge? Once you know how many protons and neutrons are in an atom, you can<br />

also figure out its mass.<br />

In this skill sheet, you will learn about isotopes, which are atoms that have the same number of protons but<br />

different numbers of neutrons.<br />

What are isotopes?<br />

In addition to its atomic number, every atom can also be described by its mass number:<br />

mass number = number of protons + number of neutrons<br />

Atoms of the same element always have the same number of protons, but can have different numbers of neutrons.<br />

These different forms of the same element are called isotopes.<br />

Sometimes the mass number for an element is included in its symbol. When the<br />

symbol is written in this way, we call it isotope notation. The isotope notation for<br />

carbon-12 is shown to the right. How many neutrons does an atom of carbon-12<br />

have? To find out, simply take the mass number and subtract the atomic number:<br />

12 – 6 = 6 neutrons.<br />

Hydrogen has three isotopes as shown below.<br />

1. How many neutrons does protium have? What about deuterium and tritium?<br />

2. Use the diagram of an atom to answer the questions:<br />

a. What is the atomic number of the element?<br />

b. What is the name of the element?<br />

c. What is the mass number of the element?<br />

d. Write the isotope notation for this isotope.<br />

12.1


Page 2 of 2<br />

What is the atomic mass?<br />

If you look at a periodic table, you will notice that the atomic number increases by one whole number at a time.<br />

This is because you add one proton at a time for each element. The atomic mass however, increases by amounts<br />

greater than one. This difference is due to the neutrons in the nucleus. The value of the atomic mass reflects the<br />

abundance of the stable isotopes for an element that exist in the universe.<br />

Since silver has an atomic mass of 107.87, this means that most of the stable isotopes that exist have a mass<br />

number of 108. In other words, the most common silver isotope is “silver-108.” To figure out the most common<br />

isotope for an element, round the atomic mass to the nearest whole number.<br />

1. Look up bromine on the periodic table. What is the most common isotope of bromine?<br />

2. Look up potassium. How many neutrons does the most common isotope of potassium have?<br />

3. Look up lithium. What is its most common isotope?<br />

4. How many neutrons does the most common isotope of neon have?<br />

12.1


Name: Date:<br />

12.1 Ernest Rutherford<br />

12.1<br />

Ernest Rutherford initiated a new and radical view of the atom. He explained the mysterious<br />

phenomenon of radiation as the spontaneous disintegration of atoms. He was the first to describe the atom’s<br />

internal structure and performed the first successful nuclear reaction.<br />

Ambitious immigrants<br />

Ernest Rutherford was born<br />

in rural New Zealand on<br />

August 31, 1871. His father<br />

was a Scottish immigrant, his<br />

mother English. Both valued<br />

education and instilled a<br />

strong work ethic in their 12<br />

children. Ernest enjoyed the<br />

family farm, but was<br />

encouraged by his parents<br />

and teachers to pursue<br />

scholarships. He first received a scholarship to a<br />

secondary school, Nelson College. Then, in 1890,<br />

after twice taking the qualifying exam, he received a<br />

scholarship to Canterbury College of the University of<br />

New Zealand.<br />

Investigating radioactivity<br />

After earning three degrees in his homeland,<br />

Rutherford traveled to Cambridge, England, to pursue<br />

graduate research under the guidance of the man who<br />

discovered the electron, J. J. Thomson. Through his<br />

research with Thomson, Rutherford became interested<br />

in studying radioactivity. In 1898 he described two<br />

kinds of particles emitted from radioactive atoms,<br />

calling them alpha and beta particles. He also coined<br />

the term half-life to describe the amount of time taken<br />

for radioactivity to decrease to half its original level.<br />

An observer of transformations<br />

Rutherford accepted a professorship at McGill<br />

University in Montreal, Canada, in 1898. It was there<br />

that he proved that atoms of a radioactive element<br />

could spontaneously decay into another element by<br />

expelling a piece of the atom. This was surprising to<br />

the scientific community—the idea that atoms could<br />

change into other atoms had been scorned as alchemy.<br />

In 1908 Rutherford received the Nobel Prize in<br />

chemistry for “his investigations into the<br />

disintegration of the elements and the chemistry of<br />

radioactive substances.” He considered himself a<br />

physicist and joked that, “of all the transformations I<br />

have seen in my lifetime, the fastest was my own<br />

transformation from physicist to chemist.”<br />

Exploring atomic space<br />

Rutherford had returned to England in 1907, to<br />

Manchester University. There, he and two students<br />

bombarded gold foil with alpha particles. Most of the<br />

particles passed through the foil, but a few bounced<br />

back. They reasoned these particles must have hit<br />

denser areas of foil.<br />

Rutherford hypothesized that the atom must be mostly<br />

empty space, through which the alpha particles<br />

passed, with a tiny dense core he called the nucleus,<br />

which some of the particles hit and bounced off. From<br />

this experiment he developed a new “planetary<br />

model” of the atom. The inside of the atom,<br />

Rutherford suggested, contained electrons orbiting a<br />

small nucleus the way the planets of our solar system<br />

orbit the sun.<br />

‘Playing with marbles’<br />

In 1917, Rutherford made another discovery. He<br />

bombarded nitrogen gas with alpha particles and<br />

found that occasionally an oxygen atom was<br />

produced. He concluded that the alpha particles must<br />

have knocked a positively charged particle (which he<br />

named the proton) from the nucleus. He called this<br />

“playing with marbles” but word quickly spread that<br />

he had become the first person to split an atom.<br />

Rutherford, who was knighted in 1914 (and later<br />

elevated to the peerage, in 1931) returned to<br />

Cambridge in 1919 to head the Cavendish Laboratory<br />

where he had begun his research in radioactivity. He<br />

remained there until his death at 66 in 1937.


Page 2 of 2<br />

Reading reflection<br />

1. What are alpha and beta particles? Use your text<strong>book</strong> to find the definitions of these terms. Make a<br />

diagram of each particle; include labels in your diagram.<br />

2. The term “alchemy” refers to early pseudoscientific attempts to transform common elements into more<br />

valuable elements (such as lead into gold). For one kind of atom to become another kind of atom, which<br />

particles of the atom need to be expelled or gained?<br />

3. Make a diagram of the “planetary model” of the atom. Include the nucleus and electrons in your diagram.<br />

4. Compare and contrast Rutherford’s “planetary model” of the atom with our current understanding of an<br />

atom’s internal structure.<br />

5. Why did Rutherford say that bombarding atoms with particles was like “playing with marbles”? What<br />

subatomic particle did Rutherford discover during this phase of his work?<br />

6. Choose one of Rutherford’s discoveries and explain why it intrigues you.<br />

12.1


Name: Date:<br />

12.2 Electrons and Energy Levels<br />

Danish physicist Neils Bohr (1885–1962) proposed the concept of energy levels to explain electron behavior in<br />

atoms. While we know today that his model doesn’t explain everything about how electrons behave, it is still a<br />

useful starting point for understanding what is happening inside atoms.<br />

The number of electrons in an atom is equal to the number of protons in the nucleus. That means each element<br />

has a different number of electrons and therefore fills the energy levels to a different point. The innermost energy<br />

level is filled first, and then each additional electron occupies the lowest unfilled energy level in the atom.<br />

An atom of the element carbon has six electrons. Show how the electrons are arranged in energy levels.<br />

Solution<br />

Note: remember that atoms are actually three-dimensional, and that it is<br />

impossible to show the exact location of a particular electron within the<br />

electron cloud. Therefore, as long as the entire first energy level is filled,<br />

and four of the spaces in the second level are filled, the answer is<br />

acceptable. However, because electrons repel each other, it is standard<br />

procedure to show the electrons evenly spaced within the energy level.<br />

Answer the following questions about electrons and energy levels. Use section 12.2 of your text if needed.<br />

1. Who proposed the concept of energy levels inside atoms?<br />

2. Explain how energy levels are like a set of stairs. Where can electrons exist?<br />

3. In a stable atom, how are the energy levels filled?<br />

4. Show how the electrons of the following atoms are arranged in energy levels.<br />

a. Chlorine-17 electrons b. Oxygen-8 electrons c. Aluminum- 13 electrons d. Argon- 18 electrons<br />

e. Copper- 29 electrons f. Calcium-20 electrons<br />

12.2


Name: Date:<br />

12.2 Niels Bohr<br />

12.2<br />

Danish physicist Niels Bohr first proposed the idea that electrons exist in specific orbits around the<br />

atom’s nucleus. He showed that when an electron falls from a higher orbital to a lower one, it releases energy in<br />

the form of visible light.<br />

At home among ideas<br />

Niels Bohr was born October<br />

7, 1885, in Copenhagen,<br />

Denmark. His father was a<br />

physiology professor at the<br />

University of Copenhagen,<br />

his mother the daughter of a<br />

prominent Jewish politician<br />

and businessman. His parents<br />

often invited professors to the<br />

house for dinners and<br />

discussions.<br />

Niels and his sister and brother were invited to join<br />

this friendly exchange of ideas. (Niels and his brother<br />

also shared a passion for soccer, which they both<br />

played, and for which Harald, later a world-famous<br />

mathematician, was to win an Olympic silver medal.)<br />

Bohr entered the University of Copenhagen in 1903 to<br />

study physics. Because the university had no physics<br />

laboratory, Bohr conducted experiments in his father’s<br />

physiology lab. He graduated with a doctorate in 1911.<br />

Meeting of great minds<br />

In 1912, Bohr went to Manchester, England, to study<br />

under Ernest Rutherford, who became a lifelong<br />

friend. Rutherford had recently published his new<br />

planetary model of the atom, which explained that an<br />

atom contains a tiny dense core surrounded by<br />

orbiting electrons.<br />

Bohr began researching the orbiting electrons, hoping<br />

to describe their behavior in greater detail.<br />

Electrons and the atom’s chemistry<br />

Bohr studied the quantum ideas of Max Planck and<br />

Albert Einstein as he attempted to describe the<br />

electrons’ orbits. In 1913 he published his results. He<br />

proposed that electrons traveled only in specific orbits.<br />

The orbits were like rungs on a ladder— electrons<br />

could move up and down orbits, but did not exist in<br />

between the orbital paths.<br />

He explained that outer orbits could hold more<br />

electrons than inner orbits, and that many chemical<br />

properties of the atom were determined by the number<br />

of electrons in the outer orbit.<br />

Bohr also described how atoms emit light. He<br />

explained that an electron needs to absorb energy to<br />

jump from an inner orbit to an outer one. When the<br />

electron falls back to the inner orbit, it releases that<br />

energy in the form of visible light.<br />

An institute, then a Nobel Prize<br />

In 1916, Bohr accepted a position as professor of<br />

physics at the University of Copenhagen. The<br />

University created the Institute of Theoretical Physics<br />

that Bohr directed for the rest of his life. In 1922, he<br />

was awarded the Nobel Prize in physics for his work<br />

in atomic structure and radiation.<br />

In 1940, World War II spread across Europe and<br />

Germany occupied Denmark. Though he had been<br />

baptized a Christian, Bohr’s family history and his<br />

own anti-Nazi sentiments made life difficult.<br />

In 1943, he escaped in a fishing boat to Sweden,<br />

where he convinced the king to offer sanctuary to all<br />

Jewish refugees from Denmark. The British offered<br />

him a position in England to work with researchers on<br />

the atomic bomb. A few months later, the team went to<br />

Los Alamos, New Mexico, to continue their work.<br />

A warrior for peace<br />

Although Bohr believed the creation of the atomic<br />

bomb was necessary in the face of the Nazi threat, he<br />

was deeply concerned about its future implications.<br />

Bohr promoted disarmament efforts through the<br />

United Nations and won the first U.S. Atoms for<br />

Peace Award in 1957, the same year his son Aage.<br />

shared the Nobel Prize in physics. He died in 1962 in<br />

Copenhagen.


Page 2 of 2<br />

Reading reflection<br />

1. How did Niels Bohr’s model of the atom compare with Ernest Rutherford’s?<br />

2. Name two specific contributions Bohr made to our understanding of atomic structure.<br />

3. Make a drawing of Bohr’s model of the atom.<br />

4. In your own words describe how atoms emit light.<br />

5. Why do you think Bohr was concerned with the future implications of his work on atomic bombs?<br />

12.2


Name: Date:<br />

12.3 The Periodic Table<br />

Many science laboratories have a copy of the periodic table of the elements on display. This important<br />

chart holds an amazing amount of information. In this skill sheet, you will use a periodic table to identify<br />

information about specific elements, make calculations, and make predictions.<br />

Periodic table primer<br />

To work through this skill sheet, you will use the periodic table of the<br />

elements. The periodic table shows five basic pieces of information.<br />

Four are labeled on the graphic at right; the fifth piece of information is<br />

the location of the element in the table itself. The location shows the<br />

element group, chemical behavior, approximate atomic mass and size,<br />

and other characteristic properties.<br />

Review: Atomic number, Symbol, and Atomic Mass<br />

Use the periodic table to find the answers to the following questions. As you become more familiar with the<br />

layout of the periodic table, you’ll be able to find this information quickly.<br />

Atomic Number: Write the name of the element that corresponds to each of the following atomic numbers.<br />

1. 9 2. 18 3. 25 4. 15 5. 43<br />

6. What does the atomic number tell you about an element?<br />

Symbol and atomic mass: For each of the following, write the element name that corresponds to the symbol. In<br />

addition, write the atomic mass for each element.<br />

7. Fe 8. Cs 9. Si 10. Na 11. Bi<br />

12. What does the atomic mass tell you about an element?<br />

13. Why isn’t the atomic mass always a whole number?<br />

14. Why don’t we include the mass of an atom’s electrons in the atomic mass?<br />

12.3


Page 2 of 2<br />

Periodic Table Groups<br />

The periodic table’s vertical columns are called groups. Groups of elements have similar properties. Use the<br />

periodic table and the information found in Chapter 15 of your text to answer the following questions:<br />

15. The first group of the periodic table is known by what name?<br />

16. Name two characteristics of the elements in the first group.<br />

17. Name three members of the halogen group.<br />

18. Describe two characteristics of halogens.<br />

19. Where are the noble gases found on the periodic table?<br />

20. Why are the noble gases sometimes called the inert gases?<br />

Periodic Table Rows<br />

The rows of the periodic table correspond to the energy levels in the atom. The<br />

first energy level can accept up to two electrons. The second and third energy<br />

levels can accept up to eight electrons each. The example to the right shows how<br />

the electrons of an oxygen atom fill the energy level.<br />

Show how the electrons are arranged in energy levels in the following atoms:.<br />

21. He 22. N 23. Ne 24. Al 25. Ar<br />

Identify each of the following elements:.<br />

26. 27. 28. 29. 30.<br />

12.3


Name: Date:<br />

13.1 Dot Diagrams<br />

You have learned that atoms are composed of protons, neutrons, and electrons. The electrons occupy energy<br />

levels that surround the nucleus in the form of an “electron cloud.” The electrons that are involved in forming<br />

chemical bonds are called valence electrons. Atoms can have up to eight valence electrons. These electrons exist<br />

in the outermost region of the electron cloud, often called the “valence shell.”<br />

The most stable atoms have eight valence electrons. When an atom has eight valence electrons, it is said to have<br />

a complete octet. Atoms will gain or lose electrons in order to complete their octet. In the process of gaining or<br />

losing electrons, atoms will form chemical bonds with other atoms. One method we use to show an atom’s<br />

valence state is called a dot diagram, and you will be able to practice drawing these in the following exercise.<br />

What is a dot diagram?<br />

Dot diagrams are composed of two parts—the chemical symbol for the element and the dots surrounding the<br />

chemical symbol. Each dot represents one valence electron.<br />

• If an element, such as oxygen (O), has six valence electrons, then six dots will surround the<br />

chemical symbol as shown to the right.<br />

• Boron (B) has three valence electrons, so three dots surround the chemical symbol for boron as<br />

shown to the right.<br />

There can be up to eight dots around a symbol, depending on the number of valence electrons the atom<br />

has. The first four dots are single, and then as more dots are added, they fill in as pairs.<br />

Using a periodic table, complete the following chart. With this information, draw a dot diagram for each element<br />

in the chart. Remember, only the valence electrons are represented in the diagram, not the total number of<br />

electrons.<br />

Element Chemical<br />

symbol<br />

Potassium K<br />

Nitrogen N<br />

Carbon C<br />

Beryllium Be<br />

Neon Ne<br />

Sulfur S<br />

Total number of<br />

electrons<br />

Number of valence<br />

electrons<br />

Dot diagram<br />

13.1


Page 2 of 2<br />

Using dot diagrams to represent chemical reactivity<br />

Once you have a dot diagram for an element, you can predict how an atom will achieve a full valence shell. For<br />

instance, it is easy to see that chlorine has one empty space in its valence shell. It is likely that chlorine will try to<br />

gain one electron to fill this empty space rather than lose the remaining seven. However, potassium has a single<br />

dot or electron in its dot diagram. This diagram shows how much easier it is to lose this lone electron than to find<br />

seven to fill the seven empty spaces. When the potassium loses its electron, it becomes positively charged. When<br />

chlorine gains the electron, it becomes negatively charged. Opposite charges attract, and this attraction draws the<br />

atoms together to form what is termed an ionic bond, a bond between two charged atoms or ions.<br />

Because chlorine needs one electron, and potassium needs to lose one electron, these two elements can achieve a<br />

complete set of eight valence electrons by forming a chemical bond. We can use dot diagrams to represent the<br />

chemical bond between chlorine and potassium as shown above.<br />

For magnesium and chlorine, however, the situation is a bit different. By examining the electron or Lewis dot<br />

diagrams for these atoms, we see why magnesium requires two atoms of chlorine to produce the compound,<br />

magnesium chloride, when these two elements chemically combine.<br />

Magnesium can easily donate one of its valence electrons to the chlorine to fill chlorine’s valence shell, but this<br />

still leaves magnesium unstable; it still has one lone electron in its valence shell. However, if it donates that<br />

electron to another chlorine atom, the second chlorine atom has a full shell, and now so does the magnesium.<br />

The chemical formula for potassium chloride is KCl. This means that one unit of the compound is made of one<br />

potassium atom and one chlorine atom.<br />

The formula for magnesium chloride is MgCl2. This means that one unit of the compound is made of one<br />

magnesium atom and two chlorine atoms.<br />

Now try using dot diagrams to predict chemical formulas. Fill in the table below:<br />

Elements Dot diagram<br />

for each element<br />

Na and F<br />

Br and Br<br />

Mg and O<br />

Dot diagram<br />

for compound formed<br />

Chemical<br />

formula<br />

13.1


Name: Date:<br />

13.2 Finding the Least Common Multiple<br />

Knowing how to find the least common multiple of two or more numbers is helpful in physical science classes.<br />

You need to find the least common multiple in order to add fractions with different denominators, or to predict<br />

the chemical formula of many common compounds.<br />

The least common multiple is the smallest multiple of two or more whole numbers. To find the least common<br />

multiple of 3 and 4, simply list the multiples of each number:<br />

multiples of 3: 3, 6, 9, 12, 15...<br />

multiples of 4: 4, 8, 12, 16, 20...<br />

Then, look for the smallest multiple that occurs in both lists. In this case, the least common multiple is 12.<br />

Sometimes it’s a little trickier to find the least common multiple. Suppose you are asked to find the least common<br />

multiple of 15 and 36. Rather than making a long list of multiples, you can use the prime factorization method.<br />

First, factor each number into primes (remember that prime numbers are numbers that can’t be divided evenly by<br />

any whole number except one).<br />

Prime factorization of 15: 3 × 5<br />

Prime factorization of 36: 3 × 3 × 2 × 2<br />

Next, create a Venn diagram. Show the factors unique to each number in the separate parts of the circles and the<br />

factors common to both in the overlapping circles. Since 15 and 36 each have one 3, put one 3 in the middle.<br />

Finally, multiply all the factors in your diagram from left to right:<br />

5 × 3 × 3 × 2 × 2 = 180. The least common multiple of 15 and 36 is 180.<br />

13.2


Page 2 of 2<br />

13.2<br />

Important note: If the two numbers each have more than one copy of a certain prime factor, place the<br />

factor in the overlapping circles as many times as necessary. To find the least common multiple of 60 and 72:<br />

Prime factorization of 60: 2 × 2 × 3 × 5<br />

Prime factorization of 72: 2 × 2 × 3 × 2 × 3<br />

Notice that 2 appears twice in the overlapping circles because 60 and 72 have two 2’s apiece.<br />

5 × 3 × 2 × 2 × 2 × 3 = 360. The least common multiple of 60 and 72 is 360.<br />

Find the least common multiple of each of the following pairs of numbers:<br />

1. 3 and 7<br />

2. 6 and 8<br />

3. 9 and 15<br />

4. 10 and 25<br />

5. 16 and 40<br />

6. 21 and 49<br />

7. 36 and 54<br />

8. 45 and 63<br />

9. 55 and 80<br />

10. 64 and 96


Name: Date:<br />

13.2 Chemical Formulas<br />

Compounds have unique names that we use to identify them when we study chemical properties and changes.<br />

Chemists have devised a shorthand way of representing chemical names that provides important information<br />

about the substance. This shorthand representation for a compound’s name is called a chemical formula. You will<br />

practice writing chemical formulas in the following activity.<br />

What is a chemical formula?<br />

Chemical formulas have two important parts: chemical symbols for the elements in the compound and subscripts<br />

that tell how many atoms of each element are needed to form the compound. The chemical formula for water,<br />

H 2O, tells us that a water molecule is made of the elements hydrogen (H) and oxygen (O) and that it takes two<br />

atoms of hydrogen and one atom of oxygen to build the molecule. For sodium nitrate, NaNO 3, the chemical<br />

formula tells us there are three elements in the compound: sodium (Na), nitrogen (N), and oxygen (O). To make a<br />

molecule of this compound, you need one atom of sodium, one atom of nitrogen, and three atoms of oxygen.<br />

How to write chemical formulas<br />

How do chemists know how many atoms of each element are needed to build a molecule? For ionic compounds,<br />

oxidation numbers are the key. An element’s oxidation number is the number of electrons it will gain or lose in a<br />

chemical reaction. We can use the periodic table to find the oxidation number for an element. When we add up<br />

the oxidation numbers of the elements in an ionic compound, the sum must be zero. Therefore, we need to find a<br />

balance of negative and positive ions in the compound for the molecule to form.<br />

Example 1:<br />

A compound is formed by the reaction between magnesium and chlorine. What is the chemical formula for this<br />

compound?<br />

From the periodic table, we find that the oxidation number of magnesium is 2+. Magnesium loses 2 electrons in<br />

chemical reactions. The oxidation number for chlorine is 1-. Chlorine tends to gain one electron in a chemical<br />

reaction.<br />

Remember that the sum of the oxidation numbers of the elements in a molecule will equal zero. This compound<br />

requires one atom of magnesium with an oxidation number of 2+ to combine with two atoms of chlorine, each<br />

with an oxidation number of 1–, for the sum of the oxidation numbers to be zero.<br />

(2 + ) + 2(1 − ) =<br />

0<br />

To write the chemical formula for this compound, first write the chemical symbol for the positive ion (Mg) and<br />

then the chemical symbol for the negative ion (Cl). Next, use subscripts to show how many atoms of each<br />

element are required to form the molecule. When one atom of an element is required, no subscript is used.<br />

Therefore, the correct chemical formula for magnesium chloride is MgCl 2.<br />

13.2


Page 2 of 2<br />

Example 2:<br />

Aluminum and bromine combine to form a compound. What is the chemical formula for the compound they<br />

form?<br />

From the periodic table, we find that the oxidation number for aluminum (Al) is 3+. The oxidation number for<br />

bromine (Br) is 1–. In order for the oxidation numbers of this compound to add up to zero, one atom of aluminum<br />

must combine with three atoms of bromine:<br />

The correct chemical formula for this compound, aluminum bromide, is AlBr 3.<br />

Practice writing chemical formulas for ionic compounds<br />

Use the periodic table to find the oxidation numbers of each element. Then write the correct chemical formula for<br />

the compound formed by the following elements:<br />

Element Oxidation<br />

Number<br />

Potassium (K) Chlorine (Cl)<br />

Calcium (Ca) Chlorine (Cl)<br />

Sodium (Na) Oxygen (O)<br />

Boron (B) Phosphorus (P)<br />

Lithium (Li) Sulfur (S)<br />

Aluminum (Al) Oxygen (O)<br />

Beryllium (Be) Iodine (I)<br />

Calcium (Ca) Nitrogen (N)<br />

Sodium (Na) Bromine (Br)<br />

(3 + ) + 3(1 − ) =<br />

0<br />

Element Oxidation<br />

Number<br />

Chemical Formula for<br />

Compound<br />

13.2


Name: Date:<br />

13.2 Naming Compounds<br />

Compounds have unique names that identify them for us when we study chemical properties and changes.<br />

Predicting the name of a compound is fairly easy provided certain rules are kept in mind. In this skill sheet, you<br />

will practice naming a variety of chemical compounds.<br />

Chemical Formulas and Compound Names<br />

Chemical formulas tell a great deal of information about a compound—the types of elements forming the<br />

compound, the numbers of atoms of each element in one molecule, and even some indication, perhaps, of the<br />

arrangement of the atoms when they form the molecule.<br />

In addition to having a unique chemical formula, each compound has a unique name. These names provide<br />

scientists with valuable information. Just like chemical formulas, chemical names tell which elements form the<br />

compound. However, the names may also identify a “family” or group to which the compound belongs. It is<br />

useful for scientists, therefore, to recognize and understand both a compound’s formula and its name.<br />

Naming Ionic Compounds<br />

Naming ionic compounds is relatively simple, especially if the compound is formed only from monoatomic ions.<br />

Follow these steps:<br />

1. Write the name of the first element or the positive ion of the compound.<br />

2. Write the root of the second element or negative ion of the compound.<br />

3. For example, write fluor- to represent fluorine, chlor- to represent chlorine.<br />

4. Replace the ending of the negative ion's name with the suffix -ide.<br />

5. Fluorine → Fluoride; Chlorine → Chloride<br />

A compound containing potassium (K 1+ ) and iodine (I 1– ) would be named potassium iodide.<br />

Lithium (Li 1+ ) combined with sulfur (S 2– ) would be named lithium sulfide.<br />

Naming Compounds with Polyatomic Ions<br />

Naming compounds that contain polyatomic ions is even easier. Just follow these two steps:<br />

1. Write the name of the positive ion first. Use the periodic table or an ion chart to find the name.<br />

2. Write the name of the negative ion second. Again, use the periodic table or an ion chart to find the name.<br />

A compound containing aluminum (Al 1+ ) and sulfate (SO 4 2– ) would be called aluminum sulfate.<br />

A compound containing magnesium (Mg 2+ ) and carbonate (CO 3 2– ) would be called magnesium carbonate.<br />

13.2


Page 2 of 2<br />

13.2<br />

Predict the name of the compound formed from the reaction between the following elements and/or<br />

polyatomic ions. Use the periodic table and the polyatomic ion chart in section 13.2 of your student text to help<br />

you name the ions.<br />

Combination Compound Name<br />

Al + Br<br />

Be + O<br />

K+N<br />

Ba + CrO 4 2–<br />

Cs + F<br />

NH 3 1+ +S<br />

Mg + Cl<br />

B+I<br />

Na + SO 4 2–<br />

Si + C 2H 3O 2 1–


Name: Date:<br />

13.2 Families of Compounds<br />

Certain compounds have common characteristics, so we place them into groups or families. The group called<br />

“enzymes” contains thousands of representative chemicals, but all share certain critical features that allow them<br />

to be placed into this group.<br />

The name of a compound often identifies the family of chemical to which it belongs. The clue is usually found in<br />

the suffix for the compound's name. The table below lists suffixes for some common chemical families.<br />

Chemical Family Suffix<br />

Sugars -ose<br />

Alcohols -ol<br />

Enzymes -ase<br />

Ketones -one<br />

Organic acids -oic or -ic acid<br />

Alkanes -ane<br />

Glucose, the compound used by your brain as its primary fuel, is a sugar. The suffix -ose indicates its<br />

membership in the sugar family. Propane, the compound used to operate your gas barbecue grill, is an alkane, a<br />

compound formed from carbon and hydrogen atoms that are covalently bonded with single pairs of electrons. We<br />

know this from the suffix -ane.<br />

Knowing such information about a compound can be very useful when you are reading the labels of consumer<br />

products. Compound names can be found in the ingredients list on the label. If you are purchasing a hand lotion<br />

to alleviate dry skin, you should avoid one that lists a compound with an -ol suffix early in the ingredients list.<br />

The ingredients are listed from largest amount to smallest amount. The earlier a compound is listed, the greater<br />

the amount of that compound in the product. A compound with an -ol suffix is an alcohol. Hand lotions with high<br />

percentages of alcohols are less effective since alcohols tend to dry out rather than moisturize the skin!<br />

In later chemistry courses, you will learn more about the names and characteristics of “families” of compounds.<br />

This knowledge will provide you with a powerful tool for making informed consumer decisions.<br />

13.2


Page 2 of 2<br />

Using the information in the table on the previous page to predict the chemical family to which the<br />

following compounds are members:<br />

Compound Name Chemical Family<br />

Lipase<br />

Methanol<br />

Formic Acid<br />

Butane<br />

Sucrose<br />

Acetone<br />

Acetic Acid<br />

13.2


Name: Date:<br />

14.1 Chemical Equations<br />

Chemical symbols provide us with a shorthand method of writing the name of an element. Chemical formulas do<br />

the same for compounds. But what about chemical reactions? To write out, in words, the process of a chemical<br />

change would be long and tedious. Is there a shorthand method of writing a chemical reaction so that all the<br />

information is presented correctly and is understood by all scientists? Yes! This is the function of chemical<br />

equations. You will practice writing and balancing chemical equations in this skill sheet.<br />

What are chemical equations?<br />

Chemical equations show what is happening in a chemical reaction. They provide you with the identities of the<br />

reactants (substances entering the reaction) and the products (substances formed by the reaction). They also tell<br />

you how much of each substance is involved in the reaction. Chemical equations use symbols for elements and<br />

formulas for compounds. The reactants are written to the left of the arrow. Products go on the right side of the<br />

arrow.<br />

The arrow should be read as “yields” or “produces.” This equation, therefore, says that hydrogen gas (H 2) plus<br />

oxygen gas (O 2) yields or produces the compound water (H 2O).<br />

Write chemical equations for the following reactions:<br />

Reactants Products Unbalanced<br />

Hydrochloric acid<br />

HCl<br />

and<br />

Sodium hydroxide<br />

NaOH<br />

Calcium carbonate<br />

CaCO 3<br />

and<br />

Potassium iodide<br />

KI<br />

Aluminum fluoride<br />

AlF 3<br />

and<br />

Magnesium nitrate<br />

Mg(NO 3 ) 2<br />

Water<br />

H 2 O<br />

and<br />

Sodium chloride<br />

NaCl<br />

H2 + O2 →<br />

H2O Potassium carbonate<br />

K 2 CO 3<br />

and<br />

Calcium iodide<br />

CaI 2<br />

Aluminum nitrate<br />

Al(NO 3 ) 3<br />

and<br />

Magnesium fluoride<br />

MgF 2<br />

Chemical Equation<br />

14.1


Page 2 of 3<br />

Conservation of atoms<br />

Take another look at the chemical equation for making water:<br />

Did you notice that something has been added?<br />

The large number in front of H 2 tells how many molecules of H 2 are required for the reaction to proceed. The<br />

large number in front of H 2O tells how many molecules of water are formed by the reaction. These numbers are<br />

called coefficients. Using coefficients, we can balance chemical equations so that the equation demonstrates<br />

conservation of atoms. The law of conservation of atoms says that no atoms are lost or gained in a chemical<br />

reaction. The same types and numbers of atoms must be found in the reactants and the products of a chemical<br />

reaction.<br />

Coefficients are placed before the chemical symbol for single elements and before the chemical formula of<br />

compounds to show how many atoms or molecules of each substance are participating in the chemical reaction.<br />

When counting atoms to balance an equation, remember that the coefficient applies to all atoms within the<br />

chemical formula for a compound. For example, 5CH 4 means that 5 atoms of carbon and 20 atoms (5 × 4) of<br />

hydrogen are contributed to the chemical reaction by the compound methane.<br />

Balancing chemical equations<br />

2H2 + O2 → 2H2O To write a chemical equation correctly, first write the equation using the correct chemical symbols or formulas<br />

for the reactants and products.<br />

The displacement reaction between sodium chloride and iodine to form sodium iodide and chlorine gas is written<br />

as:<br />

NaCl + I2 →<br />

NaI + Cl2 Next, count the number of atoms of each element present on the reactant and product side of the chemical<br />

equation:<br />

Reactant Side of Equation Element Product Side of Equation<br />

1 Na 1<br />

1 Cl 2<br />

2 I 1<br />

For the chemical equation to be balanced, the numbers of atoms of each element must be the same on either side<br />

of the reaction. This is clearly not the case with the equation above. We need coefficients to balance the equation.<br />

14.1


Page 3 of 3<br />

First, choose one element to balance. Let’s start by balancing chlorine. Since there are two atoms of<br />

chlorine on the product side and only one on the reactant side, we need to place a “2” in front of the<br />

substance containing the chlorine, the NaCl.<br />

This now gives us two atoms of chlorine on both the reactant and product sides of the equation. However, it also<br />

give us two atoms of sodium on the reactant side! This is fine—often balancing one element will temporarily<br />

unbalance another. By the end of the process, however, all elements will be balanced.<br />

We now have the choice of balancing either the iodine or the sodium. Let's balance the iodine. (It doesn’t matter<br />

which element we choose.)<br />

There are two atoms of iodine on the reactant side of the equation and only one on the product side. Placing a<br />

coefficient of “2” in front of the substance containing iodine on the product side:<br />

There are now two atoms of iodine on either side of the equation, and at the same time we balanced the number<br />

of sodium atoms!<br />

In this chemical reaction, two molecules of sodium chloride react with one molecule of iodine to produce two<br />

molecules of sodium iodide and one molecule of chlorine. Our equation is balanced.<br />

Balance the following equations using the appropriate coefficients. Remember that balancing one element may<br />

temporarily unbalance another. You will have to correct the imbalance in the final equation. Check your work by<br />

counting the total number of atoms of each element—the numbers should be equal on the reactant and product<br />

sides of the equation. Remember, the equations cannot be balanced by changing subscript numbers!<br />

1. Al + O2 → Al2O3 2. CO + H2 → H2O + CH4 3. HgO → Hg + O2 4. CaCO3 → CaO + CO2 5. C + Fe2O3 → Fe + CO2 6. N2 + H2 → NH3 7. K + H2O → KOH + H2 8. P + O2 → P2O5 9. Ba(OH) 2 + H2SO4 → H2O + BaSO4 10. CaF2 + H2SO4 → CaSO4 + HF<br />

11. KClO3 → KClO4 + KCl<br />

2NaCl+ I2 → NaI + Cl2 2NaCl+ I2 →<br />

2NaI + Cl2 14.1


Name: Date:<br />

14.1 The Avogadro Number<br />

Atoms are so small that you could fit millions of them on the head of a pin. As you have learned, the masses of<br />

atoms and molecules are measured in atomic mass units. Working with atomic mass units in the laboratory is<br />

very difficult because each atomic mass unit has a mass of 1 / 12 the mass of one carbon atom.<br />

In order to make atomic mass units more useful, it would be convenient to relate the value of one atomic mass<br />

unit to one gram. One gram is an amount of matter we can actually see. For example, the mass of one paper clip<br />

is about 2.5 grams. The Avogadro number, 6.02 × 10 23 , allows us to convert atomic mass units to grams.<br />

What is a mole?<br />

In chemistry, the term “mole” does not refer to a furry animal that lives underground. In chemistry, a mole is<br />

quantity of something and is used just like we use the term “dozen.” One dozen is equal to 12. One mole is equal<br />

to 6.02 × 10 23 , or the Avogadro number. If you have a dozen oranges, you have 12. If you have a mole of<br />

oranges, you have 6.02 × 10 23 . This would be enough oranges to cover the entire surface of Earth seven feet deep<br />

in oranges!<br />

Could you work with only a dozen atoms in the laboratory? You cannot see 12 atoms without the aid of a very<br />

powerful microscope. A mole of atoms would be much easier to work with in the laboratory because the mass of<br />

one mole of atoms can be measured in grams. Moles allow us to convert atomic mass units to grams. This<br />

relationship is illustrated below:<br />

1 carbon atom = 12.0 amu<br />

To calculate the mass of one mole of any substance (the molar mass), you use the periodic table to find the<br />

atomic mass (not the mass number) for the element or for the elements that create the compound. You then<br />

express this value in grams.<br />

Substance Elements in<br />

substance<br />

Atomic<br />

mass of<br />

element<br />

(amu)<br />

No. of atoms of<br />

each element<br />

Formula mass<br />

(amu)<br />

Molar mass<br />

(g)<br />

Na Na 22.99 1 22.99 22.99<br />

U U 238.03 1 238.03 238.03<br />

H2O H<br />

1.01<br />

2<br />

18.02 18.02<br />

O<br />

16.00<br />

1<br />

CaCO3 Ca 40.08<br />

1<br />

100.09 100.09<br />

C<br />

12.01<br />

1<br />

O<br />

16.00<br />

3<br />

Al(NO3) 3<br />

213.01 213.01<br />

Al<br />

N<br />

O<br />

1 mole of carbon atoms 6.02 10 23 = × atoms =<br />

12.0 grams<br />

26.98<br />

14.01<br />

16.00<br />

1<br />

3<br />

9<br />

14.1


Page 2 of 4<br />

For the following elements and compounds, complete the following table to calculate the mass of one<br />

mole of the substance:<br />

Substance Elements in<br />

substance<br />

Sr<br />

Ne<br />

Ca(OH) 2<br />

NaCl<br />

O 3<br />

C 6H 12O<br />

Atomic<br />

mass of<br />

element<br />

(amu)<br />

The molar mass of a substance can be used to calculate the number of particles (atoms or molecules) present in<br />

any given mass of a substance. You can determine the number of particles present by using the Avogadro<br />

number.<br />

Using the Avogadro number<br />

No. of atoms of<br />

each element<br />

The Avogadro number states that for one mole of any substance, whether element or compound, there are<br />

6.02 × 10 23 particles present in the sample. Those particles are atoms if the substance is an element and<br />

molecules if the substance is a compound. If we look again at our previous examples we see that every substance<br />

has a different molar mass:<br />

14.1<br />

Formula mass (amu) Molar mass<br />

(g)


Page 3 of 4<br />

Substance Elements in<br />

substance<br />

Atomic<br />

mass of<br />

element<br />

(amu)<br />

No. of atoms<br />

of each<br />

element<br />

However, one mole of each of these substances contains exactly the same number of fundamental particles,<br />

6.02 × 10 23 . The difference is that each of these fundamental particles, atoms, and molecules, has a different<br />

mass based on its composition (number of protons and neutrons, numbers and types of atoms). Therefore, the<br />

number of particles in one mole of any substance is identical; however, the mass of one mole of substances varies<br />

based on the formula mass for that substance.<br />

When a substance’s mass is reported in grams and you need to find the number of particles present in the sample,<br />

you must first convert the mass in grams to the mass in moles. By using proportions and ratios, you can easily<br />

calculate the molar mass of any given amount of substance.<br />

How many molecules are in a sample of NaCl that has a mass of 38.9 grams?<br />

First, determine the molar mass of NaCl:<br />

Next, determine how many particles are in 38.9 g of NaCl:<br />

Formula mass<br />

(amu)<br />

Molar mass<br />

(g)<br />

Na Na 22.99 1 22.99 22.99<br />

U U 238.03 1 238.03 238.03<br />

H2O H<br />

1.01<br />

2<br />

18.02 18.02<br />

O<br />

16.00<br />

1<br />

CaCO3 Ca 40.08<br />

1<br />

100.09 100.09<br />

C<br />

12.01<br />

1<br />

O<br />

16.00<br />

3<br />

Al(NO3) 3 Al<br />

26.98<br />

1<br />

313.1 313.1<br />

N<br />

14.01<br />

3<br />

O<br />

16.00<br />

9<br />

Element Atomic mass (amu) No. of atoms Molar mass (g)<br />

Sodium (Na) 22.99 1 22.99<br />

Chlorine (Cl) 35.45 1 35.45<br />

Molar mass of NaCl 58.44 g<br />

We know that 58.44 g of NaCl contains 6.02 × 10 23 molecules of NaCl. Therefore, we can set up a proportion to<br />

determine the number of molecules in 38.9 g of NaCl:<br />

58.44 g NaCl<br />

6.02 10 23<br />

------------------------------- =<br />

×<br />

38.9 g NaCl<br />

---------------------------x<br />

Solving for x using cross-multiplication gives us a value of 4.0 × 10 23 molecules of NaCl.<br />

14.1


Page 4 of 4<br />

14.1<br />

Complete the following table by determining the molar mass of each listed substance and either<br />

providing the number of particles in the given mass of sample or the mass of the sample that contains the given<br />

number of particles.<br />

Substance Molar Mass<br />

(g)<br />

MgCO 3<br />

Mass of Sample<br />

(g)<br />

12.75<br />

Number of<br />

Particles Present<br />

H 2O 296 × 10 50<br />

N 2<br />

Yb 0.00038<br />

Al 2(SO 3) 3<br />

K 2CrO 4<br />

4657<br />

7.1 × 10 8<br />

0.23 × 10 19


Name: Date:<br />

14.1 Formula Mass<br />

A chemical formula gives you useful information about a compound. First, it tells you which types of atoms and<br />

how many of each are present. Second, it lets you know which types of ions are present in a compound. Finally,<br />

it allows you to determine the mass of one molecule of a compound, relative to the mass of other compounds. We<br />

call this the formula mass. This skill sheet will show you how to calculate the formula mass of a compound.<br />

Calculating formula mass: a step-by-step approach<br />

A common ingredient in toothpaste is a compound called sodium phosphate. If you examine a tube of toothpaste,<br />

you will find that it is usually listed as trisodium phosphate.<br />

• What is the formula mass of sodium phosphate?<br />

Step 1: Determine the formulas and oxidation numbers of the ions in the compound.<br />

Sodium phosphate is made up of the sodium ion and the phosphate ion. The oxidation number for the sodium ion<br />

can be determined from the periodic table. Since sodium, Na, is located in group 1 of the periodic table, it has an<br />

oxidation number of 1+ like all of the elements in group 1.<br />

The chemical formula and oxidation number for sodium is: Na +<br />

To find the formula and oxidation number for the phosphate ion, use the ion chart in Chapter 16 of your text<strong>book</strong>.<br />

The chemical formula and oxidation number for the phosphate ion is: PO 4 3-<br />

Step 2: Write the chemical formula of the compound.<br />

Remember that compounds must be neutral that is, the oxidation numbers of the elements and ions must be equal<br />

to zero. Since sodium = Na + and phosphate = PO 4 3- how many of each do you need to make a neutral<br />

compound? You need three sodium ions for each phosphate ion to make a neutral compound.<br />

The chemical formula of sodium phosphate is: Na 3 PO 4 .<br />

Step 3: List the type of atom, quantity, atomic mass, and total mass of each atom.<br />

Atom Quantity Atomic mass<br />

(from the periodic table)<br />

Step 4: Add up the values and calculate the formula mass of the compound.<br />

68.97 amu + 30.97 amu + 64.00 amu = 163.94 amu<br />

The formula mass of sodium phosphate is 163.94 amu<br />

Total mass<br />

(number × atomic mass)<br />

Na 3 22.99 amu 3 × 22.99 = 68.97 amu<br />

P 1 30.97 amu 1 × 30.97 = 30.97 amu<br />

O 4 16.00 amu 4 × 16.00 = 64.00 amu<br />

14.1


Page 2 of 2<br />

Now try one on your own:<br />

Eggshells are made mostly of a brittle compound called calcium phosphate. What is the formula mass of this<br />

compound?<br />

1. Write the chemical formula and oxidation number of each ion in the compound:<br />

First ion: Second ion:<br />

2. Write the chemical formula of the compound:<br />

3. List the type of atom, quantity, atomic mass, and total mass of each atom.<br />

Atom Quantity Atomic mass<br />

(from the periodic table)<br />

4. Add up the values to calculate the formula mass of the compound.<br />

Write the chemical formula and the formula mass for each of the compounds below. Use separate paper and show<br />

all of your work.<br />

1. barium chloride<br />

2. sodium hydrogen carbonate<br />

3. magnesium hydroxide<br />

4. ammonium nitrate<br />

5. strontium phosphate<br />

Total mass<br />

(number × atomic mass)<br />

14.1


Name: Date:<br />

14.2 Classifying Reactions<br />

Chemical reactions may be classified into different groups according to the reactants and products. The five<br />

major groups of chemical reactions are summarized below.<br />

Synthesis reactions - when two or more substances combine to form a new compound.<br />

• General equation: A + B → AB<br />

• Example: When rust forms, iron reacts with oxygen to form iron oxide (rust).<br />

4Fe (s) + 3O 2 (g) →2 Fe 2O 3 (s)<br />

Decomposition reactions - when a single compound is broken down to produce two or more smaller<br />

compounds.<br />

• General equation: AB → A + B<br />

• Example: Water can be broken down into hydrogen and oxygen gases.<br />

2H 2O (l) → 2H 2 (g) + O 2 (g)<br />

Single displacement reactions - when one element replaces a similar element in a compound.<br />

• General equation: A + BX → AX + B<br />

• Example: When iron is added to a solution of copper chloride, iron replaces copper in the solution and<br />

copper falls out of the solution.<br />

Fe (s) + CuCl 2 (aq) → Cu (s) + FeCl 2 (aq)<br />

Double displacement reactions - when ions from two compounds in solution exchange places to produce two<br />

new compounds.<br />

• General equation: AX + BY → AY + BX<br />

• Example: When carbon dioxide gas is bubbled into lime water, a precipitate of calcium carbonate is formed<br />

along with water.<br />

CO 2 (g) + CaO 2H 2 (aq) → CaCO 3 (s) + H 2O (l)<br />

Combustion reactions - when a carbon compound reacts with oxygen gas to produce carbon dioxide and water<br />

vapor. Energy is released from the reaction.<br />

• General equation: Carbon Compound + O2 → CO2 + H2O + energy<br />

• Example: The combustion of methane gas.<br />

CH 4 (g) + 2O 2 → CO 2 (g) + 2H 2O (g)<br />

Classify the following reaction as synthesis, decomposition, single displacement, double displacement, or<br />

combustion. Explain your answer.<br />

Mg (s) + CuSO 4 (s) → MgSO 4 (aq) + Cu (s)<br />

Answer: Displacement. Magnesium replaces copper in the compound.<br />

14.2


Page 2 of 2<br />

Classify the reactions below as synthesis, decomposition, single displacement, double<br />

displacement, or combustion. Explain your answers.<br />

1. CO 2 (g) + H 2O (l) → H 2CO 3 (aq)<br />

2. Cl 2 (g) + 2KI (aq) → 2KCl (aq) + I 2 (g)<br />

3. H 2O 2 (l) → H 2O (l) + O 2 (g)<br />

4. MnSO 4 (s) → MnO (s) + SO 3 (g)<br />

5. C 6H 12O 6 (s) + 6O 2 (g) → 6CO 2 (g) + 6H 2O (g)<br />

6. CaCl 2 (aq) + 2AgNO 3 (aq) → Ca(NO 3) 2 (aq) + 2AgCl (s)<br />

7. 2NaCl (aq) + CuSO 4 (aq) → Na 2SO 4 (aq) + CuCl 2 (s)<br />

8. CaCl 2 (aq) + 2Na (s) → Ca (s) + 2NaCl (aq)<br />

9. CaCO 3 (s) → CaO (s) + CO 2 (g)<br />

10. C 3H 8 (g) + 5O 2 (g) → 3CO 2 (g) + 4H 2O (g)<br />

Answer the following questions.<br />

11. You mix two clear solutions. Instantly, you see a bright yellow precipitate form. What type of reaction did<br />

you just observe? Explain your answer.<br />

12. What type of reaction occurs when you strike a match?<br />

13. Solid sodium reacts violently with chlorine gas. The product formed in the reaction is sodium chloride, also<br />

known as table salt. What type of reaction is this? Explain your answer.<br />

14. Hydrogen-powered cars burn hydrogen gas to produce water and energy. The reaction is:<br />

2H 2 (g) + O 2 (g) → 2H 2O (g) + Energy<br />

While this reaction can be classified as a synthesis reaction, it is sometimes referred to as combustion. What<br />

characteristics does this reaction share with other combustion reactions? How is it different?<br />

14.2


Name: Date:<br />

14.2 Predicting Chemical Equations<br />

Chemical reactions cause chemical changes. Elements and compounds enter into a reaction, and new substances<br />

are formed as a result. Often, we know the types of substances that entered the reaction and can tell what types of<br />

substance(s) were formed. Sometimes, though, it might be helpful if we could predict the products of the<br />

chemical reaction—know in advance what would be formed and how much of it would be produced.<br />

For certain chemical reactions, this is possible, using our knowledge of oxidation numbers, types of chemical<br />

reactions, and how equations are balanced. In this skill sheet, you will practice writing a complete balanced<br />

equation for chemical reactions when only the identities of the reactants are known.<br />

Review: Chemical equations<br />

Recall that chemical equations show the process of a chemical reaction. The equation reads from left to right with<br />

the reactants separated from the products by an arrow that indicates “yields” or “produces.”<br />

In the chemical equation:<br />

Two atoms of lithium combine with one molecule of barium chloride to yield two molecules of lithium chloride<br />

and one atom of barium. The equation fully describes the chemical change for this reaction.<br />

For reactions such as the one above, a single displacement reaction, we are often able to predict the products in<br />

advance and write a completely balanced equation for the chemical change. Here are the steps involved:<br />

1. Predict the replacements for the reaction.<br />

In single displacement reactions, one element is replaced by a similar element in a compound. The pattern for<br />

this replacement is easily predictable: if the element doing the replacing forms a positive ion, it replaces the<br />

element in the compound that forms a positive ion. If the substance doing the replacing forms a negative ion, it<br />

replaces the element in the compound that forms a negative ion.<br />

For the reaction described above, we could predict that the lithium would replace the barium in the compound<br />

barium chloride since both lithium and barium have positive oxidation numbers. The resulting product would<br />

pair lithium (1+) and chlorine (1-): the positive/negative combination required for ionic compounds.<br />

2. Determine the chemical formula for the products.<br />

Once you have determined which elements will be swapped to form the products, you can use oxidation numbers<br />

and the fact that the sum of the oxidation numbers for an ionic compound must equal zero in order to determine<br />

the chemical formula for the reaction products.<br />

3. Balance the chemical equation<br />

2Li + BaCl2 →<br />

2LiCl + Ba<br />

Once you have determined the nature and formulas of the products for a chemical reaction, the final step is to<br />

write a balanced equation for the reaction.<br />

14.2


Page 2 of 2<br />

• If beryllium (Be) combines with potassium iodide (KI) in a chemical reaction, what are the<br />

products?<br />

Solution:<br />

First, we decide which element of KI will be replaced by the beryllium. Since beryllium has an oxidation<br />

number of 2+, it replaces the element in KI that also has a positive oxidation number—the potassium (K 1+ ).<br />

It will therefore combine with the iodine to form a new compound.<br />

Because beryllium has an oxidation number of 2+ and iodine's oxidation number is 1–, it is necessary for<br />

two atoms of iodine to combine with one atom of beryllium to form an electrically neutral compound. The<br />

resulting chemical formula for beryllium iodide is BeI2. In single-displacement reactions, the component of the compound that has been replaced by the uncombined<br />

reactant now stands alone and uncombined. The resulting products of this chemical reaction, therefore, are<br />

BeI2 and K. Balancing the equation give us:<br />

Predict replacements<br />

Be + 2KI → BeI 2 + 2K<br />

1. If Na 1+ were to combine with CaCl 2, what component of CaCl 2 would be replaced by the Na 1+ ?<br />

2. If Fe 2+ were to combine with K 2Br, what component of K 2Br would be replaced by the Fe 2+ ?<br />

3. If Mg 2+ were to combine with AlCl 3, what component of AlCl 3 would be replaced by the Mg 2+ ?<br />

Predict product formulas<br />

For the following combinations of reactants, predict the formulas of the products:<br />

4. Li + AlCl 3<br />

5. K + CaO<br />

6. F 2 + KI<br />

Predicting chemical equations for displacement reactions<br />

Write complete balanced equations for the following combinations of reactants.<br />

7. Ca and K 2S<br />

8. Mg and Fe 2O 3<br />

9. Li and NaCl<br />

14.2


Name: Date:<br />

14.2 Percent Yield<br />

You can predict the amount of product to expect from a reaction if you know how much reactant you started with.<br />

For example, if you start out with one mole of limiting reactant, you can expect to produce one mole of product.<br />

In real-world chemical reactions, the actual amount of product is usually less than the predicted amount. This is<br />

due to experimental error and other factors (such as the fact that some product is difficult to collect and measure).<br />

The amount of product you expect to produce is called the predicted yield. The amount of product that you are<br />

able to measure after the reaction is called the actual yield. The percent yield is the actual yield divided by the<br />

predicted yield and then multiplied by 100.<br />

Actual yield<br />

Percent yield =<br />

----------------------------------- × 100<br />

Predicted yield<br />

The percent yield can provide information about how carefully the experiment was performed. If a percent yield<br />

is low, chemical engineers look for sources of error. Manufacturers of chemical products try to maximize their<br />

percent yield so that they can get the maximum amount of product to sell from the reactants that they purchased.<br />

I<br />

• In the reaction below, potassium and water are combined in a chemical reaction that produces potassium<br />

hydroxide and hydrogen gas. If two moles of potassium (the limiting reactant) are used, what is the predicted<br />

yield of potassium hydroxide (KOH) in grams?<br />

2K + 2H 2O → 2KOH + H 2<br />

1. Looking for: Predicted yield of KOH in grams<br />

2. Given: Two moles of the limiting reactant (K) are used<br />

3. Relationships: Two moles of limiting reactant should produce two moles of product. Two moles of<br />

the product will have a mass twice its molar mass.<br />

4. Solution: Molar mass of KOH = atomic mass of K + atomic mass of O + atomic mass of H<br />

From periodic table, molar mass of KOH = 39.10 + 16.00 + 1.01 = 56.11 grams<br />

Because we started with two moles of limiting reactant, we should end up with two<br />

moles, or 112.22 grams, of KOH.<br />

• If the actual yield of KOH was 102.5 grams, what was the percent yield for this reaction?<br />

1. Looking for: Percent yield of KOH<br />

2. Given: Actual yield = 102.5 g; predicted yield = 122.22 g<br />

3. Relationships: Percent yield = actual yield ÷ predicted yield × 100<br />

4. Solution: Percent yield = 102.5 g ÷ 112.22 g × 100 = 91.3%<br />

14.2


Page 2 of 2<br />

1. In the balanced reaction below, hydrochloric acid reacts with calcium carbonate to produce calcium<br />

chloride, carbon dioxide, and water.<br />

2HCL + CaCO 3 → CaCl 2 + CO 2 + H 2O<br />

If one mole of calcium carbonate (the limiting reactant) is used, how much calcium chloride should the<br />

reaction produce? Give your answer in grams. (Hint: use your periodic table to find the atomic masses of<br />

calcium and chlorine).<br />

2. If the actual yield of calcium chloride in the reaction is 97.6 grams, what is the percent yield?<br />

3. In order to get a 94% actual yield for this reaction, how many grams of calcium chloride would the reaction<br />

need to produce?<br />

4. If you put an iron nail into a beaker of copper (II) chloride, you will begin to see a reddish precipitate<br />

forming on the nail. In this reaction, iron replaces copper in the solution and copper falls out of the solution<br />

as a metal. Here is the balanced reaction:<br />

Fe + CuCl 2 → FeCl 2 + Cu<br />

If you start out with one mole of your limiting reactant (Fe), how many grams of copper can you expect to<br />

produce through this reaction?<br />

5. If the actual yield of copper in the reaction is 55.9 grams, what is the percent yield?<br />

6. How much copper would the reaction need to produce to achieve a 96% yield?<br />

7. When sodium hydroxide and sulfuric acid react, sodium sulfate and water are produced. Here is the balanced<br />

reaction:<br />

2NaOH + H 2SO 4 → NaSO 4 + 2H 2O<br />

If two moles of sodium hydroxide (the limiting reactant) are used, one mole of NaSO 4 should be produced.<br />

How many grams of NaSO 4 should be produced?<br />

8. If 100.0 grams of NaSO 4 are actually produced, what is the percent yield?<br />

9. How much NaSO 4 would have to be produced to achieve a 90% yield?<br />

10. Name two reasons why the actual yield in a reaction is usually lower than the predicted yield.<br />

14.2


Name: Date:<br />

14.4 Lise Meitner<br />

Lise Meitner identified and explained nuclear fission, proving it was possible to split an atom.<br />

Prepared to learn<br />

allowed to attend.<br />

Lise Meitner was born in<br />

Vienna on November 7,<br />

1878, one of eight children;<br />

her father was among the first<br />

Jews to practice law in<br />

Austria. At 13, she completed<br />

the schooling provided to<br />

girls. Her father hired a tutor<br />

to help her prepare for a<br />

university education,<br />

although women were not yet<br />

The preparation was worthwhile. When the University<br />

of Vienna opened its doors to women in 1901, Meitner<br />

was ready. She found a mentor there in physics<br />

professor Ludwig Boltzmann, who encouraged her to<br />

pursue a doctoral degree. Physicist Otto Robert Frisch,<br />

Meitner’s nephew, wrote that “Boltzmann gave her the<br />

vision of physics as a battle for ultimate truth, a vision<br />

she never lost.”<br />

Pioneer in radioactivity<br />

In 1906 Meitner went to Berlin after earning her<br />

doctorate, only the second in physics awarded to a<br />

woman by the university. There was great interest in<br />

theoretical physics in Berlin. There she began a<br />

30-year collaboration with chemist Otto Hahn.<br />

Together, they studied radioactive substances. One of<br />

their first successes was the development of a new<br />

technique for purifying radioactive material.<br />

During World War I, Meitner volunteered as an X-ray<br />

nurse-technician with the Austrian army. She<br />

pioneered cautious handling techniques for<br />

radioactive substances, and when she was off duty,<br />

continued her work with Hahn.<br />

Elemental discoveries<br />

In 1917, they discovered the element protactinium.<br />

Afterward, Meitner was appointed head of the physics<br />

department at the Kaiser Wilhelm Institute for<br />

Chemistry in Berlin, where Hahn was head of the<br />

chemistry department. The two continued their study<br />

of radioactivity, and Meitner became the first to<br />

explain how conversion electrons were produced<br />

when gamma rays were used to remove orbital<br />

electrons.<br />

Atomic-age puzzles<br />

14.4<br />

In 1934, when Enrico Fermi produced radioactive<br />

isotopes of uranium by neutron bombardment, he was<br />

puzzled by the products. Meitner, Hahn, and German<br />

chemist Fritz Strassmann began looking for answers.<br />

Their research was interrupted when Nazi Germany<br />

annexed Austria in 1938 and restrictions on “non-<br />

Aryan” academics tightened. Meitner, though she had<br />

been baptized and raised a Protestant, went into exile<br />

in Sweden. She continued to correspond with her<br />

collaborators and suggested that they perform further<br />

tests on a product of the uranium bombardment.<br />

When tests showed it was barium, the group was<br />

puzzled. Barium was so much smaller than uranium.<br />

Hahn wrote to Meitner that uranium “can’t really<br />

break into barium … try to think of some other<br />

possible explanation.”<br />

Meitner and Frisch (who was also in Sweden) worked<br />

on the problem and proved that splitting the uranium<br />

atom was energetically possible. Using Neils Bohr’s<br />

model of the nucleus, they explained how the neutron<br />

bombardment could cause the nucleus to elongate into<br />

a dumbbell shape. Occasionally, they explained, the<br />

narrow center of the dumbbell could separate, leaving<br />

two nuclei. Meitner and Frisch called this process<br />

nuclear fission.<br />

Meitnerium honors achievement<br />

In 1944, Hahn received the Nobel Prize in chemistry<br />

for the discovery of nuclear fission. Meitner’s role was<br />

overlooked or obscured.<br />

In 1966, she, Hahn, and Strassman shared the Enrico<br />

Fermi Award, given by President Lyndon B. Johnson<br />

and the Department of Energy. Meitner died two years<br />

later, just days before her 90th birthday. In 1992,<br />

element 109 was named meitnerium to honor her<br />

work.


Page 2 of 2<br />

Reading reflection<br />

1. Research: Ludwig Boltzmann was an important mentor to Lise Meitner. Who was Boltzmann?<br />

Research and list one of his contributions to science.<br />

2. What element did Meitner and Otto Hahn discover? Using the periodic table, list the atomic number and<br />

mass number of this element. Does this element have stable isotopes?<br />

3. What is nuclear fission? Explain this event in your own words and draw a diagram showing how fission<br />

occurs in a uranium nucleus.<br />

4. Research and describe at least two ways nuclear fission was used in the twentieth century.<br />

5. Meitner did not receive the Nobel Prize for her work on nuclear fission, though she was honored in other<br />

ways. List how she was honored for her work in physics.<br />

6. On a separate sheet of paper, compose a letter to the Nobel Prize Committee explaining why Meitner<br />

deserved this prize for her work. Be sure to explain your reasoning clearly and be sure to use formal<br />

language and correct grammar in your letter.<br />

14.4


Name: Date:<br />

14.4 Marie and Pierre Curie<br />

14.4<br />

Marie and Pierre Curie’s pioneering studies of radioactivity had a dramatic impact on the development<br />

of twentieth-century science. Marie Curie’s bold view that uranium rays seemed to be an intrinsic part of uranium<br />

atoms encouraged physicists to explore the possibility that atoms might have an internal structure. Out of this<br />

idea the field of nuclear physics was born. Together the Curies discovered two radioactive elements, polonium<br />

and radium. Through Pierre Curie’s study of how living tissue responds to radiation, a new era in cancer<br />

treatment was born.<br />

The allure of learning<br />

Marya Sklodowska was born<br />

on November 7, 1867, in<br />

Russian-occupied Warsaw,<br />

Poland. She was the<br />

youngest of five children of<br />

two teachers, her father a<br />

teacher of physics and<br />

mathematics, her mother<br />

also a singer and pianist.<br />

Marya loved school, and especially liked math and<br />

science. However, in Poland, as in much of the rest of<br />

the world, opportunities for higher education were<br />

limited for women. At 17, she and one of her sisters<br />

enrolled in an illegal, underground “floating<br />

university” in Warsaw.<br />

After these studies, she worked for three years as a<br />

governess. Her employer allowed her to teach reading<br />

to the children of peasant workers at his beet-sugar<br />

factory. This was forbidden under Russian rule. At the<br />

same time, she took chemistry lessons from the<br />

factory’s chemist, mathematics lessons from her father<br />

by mail, and studied on her own.<br />

By fall 1891, Sklodowska had saved enough money to<br />

enroll at the University of Paris (also called the<br />

Sorbonne). She earned two master’s degrees, in<br />

physics and mathematics.<br />

A Polish friend introduced Marie, as she was called in<br />

French, to Pierre Curie, the laboratory chief at the<br />

Sorbonne’s Physics and Industrial Chemistry <strong>School</strong>s.<br />

The piezoelectric effect<br />

Pierre Curie’s early research<br />

centered on properties of<br />

crystals. He and his brother<br />

Jacques discovered the<br />

piezoelectric effect, which<br />

describes how a crystal will<br />

oscillate when electric<br />

current is applied. The<br />

oscillation of crystals is now<br />

used to precisely control<br />

timing in computers and watches and many other<br />

devices.<br />

Pierre Curie and Marie Sklodowska found that despite<br />

their different nationalities and background, they had<br />

the same passion for scientific research and shared the<br />

desire to use their discoveries to promote<br />

humanitarian causes. They married in 1895.<br />

Crystals and uranium rays<br />

Pierre continued his pioneering research in crystal<br />

structures, while Marie pursued a physics doctorate.<br />

She chose uranium rays as her research topic.<br />

Uranium rays had been discovered only recently by<br />

French physicist Henri Becquerel.<br />

Becquerel’s report explained that uranium compounds<br />

emitted some sort of ray that fogged photographic<br />

plates. Marie Curie decided to research the effect these<br />

rays had on the air’s ability to conduct electricity. To<br />

measure this effect, she adapted a device that Pierre<br />

and Jacques Curie had invented 15 years earlier.


Page 2 of 3<br />

Marie Curie confirmed that the electrical effects of<br />

uranium rays were similar to the photographic effects<br />

that Becquerel reported—both were present whether<br />

the uranium was solid or powdered, pure or in<br />

compound, wet or dry, exposed to heat or to light.<br />

She concluded that the emission of rays by uranium<br />

was not the product of a chemical reaction, but could<br />

be something built into the very structure of uranium<br />

atoms.<br />

Allies behind a revolutionary idea<br />

Marie Curie’s idea was revolutionary because atoms<br />

were still believed to be tiny, featureless particles. She<br />

decided to test every known element to see if any<br />

others would, like uranium, improve the air’s ability to<br />

conduct electricity. She found that the element<br />

thorium had this property.<br />

Pierre Curie decided to join Marie after she found that<br />

two different uranium ores (raw materials gathered<br />

from uranium mines) caused the air to conduct<br />

electricity much better than even pure uranium or<br />

thorium. They wondered if an undiscovered element<br />

might be mixed into each ore.<br />

They worked to separate the chemicals in the ores and<br />

found two substances that were responsible for the<br />

increased conductivity. They called these elements<br />

polonium, in honor of Marie’s native country, and<br />

radium, from the Greek word for ray.<br />

A new field of medicine<br />

While Marie Curie searched for ways to extract these<br />

pure elements from the ores, Pierre turned his<br />

attention to the properties of the rays themselves. He<br />

tested the radiation on his own skin and found that it<br />

damaged living tissue.<br />

As Pierre published his findings, a whole new field of<br />

medicine developed, using targeted rays to destroy<br />

cancerous tumors and cure skin diseases.<br />

Unfortunately, both Curies became ill from<br />

overexposure to radiation.<br />

Curies share the Nobel Prize<br />

In June 1903, Madame Curie became the first woman<br />

in Europe to receive a doctorate in science. In<br />

December of that year, the Curies and Becquerel<br />

shared the Nobel Prize in physics.<br />

The Curies were honored for their work on<br />

the spontaneous radiation that Becquerel had<br />

14.4<br />

discovered. Marie Curie called spontaneous<br />

radiation “radioactivity.” She was the first woman to<br />

win the Nobel in physics. And in 1904, her second<br />

daughter, Eve, was born. The elder daughter, Irene,<br />

was seven.<br />

Tragedy intrudes<br />

In April 1906, Pierre was killed by a horse-drawn<br />

wagon in a Paris street accident. A month later, the<br />

Sorbonne asked Madame Curie to take over her<br />

husband’s position there. She agreed, in hopes of<br />

creating a state-of-the art research center in her<br />

husband’s memory.<br />

Marie Curie threw herself into the busy academic<br />

schedule of teaching and conducting research (she was<br />

the first woman to lecture, the first to be named<br />

professor, and the first to head a laboratory at the<br />

Sorbonne), and found time to work on raising money<br />

for the new center. The Radium Institute of the<br />

University of Paris opened in 1914 and Madame Curie<br />

was named director of its Curie Laboratory.<br />

The scientist-humanitarian<br />

In 1911, Curie received a second Nobel Prize (the first<br />

person so honored), this time in chemistry for her<br />

work in finding elements and determining the atomic<br />

weight of radium.<br />

With the start of World War I in 1914, she turned her<br />

attention to the use of radiation to help wounded<br />

soldiers. Assisted by her daughter Irene, she created a<br />

fleet of 20 mobile x-ray units to help medics quickly<br />

determine and then treat injuries in the field. Next, she<br />

set up nearly 200 x-ray labs in hospitals and trained<br />

150 women to operate the equipment.<br />

Legacy continues<br />

After the war, Curie went back to direct the Radium<br />

Institute, which grew to two centers, one devoted to<br />

research and the other to treatment of cancer. In July<br />

1934, she died at 66 of radiation-induced leukemia.<br />

The next year, Irene Joliot-Curie and her husband,<br />

Frederic Joliot-Curie, were awarded the Nobel Prize in<br />

chemistry for their discovery of artificial radiation.


Page 3 of 3<br />

Reading reflection<br />

1. Why might Marie Curie have been motivated to teach the children of beet workers? Recall that this<br />

was forbidden by Russian rule.<br />

2. What fundamental change in our understanding of the atom was brought about by the work of Marie Curie?<br />

3. Describe how Marie and Pierre Curie discovered two elements.<br />

4. Name at least three new fields of science that stem from the work of Marie and/or Pierre Curie.<br />

5. Research: In your own words, describe Marie Curie as a role model for women in science. Use your library<br />

or the Internet to research how she worked to balance a scientific career and motherhood.<br />

14.4


Name: Date:<br />

14.4 Rosalyn Sussman Yalow<br />

14.4<br />

Rosalyn Sussman Yalow and her research partner, Solomon Berson, developed radioimmunoassay, or<br />

RIA. This important medical diagnostic tool uses radioactive isotopes to trace hormones, enzymes, and<br />

medicines that exist in such low concentrations in blood that they were previously impossible to detect using<br />

other laboratory methods.<br />

Encouraged and inspired<br />

Rosalyn Sussman was born in<br />

1921 in New York City.<br />

Neither of her parents<br />

attended school beyond eighth<br />

grade, but they encouraged<br />

Rosalyn and her older brother<br />

to value education. In the early<br />

grades, Rosalyn enjoyed math,<br />

but in high school her<br />

chemistry teacher encouraged<br />

her interest in science.<br />

She stayed in New York after high school, studying<br />

physics and chemistry at Hunter College. After her<br />

graduation in 1941, she took a job as a secretary at<br />

Columbia University. There were few opportunities<br />

for women to attend graduate school, and Sussman<br />

hoped that by working at Columbia, she might be able<br />

to sit in on some courses.<br />

A wartime opportunity<br />

However, as the United States began drafting large<br />

numbers of men in preparation for war, universities<br />

began to accept women rather than close down. In fall<br />

1941, Sussman arrived at the University of Illinois<br />

with a teaching assistantship in the <strong>School</strong> of<br />

Engineering, where she was the only woman.<br />

There, she specialized in the construction and use of<br />

devices for measuring radioactive substances. By<br />

January 1945 she had earned her doctorate, with<br />

honors, in nuclear physics, and married Aaron Yalow,<br />

a fellow student.<br />

From medical physics to ‘radioimmunoassay’<br />

From 1946–50, Yalow taught physics at Hunter<br />

College, which had only introduced it as a major her<br />

senior year and which now admitted men. In 1947, she<br />

also began working part time at the Veterans<br />

Administration Hospital in the Bronx, which was<br />

researching medical uses of radioactive substances.<br />

In 1950 she joined the hospital full time and began a<br />

research partnership with Solomon A. Berson, an<br />

internist. Together they developed the basic science,<br />

instruments, and mathematical analysis necessary to<br />

use radioactive isotopes to measure tiny<br />

concentrations of biological substances and certain<br />

drugs in blood and other body fluids. They called their<br />

technique radioimmunoassay, or RIA. (Yalow also had<br />

two children by 1954.)<br />

RIA helps diabetes research<br />

One early application of RIA was in diabetes research,<br />

which was especially significant to Yalow because her<br />

husband was diabetic. Diabetes is a condition in which<br />

the body is unable to regulate blood sugar levels. This<br />

is normally accomplished through the release of a<br />

hormone called insulin from the pancreas.<br />

Using RIA, they showed that adult diabetics did not<br />

always lack insulin in their blood, and that, therefore,<br />

something must be blocking their insulin’s normal<br />

action. They also studied the body’s immune system<br />

response to insulin injected into the bloodstream.<br />

Commercial applications, not commerce<br />

RIA’s current uses include screening donated blood,<br />

determining effective doses of medicines, detecting<br />

foreign substances in the blood, testing hormone<br />

levels in infertile couples, and treating certain children<br />

with growth hormones.<br />

Yalow and Berson changed theoretical immunology<br />

and could have made their fortunes had they chosen to<br />

patent RIA, but instead, Yalow explained, “Patents are<br />

about keeping things away from people for the<br />

purpose of making money. We wanted others to be<br />

able to use RIA.” Berson died unexpectedly in 1972;<br />

Yalow had their VA research laboratory named after<br />

him, and lamented later that his death had excluded<br />

him from sharing the team’s greatest recognition.<br />

A rare Nobel winner<br />

Yalow was awarded the Nobel Prize in Physiology or<br />

Medicine in 1977. She was only the second woman to<br />

win in that category, for her work on radioimmunoassay<br />

of peptide hormones.


Page 2 of 2<br />

Reading reflection<br />

1. Rosalyn Yalow has said that Eve Curie’s biography of her mother, Marie Curie, helped spark her<br />

interest in science. Compare the lives of these two scientists.<br />

2. Describe radioimmunoassay in your own words.<br />

3. What information about adult diabetes was discovered using RIA?<br />

4. Find out more about the role of patents in medical research. Do you agree or disagree with Yalow’s<br />

statement? Why?<br />

14.4


Name: Date:<br />

14.4 Chien-Shiung Wu<br />

14.4<br />

During World War II, Chinese-American physicist Chien-Shiung Wu played an important role in the<br />

Manhattan Project, the Army’s secret work to develop the atomic bomb. In 1957, she overthrew what was<br />

considered an indisputable law of physics, changing the way we understand the weak nuclear force.<br />

Determined to learn<br />

Chien-Shiung Wu was born on<br />

May 31, 1912, in a small town<br />

outside Shanghai, China. Her<br />

father had opened the region’s<br />

first school for young girls,<br />

which Chien-Shiung finished<br />

at age 10.<br />

She then attended a girls<br />

boarding school in Suzhou<br />

that had two sections—a<br />

teacher training school and an academic school with a<br />

standard Western curriculum. Chien-Shiung enrolled<br />

in teacher training, because tuition was free and<br />

graduates were guaranteed jobs.<br />

Students from both sections lived in the dormitory,<br />

and as Chien-Shiung became friends with girls in the<br />

academic school, she learned that their science and<br />

math curriculum was more rigorous than hers. She<br />

asked to borrow their <strong>book</strong>s and stayed up late<br />

teaching herself the material.<br />

Chien-Shiung Wu graduated first in her class and was<br />

invited to attend prestigious National Central<br />

University in Nanjing. There, she earned a bachelor’s<br />

degree in physics and did research for two years. In<br />

1936 Wu emigrated from China to the United States.<br />

She earned her doctorate from the University of<br />

California at Berkeley in 1940.<br />

A key scientist in the Manhattan Project<br />

Wu taught at Smith College and Princeton University<br />

until 1944, when she went to Columbia University as a<br />

senior scientist and researcher and was asked to join<br />

the Manhattan Project. There she helped develop the<br />

process to enrich uranium ore.<br />

In the course of the project, her renowned colleague<br />

Enrico Fermi turned to Wu for help with a fission<br />

experiment. A rare gas which she had studied in<br />

graduate school was causing the problem. With Wu’s<br />

assistance, Fermi was able to solve the problem and<br />

continue his work.<br />

Right and left in nature?<br />

After the war, Wu continued her research in nuclear<br />

physics at Columbia. In 1956, she and two colleagues,<br />

Tsung-Dao Lee of Columbia and Chen Ning Yang of<br />

Princeton, reconsidered the law of conservation of<br />

parity. This law stated that nature does not distinguish<br />

between left and right in nuclear reactions. They<br />

wondered if the law might not be valid for interactions<br />

of subatomic particles involving the weak nuclear<br />

force.<br />

Wu was a leading specialist in beta decay. She figured<br />

out a means of testing their theory. She cooled cobalt-<br />

60, a radioactive isotope, to 0.01 degree above<br />

absolute zero. Next, she placed the cobalt-60 in a<br />

strong magnetic field so that the cobalt nuclei lined up<br />

and spun along the same axis. She observed what<br />

happened as the cobalt-60 broke down and gave off<br />

electrons.<br />

According to the law of conservation of parity, equal<br />

numbers of electrons should have been given off in<br />

each direction. However, Wu found that many more<br />

electrons flew off in the direction opposite the spin of<br />

the cobalt-60 nuclei. She proved that in beta decay,<br />

nature does in fact distinguish between left and right.<br />

Always a landmark achiever<br />

Unfortunately, when Lee and Yang were awarded the<br />

Nobel Prize in physics in 1957, Wu’s contribution to<br />

the project was overlooked. However, among her<br />

many honors and awards, she later received the<br />

National Medal of Science, the nation’s highest award<br />

for science achievement.<br />

In 1973, she became the first female president of the<br />

American Physical Society. Wu died at 84 in 1997,<br />

leaving a husband and son who were both physicists.


Page 2 of 2<br />

Reading reflection<br />

1. Use a dictionary to look up the meaning of each boldface word. Write a definition for each word.<br />

Be sure to credit your source.<br />

2. How did Chien-Shiung Wu’s work in graduate school help her with her work on the Manhattan Project?<br />

3. From the reading, define the law of conservation of parity in your own words.<br />

4. How many protons and neutrons does cobalt-60 have? List the nonradioactive isotopes of cobalt.<br />

5. Briefly describe Wu’s elegant experiment that proved that nature distinguishes between right and left.<br />

6. Research: Wu was the first woman recipient of the National Medal of Science in physical science. Two<br />

other women have since received this award. Who were they and what did they do?<br />

7. What are three questions that you have about Wu and her work?<br />

8. Suggest some possible reasons why Wu did not receive the Nobel Prize for her work.<br />

14.4


Name: Date:<br />

14.4 Radioactivity<br />

There are three main types of radiation that involve the decay of the nucleus of an atom:<br />

• alpha radiation (α): release of a helium-4 nucleus (two protons and two neutrons). We can represent<br />

4<br />

helium-4 using isotope notation: 2He<br />

. The top number, 4, represents the mass number, and the bottom<br />

number represents the atomic number for helium, 2.<br />

• beta radiation (β): release of an electron.<br />

• gamma radiation (γ): release of an electromagnetic wave.<br />

Half-life<br />

The time it takes for half of the atoms in a sample to decay is called the half-life. Four kilograms of a certain<br />

substance undergo radioactive decay. Let’s calculate the amount of substance left over after 1, 2, and 3 half-lives.<br />

• After one half-life, the substance will be reduced by half, to 2 kilograms.<br />

• After two half-lives, the substance will be reduced by another half, to 1 kilogram.<br />

• After three half-lives, the substance will be reduced by another half, to 0.5 kilogram.<br />

So, if we start with a sample of mass m that decays, after a few half-lives, the mass of the sample will be:<br />

Number of half-lives Mass left<br />

1 ----m<br />

1<br />

=<br />

2 1<br />

2 ----m<br />

1<br />

=<br />

2 2<br />

3 ----m<br />

1<br />

=<br />

2 3<br />

4 ----m<br />

1<br />

=<br />

2 4<br />

1<br />

--m<br />

2<br />

1<br />

--m<br />

4<br />

1<br />

--m<br />

8<br />

-----m<br />

1<br />

16<br />

14.4


Page 2 of 2<br />

1. The decay series for uranium-238 and plutonium-240 are listed below. Above each arrow, write “a”<br />

for alpha decay or “b” for beta decay to indicate which type of decay took place at each step.<br />

a. 238 → 234 → 234 → 234 → 230 →<br />

92 U<br />

226<br />

88 Ra<br />

214<br />

84 Po<br />

240<br />

94 Pu<br />

90 Th<br />

222<br />

86 Rn<br />

210<br />

82 Pb<br />

91 Pa<br />

218<br />

84 Po<br />

→ → → → →<br />

210<br />

83 Bi<br />

→ → → →<br />

240<br />

95Am 236<br />

93 Np<br />

b. → → → → →<br />

228<br />

90 Bi<br />

212<br />

83 Bi<br />

224<br />

88 Ra<br />

224<br />

89 Ac<br />

→ → → → →<br />

212<br />

84 Po<br />

208<br />

82 Pb<br />

→ → →<br />

18<br />

92 U<br />

214<br />

82 Pb<br />

210<br />

84 Po<br />

232<br />

91 Pa<br />

220<br />

87 Fr<br />

208<br />

83 Bi<br />

90 Th<br />

214<br />

83 Bi<br />

206<br />

82 Pb<br />

232<br />

92 U<br />

216<br />

85 At<br />

2. Fluorine-18 ( 9F<br />

) has a half-life of 110 seconds. This material is used extensively in medicine. The hospital<br />

18<br />

laboratory starts the day at 9 a.m. with 10 grams of 9F<br />

.<br />

a. How many half-lives for fluorine-18 occur in 11 minutes (660 seconds)?<br />

b. How much of the 10-gram sample of fluorine-18 would be left after 11 minutes?<br />

14<br />

3. The isotope 6C<br />

has a half-life of 5,730 years. What is the fraction of 6C<br />

in a sample with mass, m, after<br />

28,650 years?<br />

4. What is the half-life of this radioactive isotope that decreases to one-fourth its original amount in 18 months?<br />

5. This diagram illustrates a formula that is used to calculate the intensity of radiation from a radioactive<br />

source. Radiation “radiates” from a source into a spherical area. Therefore, you can calculate intensity using<br />

the area of a sphere ( 4πr ). Use the formula and the diagram to help you answer the questions below.<br />

2<br />

a. A radiation source with a power of 1,000. watts is located at a point in space. What is the intensity of<br />

radiation at a distance of 10. meters from the source?<br />

b. The fusion reaction releases 2.8 × 10 –12 2<br />

1H<br />

H<br />

joules of energy. How many such<br />

reactions must occur every second in order to light a 100-watt light bulb? Note that one watt equals one<br />

joule per second.<br />

3 4<br />

+ 1 →<br />

2He + n + energy<br />

14<br />

14.4


Page 1 of 2<br />

15.2 Svante Arrhenius<br />

15.2<br />

Svante Arrhenius was a Swedish Chemist who won the Nobel Prize in 1903 for his work on acid and<br />

base chemistry. He is also known for recognizing that carbon dioxide (CO2 ) is added to the atmosphere when<br />

fossil fuels are burned, and that it is a greenhouse gas. Arrhenius calculated that doubling the CO2 in the<br />

atmosphere would increase Earth’s temperature by 4-5°C. His prediction, made without computers or modern<br />

scientific equipment, is close to current estimates!<br />

Young scholar<br />

Svante Arrhenius was born in<br />

1859 in Wijk, Sweden. His<br />

father was a land surveyor.<br />

Svante taught himself to read<br />

at the age of three. He was a<br />

strong student and especially<br />

enjoyed math and physics. He<br />

graduated at the top of his high<br />

school class, although he was<br />

the youngest student.<br />

Arrhenius went on to study<br />

mathematics, chemistry, and physics at the University of<br />

Uppsala in Sweden. In 1881 he moved to Stockholm to<br />

study with Professor E. Edlund at the Academy of<br />

Sciences. Arrhenius was especially interested in what<br />

happens when electricity is passed through solutions. For<br />

his doctoral thesis, he proposed that molecules in solutions<br />

could break up into electrically charged fragments called<br />

ions.<br />

Setback, perseverance, and recognition<br />

Unfortunately, the value of Arhenius’s work was not<br />

recognized by the faculty at the University of Uppsala,<br />

where he defended his dissertation. The idea that a<br />

molecule could break up in water was difficult to accept.<br />

Finally, Arrhenius was given a “fourth rank” degree--which<br />

meant that he barely passed. Arrhenius could not hope to<br />

obtain a university professorship with that degree!<br />

Arrhenius’s mentor, Professor Edlund, helped him obtain a<br />

travel grant to meet and work with leading scientists in the<br />

field of physical chemistry. They helped Arrhenius clarify<br />

his ionic theory. In the late 1890’s, when electrically<br />

charged subatomic particles were discovered, the<br />

importance of Arrhenius’s work was finally recognized.<br />

Arrhenius was awarded the Nobel Prize for Chemistry in<br />

1903.<br />

A man of many interests<br />

Arrhenius was fascinated by many branches of science. He<br />

studied electrolytes in the human body, publishing papers<br />

about their role in digestion and absorption, and about their<br />

function as antitoxins.<br />

Along with his scientific publications, Arrhenius wrote<br />

<strong>book</strong>s intended to introduce the general public to advances<br />

in various scientific fields. These included Smallpox and its<br />

Combating (1913) and Chemistry and Modern Life (1919).<br />

Arrhenius was also interested in Astronomy. In 1908 he put<br />

forth the theory of panspermy--which suggested that life<br />

may spread through the universe when spores from a lifebearing<br />

planet escape their atmosphere and are then driven<br />

by radiation pressure across long expanses of space, until<br />

they come to rest on another planet where hospitable<br />

conditions allow them to flourish. While this theory hasn’t<br />

withstood the test of time, Arrhenius did contribute to our<br />

understanding of the phenomenon known as Aurora<br />

Borealis, or northern lights.<br />

Pioneering climate research<br />

Arrhenius was curious about what caused the beginning<br />

and end of Earth’s ice ages. In 1895, he presented a paper<br />

to the Stockholm Physical Society called “On the Influence<br />

of Carbonic Acid (CO2 ) in the Air upon the Temperature of<br />

the Ground.” He proposed that variations in the amount of<br />

CO2 in the atmosphere could influence climate.<br />

In 1903, he wrote a <strong>book</strong> called Worlds in the Making in<br />

which he explained that atmospheric gases like carbon<br />

dioxide trap heat near Earth’s surface, increasing its<br />

average temperature. In 1904, he suggested that human<br />

activity could affect Earth’s climate, if industrial emissions<br />

increased the amount of CO2 in the atmosphere. He was not<br />

concerned about this increase; in fact he thought that it<br />

might be beneficial for growing crops to feed a larger<br />

human population.<br />

Arrhenius died in Stockholm in 1927.


Page 2 of 2<br />

Reading reflection<br />

1. Describe Arrhenius’s doctoral thesis.<br />

2. What was the setback that Arrhenius had to overcome early in his career?<br />

3. Name three fields of science that interested Arrhenius.<br />

4. Describe a scientific theory proposed by Arrhenius that has never received widespread acceptance.<br />

5. Why is Arrhenius considered a pioneer in the field of climate change study?<br />

15.2


Name: Date:<br />

16.1 Open and Closed Circuits<br />

Where is the current flowing?<br />

You have built and tested different kinds of circuits in the lab. Now you can use what you learned to make<br />

predictions about circuits you haven’t seen before. Use the circuit diagrams pictured below to answer the<br />

questions. You may wish to write on the diagrams in order to keep track where the current is flowing. As a result,<br />

each diagram is repeated several times.<br />

1. Which devices (A, B, C, or D) in the circuit pictured below will be on when the following conditions are<br />

met? For your answer, give the letter of the device or devices.<br />

a. Switch 3 is open, and all other switches are closed.<br />

b. Switch 2 is open, and all other switches are closed.<br />

c. Switch 4 is open, and all other switches are closed.<br />

d. Switch 1 is open, and all other switches are closed.<br />

e. Bulb C blows out, and all switches are closed.<br />

f. Bulb A blows out, and all switches are closed.<br />

.<br />

g. Switches 2 and 4 are open, and switches 1 and 3 are closed.<br />

h. Switches 2 and 3 are open, and switches 1 and 4 are closed.<br />

16.1


.<br />

Page 2 of 3<br />

i. Switches 2, 3, and 4 are open, and switch 1 is closed.<br />

j. Switches 1 and 2 are open, and switches 3 and 4 are closed.<br />

2. Which of the devices (A-G) in the circuit below will be on when the following conditions are met? For your<br />

answer, give the letter of the device or devices.<br />

a. Switch 5 is open, and all other switches are closed.<br />

b. Switch 6 is open, and all others are closed.<br />

c. Switch 7 is open, and all others are closed.<br />

.<br />

d. Switch 4 is open, and all others are closed.<br />

e. Switch 3 is open, and all others are closed.<br />

f. Switch 2 is open, and all others are closed.<br />

g. Switch 1 is open, and all others are closed.<br />

h. Switches 2 and 4 are open, and all others are closed.<br />

16.1


.<br />

Page 3 of 3<br />

i. Switches 4 and 6 are open, and all others are closed.<br />

j. Switches 4 and 7 are open, and all others are closed.<br />

k. Switches 5 and 7 are open, and all others are closed.<br />

l. Switches 2 and 3 are open, and all others are closed.<br />

m. Bulb D blows out with all switches closed.<br />

n. Bulbs A and B blow out with all switches closed.<br />

o. Bulbs A and D blow out with all switches closed.<br />

3. Use arrows to draw the direction of the current in each of the circuits below. Make sure to show current<br />

direction in all paths of the circuits within each diagram.<br />

4. How many possible paths are there in circuit diagrams in questions (1) and (2)?<br />

5. Draw a circuit of your own. Use one battery, show at least 4 devices (bulbs and bells), and divide the current<br />

at some point in the circuit. Finally, use arrows to show the direction of the current in all parts of your circuit.<br />

16.1


Name: Date:<br />

16.1 Benjamin Franklin<br />

Benjamin Franklin overcame a lack of formal education to become a prominent businessman,<br />

community leader, inventor, scientist, and statesman. His study of “electric fire” changed our basic<br />

understanding of how electricity works.<br />

An eye toward inventiveness<br />

Benjamin Franklin was born<br />

in Boston in 1706. With only<br />

one year of schooling he<br />

became an avid reader and<br />

writer. He was apprenticed at<br />

age 12 to his brother James, a<br />

printer. The siblings did not<br />

always see eye to eye, and at<br />

17, Ben ran away to<br />

Philadelphia.<br />

In his new city, Franklin developed his own printing<br />

and publishing business. Over the years, he became a<br />

community leader, starting the first library, fire<br />

department, hospital, and fire insurance company. He<br />

loved gadgets and invented some of his own: the<br />

Franklin stove, the glass armonica (a musical<br />

instrument), bifocal eyeglasses, and swim fins.<br />

‘Electric fire’<br />

In 1746, Franklin saw some demonstrations of static<br />

electricity that were meant for entertainment. He<br />

became determined to figure out how this so-called<br />

“electric fire” worked.<br />

Undeterred by his lack of science education, Franklin<br />

began experimenting. He generated static electricity<br />

using a glass rod and silk cloth, and then recorded how<br />

the charge could attract and repel lightweight objects.<br />

Franklin read everything he could about this “electric<br />

fire” and became convinced that a lightning bolt was a<br />

large-scale example of the same phenomenon.<br />

Father and son experiment<br />

In June 1752, Franklin and his 21-year-old son,<br />

William, conducted an experiment to test his theory.<br />

Although there is some debate over the details, most<br />

historians agree that Franklin flew a kite on a stormy<br />

day in order to collect static charges.<br />

16.1<br />

Franklin explained that he and his son constructed a<br />

kite of silk cloth and two cedar strips. They attached a<br />

metal wire to the top. Hemp string was used to fly the<br />

kite. A key was tied near the string’s lower end. A silk<br />

ribbon was affixed to the hemp, below the key.<br />

Shocking results<br />

It is probable that Franklin and his son were under<br />

some sort of shelter, to keep the silk ribbon dry. They<br />

got the kite flying, and once it was high in the sky they<br />

held onto it by the dry silk ribbon, not the wet hemp<br />

string. Nothing happened for a while. Then they<br />

noticed that the loose threads of the hemp suddenly<br />

stood straight up.<br />

The kite probably was not struck directly by lightning,<br />

but instead collected charge from the clouds. Franklin<br />

touched his knuckle to the key and received a static<br />

electric shock. He had proved that lightning was a<br />

discharge of static electricity.<br />

Those are charged words<br />

Through his experiments, Franklin determined that<br />

“electric fire” was a single “fluid” rather than two<br />

separate fluids, as European scientists had thought.<br />

He proposed that this “fluid” existed in two states,<br />

which he called “positive” and “negative.” Franklin<br />

was the first to explain that if there is an excess<br />

buildup of charge on one item, such as a glass rod, it<br />

must be exactly balanced by a lack of charge on<br />

another item, such as the silk cloth. Therefore, electric<br />

charge is conserved. He also explained that when there<br />

is a discharge of static electricity between two items,<br />

the charges become balanced again.<br />

Many of Franklin’s electrical terms remain in use<br />

today, including battery, charge, discharge, electric<br />

shock, condenser, conductor, plus and minus, and<br />

positive and negative.


Page 2 of 2<br />

Reading reflection<br />

1. Although Ben Franklin had only one year of schooling, he became a highly educated person.<br />

Describe how Franklin learned about the world.<br />

2. What hypothesis did Franklin test with his kite experiment?<br />

3. Describe the results and conclusion of Franklin’s kite experiment.<br />

4. Franklin’s kite experiment was dangerous. Explain why.<br />

5. Silk has an affinity for electrons. When you rub a glass rod with silk, the glass is left with a positive charge.<br />

Make a diagram that shows the direction that charges move in this example. Illustrate and label positive and<br />

negative charges on the silk and glass rod in your diagram. Note: Show the same number of positive and<br />

negative charges in your diagram.<br />

6. Research: Among Franklin’s many inventions is the lightning rod. Find out how this device works, and<br />

create a model or diagram to show how it functions.<br />

16.1


Name: Date:<br />

16.2 Using an Electric Meter<br />

What do you measure in a circuit and how do you measure it? This skill sheet gives you useful tips to help you<br />

use an electric meter and understand electrical measurements.<br />

The digital multimeter<br />

Most people who work with electric circuits use a digital multimeter to measure electrical quantities. These<br />

measurements help them analyze circuits. Most multimeters measure voltage, current, and resistance. A typical<br />

multimeter is shown below:<br />

16.2


Page 2 of 4<br />

Using the digital multimeter<br />

This table summarizes how to use and interpret any digital meter in a battery circuit. Note: A component<br />

is any part of a circuit, such as a battery, a bulb, or a wire.<br />

Measuring Voltage Measuring Current Measuring Resistance<br />

Circuit is ON Circuit is ON Circuit is OFF<br />

Turn dial to voltage, labeled Turn dial to current, labeled Turn dial to resistance, labeled Ω<br />

Connect leads to meter following<br />

meter instructions<br />

Place leads at each end of<br />

component (leads are ACROSS<br />

the component)<br />

Connect leads to meter following<br />

meter instructions<br />

Break circuit and place leads on<br />

each side of the break (meter is IN<br />

the circuit)<br />

Connect leads to meter following<br />

meter instructions<br />

Place leads at each end of<br />

component (leads are ACROSS<br />

the component)<br />

Measurement in VOLTS (V) Measurement in AMPS (A) Measurement in OHMS (Ω)<br />

Battery measurement shows<br />

relative energy provided<br />

Component measurement shows<br />

relative energy used by that<br />

component<br />

Measurement shows the value of<br />

current at the point where meter is<br />

placed<br />

Current is the flow of charge<br />

through the wire<br />

Measurement shows the<br />

resistance of the component<br />

When the resistance is too high,<br />

the display shows OL (overload)<br />

or ∝ (infinity)<br />

16.2


Page 3 of 4<br />

Build a series circuit with 2 batteries and 2 bulbs.<br />

1. Measure and record the voltage across each battery:<br />

2. Measure and record the voltage across each bulb:<br />

3. Measure and record the voltage across both batteries:<br />

4. Draw a circuit diagram or sketch that shows all the posts in the circuit (posts are where wires and holders<br />

connect together).<br />

5. Break the circuit at one post. Measure the current and record the value below. Repeat until you have<br />

measured the current at every post.<br />

16.2


Page 4 of 4<br />

6. Create a set of instructions on how to use the meter to do a task. Find someone unfamiliar with the<br />

meter. See if he or she can follow your instructions.<br />

7. A fuse breaks a circuit when current is too high. A fuse must be replaced when it breaks a circuit. Explain<br />

how measuring the resistance of a fuse can tell you if it is defective.<br />

8. You suspect that a wire is defective but can't see a break in it. Explain how measuring the resistance of the<br />

wire can tell you if it has a break.<br />

16.2


Name: Date:<br />

16.3 Voltage, Current, and Resistance<br />

Electricity is one of the most fascinating topics in physical science. It’s also one of the most useful to understand,<br />

since we all use electricity daily. This skill sheet reviews some of the important terms in the study of electricity.<br />

In the reading section, you’ll find questions that check your understanding. If you’re not sure of the answer, go<br />

back and read that section again. In the practice section, you will have an opportunity to show that you know how<br />

voltage, current, and resistance are related in real-world situations.<br />

What is voltage?<br />

What is current?<br />

16.3<br />

You know that water will flow from a higher tank through a hose into a<br />

lower tank. The water in the higher tank has greater potential energy than the<br />

water in the lower tank. A similar thing happens with the flow of charges in<br />

an electric circuit.<br />

Charges flow in a circuit when there is a difference in energy level from one<br />

end of the battery (or any other energy source) to the other. This energy<br />

difference is measured in volts. The energy difference causes the charges to<br />

move from a higher to a lower voltage in a closed circuit.<br />

Think of voltage as the amount of “push” the electrical source supplies to the<br />

circuit. A meter is used to measure the amount of energy difference or<br />

“push” in a circuit. The meter reads the voltage difference (in volts) between<br />

the positive and the negative ends of the power source (the battery). This<br />

voltage difference supplies the energy to make charges flow in a circuit.<br />

1.What is the difference between placing a 1.5-volt battery in a circuit and<br />

placing a 9-volt battery in a circuit?<br />

Current describes the flow of electric charges. Current is the actual measure of how many charges are flowing<br />

through the circuit in a certain amount of time. Current is measured in units called amperes.<br />

Just as the rate of water flowing out of a faucet can be fast or slow, electrical current can move at different rates.<br />

The type, length, and thickness of wire all effect how much current flows in a circuit. Resistors slow the flow of<br />

current. Adding voltage causes the current to speed up.<br />

2. What could you do to a closed circuit consisting of a battery, a light bulb, and a switch that would increase<br />

the amount of current? Explain your answer.<br />

3. What could you do to a closed circuit consisting of a battery, a light bulb, and a switch that would decrease<br />

the amount of current? Explain your answer.


Page 2 of 3<br />

What is resistance?<br />

16.3<br />

Resistance is the measure of how easily charges flow through a circuit. <strong>High</strong> resistance means it is<br />

difficult for charges to flow. Low resistance means it is easy for charges to flow. Electrical resistance is measured<br />

in units called ohms (abbreviated with the symbol Ω).<br />

Resistors are items that reduce the flow of charge in a circuit. They act like “speed bumps” in a circuit. A light<br />

bulb is an example of a resistor.<br />

4. Describe one thing that you could do to the wire used in a circuit to decrease the amount of resistance<br />

presented by the wire.<br />

How are voltage, current, and resistance related?<br />

When the voltage (push) increases, the current (flow of charges) will also increase, and when the voltage<br />

decreases, the current likewise decreases. These two variables, voltage and current, are said to be directly<br />

proportional.<br />

When the resistance in an electric circuit increases, the flow of charges (current) decreases. These two variables,<br />

resistance and current, are said to be inversely proportional. When one goes up, the other goes down, and vice<br />

versa.<br />

The law that relates these three variables is called Ohm’s Law. The formula is:<br />

Voltage (volts)<br />

Current (amps) =<br />

Resistance (ohms, Ω)<br />

5. In your own words, state the relationship between resistance and current, as well as the relationship between<br />

voltage and current.<br />

• In a circuit, how many amps of current flow through a resistor such as a 6-ohm light bulb when using four<br />

1.5-volt batteries as an energy supply?<br />

Solution:<br />

4× 1.5 volts 6 volts<br />

Current =<br />

=<br />

6 ohms 6 ohms<br />

Current = 1 amp


Page 3 of 3<br />

Now you will have the opportunity to demonstrate your understanding of the relationship between current,<br />

voltage and resistance. Answer each of the following questions and show your work.<br />

1. How many amps of current flow through a circuit that includes a 9-volt battery and a bulb with a resistance<br />

of 6 ohms?<br />

2. How many amps of current flow through a circuit that includes a 9-volt battery and a bulb with a resistance<br />

of 12 ohms?<br />

3. How much voltage would be necessary to generate 10 amps of current in a circuit that has 5 ohms of<br />

resistance?<br />

4. How many ohms of resistance must be present in a circuit that has 120 volts and a current of 10 amps?<br />

16.3


Name: Date:<br />

16.3 Ohm’s Law<br />

A German physicist, Georg S. Ohm, developed this mathematical relationship, known as Ohm’s Law, which is<br />

present in most circuits. It states that if the voltage in a circuit increases, so does the current. If the resistance<br />

increases, the current decreases.<br />

To work through this skill sheet, you will need the symbols used to depict circuits in<br />

diagrams. The symbols that are most commonly used for circuit diagrams are<br />

provided to the right.<br />

If a circuit contains more than one battery, the total voltage is the sum of the<br />

individual voltages. A circuit containing two 6 V batteries has a total voltage of 12 V.<br />

[Note: The batteries must be connected positive to negative for the voltages to add.]<br />

• If a toaster produces 12 ohms of resistance in a 120-volt circuit, what is the<br />

amount of current in the circuit?<br />

Solution:<br />

The current in the toaster circuit is 10 amps.<br />

Voltage (volts)<br />

Current (amps) =<br />

Resistance (ohms, Ω)<br />

V 120 volts<br />

I = = = 10 amps<br />

R 12 ohms<br />

Note: If a problem asks you to calculate the voltage or resistance, you must rearrange the equation to solve<br />

for V or R. All three forms of the equation are listed below.<br />

V V<br />

I = V = IR R =<br />

R I<br />

Answer the following question using Ohm’s law. Don’t forget to show your work.<br />

1. How much current is in a circuit that includes a 9-volt battery and a bulb with a resistance of 3 ohms?<br />

2. How much current is in a circuit that includes a 9-volt battery and a bulb with a resistance of 12 ohms?<br />

3. A circuit contains a 1.5 volt battery and a bulb with a resistance of 3 ohms. Calculate the current.<br />

4. A circuit contains two 1.5 volt batteries and a bulb with a resistance of 3 ohms. Calculate the current.<br />

5. What is the voltage of a circuit with 15 amps of current and toaster with 8 ohms of resistance?<br />

6. A light bulb has a resistance of 4 ohms and a current of 2 A. What is the voltage across the bulb?<br />

16.3


Page 2 of 2<br />

7. How much voltage would be necessary to generate 10 amps of current in a circuit that has 5 ohms<br />

of resistance?<br />

8. How many ohms of resistance must be present in a circuit that has 120 volts and a current of 10<br />

amps?<br />

9. An alarm clock draws 0.5 A of current when connected to a 120 volt circuit. Calculate its resistance.<br />

10. A portable CD player uses two 1.5 V batteries. If the current in the CD player is 2 A, what is its resistance?<br />

11. You have a large flashlight that takes 4 D-cell batteries. If the current in the flashlight is 2 amps, what is the<br />

resistance of the light bulb? (Hint: A D-cell battery has 1.5 volts.)<br />

12. Use the diagram below to answer the following problems.<br />

a. What is the total voltage in each circuit?<br />

b. How much current would be measured in each circuit if the light bulb has a resistance of 6 ohms?<br />

c. How much current would be measured in each circuit if the light bulb has a resistance of 12 ohms?<br />

d. Is the bulb brighter in circuit A or circuit B? Why?<br />

13. What happens to the current in a circuit if a 1.5-volt battery is removed and is replaced by a 9-volt battery?<br />

14. In your own words, state the relationship between resistance and current in a circuit.<br />

15. In your own words, state the relationship between voltage and current in a circuit.<br />

16. What could you do to a closed circuit consisting of 2 batteries, 2 light bulbs, and a switch to increase the<br />

current? Explain your answer.<br />

17. What could you do to a closed circuit consisting of 2 batteries, 2 light bulbs, and a switch to decrease the<br />

current? Explain your answer.<br />

18. You have four 1.5 V batteries, a 1 Ω bulb, a 2 Ω bulb, and a 3 Ω bulb. Draw a circuit you could build to<br />

create each of the following currents. There may be more than one possible answer for each.<br />

a. 1 ampere<br />

b. 2 amperes<br />

c. 3 amperes<br />

d. 6 amperes<br />

16.3


Name: Date:<br />

16.4 Series Circuits<br />

In a series circuit, current follows only one path from the positive end of the battery toward the negative end. The<br />

total resistance of a series circuit is equal to the sum of the individual resistances. The amount of energy used by<br />

a series circuit must equal the energy supplied by the battery. In this way, electrical circuits follow the law of<br />

conservation of energy. Understanding these facts will help you solve problems that deal with series circuits.<br />

To answer the questions in the practice section, you will have to use Ohm’s law. Remember that:<br />

Voltage (volts)<br />

Current (amps) =<br />

Resistance (ohms, Ω)<br />

Some questions ask you to calculate a voltage drop. We often say that each resistor (or light bulb) creates a<br />

separate voltage drop. As current flows along a series circuit, each resistor uses up some energy. As a result, the<br />

voltage gets lower after each resistor. If you know the current in the circuit and the resistance of a particular<br />

resistor, you can calculate the voltage drop using Ohm’s law.<br />

Voltage drop across resistor (volts) =<br />

Current through resistor (amps) × Resistance of one resistor (ohms)<br />

1. Use the series circuit pictured to the right to answer<br />

questions (a)–(e).<br />

a. What is the total voltage across the bulbs?<br />

b. What is the total resistance of the circuit?<br />

c. What is the current in the circuit?<br />

d. What is the voltage drop across each light bulb?<br />

(Remember that voltage drop is calculated by<br />

multiplying current in the circuit by the resistance of a<br />

particular resistor: V = IR.)<br />

e. Draw the path of the current on the diagram.<br />

2. Use the series circuit pictured to the right to answer<br />

questions (a)–(e).<br />

a. What is the total voltage across the bulbs?<br />

b. What is the total resistance of the circuit?<br />

c. What is the current in the circuit?<br />

d. What is the voltage drop across each light bulb?<br />

e. Draw the path of the current on the diagram.<br />

3. What happens to the current in a series circuit as more light bulbs are added? Why?<br />

4. What happens to the brightness of each bulb in a series circuit as additional bulbs are added? Why?<br />

16.4


Page 2 of 2<br />

5. Use the series circuit pictured to the right to<br />

answer questions (a), (b), and (c).<br />

a. What is the total resistance of the circuit?<br />

b. What is the current in the circuit?<br />

c. What is the voltage drop across each resistor?<br />

6. Use the series circuit pictured to the right to<br />

answer questions (a)–(e).<br />

a. What is the total voltage of the circuit?<br />

b. What is the total resistance of the circuit?<br />

c. What is the current in the circuit?<br />

d. What is the voltage drop across each light bulb?<br />

e. Draw the path of the current on the diagram.<br />

7. Use the series circuit pictured right to answer questions<br />

(a), (b), and (c). Consider each resistor equal to all others.<br />

a. What is the resistance of each resistor?<br />

b. What is the voltage drop across each resistor?<br />

c. On the diagram, show the amount of voltage in the<br />

circuit before and after each resistor.<br />

8. Use the series circuit pictured right to answer questions<br />

(a)–(d).<br />

a. What is the total resistance of the circuit?<br />

b. What is the current in the circuit?<br />

c. What is the voltage drop across each resistor?<br />

d. What is the sum of the voltage drops across the three<br />

resistors? What do you notice about this sum?<br />

9. Use the diagram to the right to answer<br />

questions (a), (b), and (c).<br />

a. How much current would be measured<br />

in each circuit if each light bulb has a<br />

resistance of 6 ohms?<br />

b. How much current would be measured<br />

in each circuit if each light bulb has a<br />

resistance of 12 ohms?<br />

c. What happens to the amount of current<br />

in a series circuit as the number of<br />

batteries increases?<br />

16.4


Name: Date:<br />

16.4 Parallel Circuits<br />

A parallel circuit has at least one point where the circuit divides, creating more than one path for current. Each<br />

path is called a branch. The current through a branch is called branch current. If current flows into a branch in a<br />

circuit, the same amount of current must flow out again. This rule is known as Kirchoff’s current law.<br />

Because each branch in a parallel circuit has its own path to the battery, the voltage across each branch is equal to<br />

the battery’s voltage. If you know the resistance and voltage of a branch you can calculate the current with Ohm’s<br />

Law (I = V/R).<br />

1. Use the parallel circuit pictured right to answer<br />

questions (a)–(d).<br />

a. What is the voltage across each bulb?<br />

b. What is the current in each branch?<br />

c. What is the total current provided by the battery?<br />

d. Use the total current and the total voltage to<br />

calculate the total resistance of the circuit.<br />

2. Use the parallel circuit pictured right to answer<br />

questions (a)–(d).<br />

a. What is the voltage across each bulb?<br />

b. What is the current in each branch?<br />

c. What is the total current provided by the battery?<br />

d. Use the total current and the total voltage to<br />

calculate the total resistance of the circuit.<br />

3. Use the parallel circuit pictured right to answer<br />

questions (a)–(d).<br />

a. What is the voltage across each resistor?<br />

b. What is the current in each branch?<br />

c. What is the total current provided by the<br />

batteries?<br />

d. Use the total current and the total voltage to calculate the total resistance of the circuit.<br />

4. Use the parallel circuit pictured right to answer<br />

questions (a)–(c).<br />

a. What is the voltage across each resistor?<br />

b. What is the current in each branch?<br />

c. What is the total current provided by the<br />

battery?<br />

16.4


Page 2 of 2<br />

In part (d) of problems 1, 2, and 3, you calculated the total resistance of each circuit. This required you to first<br />

find the current in each branch. Then you found the total current and used Ohm’s law to calculate the total<br />

resistance. Another way to find the total resistance of two parallel resistors is to use the formula shown below.<br />

Calculate the total resistance of a circuit containing two 6 ohm resistors.<br />

Given<br />

The circuit contains two 6 Ω resistors in parallel.<br />

Looking for<br />

The total resistance of the circuit.<br />

Relationships<br />

R<br />

total<br />

1. Calculate the total resistance of a circuit containing each of the following combinations of resistors.<br />

a. Two 8 Ω resistors in parallel<br />

b. Two 12 Ω resistors in parallel<br />

c. A 4 Ω resistor and an 8 Ω resistor in parallel<br />

d. A 12 Ω resistor and a 3 Ω resistor in parallel<br />

2. To find the total resistance of three resistors A, B, and C in parallel, first use the formula to find the total of<br />

resistors A and B. Then use the formula again to combine resistor C with the total of A and B. Use this<br />

method to find the total resistance of a circuit containing each of the following combinations of resistors.<br />

a. Three 8 Ω resistors in parallel<br />

=<br />

R × R<br />

R +<br />

R<br />

1 2<br />

1 2<br />

total<br />

b. Two 6 Ω resistors and a 2 Ω resistor in parallel<br />

c. A 1 Ω, a 2 Ω, and a 3 Ω resistor in parallel<br />

R<br />

=<br />

R × R<br />

R + R<br />

1 2<br />

1 2<br />

Solution<br />

R<br />

R<br />

total<br />

total<br />

6 Ω× 6 Ω<br />

=<br />

6 Ω+ 6 Ω<br />

= 3 Ω<br />

The total resistance is 3 ohms.<br />

16.4


Name: Date:<br />

16.4 Thomas Edison<br />

16.4<br />

Thomas Alva Edison holds the record for the most patents issued to an individual in the United States:<br />

1,093. He is famous for saying that “Genius is one percent inspiration and ninety-nine percent perspiration.”<br />

Edison’s hard work and imagination brought us the phonograph, practical incandescent lighting, motion pictures,<br />

and the alkaline storage battery.<br />

The young entrepreneur<br />

Thomas Alva Edison was born<br />

February 11, 1847, in Milan, Ohio, the<br />

youngest of seven children. His family<br />

moved to Port Huron, Michigan, in<br />

1854 and Thomas attended school<br />

there—for a few months. He was<br />

taught reading, writing, and simple arithmetic by his<br />

mother, a former teacher, and he read widely and<br />

voraciously. The basement was his first laboratory.<br />

When he was 13, Thomas started selling newspapers<br />

and candy on the train from Port Huron to Detroit.<br />

Waiting for the return train, he often read science and<br />

technology <strong>book</strong>s. He set up a chemistry lab in an<br />

empty boxcar, until he accidentally set the car on fire.<br />

At 16, Thomas learned to be a telegraph operator and<br />

began to travel the country for work. His interest in<br />

experiments and gadgets grew and he invented an<br />

automatic timer to send telegraph messages while he<br />

slept. About this time his hearing was deteriorating; he<br />

was left with only about 20 percent hearing in one ear.<br />

First a patent, then business<br />

In 1868 Edison arrived in Boston. His first patent was<br />

issued there for an electronic vote recorder. While the<br />

device worked very well, it was a commercial failure.<br />

Edison vowed that, in the future, he would only invent<br />

things he was certain the public would want.<br />

He moved on to New York, where he invented a<br />

“Universal Stock Printer” for which he was paid<br />

$40,000, a huge sum he found hard to comprehend.<br />

After developing some devices to improve telegraph<br />

communications, Edison had enough money to build a<br />

research lab in Menlo Park, New Jersey.<br />

The invention factory<br />

Edison’s facility had everything he needed for<br />

inventing: machine and carpentry shops, a lab, offices,<br />

and a library. He hired assistants who specialized<br />

where he felt he was lacking, in mathematics, for<br />

instance.<br />

The concept of a commercial research facility—an<br />

“invention factory” of sorts—was new. Some consider<br />

Menlo Park itself to be one of Edison’s most important<br />

inventions.<br />

It was there Edison invented the tin foil phonograph,<br />

the first machine to record and play back sounds.<br />

Next, he developed a practical, safe, and affordable<br />

incandescent light. The company he formed to<br />

manufacture and market this invention eventually<br />

became General Electric.<br />

In 1888, Edison opened an even larger research<br />

complex in West Orange, New Jersey. Here he<br />

improved the phonograph and created a device that<br />

“does for the eye what the phonograph does for the<br />

ear.” This was the first motion picture player.<br />

Not a man to be discouraged<br />

Not all of Edison’s ideas were successful. In the 1890s<br />

he sold all his stock in General Electric and invested<br />

millions to develop better methods of mining iron ore.<br />

He never was able to come up with a workable process<br />

and the investment was a loss.<br />

One of the most remarkable aspects of Edison’s<br />

character was his refusal to be discouraged by failure.<br />

The 3,500 note<strong>book</strong>s he kept illustrate his typical<br />

approach to inventing: brainstorm as many avenues as<br />

possible to create a product, try anything that seems<br />

remotely workable, and record everything. Failed<br />

experiments, he said, helped direct his thinking toward<br />

more useful designs.<br />

Edison also worked to create an efficient storage<br />

battery to use in electric cars. By the time his alkaline<br />

battery was ready, electric cars were uncommon. But<br />

the invention proved useful in other devices, like<br />

lighting railway cars and miners’ lamps. Edison’s last<br />

patent was granted when he was 83, the year before he<br />

died, and his last big undertaking was an attempt, at<br />

Henry Ford’s request, to develop an alternative source<br />

of rubber. He was still working on the project when he<br />

died in 1931.


Page 2 of 2<br />

Reading reflection<br />

1. Name three different avenues by which Thomas Edison received an education.<br />

2. What did Edison learn from his attempts to sell his first patented invention?<br />

3. Describe Edison’s “invention factory.”<br />

4. Name two important inventions that came out of Menlo Park.<br />

5. Describe the process Edison used to invent things.<br />

6. How did Edison view his projects that failed?<br />

7. How do you think the tin foil phonograph worked? Discuss and compare your ideas with a fellow member of<br />

your class.<br />

8. Research: Edison holds the record for the most patents issued to an individual in the United States. Use a<br />

library or the Internet to research three of his inventions that are not mentioned in this biography, and briefly<br />

describe each one.<br />

16.4


Page 1 of 2<br />

16.4 George Westinghouse<br />

16.4<br />

George Westinghouse was both an imaginative tinkerer and a bold entrepreneur. His inventions had a<br />

profound effect on nineteenth-century transportation and industrial development in the United States. His air<br />

brakes and signaling systems made railway systems safer at higher speeds, so that railroads became a<br />

practical method of transporting goods across the country. He promoted alternating current as the best means of<br />

providing electric power to businesses and homes, and his method became the worldwide standard.<br />

Westinghouse obtained 361 patents over the course of his life.<br />

A boyhood among machines<br />

George Westinghouse was born<br />

October 6, 1846, in Central Bridge,<br />

New York. When he was 10, his family<br />

moved to Schenectady, where his<br />

father opened a shop that<br />

manufactured agricultural machinery.<br />

George spent a great deal of time working and<br />

tinkering there.<br />

After serving in both the Union Army and Navy in the<br />

Civil War, Westinghouse attended college for three<br />

months. He dropped out after receiving his first patent<br />

in 1865, for a rotary steam engine he had invented in<br />

his father’s shop.<br />

An inventive train of thought<br />

In 1866, Westinghouse was aboard a train that had to<br />

come to a sudden halt to avoid colliding with a<br />

wrecked train. To stop the train, brakemen manually<br />

applied brakes to each individual car based on a signal<br />

from the engineer.<br />

Westinghouse believed there could be a safer way to<br />

stop these heavy trains. In April 1869, he patented an<br />

air brake that enabled the engineer to stop all the cars<br />

in tandem. That July he founded the Westinghouse Air<br />

Brake Company, and soon his brakes were used by<br />

most of the world’s railways.<br />

The new braking system made it possible for trains to<br />

travel safely at much higher speeds. Westinghouse<br />

next turned his attention to improving railway<br />

signaling and switching systems. Combining his own<br />

inventions with others, he created the Union Switch<br />

and Signal Company.<br />

Long-distance electricity<br />

Next, Westinghouse became interested in transmitting<br />

electricity over long distances. He saw the potential<br />

benefits of providing electric power to individual<br />

homes and businesses, and in 1884 formed the<br />

Westinghouse Electric Company. Westinghouse<br />

learned that Nikola Tesla had developed alternating<br />

current and he persuaded Tesla to join the company.<br />

Initially, Westinghouse met with resistance from<br />

Thomas Edison and others who argued that direct<br />

current was a safer alternative. But direct current<br />

could not be transmitted over distances longer than<br />

three miles. Westinghouse demonstrated the potential<br />

of alternating current by lighting the streets of<br />

Pittsburgh, Pennsylvania, and, in 1893, the entire<br />

Chicago World’s Fair. Afterward, alternating current<br />

became the standard means of transmitting electricity.<br />

From waterfalls to elevated railway<br />

Also in 1893, Westinghouse began yet another new<br />

project: the construction of three hydroelectric<br />

generators to harness the power of Niagara Falls on<br />

the New York-Canada border. By November 1895,<br />

electricity generated there was being used to power<br />

industries in Buffalo, some 20 miles away.<br />

Another Westinghouse interest was alternating current<br />

locomotives. He introduced this new technology first<br />

in 1905 with the Manhattan Elevated Railway in New<br />

York City, and later with the city’s subway system.<br />

An always inquiring mind<br />

The financial panic of 1907 caused Westinghouse to<br />

lose control of his companies. He spent much of his<br />

last years in public service. Westinghouse died in 1914<br />

and left a legacy of 361 patents in his name—the final<br />

one received four years after his death.


Page 2 of 2<br />

Reading reflection<br />

1. Where did George Westinghouse first develop his talent for inventing things?<br />

2. How did Westinghouse make it possible for trains to travel more safely at higher speeds?<br />

3. Why did Westinghouse promote alternating current over direct current for delivering electricity to<br />

businesses and homes?<br />

4. How did Westinghouse turn public opinion in favor of alternating current?<br />

5. Together with a partner, explain the difference between direct and alternating current. Write your<br />

explanation as a short paragraph and include a diagram.<br />

6. How did Westinghouse provide electrical power to the city of Buffalo, New York?<br />

7. Ordinary trains in Westinghouse’s time were coal-powered steam engines. How were Westinghouse’s<br />

Manhattan elevated trains different?<br />

8. Research: Westinghouse had a total of 361 patents to his name. Use a library or the Internet to find out about<br />

three inventions not mentioned in this brief biography, and describe each one.<br />

16.4


Name: Date:<br />

16.4 Lewis Latimer<br />

16.4<br />

Latimer, often called a “Renaissance Man,” was an accomplished African-American inventor receiving<br />

seven U.S. patents. His professional and personal achievements define him as a humanitarian, artist, and scientist.<br />

Son of former slaves<br />

Lewis Howard Latimer<br />

was born on September 4,<br />

1848 in Chelsea,<br />

Massachusetts. Latimer's<br />

parents had escaped from<br />

slavery in Virginia and<br />

moved north. In Boston,<br />

Latimer's father, George,<br />

was arrested and jailed<br />

because he was considered<br />

a fugitive. The<br />

Massachusetts Supreme Court ruled that he belonged<br />

to his owner in Virginia.<br />

The people of Boston protested and local supporters<br />

paid for his release. George was free. George and his<br />

wife settled in Chelsea where they started their family.<br />

In 1858, George, fearing he would be forced to return<br />

to slavery, went into hiding, leaving his family behind.<br />

Young Lewis Latimer attended grammar school in<br />

Chelsea and was a high-achieving student. As a<br />

teenager, Lewis lied about his age to join the Union<br />

Navy during the Civil War. After four years of military<br />

service, the war ended and Latimer was honorably<br />

discharged.<br />

Drafting a career<br />

Latimer looked for work in Boston and finally found a<br />

job as an office boy with a patent law firm, Crosby and<br />

Gould. He earned $3.00 per week. At the firm, Lewis<br />

studied the detailed patent drawings prepared by the<br />

draftsmen. Over time, he taught himself the drafting<br />

trade using the tools and <strong>book</strong>s available there.<br />

Latimer showed his drawings to his boss and secured a<br />

job as a draftsman earning $20.00 per week. He<br />

eventually became chief draftsman and worked at the<br />

firm for eleven years.<br />

During this time, Latimer created patent drawings for<br />

Alexander Graham Bell. He completed the drawings<br />

and submitted them only hours before a competing<br />

inventor. Bell was awarded the telephone patent in<br />

1876 due to Latimer's hard work and drafting skills.<br />

An enlightened inventor<br />

Latimer was not only a talented draftsman, but also a<br />

successful inventor. While at Crosby and Gould, he<br />

developed his first invention—mechanical improvements<br />

for railroad train water closets (also known as toilets!).<br />

After Crosby and Gould, Latimer worked as a<br />

draftsman at the Follandsbee Machine Shop. Here he<br />

met Hiram Maxim and was hired to work at Maxim's<br />

company, U.S. Electric Lighting. Maxim was an<br />

inventor searching for ways to improve Thomas<br />

Edison’s light bulb. Edison held the patent for the light<br />

bulb, but the life span of the bulb was very short.<br />

Maxim wanted to extend the life of the light bulb and<br />

turned to Latimer for help.<br />

Latimer taught himself the details of electricity. In<br />

1881, he invented carbon filaments to replace paper<br />

filaments in light bulbs. He then went on to improve<br />

the manufacturing process for carbon filaments. Now<br />

light bulbs lasted longer, were more affordable, and<br />

had more uses. Latimer oversaw the installation of<br />

electric street lights in North America and London. He<br />

became chief electrical engineer for U.S. Electric<br />

Lighting and supervised The Maxim-Westin Electric<br />

Lighting Company in London.<br />

Edison and beyond<br />

In 1885, Thomas Edison hired Latimer to work in the<br />

legal department of Edison Electric Light Company.<br />

Latimer was the chief draftsman and patent authority<br />

working to protect Edison's patents. He wrote the<br />

widely acclaimed electrical engineering <strong>book</strong> called<br />

Incandescent Electric Lighting: A Practical<br />

Description of the Edison System. Latimer became one<br />

of only 28 members of the “Edison Pioneers” and the<br />

only African-American member. The Edison Pioneers<br />

were the most highly regarded men in the electrical<br />

field. Edison's company eventually became the<br />

General Electric Company.<br />

Latimer’s additional inventions included an early<br />

version of the air conditioner; a locking rack for hats,<br />

coats, and umbrellas; and a <strong>book</strong> support. He was also a<br />

poet, musician, playwright, painter, civil rights activist,<br />

husband, and father. Latimer died in 1928 at age 80.


Page 2 of 2<br />

Reading reflection<br />

1. How did Lewis Latimer become a draftsman and electrical engineer?<br />

2. List Latimer's major inventions. What was his most important invention and why?<br />

3. Research: What is a “Renaissance man”? Why is Latimer referred to as a Renaissance man?<br />

4. Research: Latimer was an accomplished poet. Locate and identify the names of two of his poems.<br />

5. Research: When did the Edison Pioneers first meet? Locate an excerpt from the obituary published by the<br />

Edison Pioneers honoring Lewis Latimer.<br />

16.4


Name: Date:<br />

17.1 Magnetic Earth<br />

Earth’s magnetic field is very weak compared with the strength of the field on the surface of the ceramic magnets<br />

you probably have in your classroom. The gauss is a unit used to measure the strength of a magnetic field. A<br />

small ceramic permanent magnet has a field of a few hundred up to 1,000 gauss at its surface. At Earth’s surface,<br />

the magnetic field averages about 0.5 gauss. Of course, the field is much stronger nearer to the core of the planet.<br />

1. What is the source of Earth’s magnetic field according to what you have read in chapter 17?<br />

2. Today, Earth’s magnetic field is losing approximately 7 percent of its strength every 100 years. If the<br />

strength of Earth’s magnetic field at its surface is 0.5 gauss today, what will it be 100 years from now?<br />

3. Describe what you think might happen if Earth’s magnetic field continues to lose strength.<br />

4. The graphic to the right illustrates one piece of evidence that proves the<br />

reversal of Earth’s poles during the past millions of years. The ‘crust’ of<br />

Earth is a layer of rock that covers Earth’s surface. There are two kinds of<br />

crust—continental and oceanic. Oceanic crust is made continually (but<br />

slowly) as magma from Earth’s interior erupts at the surface. Newly formed<br />

crust is near the site of eruption and older crust is at a distance from the site.<br />

Based on what you know about magnetism, why might oceanic crust rock<br />

be a record of the reversal of Earth’s magnetic field? (HINT: What happens<br />

to materials when they are exposed to a magnetic field?)<br />

5. The terms magnetic south pole and geographic north pole refer to locations<br />

on Earth. If you think of Earth as a giant bar magnet, the magnetic south<br />

pole is the point on Earth’s surface above the south end of the magnet. The<br />

geographic north pole is the point where Earth’s axis of rotation intersects<br />

its surface in the northern hemisphere. Explain these terms by answering the<br />

following questions.<br />

a. Are the locations of the magnetic south pole and the geographic north<br />

pole near Antarctica or the Arctic?<br />

b. How far is the magnetic south pole from the geographic north pole?<br />

c. In your own words, define the difference between the magnetic south pole and the geographic north<br />

pole.<br />

6. A compass is a magnet and Earth is a magnet. How does the magnetism of a compass work with the<br />

magnetism of Earth so that a compass is a useful tool for navigating?<br />

17.1


Page 2 of 2<br />

7. The directions—north, east, south, and west—are arranged on a compass so that they align with<br />

360 degrees. This means that zero degrees (0°) and 360° both represent north. For each of the<br />

following directions by degrees, write down the direction in words. The first one is done for you.<br />

a. 45° Answer: The direction is northeast.<br />

b. 180°<br />

c. 270°<br />

d. 90°<br />

e. 135°<br />

f. 315°<br />

Magnetic declination<br />

Earth’s geographic north pole (true north) and magnetic south pole are located near each other, but they are not at<br />

the same exact location. Because a compass needle is attracted to the magnetic south pole, it points slightly east<br />

or west of true north. The angle between the direction a compass points and the direction of the geographic north<br />

pole is called magnetic declination. Magnetic declination is measured in degrees and is indicated on<br />

topographical maps.<br />

8. Let’s say you were hiking in the woods and relying on a map and compass to navigate. What would happen<br />

if you didn’t correct your compass for magnetic declination?<br />

9. Are there places on Earth where magnetic declination equals 0°? Use the Internet or your local library to find<br />

out where on Earth there is no magnetic declination.<br />

17.1


Name: Date:<br />

17.2 Model Maglev Train Project<br />

Magnetically levitating (Maglev) trains use electromagnetic force to lift the<br />

train above the tracks. This system greatly reduces wear because there are few<br />

moving parts that carry heavy loads. It’s also more fuel efficient, since the<br />

energy needed to overcome friction is greatly reduced. Although maglev<br />

technology is still in its experimental stages, many engineers believe it will<br />

become the standard for mass transit systems over the next 100 years.<br />

This project will give you an opportunity to create a model maglev train. You<br />

can even experiment with different means of providing power to your train.<br />

Materials<br />

• 52 one-inch square magnets with north and south poles on the faces, rather than ends (found at hobby shops)<br />

• One strip of 1/4-inch thick foam core, 24 inches long by 4 inches wide<br />

• Two strips of 1/4-inch thick foam core, 24 inches long by 2.5 inches wide<br />

• One strip of 1/4-inch thick foam core, 6 inches long by 3.75 inches wide<br />

• Hot melt glue and glue gun<br />

• Masking tape<br />

Directions<br />

1. Cut a strip of masking tape 24 inches long. Press a line of 24 magnets onto the tape, north sides up.<br />

2. Hold an additional magnet north side down and run it along the strip to make sure that the entire “track” will<br />

repel the magnet. Flip over any magnets that attract your test magnet.<br />

3. Glue the magnet strip along one long side of the 24-by-4-inch foam core rectangle.<br />

4. Repeat steps 1-2, then glue the<br />

second magnet strip along the<br />

opposite side to create the other<br />

track.<br />

5. Place a bead of hot glue along the<br />

cut edge and attach one 24-by-2.5<br />

inch foam core rectangle to form a<br />

short wall.<br />

17.2


Page 2 of 2<br />

6. Repeat step 5 to form the opposite wall. This keeps the train from sliding sideways off the track.<br />

7. To create your train, glue the south side of a magnet to each corner of the small foam core rectangle.<br />

8. Turn the train over so that the north side of its magnets face the tracks. Place your train above the track and<br />

watch what happens!<br />

Extensions:<br />

1. Experiment with various means to propel your train along the tracks. Consider using balloons, rubber bands<br />

and toy propellers, small motors (available at hobby stores) or even jet propulsion using vinegar and baking<br />

soda as fuel.<br />

2. Build a longer, more permanent track using plywood shelving. Use clear, flexible plastic for the front wall so<br />

that you can see the train floating above the track.<br />

3. Find out how much weight your train can carry. Are some propulsion systems able to carry more weight than<br />

others? Why?<br />

4. Have a design contest to see who can build the fastest train, or the train that can carry the most weight from<br />

one end of the track to the other.<br />

17.2


Name: Date:<br />

17.4 Michael Faraday<br />

17.4<br />

Despite little formal schooling, Michael Faraday rose to become one of England’s top research<br />

scientists of the nineteenth century. He is best known for his discovery of electromagnetic induction, which made<br />

possible the large-scale production of electricity in power plants.<br />

Reading his way to a job<br />

Michael Faraday was born<br />

on September 22, 1791, in<br />

Surrey, England, the son of<br />

a blacksmith. His family<br />

moved to London, where<br />

Michael received a<br />

rudimentary education at a<br />

local school.<br />

At 14, he was apprenticed<br />

to a <strong>book</strong>binder. He enjoyed<br />

reading the materials he<br />

was asked to bind, and found himself mesmerized by<br />

scientific papers that outlined new discoveries.<br />

A wealthy client of the <strong>book</strong>binder noticed this<br />

voracious reader and gave him tickets to hear<br />

Humphry Davy, the British chemist who had<br />

discovered potassium and sodium, give a series of<br />

lectures to the public.<br />

Faraday took detailed notes at each lecture. He bound<br />

the notes and sent them to Davy, asking him for a job.<br />

In 1812, Davy hired him as a chemistry laboratory<br />

assistant at the Royal Institution, London’s top<br />

scientific research facility.<br />

Despite his lack of formal training in science or math,<br />

Faraday was an able assistant and soon began<br />

independent research in his spare time. In the early<br />

1820s, he discovered how to liquefy chlorine and<br />

became the first to isolate benzene, an organic solvent<br />

with many commercial uses.<br />

The first electric motor<br />

Faraday also was interested in electricity and<br />

magnetism. After reading about the work of Hans<br />

Christian Oersted, the Danish physicist, chemist, and<br />

electromagnetist, he repeated Oersted’s experiments<br />

and used what he learned to build a machine that used<br />

an electromagnet to cause rotation—the first electric<br />

motor.<br />

Next, he tried to do the opposite, to use a moving<br />

magnet to cause an electric current. In 1831, he<br />

succeeded. Faraday’s discovery is called<br />

electromagnetic induction, and it is used by power<br />

plants to generate electricity even today.<br />

The Faraday effect<br />

Faraday first developed the concept of a field to<br />

describe magnetic and electric forces, and used iron<br />

filings to demonstrate magnetic field lines. He also<br />

conducted important research in electrolysis and<br />

invented a voltmeter.<br />

Faraday was interested in finding a connection<br />

between magnetism and light. In 1845 he discovered<br />

that a strong magnetic field could rotate the plane of<br />

polarized light. Today this is known as the Faraday<br />

effect.<br />

A scientist’s public education<br />

Faraday was a teacher as well as a researcher. When<br />

he became director of the Royal Institution laboratory<br />

in 1825, he instituted a popular series of Friday<br />

Evening Discourses. Here paying guests (including<br />

Prince Albert, who was Queen Victoria’s husband)<br />

were entertained with demonstrations of the latest<br />

discoveries in science.<br />

A series of lectures on the chemistry and physics of<br />

flames, titled “The Natural History of a Candle,” was<br />

among the original Christmas Lectures for Children,<br />

which continue to this day.<br />

Named in his honor<br />

Faraday continued his work at the Royal Institution<br />

until just a few years before his death in 1867. Two<br />

units of measure have been named in his honor: the<br />

farad, a unit of capacitance, and the faraday, a unit of<br />

charge.


Page 2 of 2<br />

Reading reflection<br />

1. What did Michael Faraday do to get a job with Humphry Davy? Why was this effort important in<br />

getting Faraday started in science?<br />

2. Research benzene and list two modern-day commercial uses for this chemical.<br />

3. Based on the reading, define electromagnetic induction.<br />

4. In your own words, describe the Faraday effect. In your description, explain the term “polarized light.”<br />

5. How did Faraday contribute to society during his time as the director of the Royal Institution laboratory?<br />

6. Name two ways in which Faraday’s work affects your own life in the twenty-first century.<br />

7. Imagine you could go back in time to see one of Faraday’s demonstrations. Explain why you would like to<br />

attend one of his demonstrations.<br />

8. Activity: Use iron filings and a magnet to demonstrate magnetic field lines, or prepare a simple<br />

demonstration of electromagnetic induction for your classmates.<br />

17.4


Name: Date:<br />

17.4 Transformers<br />

A transformer is a device used to change voltage and current. You may have<br />

noticed the gray electrical boxes often located between two houses or<br />

buildings. These boxes protect the transformers that “step down” high voltage<br />

from power lines (13,800 volts) to standard household voltage (120 volts).<br />

17.4 How a transformer works:<br />

1. The primary coil is connected to outside power lines. Current in<br />

the primary coil creates a magnetic field through the secondary<br />

coil.<br />

2. The current in the primary coil changes frequently because it is<br />

alternating current.<br />

3. As the current changes, so does the strength and direction of the<br />

magnetic field through the secondary coil.<br />

4. The changing magnetic field through the secondary coil induces<br />

current in the secondary coil. The secondary coil is connected to<br />

the wiring in your home.<br />

Transformers work because the primary and secondary coils have different numbers of turns. If the secondary<br />

coil has fewer turns, the induced voltage in the secondary coil is lower than the voltage applied to the primary<br />

coil. You can use the proportion below to figure out how number of turns affects voltage:<br />

17.4


Page 2 of 2<br />

A transformer steps down the power line voltage (13,800 volts) to standard household voltage<br />

(120 volts). If the primary coil has 5,750 turns, how many turns must the secondary coil have?<br />

Solution:<br />

V N<br />

=<br />

V N<br />

1 1<br />

2 2<br />

13,800 volts 5750 turns<br />

=<br />

120 volts N<br />

1. In England, standard household voltage is 240 volts. If you brought your own<br />

hair dryer on a trip there, you would need a transformer to step down the voltage<br />

before you plug in the appliance. If the transformer steps down voltage from 240<br />

to 120 volts, and the primary coil has 50 turns, how many turns does the<br />

secondary coil have?<br />

2. You are planning a trip to Singapore. Your travel agent gives you the proper<br />

transformer to step down the voltage so you can use your electric appliances<br />

there. Curious, you open the case and find that the primary coil has 46 turns and<br />

the secondary has 24 turns. Assuming the output voltage is 120 volts, what is the<br />

standard household voltage in Singapore?<br />

N<br />

3. A businessman from Zimbabwe buys a transformer so that he can use his own electric appliances on a trip to<br />

the United States. The input coil has 60 turns while the output coil has 110 turns. Assuming the input voltage<br />

is 120 volts, what is the output voltage necessary for his appliances to work properly? (This is the standard<br />

household output voltage in Zimbabwe.)<br />

4. A family from Finland, where standard household voltage is 220 volts, is planning a trip to Japan. The<br />

transformer they need to use their appliances in Japan has an input coil with 250 turns and an output coil<br />

with 550 turns. What is the standard household voltage in Japan?<br />

2<br />

5. An engineer in India (standard household voltage = 220 volts) is designing a transformer for use on her<br />

upcoming trip to Canada (standard household voltage = 120 volts). If her input coil has 240 turns, how many<br />

turns should her output coil have?<br />

6. While in Canada, the engineer buys a new electric toothbrush. When she returns home she designs another<br />

transformer so she can use the toothbrush in India. This transformer also has an input coil with 240 turns.<br />

How many turns should the output coil have?<br />

2<br />

= 50 turns<br />

17.4


Name: Date:<br />

17.4 Electrical Power<br />

How do you calculate electrical power?<br />

In this skill sheet you will review the relationship between electrical power and Ohm’s law. As you work through<br />

the problems, you will practice calculating the power used by common appliances in your home.<br />

During everyday life we hear the word watt mentioned in reference to things like light bulbs and electric bills.<br />

The watt is the unit that describes the rate at which energy is used by an electrical device. Energy is never created<br />

or destroyed, so “used” means it is converted from electrical energy into another form such as light or heat. Since<br />

energy is measured in joules, power is measured in joules per second. One joule per second is equal to one watt.<br />

To calculate the electrical power “used” by an electrical component, multiply the voltage by the current.<br />

A kilowatt (kWh) is 1,000 watts or 1,000 joules of energy per second. On an electric bill you may have noticed<br />

the term kilowatt-hour. A kilowatt-hour means that one kilowatt of power has been used for one hour. To<br />

determine the kilowatt-hours of electricity used, multiply the number of kilowatts by the time in hours.<br />

.<br />

• You use a 1,500-watt heater for 3 hours. How many kilowatt-hours of electricity did you use?<br />

You used 4.5 kilowatt-hours of electricity.<br />

1. Your oven has a power rating of 5,000 watts.<br />

Current × Voltage = Power, or P = IV<br />

1 kilowatt<br />

1,500 watts × = 1.5 kilowatts<br />

1,000 watts<br />

1.5 kilowatts × 3 hours =<br />

4.5 kilowatt-hours<br />

a. How many kilowatts is this?<br />

b. If the oven is used for 2 hours to bake cookies, how many kilowatt-hours (kWh) are used?<br />

c. If your town charges $0.15 per kWh, what is the cost to use the oven to bake the cookies?<br />

2. You use a 1,200-watt hair dryer for 10 minutes each day.<br />

a. How many minutes do you use the hair dryer in a month? (Assume there are 30 days in the month.)<br />

b. How many hours do you use the hair dryer in a month?<br />

c. What is the power of the hair dryer in kilowatts?<br />

d. How many kilowatt-hours of electricity does the hair dryer use in a month?<br />

e. If your town charges $0.15 per kWh, what is the cost to use the hair dryer for a month?<br />

3. Calculate the power rating of a home appliance (in kilowatts) that uses 8 amps of current when plugged into<br />

a 120-volt outlet.<br />

17.4


Page 2 of 2<br />

4. Calculate the power of a motor that draws a current of 2 amps when connected to a 12 volt battery.<br />

5. Your alarm clock is connected to a 120 volt circuit and draws 0.5 amps of current.<br />

a. Calculate the power of the alarm clock in watts.<br />

b. Convert the power to kilowatts.<br />

c. Calculate the number of kilowatt-hours of electricity used by the alarm clock if it is left on for one year.<br />

d. Calculate the cost of using the alarm clock for one year if your town charges $0.15 per kilowatt-hour.<br />

6. Using the formula for power, calculate the amount of current through a 75-watt light bulb that is connected<br />

to a 120-volt circuit in your home.<br />

7. The following questions refer to the diagram.<br />

a. What is the total voltage of the circuit?<br />

b. What is the current in the circuit?<br />

c. What is the power of the light bulb?<br />

8. A toaster is plugged into a 120-volt household circuit. It draws 5 amps of<br />

current.<br />

a. What is the resistance of the toaster?<br />

b. What is the power of the toaster in watts?<br />

c. What is the power in kilowatts?<br />

9. A clothes dryer in a home has a power of 4,500 watts and runs on a special 220-volt household circuit.<br />

a. What is the current through the dryer?<br />

b. What is the resistance of the dryer?<br />

c. How many kilowatt-hours of electricity are used by the dryer if it is used for 4 hours in one week?<br />

d. How much does it cost to run the dryer for one year if it is used for 4 hours each week at a cost of $0.15<br />

per kilowatt-hour?<br />

10. A circuit contains a 12-volt battery and two 3-ohm bulbs in series.<br />

a. Calculate the total resistance of the circuit.<br />

b. Calculate the current in the circuit.<br />

c. Calculate the power of each bulb.<br />

d. Calculate the power supplied by the battery.<br />

11. A circuit contains a 12-volt battery and two 3-ohm bulbs in parallel.<br />

a. What is the voltage across each branch?<br />

b. Calculate the current in each branch.<br />

c. Calculate the power of each bulb.<br />

d. Calculate the total current in the circuit.<br />

e. Calculate the power supplied by the battery.<br />

17.4


Page 1 of 2<br />

18.1 Andrew Ellicott Douglass<br />

Douglass, a successful American astronomer, is better known as the father of dendrochronology.<br />

His accomplishments in tree ring analysis and cross-dating allowed him to create a tree calendar<br />

dating back to AD 700 for the American Southwest.<br />

Vermont Native<br />

Andrew Ellicott Douglas<br />

was born on July 5, 1867 in<br />

Windsor, Vermont. Andrew<br />

was one of five children<br />

born to Sarah and Malcolm<br />

Douglass. Malcolm, an<br />

Episcopalian minister, and<br />

his wife moved frequently.<br />

They settled for a period of<br />

time in Windsor where<br />

Malcolm became a minister<br />

for St. Pauls Church and<br />

they raised their children.<br />

Douglass attended Trinity College in Hartford,<br />

Connecticut. An astronomer, Douglass worked at<br />

Harvard College Observatory from 1889-1894. While<br />

working for the observatory, he traveled to Peru to<br />

find a suitable location for another observatory. He<br />

helped to establish the Harvard Southern Hemisphere<br />

Observatory in Arequipa, Peru.<br />

From sunspots to tree rings<br />

When Douglass returned home, he met astronomer<br />

Percival Lowell of Boston, Massachusetts. Working for<br />

Lowell, Douglass set out again to find a location for an<br />

observatory, but this time in Arizona. In 1894, he found<br />

a site on a Flagstaff mesa and oversaw the building of<br />

the Lowell Observatory. While at the observatory,<br />

Douglass was Lowell’s chief assistant and worked with<br />

Lowell to observe the planet Mars. However, Douglass<br />

and Lowell did not always agree on how to use the<br />

gathered data and Lowell fired Douglass.<br />

Douglass remained in Flagstaff to teach Spanish and<br />

geography at what is now known as Northern Arizona<br />

University. While in Flagstaff, he became interested in<br />

tree rings and their possible connection to sunspot<br />

cycles. While researching the eleven-year sunspot<br />

cycle, he examined ponderosa pine tree rings. He<br />

noted that rings held information about weather<br />

patterns and hoped he could find a link between<br />

periods of drought and sunspot activity.<br />

18.1<br />

In 1906, Douglass moved to Tucson, Arizona and<br />

taught at the University of Arizona. Here, he created<br />

the science of dendrochronology. He found that<br />

differences in tree ring width corresponded to weather<br />

patterns. A period of drought produced narrower rings<br />

than a time of increased rainfall. In 1929, Douglass<br />

was able to place a date on Native American ruins in<br />

Arizona with accuracy. He studied Ponderosa pine tree<br />

rings dating back to the time of these Native American<br />

dwellings. He matched wooden beam samples against<br />

pine tree rings to determine a precise date for the<br />

ancient ruins. Douglass development of this crossdating<br />

technique was a tremendous breakthrough in<br />

the field of archaeology. Archaeologists now had a<br />

tool to date ancient ruins.<br />

Despite his work in tree ring analysis, Douglass<br />

remained an active astronomer. From 1918 to 1937,<br />

Douglass worked at the Steward Observatory at the<br />

University of Arizona. Within this period of time, he<br />

also wrote Climate Cycles and Tree Growth, Volumes<br />

I, II, and III. In 1937, he retired as director of the<br />

observatory and devoted his time to tree ring research.<br />

Dendrochronology and beyond<br />

Douglass quickly discovered that tree ring studies<br />

required time and physical space. He asked the<br />

University of Arizona president for a tree ring<br />

research facility. In 1938, Douglass became the first<br />

directory of the Laboratory of Tree-Ring Research at<br />

the University of Arizona. The Laboratory of Tree-<br />

Ring Research has the largest number of tree ring<br />

samples in the world. He remained director of the<br />

laboratory until 1958.<br />

In 1984, an asteroid was identified and named Minor<br />

Planet or Asteroid (2196) Ellicott, after Douglass<br />

middle name. Douglass died on March 20, 1962 at age<br />

94. Later, Spacewatch astronomer Tom Gehrels<br />

discovered an asteroid in 1998 using a telescope that<br />

Douglass had dedicated to the Steward Observatory<br />

many years earlier. A second asteroid was then named<br />

after Douglass. On the planet Mars, a crater has also<br />

been named in honor of Douglass.


Page 2 of 2<br />

Reading reflection<br />

1. How did Douglass move from studying planets and stars to studying trees?<br />

2. What is the name of the science and specific technique that Douglass discovered?<br />

3. How has Douglass work with tree rings been useful to archaeologists?<br />

4. Research: The first asteroid named after Douglass is called Minor Planet (2196) Ellicott. What is the name<br />

of the second asteroid named after Douglass?<br />

5. Research: The Harvard Southern Hemisphere Observatory, also called the Boyden Observatory, was<br />

originally located in Arequipa, Peru. It has moved. Where is the observatory now located?<br />

6. Research: Tom Gehrels is an astronomer associated with the Spacewatch program. What is the Spacewatch<br />

program?<br />

18.1


Name: Date:<br />

18.2 Relative Dating<br />

f<br />

Earth is very old and many of its features were formed before people came along to study them. For that reason,<br />

studying Earth now is like detective work—using clues to uncover fascinating stories. The work of geologists<br />

and paleontologists is very much like the work of forensic scientists at a crime scene. In all three fields, the<br />

ability to put events in their proper order is the key to unraveling the hidden story.<br />

Relative dating is a method used to determine the general age of a rock, rock formation, or fossil. When you use<br />

relative dating, you are not trying to determine the exact age of something. Instead, you use clues to sequence<br />

events that occurred first, then second, and so on. A number of concepts are used to identify the clues that<br />

indicate the order of events that made a rock formation.<br />

Sequencing events after a thunderstorm<br />

Carefully examine this illustration. It contains evidence of the following events:<br />

• The baking heat of the sun caused cracks to formed in the dried mud puddle.<br />

• A thunderstorm began.<br />

• The mud puddle dried.<br />

• A child ran through the mud puddle.<br />

• Hailstones fell during the thunderstorm.<br />

1. From the clues in the illustration, sequence the events listed above in the order in which they happened.<br />

2. Write a brief story that explains the appearance of the dried mud puddle and includes all the events. In your<br />

story, justify the order of the events.<br />

18.2


Page 2 of 4<br />

Determining the relative ages of rock formations<br />

Relative dating is an earth science term that describes the set of principles and techniques used to<br />

sequence geologic events and determine the relative age of rock formations. Below are graphics that<br />

illustrate some of these basic principles used by geologists. You will find that these concepts are easy to<br />

understand.<br />

Match each principle to its explanation. One relative dating term will be new to you! Which is it? There is one<br />

explanation that does not have a matching picture. Write the name of this explanation.<br />

Explanations<br />

3. In undisturbed rock layers, the oldest layer is at the bottom and the youngest layer is at the top.<br />

4. In some rock formations, layers or parts of layers may be missing. This is often due to erosion. Erosion by<br />

water or wind removes sediment from exposed surfaces. Erosion often leaves a new flat surface with some<br />

of the original material missing.<br />

5. Sediments are originally deposited in horizontal layers.<br />

6. Any feature that cuts across rock layers is younger than the layers.<br />

7. Sedimentary layers or lava flows extend sideways in all directions until they thin out or reach a barrier.<br />

8. Any part of a previous rock layer, like a piece of stone, is older than the layer containing it.<br />

9. Fossils can be used to identify the relative ages of the layers of a rock formation.<br />

18.2


Page 3 of 4<br />

Sequencing events in a geologic cross-section<br />

18.2<br />

Understanding how a land formation with its many layers of soil was created begins with the same<br />

time-ordering process you used earlier in this skill sheet. Geologists use logical thinking and geology principles<br />

to determine the order of events for a geologic formation. Cross-sections of Earth, like the one shown below, are<br />

our best records of what has happened in the past.<br />

Rock bodies in this cross-section are labeled A through H. One of these rock bodies is an intrusion. Intrusions<br />

occur when molten rock called magma penetrates into layers from below. The magma is always younger than the<br />

layers that it penetrates. Likewise, a fault is always younger than the layers that have faulted. A fault is a crack or<br />

break that occurs across rock layers, and the term faulting is used to describe the occurrence of a fault. The<br />

broken layers may move so that one side of the fault is higher than the other. Faulted layers may also tilt.<br />

10. List the rock bodies illustrated below in order based on when they formed.<br />

11. Relative to the other rock bodies, when did the fault occur?<br />

12. Compared with the formation of the rock bodies, when did the stream form? Justify your answer.


Page 4 of 4<br />

Extension—Creating clues for a story<br />

Collect some materials to use to create a set of clues that will tell a story. Examples of materials:<br />

construction paper, colored markers, tape, glue, scissors, different colors of modeling clay, different<br />

colors of sand or soil, rocks, an empty shoe box or a clear tank for clues.<br />

Then, give another group in your class the opportunity to sequence the clues into a story. Follow these guidelines<br />

in setting up your story:<br />

• Set up a situation that includes clues that represent at least five events.<br />

• Each of the five events must happen independently. In other words, two events cannot have happened at the<br />

same time.<br />

• Use at least one geology principle that you learned through this skill sheet.<br />

• Answer the questions below.<br />

13. Describe your set of clues in a paragraph. Include enough details in your paragraph so that someone can recreate<br />

the set of clues.<br />

14. What relative dating principles are represented with your set of clues? Explain how these principles are<br />

represented.<br />

15. Now, have a group of your classmates put your set of clues in order. When they are done, evaluate their<br />

work. Write a short paragraph that explains how they did and whether or not they figured out the correct<br />

sequence of clues. Describe the clue they missed if they made an error.<br />

18.2


Page 1 of 2<br />

18.2 Nicolas Steno<br />

18.2<br />

Nicolas Steno was a keen observer of nature at a time when many scientists were content to learn<br />

about the world by reading <strong>book</strong>s. Through dissection, Steno made important advances in the field of medicine.<br />

Later he applied his observational skills to the field of geology, identifying three important principles that<br />

geologists still use to determine the order in which geological events occurred.<br />

Steno’s childhood<br />

Nicolas Steno was born in<br />

1638 in Copenhagen,<br />

Denmark. He became ill at age<br />

three and spent most of his<br />

time indoors until age six. He<br />

saw few children, but spent<br />

time listening to adults discuss<br />

religion. Religion later<br />

became an important part of<br />

his life.<br />

Steno, the son of a goldsmith,<br />

had skillful hands like his father. However, his skill<br />

was not in making jewelry. He was an expert in<br />

dissecting animals to learn about anatomy. He was<br />

fascinated by the structure of living things.<br />

The young scientist<br />

When Nicolas was not yet ten years old, his father<br />

died. He spent his teen years living in Copenhagen<br />

with a half-sister and her husband. Steno was smart,<br />

curious, and a good listener. He gained the attention of<br />

two scholars in Copenhagen.<br />

The first scholar, Ole Borch, welcomed Steno into his<br />

alchemy laboratory. There, Steno watched as<br />

sediments settled out of liquid solutions. He thought it<br />

was interesting that even when the bottom of the jar<br />

was bumpy, the sediments formed a smooth horizontal<br />

layer on top of the bumpy surface.<br />

Thomas Bartholin, a famous anatomist from the<br />

University of Copenhagen, also mentored Steno.<br />

Perhaps through this friendship, Steno developed a<br />

keen interest in dissection and anatomy. In 1660, he<br />

left Denmark to study medicine at the University of<br />

Leiden in the Netherlands. There, through careful<br />

dissection of mammals, he made discoveries related to<br />

glands, ducts, the heart, brain, and muscles.<br />

A shark’s tooth unlocks a mystery<br />

In 1665, Steno moved to Italy. The following year,<br />

fishermen there captured a great white shark. The<br />

Italian Duke Ferdinand sent the head to Steno for<br />

dissection. Steno carefully observed the shark’s teeth.<br />

They looked like glossopetrae or “tongue stones,”<br />

common stony items found inside rocks.<br />

While we now know that these tongue stones are<br />

fossilized remains of living things, in Steno’s time<br />

many people believed tongue stones either grew inside<br />

rocks, fell from the sky, or even fell from the Moon.<br />

Steno suggested a different explanation for the tongue<br />

stones. He said they had once been actual shark teeth!<br />

Then Steno started to think about how a solid object,<br />

like a shark tooth, could get inside another solid<br />

object, like a rock.<br />

Three important principles<br />

Based on his work, Steno came up with three<br />

important principles of geology.<br />

• The principle of superposition says that layers of<br />

sediment settle on top of each other. The oldest layers<br />

are on the bottom and the more recent layers are on<br />

top.<br />

• The principle of original horizontality says that<br />

sedimentary rock layers form in horizontal patterns,<br />

even if they form on a bumpy surface.<br />

• The principle of lateral continuity says that sediment<br />

layers spread out until they reach something that stops<br />

the spreading.<br />

Steno explained that the shark teeth had been in soft<br />

sediment that eventually hardened into a layer of rock.<br />

Steno used his principles to write a <strong>book</strong> about the<br />

geology of a region of Italy called Tuscany. Even<br />

today, geologists use Steno’s principles to determine<br />

the order in which geologic events occurred.<br />

Father Steno<br />

In 1675, Steno gave up science to become a priest. He<br />

died in 1686 at the age of 48. In 1988, Pope John Paul<br />

II beatified Steno, the first step in the process of<br />

naming someone a saint. Today, the Steno Museum in<br />

Denmark and craters on both Mars and the Moon bear<br />

his name.


Page 2 of 2<br />

Reading reflection<br />

1. Name and briefly describe the three important principles of geology developed by Steno.<br />

2. How did most people in the 1600s explain the origin of fossils?<br />

3. How did Steno explain the existence of tongue stones or shark teeth in rocks?<br />

4. How did Steno’s medical background and skills help him with his geological discoveries?<br />

5. Observing is very important in science. What things do you like to observe? What have you learned through<br />

observation?<br />

6. Research: Steno’s father was a goldsmith and one of his teachers was interested in alchemy. What does a<br />

goldsmith do? What is alchemy? How could these two fields have been helpful to Steno’s work?<br />

18.2


Name: Date:<br />

18.3 The Rock Cycle<br />

In Section 18.3 of your student text, you will learn about the rock cycle. Place the three main groups of rocks in<br />

the ovals below. Then, fill in the blank lines with the materials or processes at work in each stage of the rock<br />

cycle. Use this diagram as a study aid. Describe to a friend or family member what is happening at each stage.<br />

18.3


Name: Date:<br />

19.1 Earth’s Interior<br />

19.1


Page 1 of 2<br />

19.1 Charles Richter<br />

19.1<br />

Richter is the most recognized name in seismology due to the earthquake magnitude scale he<br />

codeveloped. But Earth science was never a subject of interest to this bright young physicist, until a mentor<br />

made an interesting suggestion and a “happy accident” introduced him to seismology.<br />

The unexpected path<br />

Charles F. Richter was born<br />

on April 26, 1900 in<br />

Hamilton, Ohio. When he<br />

was 16, Charles and his<br />

mother left their Ohio farm<br />

and moved to Los Angeles.<br />

Richter attended the<br />

University of Southern<br />

California from 1916–1917,<br />

and then earned a bachelor’s<br />

degree in physics at Stanford<br />

University.<br />

It was during his Ph.D. studies in theoretical physics<br />

at the California Institute of Technology (Caltech) that<br />

Richter began his work in seismology, quite by<br />

accident.<br />

In 1927, Richter was working on his Ph.D. under the<br />

Nobel Prize winning physicist Dr. Robert Millikan.<br />

One day, Dr. Millikan called Richter into his office<br />

and presented him with an opportunity. The Caltech<br />

Seismology Laboratory was in need of a physicist, and<br />

although Richter had never done any Earth science<br />

work, Dr. Millikan thought he might be a good person<br />

for the job.<br />

Richter was a little surprised, but decided to talk to<br />

Harry Wood, the lead scientist in charge of the lab.<br />

Richter became intrigued and decided to join the<br />

seismology lab located in Pasadena, California.<br />

Richter described this introduction to the science that<br />

would become his life’s work as a “happy accident.”<br />

Doing something ordinary<br />

One of Charles Richter’s most famous sayings is<br />

based on looking back at his own life: “Don’t wait for<br />

extraordinary circumstance to do good; try to use<br />

ordinary situations.”<br />

When he first started at the seismology lab, Richter<br />

was busy with the routine work of measuring<br />

seismograms and locating earthquakes, so that a<br />

catalog of epicenters and occurrence times could be<br />

set up. At the time, this kind of earthquake study was<br />

new. Harry Wood was leading the effort to use<br />

southern California’s active seismic setting to gain a<br />

better understanding of earthquakes.<br />

This creative setting allowed Richter to attempt to<br />

develop new ways to “rate” earthquakes based on the<br />

seismic waves they produced. Since the lab used seven<br />

seismographs to record activity, Richter suggested that<br />

they compare quakes to one another using the amplitude<br />

of each quake measured at all seven locations. To do this,<br />

the seismic readings needed to be corrected to take into<br />

account the differences in distance from the epicenters.<br />

Richter had learned of a method to do this based on large<br />

earthquakes, but the magnitudes that Richter was<br />

studying ranged from tiny to very large.<br />

Collaboration and success<br />

Richter thought that the size difference in the<br />

magnitudes was unmanageably large. Fellow scientist<br />

Dr. Beno Gutenberg suggested that they plot the<br />

magnitudes using powers of 10. A magnitude two<br />

earthquake would represent 10 times the amplitude of<br />

ground motion of a magnitude one. A magnitude three<br />

would be 100 times a magnitude one, a four would<br />

be 1,000 times a magnitude one, and so on.<br />

Richter realized this was the obvious answer to his<br />

problem. When he used this method and graphed the<br />

results, it worked! At first it could be used only for<br />

southern California, because the system was only<br />

meant to compare quakes of that region using the<br />

seven seismographs in their lab.<br />

A new way to rate earthquakes<br />

In 1935, Richter and Gutenberg published their<br />

magnitude scale system. By 1936, they had worked<br />

out how their system could be used in all parts of the<br />

world, with any type of instrument. Before this, the<br />

Mercalli scale had been used to rate the magnitude of<br />

earthquakes, but it was based on local damage to<br />

buildings and people’s reactions to a quake.<br />

Richter and Gutenberg’s scale allowed for a more<br />

absolute and scientific method to be used by anyone,<br />

anywhere in the world.


Page 2 of 2<br />

Reading reflection<br />

1. Look up the definition of each boldface word in the article. Write down the definitions and be sure<br />

to credit your source.<br />

2. What do you think you would feel like if a world reknown scientist like Dr. Robert Millikan recommended<br />

you for a job? How would you feel if accepting that job meant that you could no longer work closely with<br />

Dr. Millikan?<br />

3. How did Richter respond to his new job?<br />

4. Who helped Richter refine his idea into a working model?<br />

5. Name a scale other than the Richter scale that scientists use to evaluate earthquakes.<br />

6. Research: Why do scientists use different scales to rate earthquakes?<br />

7. Research: What is the difference between a seismograph and a seismometer?<br />

19.1


Page 1 of 2<br />

19.1 Jules Verne<br />

19.1<br />

Jules Verne was an enormously successful nineteenth century author. He introduced the world to<br />

science fiction. His stories of adventure and imaginative methods of travel were decades ahead of their time. His<br />

ideas have entertained and inspired generations of readers. Several of his <strong>book</strong>s have been made into popular<br />

movies.<br />

A great imagination yearning for adventure<br />

Jules Verne was born on<br />

February 8, 1828 in the<br />

busy port city of Nantes,<br />

France. The oldest of five<br />

children, Jules came from a<br />

family with a strong<br />

seafaring tradition rich with<br />

the spirit for travel and<br />

adventure.<br />

The family’s summer home<br />

just outside the city of<br />

Nantes may have inspired Jules to search for adventure.<br />

The house was on the banks of the Loire River. Jules and<br />

his younger brother Paul would often play outside and<br />

watch ships from all over the world sail down the river.<br />

The boys would make up stories about these ships;<br />

where they were from, where they were going, the<br />

characters aboard the vessels, and especially the wild<br />

escapades they had during their journeys.<br />

While Jules’ father was part of a family that included<br />

many travelers, he did not intend his sons to follow in<br />

those footsteps. Both Jules and Paul were sent to a<br />

boarding school, right in their hometown of Nantes.<br />

There they were expected to get an education that<br />

would take them out of the seafaring class and into<br />

wealthy society.<br />

Expectations and creativity clash<br />

After graduating from the boarding school, Verne’s<br />

father sent him to Paris in 1847, where he was<br />

expected to study law. While he studied and prepared<br />

for the bar exam, Verne found his time was<br />

increasingly spent writing.<br />

An uncle that had been asked to check up on Verne<br />

saw that he was having some quiet success writing<br />

the words for operas. This uncle understood Verne’s<br />

true calling. He began to introduce Verne to people<br />

involved with Paris’ literary circles.<br />

Verne managed to get a few plays published and even<br />

performed. Although busy, he still was able to get his<br />

law degree. This came in handy, because as soon as<br />

Verne’s father found out about his writing, he<br />

furiously stopped sending his son money. With his<br />

money supply gone, Verne took a job as a stockbroker.<br />

He hated this job, yet was quite good at it.<br />

A career takes off<br />

Around this time Verne began to meet important<br />

authors like Alexander Dumas and Victor Hugo. They<br />

offered advice to the young writer. In 1857 Verne<br />

married, and was encouraged by his wife to pursue his<br />

dream of writing.<br />

Verne became a fan of Edgar Allen Poe, modelling<br />

some of his early work on Poe’s style, and in 1897 he<br />

wrote a sequel to one of Poe’s unfinished novels. In<br />

1862 Verne met Pierre-Jules Hetzel, an editor with a<br />

keen eye and feel for what a story needed to be<br />

successful.<br />

Verne’s writing had often been criticized for being too<br />

scientific. Hetzel knew how to make Verne’s stories<br />

appeal to the common person. In 1863, Verne began<br />

publishing his “Extraordinary Voyages” series of<br />

novels and thankfully quit his stockbroking job.<br />

In rapid succession Verne tackled the sky, the sea, the<br />

land, and even space in his novels. In 1863 he wrote<br />

Five Weeks in a Balloon, a story about exploring<br />

Africa in a hot air balloon. In 1864 he wrote Journey<br />

to the Center of the Earth, a trek by scientists down a<br />

volcano on their way to Earth’s core. In 1865 he wrote<br />

From Earth to the Moon, a visionary work that<br />

preceded NASA missions by 100 years. He published<br />

20,000 Leagues Under the Sea in 1869, introducing<br />

the world to Captain Nemo, a mysterious genius who<br />

built the futuristic submarine The Nautilus.<br />

Jules Verne’s 65 novels took readers on marvelous<br />

adventures, introducing futuristic ideas that while not<br />

always based on scientific facts, incorporated concepts<br />

that inspired future thinkers and entertained millions.<br />

Verne died in 1905, as the world’s most translated<br />

author, making up for his lack of scientific training<br />

and actual travel experience with a vivid imagination.


Page 2 of 2<br />

Reading reflection<br />

1. Why do you think Jules Verne’s novels appealed so widely to readers around the world?<br />

2. Research which novels written by Verne have been made into movies. Have any of them won awards?<br />

3. Research the bar exam. Why would Jules Verne need to pass it?<br />

4. Research Victor Hugo and explain why meeting him may have been important to Verne.<br />

5. Research some of the machines, ideas, and predictions Verne made in his novels that have come to exist<br />

today.<br />

19.1


Page 1 of 2<br />

19.2 Alfred Wegener<br />

19.2<br />

Alfred Wegener was a man ahead of his time. He was an astronomer and a meteorologist, yet his<br />

greatest work was in the field of earth science. His theory of plate tectonics is widely accepted today. Yet, in<br />

1912 when he proposed the idea, he was ridiculed. It took fifty years for other scientists to find the evidence that<br />

would prove his theory.<br />

The young man<br />

Alfred Wegener was born<br />

in Berlin in 1880. He was<br />

the son of a German<br />

minister who ran an<br />

orphanage. As a boy, he<br />

became interested in<br />

Greenland, and as a<br />

scientist, he went to<br />

Greenland several times to<br />

study the movement of air<br />

masses over the ice cap.<br />

This was at a time when<br />

most scientists doubted the existence of the jet stream.<br />

Just after his fiftieth birthday, he died there in a<br />

blizzard during one of his expeditions.<br />

Wegener graduated from the University of Berlin in<br />

1905 with a degree in astronomy. Soon, however, his<br />

interest shifted to meteorology. This was a new and<br />

exciting field of science. Wegener was one of the first<br />

scientists to track air masses using weather balloons.<br />

No doubt, he got the idea from his hobby of flying in<br />

hot air balloons. In 1906, he and his brother set a<br />

world record by staying up in a balloon for over fiftytwo<br />

hours.<br />

The search for evidence<br />

In 1910, in a letter to his future bride, Wegener wrote<br />

about the way that South America and Africa seemed<br />

to fit together like pieces of a puzzle. To Wegener, this<br />

was not just an odd coincidence. It was a mystery that<br />

he felt he must solve. He began to look for evidence to<br />

prove that the continents had once been joined<br />

together and had moved apart.<br />

Fossils of a small reptile had been found on the west<br />

coast of Africa and the east coast of South America.<br />

That meant that this reptile had lived in both places at<br />

the same time millions of years ago. Wegener figured<br />

that the only way this was possible was if the two<br />

continents were connected when animals were alive.<br />

They could not have traveled across the ocean.<br />

Geological evidence<br />

There was also geological evidence. The rock<br />

structures and types of rocks on the coasts of these two<br />

continents were identical. Again, Wegener could find<br />

no explanation for how this could have happened by<br />

accident on opposite sides of the ocean. The rock<br />

structures had to have been formed at the same time<br />

and place under the same conditions.<br />

A study of climates produced other evidence. Coal<br />

deposits had been found in Antarctica and in England.<br />

Since coal is formed only from plants that grow in<br />

warm, wet climates, Wegener concluded that those<br />

land masses must have once been near the equator, far<br />

from their locations today.<br />

Ridiculed and rejected<br />

Wegener explained that all of the continents had been<br />

part of one large land mass about 300 million years<br />

ago. This super-continent was called Pangaea, a Greek<br />

word that means “all earth.” It broke up over time, and<br />

the pieces have been drifting apart ever since.<br />

Wegener compared the drifting continents to icebergs.<br />

Wegener’s peers called his theory “utter rot!” Many<br />

scientists attacked him with rage and hostility.<br />

Wegener had two main problems. First, he was an<br />

unknown outsider, not a geologist, who was<br />

challenging everything that scientists believed at the<br />

time. Second, he was not able to explain what caused<br />

the continents to drift. While there seemed to be<br />

evidence to show that they had indeed moved, he<br />

could not identify a force that made it happen.<br />

About fifty years after Wegener proposed his theory, a<br />

scientist named Harry Hess made a discovery about<br />

sea floor spreading that seemed to support Wegener’s<br />

ideas. As a result, the theory of plate tectonics was<br />

finally accepted by most scientists.


Page 2 of 2<br />

Reading reflection<br />

1. Explain the significance of Greenland in Wegener’s life.<br />

2. What world record did Wegener set in 1906?<br />

3. Why could Wegener be called an interdisciplinary scientist? Identify the fields of science of which he was<br />

knowledgeable.<br />

4. Explain how the fossil of a small reptile provided evidence to help prove Wegener’s theory of drifting<br />

continents.<br />

5. How did the discovery of coal deposits in England and Antarctica strengthen Wegener’s argument?<br />

6. Research: In his search for evidence to support his theory of drifting continents, Wegener studied the rock<br />

strata in the Karroo section of South Africa and the Santa Catarina section of Brazil. He also studied the<br />

Appalachian Mountains in North America and the Scottish <strong>High</strong>lands. Use a library or the Internet to<br />

research these areas. What evidence do they provide for Wegener’s theory? Share your findings with the<br />

class.<br />

7. What were the two main problems that Wegener faced when he tried to convince others that his theory of<br />

drifting continents was valid?<br />

8. Research: Wegener and some colleagues drew maps of what they thought the world looked like at different<br />

times as the super continent broke up and the continents drifted apart. Use a library or the Internet to find<br />

pictures of these maps. Make a poster displaying Wegener’s vision of the world at:<br />

• 300 million years ago (Pangaea)<br />

• 225 million years ago (Permian period)<br />

• 200 million years ago (Triassic period)<br />

• 135 million years ago (Jurassic period)<br />

• 65 million years ago (Cretaceous period)<br />

• Today<br />

19.2


Page 1 of 2<br />

19.2 Harry Hess<br />

19.2<br />

Harry Hammond Hess was a geology professor at Princeton University and served many years in the<br />

U.S. Navy. In 1962, Hess published a landmark paper that described his theory of sea floor spreading. Hess<br />

also made major contributions to our national space program.<br />

A globe-trotting geologist<br />

Harry Hammond Hess was<br />

born in New York City on<br />

May 24, 1906. He first<br />

studied electrical<br />

engineering at Yale<br />

University, but later<br />

changed his major to<br />

geology. He received his<br />

degree in 1927.<br />

After graduation, Hess<br />

worked for two years as a<br />

mineral prospector in southern Rhodesia (currently<br />

Zimbabwe, Africa). He then returned to the United<br />

States to study at Princeton University. In 1932, Hess<br />

became a professor of geology at Princeton. Years<br />

later, his geological research took him to the far depths<br />

of the Pacific Ocean floor.<br />

The Navy commander<br />

Harry Hess was part of the Naval Reserve. In 1941 he<br />

was called to active duty. His first duty during World<br />

War II was in New York City where he tracked enemy<br />

positions in the North Atlantic. He later commanded<br />

an attack transport ship in the Pacific.<br />

Although he was a Naval commander, Hess seized the<br />

opportunity of being on a ship to further his geological<br />

research. Between battles, Hess and his crew gathered<br />

data about the structure of the ocean floor using the<br />

ship’s sounding equipment. They recorded thousands<br />

of miles worth of depth recordings.<br />

In 1945, Hess measured the deepest point of the ocean<br />

ever recorded—nearly 7 miles deep. He also<br />

discovered hundreds of flat-topped mountains lining<br />

the Pacific Ocean floor. He named these unusual<br />

mountains “guyouts” (after his first geology professor<br />

at Princeton).<br />

A ground breaking theory<br />

After the war, Hess continued to study guyouts,<br />

midocean ridges, and minerals. In 1959, his research<br />

led him to propose the ground breaking theory of sea<br />

floor spreading. At first, Hess’ idea was met with<br />

some resistance because little information was<br />

available to test this concept.<br />

In 1962, his sea floor spreading theory was published<br />

in a paper titled “History of Ocean Basins.” Hess<br />

explained that sea floor spreading occurs when molten<br />

rock (or magma) oozes up from inside the Earth along<br />

the mid-oceanic ridges. This magma creates new sea<br />

floor that spreads away from the ridge and eventually<br />

sinks into the deep oceanic trenches where it is<br />

destroyed. Hess’ theory became one of the most<br />

important contributions to the study of plate tectonics.<br />

The sea floor spreading theory explained many<br />

unsolved geological questions. Most geologists at the<br />

time believed that the oceans had existed for at least 4<br />

billion years. But they wondered why there wasn’t<br />

more sediment deposited on the ocean floor after such<br />

a long time period.<br />

Hess explained that the ocean floor is continually<br />

being recycled and that sediment has been<br />

accumulating for no more than 300 million years. This<br />

is about the time period needed for the ocean floor to<br />

spread from the ridge crest to the trenches. Hess’s<br />

theory helped geologists understand why the oldest<br />

fossils found on the sea floor are 180 million years old<br />

at most, while marine fossils found on land may be<br />

much older.<br />

From the ocean to the moon<br />

Harry Hess also played a key role in developing our<br />

country’s space program. In 1962, President John F.<br />

Kennedy appointed Hess as Chairman of the Space<br />

Science Board—a NASA advisory group. During the<br />

late 1960s, Hess helped plan the first landing of<br />

humans on the moon. He was part of a committee<br />

assigned to analyze rock samples brought back by the<br />

Apollo 11 crew.<br />

Harry Hess died in August 1969, only one month after<br />

the successful Apollo 11 lunar mission. He was buried<br />

in the Arlington National Cemetery. After his death,<br />

he was awarded NASA’s Distinguished Public Service<br />

Award.


Page 2 of 2<br />

Reading reflection<br />

1. How did Harry Hess’ career in the Navy contribute to his geological research?<br />

2. What were some of the geological discoveries Hess made while aboard his attack transport ship?<br />

3. Describe Hess’ theory of sea floor spreading.<br />

4. How did Hess’ sea floor spreading theory explain why so little sediment is deposited on the ocean floor?<br />

5. What were Hess’ contributions to space research?<br />

6. Research: Harry Hess made significant contributions in the fields of geology, geophysics, and mineralogy.<br />

What scientific society established the Harry H. Hess Medal and what achievements does it recognize?<br />

19.2


Page 1 of 2<br />

19.2 John Tuzo Wilson<br />

19.2<br />

John Tuzo Wilson was a professor at the University of Toronto whose love for adventure helped him<br />

make major contributions in the field of geophysics. His research on plate tectonics explained volcanic island<br />

formation and led to the discovery of transform faults. He also described the formation of oceans, a process later<br />

named the Wilson Cycle.<br />

A noteworthy family<br />

John Tuzo Wilson was born<br />

in Ottawa, Canada on<br />

October 24, 1908. His<br />

adventurous parents helped<br />

to expand Canada’s frontiers.<br />

Wilson’s mother, Henrietta<br />

Tuzo, was a famous<br />

mountaineer. Mount Tuzo in<br />

western Canada was named<br />

in her honor after she scaled<br />

its peak. Wilson’s father, also<br />

named John, helped plan the<br />

Canadian Arctic Expedition of 1913 to 1918. He also<br />

helped develop airfields throughout Canada.<br />

In 1930, Wilson was the first graduate of geophysics<br />

from the University of Toronto. He earned a second<br />

degree from Cambridge University. In 1936, Wilson<br />

received a doctorate in geology from Princeton<br />

University.<br />

An adventurous scholar<br />

Throughout his career, Wilson enjoyed traveling to<br />

unusual locations. While a student at Princeton,<br />

Wilson became the first person to scale Mount Hague<br />

in Montana—an elevation of 12,328 feet.<br />

When World War II broke, Wilson served in the Royal<br />

Canadian Army. After the war, Wilson led an<br />

expedition called Exercise Musk-Ox. He directed ten<br />

army vehicles 3,400 miles through the Canadian<br />

Arctic. This journey proved that people could travel to<br />

Canada’s north country.<br />

In 1946, Wilson began his 30-year career as a<br />

professor of geophysics at the University of Toronto.<br />

While a professor, Wilson mapped glaciers in<br />

Northern Canada. Between 1946 and 1947, he became<br />

the second Canadian to fly over the North Pole during<br />

his search for unknown Arctic islands.<br />

Plate tectonics and a hot idea<br />

Many scientists contributed to the development of the<br />

plate tectonics theory. However, they had difficulty<br />

explaining the formation of volcanic islands. These<br />

islands, like the Hawaiian Islands, are thousands of<br />

kilometers away from plate boundaries.<br />

In the early 1960s, Wilson solved the volcanic island<br />

mystery. He explained that sometimes a single hot<br />

mantle plume will break through a plate and form a<br />

volcanic island. As the plate moves over the mantle<br />

plume, a chain of islands forms. At first this theory was<br />

rejected. Finally, in 1963, Wilson published his paper.<br />

Slipping and sliding plates<br />

In 1965, Wilson proposed that a type of plate<br />

boundary must connect ocean ridges and trenches. He<br />

suggested that a plate boundary ends abruptly and<br />

transforms into major faults that slip horizontally.<br />

Wilson called these boundaries “transform faults.”<br />

Wilson’s idea was confirmed and quickly became a<br />

major milestone in the plate tectonics theory. The San<br />

Andreas Fault of southern California is a well-known<br />

transform fault.<br />

Opening and closing ocean basins<br />

Wilson was one of the first geologists to link seafloor<br />

spreading with land geology. In 1967, Wilson<br />

published an article that described the repeated<br />

process of ocean basins opening and closing. This<br />

process later became known as the Wilson Cycle.<br />

Geologists believe that the Atlantic Ocean basin<br />

closed millions of years ago. This event led to the<br />

formation of the Appalachian and Caledonian<br />

mountain systems. The basin later re-opened to form<br />

today’s Atlantic Ocean.<br />

An honored geologist<br />

Wilson’s contributions to the field of geophysics led<br />

to many honors and awards throughout his career. In<br />

1967, Wilson became the principle of Erindale<br />

College at the University of Toronto. From 1974 to<br />

1985, Wilson served as director of the worldrenowned<br />

Ontario Science Center. On April 15, 1993,<br />

Wilson died at age 84.


Page 2 of 2<br />

Reading reflection<br />

1. How did John Tuzo Wilson’s parents contribute to his passion for the outdoors?<br />

2. Why is Wilson sometimes referred to as an adventurous scholar?<br />

3. Describe Wilson’s theory of how volcanic islands are formed.<br />

4. What did Wilson discover about plate boundaries and the formation of faults?<br />

5. What is the Wilson Cycle? Give an example of this process.<br />

6. Research: On which continent are mountains named in honor of John Tuzo Wilson?<br />

19.2


Name: Date:<br />

19.3 Earth’s Largest Plates<br />

19.3


Name: Date:<br />

19.4 Continental United States Geology<br />

You have learned about the plates that make up the surface of Earth. You have also learned how plates are formed<br />

at mid-ocean ridges and are destroyed at subduction zones. Here is a very brief look at how plate tectonics<br />

formed the land mass that we call the United States. It covers only the last chapter of the Earth history of the 48<br />

contiguous states.<br />

The full history of the surface of Earth is a very long and complicated story. To give you an idea of the difficulty<br />

of understanding the full story, imagine this: A young child is given a new box of modeling clay. In the box are<br />

four sticks of differently colored clay. The child plays with the clay for hours making different figures. First a set<br />

of animals, then a fort, and so on. Between each idea, the child balls up all of the clay. Now imagine that it’s the<br />

next day and the ball of swirled clay colors is in your hand. Your task is to figure out what the child made and in<br />

what order.<br />

That sounds impossible, and it probably is. The amazing thing is that geologists have figured out a lot of the<br />

equally difficult story of Earth’s surface. We have a pretty good idea about how the early crust was formed. And<br />

we know that there was a super continent called Rodinia that formed before Pangaea, the last super continent.<br />

But like the child’s clay figures, the further back we look, the more the clues are mixed up.<br />

The last chapter<br />

Our story begins late. Most of the history of Earth has already passed. During this time rift valleys formed that<br />

split continents into smaller pieces. First the land moved apart on both sides of a rift valley. Then, once the rift<br />

valley opened wide enough, water flooded in and a new ocean was born. Underwater, the rift valley then became<br />

a mid-ocean ridge.<br />

At the same time, subducting plates acted like conveyor belts. Anything that was part of a subducting plate was<br />

carried toward the subduction zone. In this way continents were carried together. Collisions between continents<br />

welded them together. Mountain ranges formed at the point of contact.<br />

The combination of rifting and subduction worked together to form, destroy, and reform the early continents. You<br />

can see that the result is very much like playing with modeling clay.<br />

The craton<br />

Even though most of Earth’s history had passed,<br />

it was still an incredibly long time ago. Rifting<br />

had broken up Rodinia, but subduction had not<br />

yet formed Pangaea. The break-up of Rodinia<br />

left six continents scattered across the world<br />

oceans. These continents were not the continents<br />

that we see today. One of these, Gondwanaland,<br />

was larger than the others put together.<br />

19.4


Page 2 of 4<br />

Two other continents are important to our story. They are called Baltica and Laurentia. At the center of<br />

Laurentia was a core piece that was very old even then. This core piece is called the craton. The craton<br />

had been changed again and again, but it was stable inside Laurentia. Today the craton of Laurentia<br />

forms the central United States.<br />

You may wonder where these names came from. After all, these continents were gone many millions of years<br />

before humans appeared on Earth. They are modern names proposed and adopted by geologists.<br />

The first collision<br />

Rifting and subduction caused Baltica to move<br />

in a jerky path. Eventually, Baltica collided with<br />

Laurentia to form a larger combined continent.<br />

This new continent is called Laurasia. A high<br />

mountain range formed where the colliding<br />

continents made contact. This mountain range<br />

lay deep inside Laurasia. Today the remains of<br />

this high mountain range form our northern<br />

Appalachian Mountains.<br />

Gondwanaland collides<br />

Subduction continued to bring continents<br />

together. Next mighty Gondwanaland was drawn<br />

ever closer to Laurasia. Gondwanaland collided<br />

just below where Laurentia and Baltica collided<br />

with each other. This new collision raised<br />

another set of mountains that continued the<br />

northern Appalachians into what are now the<br />

southern Appalachians. The combined<br />

Appalachians were as high as the Himalayans of<br />

today! The super continent Pangaea was then<br />

complete and the lofty Appalachian Mountains<br />

stood near its center.<br />

Pangaea breaks up<br />

Pangaea did not remain together for very long, only a few tens of millions of years. The same rifting process that<br />

broke up Rodinia split the new super continent into smaller pieces. Our future East Coast had been deep inside<br />

the central part of Pangaea. But in the break-up, a rift valley split our eastern shore away from what is now<br />

Africa. Instead of an inland place, our East Coast became an eastern shore.<br />

The East Coast after Pangaea<br />

One of the amazing things in geology is how quickly mountain ranges are eroded away. After Pangaea broke up,<br />

the Appalachians completely eroded away. All that was left was a flat plain! The sediments produced from this<br />

erosion formed deep layers on the eastern shore and near-shore waters. These coastal margin sediments make up<br />

most of the eastern states today. But wait a minute; today we see rounded mountains where there had been only<br />

flat plains. What formed the rounded Appalachian Mountains of today?<br />

19.4


Page 3 of 4<br />

When a mountain is formed, some of it is pressed deep into the mantle by the weight of the mountain<br />

above. It’s like stacking wood blocks in water. As the stack grows taller, it also sinks deeper. Erosion 19.4<br />

takes a tall mountain down quickly. With the top gone, its bottom rebounds back to the surface. In this<br />

way, the Appalachian Mountains that we see today are actually the rebounded lower section of the mountains that<br />

once had been pressed deep below Earth’s surface.<br />

The West Coast and the Ancestral Rocky Mountains<br />

There are two Rocky Mountain ranges. The first is called the Ancestral Rocky Mountains. The Ancestral Rocky<br />

Mountains were formed when subduction caused an ancient collision with Laurentia. The collision struck<br />

Laurentia on the side that would become our western states. In other words, the Ancestral Rocky Mountains<br />

already existed before Pangaea formed. The Ancestral Rockies were then heavily weathered and the sediment<br />

deposited on the surrounding plains. Today the Front Range of Colorado is part of the exposed remains of the<br />

Ancestral Rocky Mountains.<br />

Pangaea and the West Coast<br />

Our West Coast did not exist as Pangaea began to break up. The shoreline was near the present eastern border of<br />

California. What would become our West Coast states were sediments and islands scattered in the ocean to the<br />

west.<br />

North America began to move westward as it<br />

was rifted apart from Pangaea. A subduction<br />

zone appeared in front of the moving continent.<br />

As the ocean floor dove under the westwardmoving<br />

continent, these sediments, islands and<br />

even pieces of ocean floor became permanently<br />

attached to the continent. Our western shore<br />

grew in this way, forming the shape that we see<br />

today.<br />

The modern Rocky Mountains<br />

The mid-ocean ridge that was forming the subducting ocean plate was not too far away to the west. As the plate<br />

subducted, the mid-ocean ridge got closer and closer to the edge of the continent. This changed the way that the<br />

plate subducted. The result was that stronger push pressure caused the continent to buckle well back from its<br />

edge. In this way, the modern Rocky Mountains were formed near the remains of the Ancestral Rocky<br />

Mountains.


Page 4 of 4<br />

Inland volcanoes<br />

The subducting plate also caused volcanoes to form and erupt<br />

inland. These eruptions produced the Sierra Nevada<br />

Mountains to the south and <strong>High</strong> Cascades to the north.<br />

A small plate disappears<br />

The plate that had been subducting along the southern West Coast was<br />

small. Eventually it disappeared when its mid-ocean ridge was<br />

subducted. This changed the western edge of the United States from a<br />

converging boundary to a transform boundary. Now instead of one<br />

plate diving under another, the remaining Pacific Plate slides by the<br />

West Coast. Today this slide-by motion is well known as the San<br />

Andreas Fault.<br />

When subduction stopped along the lower West Coast, the Sierra<br />

volcanoes became extinct. Magma cooled and solidified below the<br />

surface. Today this cooled magma is exposed as the domes of Yosemite<br />

National Park. Further north, the Pacific Plate is still subducting under<br />

the West Coast. That subduction continues to drive the volcanoes of the<br />

<strong>High</strong> Cascades.<br />

The United States today<br />

In geologic terms, the East Coast is quiet and the West Coast is active. The contiguous United States are part of<br />

the North American Plate. The active eastern boundary of the plate lies in the middle of the Atlantic Ocean, far<br />

from our East Coast. But the active western boundary is also our western shore. The San Andreas Fault slowly<br />

moves slivers of California northward. Baja California will eventually be attached to San Diego. Map makers<br />

won’t have to redraw New England, but they will have to watch for West Coast changes. The good news is that<br />

they’ll have plenty of time to make those changes.<br />

19.4


Name: Date:<br />

20.1 Averaging<br />

The most common type of average is called the mean. To find the mean, just add all the data, then divide the total<br />

by the number of items in the data set. This type of average is used daily by many people; teachers and students<br />

use it to average grades, meteorologists use it to average normal high and low temperatures for a certain date, and<br />

sports statisticians use it to calculate batting averages, among many other things.<br />

Seven students in Mrs. Ramos’ homeroom have part time jobs on the weekends. Some of them baby sit, some<br />

mow lawns, and others help their parents with their businesses. They all listed their hourly wages to see how their<br />

own pay compares to that of the others. Here is the list: $11.00, $4.50, $12.20, $5.25, $8.77, $15.33, $5.75. What<br />

is the average (mean) hourly wage earned by students in Mrs. Ramos’ homeroom?<br />

1. Find the sum of the data: $11.00 + $4.50 + $12.20 + $5.25 + $8.77 + $15.33 + $5.75 = $62.80<br />

2. Divide the sum ($62.80) by the number of items in the data set (7): $62.80 ÷ 7 ≈ $8.97<br />

3. Solution: The average hourly wage of the students in Mrs. Ramos’ homeroom is $8.97.<br />

1. Jill’s test grades in science class so far this grading period are: 77%, 64%, 88%, and 82%. What is her<br />

average test grade so far?<br />

2. The total team salaries in 2005 for teams in a professional baseball league are as follows: Team One,<br />

$63,015,833 (24 players); Team Two, $48,107,500 (24 players); Team Three, $81,029,500 (29 players);<br />

Team Four, $62,888,192 (22 players); Team Five, $89,487,426 (18 players). What is the average amount of<br />

money spent by a team in this league on players salaries in 2005?<br />

3. During a weekend landscaping job, Raul worked 8 hours, Ben worked 15 hours, Michelle worked 22 hours,<br />

Rosa worked 5 hours, and Sammie worked 15 hours. What was the average number of hours worked by one<br />

person during this landscaping job? If each worker was paid $12.00 an hour, what was the average pay per<br />

person for the job?<br />

4. The 8th grade girls basketball team at George Washington Carver Middle <strong>School</strong> played the team from<br />

Rockwood Valley Middle <strong>School</strong> last night. The Rockwood Valley team won, 53-37. Altogether, there were<br />

three girls who scored 11 points each, four who scored 8 points each, one who scored 6 points, two who<br />

scored 4 points each, four who scored 2 points each, three who scored one point each, and two girls who did<br />

not score at all. What is the average number of points scored by a player on either team?<br />

5. During a weekend car trip that covered 220 miles each way, Rowan kept track of the price per gallon of<br />

regular unleaded gasoline at different gas stations along the way. Here is the list he kept: $2.79, $3.23, $3.99,<br />

$2.89, $3.09, $2.99, $2.97, $3.11, $2.88, $3.01, $3.00, $2.99. What was the average price per gallon of gas<br />

among the different gas stations on the list?<br />

20.1


Name: Date:<br />

20.1 Finding an Earthquake Epicenter<br />

The location of an earthquake’s epicenter can be determined if you<br />

have data from at least three seismographic stations. One method of<br />

finding the epicenter utilizes a graph and you need to know the<br />

difference between the arrival times of the P- and S-waves at each of<br />

three seismic stations. Another method uses a formula and you need to<br />

know the arrival times and speeds of the P- and S-waves. The only<br />

other items you need to find an epicenter are a calculator, a compass,<br />

and a map.<br />

Finding the epicenter using a graph<br />

Table 1 provides the arrival time difference between P- and S-waves. Use this value to find the distance to the<br />

epicenter on the graph. Record the distance values in the table in the third column from the left.<br />

1<br />

2<br />

3<br />

Station<br />

name<br />

Table 1: Seismic wave arrival time and distance to the epicenter<br />

Arrival time difference<br />

between P- and S-waves<br />

15 seconds<br />

25 seconds<br />

42 seconds<br />

Distance to epicenter<br />

in kilometers<br />

Scale distance to<br />

epicenter in<br />

centimeters<br />

20.1


Page 2 of 5<br />

Locating the epicenter on a map<br />

Once you have determined the distance to the epicenter for three stations in kilometers, you can use a<br />

map to locate the epicenter. The steps are as follows:<br />

Step 1: Determine the radius of a circle around each seismographic station on a map. The radius will be<br />

proportional to distance from the epicenter. Use the formula below to convert the distances in kilometers to<br />

distances in centimeters. For this situation, we will assume that 100 kilometers = 1 centimeter. Record the scale<br />

distances in centimeters in the fourth column of Table 1.<br />

-----------------<br />

1 cm<br />

100 km<br />

Step 2: Draw circles around each seismic station. Use a geometric compass to make circles around each station.<br />

Remember that the radius of each circle is proportional to the distance to the epicenter.<br />

Step 3: The location where the three circles intersect is the location of the epicenter.<br />

=<br />

--------------------------------------------------------------x<br />

distance to epicenter in km<br />

20.1


Page 3 of 5<br />

Finding the epicenter using a formula<br />

To calculate the distance to the epicenter for each station, you will use the equation:<br />

Table 2 lists the variables that are used in the equation for finding the distance to the epicenter. This table also<br />

lists values that are given to you.<br />

Table 2: Variables for the equation to calculate the distance to the epicenter<br />

Variable What it means Given<br />

d p distance traveled by P-waves r p = 5 km/s<br />

r p<br />

t p<br />

d s<br />

r s<br />

t s<br />

speed of P-waves<br />

travel time of P-waves<br />

distance traveled by S-waves<br />

speed of S-waves<br />

travel time of S-waves<br />

Distance = Rate × Time<br />

For each of the practice problems, assume that the speed of the P-waves will be 5 km/s and the speed of the Swaves<br />

will be 3 km/s. Also, because the P- and S-waves come from the same location, we can assume the<br />

distance traveled by both waves is the same.<br />

distance traveled by P-waves = distance traveled by S-waves<br />

d p<br />

=<br />

Since the travel time for the S-waves is longer, we can say that,<br />

d s<br />

rp × tp = rs × ts travel time of S-waves = ( travel time of P-waves)<br />

+ ( extra time)<br />

ts = tp + ( extra time)<br />

rp × tp =<br />

rs × ( tp + extra time)<br />

r s = 3 km/s<br />

d p = d s<br />

20.1


Page 4 of 5<br />

20.1<br />

For each of the practice problems, assume that the speed of the P-waves is 5 kilometers per second, and<br />

the speed of the S-waves is 3 kilometers per second. The first problem is done for you. Show your work for all<br />

problems.<br />

1. S-waves arrive to seismographic station A 85 seconds after the P-waves arrive. What is the travel time for<br />

the P-waves?<br />

2. S-waves arrive to another seismographic station B 80 seconds after the P-waves. What is the travel time for<br />

the P-waves to this station?<br />

3. A third seismographic station C records that the S-waves arrive 120 seconds after the P-waves. What is the<br />

travel time for the P-waves to this station?<br />

4. From the calculations in questions 1, 2, and 3, you know the travel times for P-waves to three seismographic<br />

stations (A, B, and C). Now, calculate the distance from the epicenter to each of the stations using the speed<br />

and travel time of the P-waves. Use the equation: distance = speed × time.<br />

5. Challenge question: You know that the travel time for P-waves to a seismographic station is 200 seconds.<br />

a. What is the difference between the arrival times of the P- and S-waves?<br />

b. What is the travel time for the S-waves to this station?<br />

6. Table 3 includes data for three seismographic stations. Using this information, perform the calculations that<br />

will help you fill in the rest of the table, except for the scale distance row.<br />

Table 3: Calculating the distance to the epicenter<br />

Variables Station 1 Station 2 Station 3<br />

Speed of P-waves rp 5 km/s 5 km/s 5 km/s<br />

Speed of S-waves rs 3 km/s 3 km/s 3 km/s<br />

Time between the arrival of P-<br />

and S-waves<br />

ts - tp 70 seconds 115 seconds 92 seconds<br />

Total travel time of P-waves t p<br />

Total travel time of S-waves ts Distance to epicenter in<br />

kilometers<br />

Scale distance to epicenter in<br />

centimeters<br />

5 km<br />

× t p<br />

s<br />

3 km<br />

= × (tp + 85 s)<br />

s<br />

⎛ 5 km ⎞<br />

⎜ ⎟tp ⎝ s ⎠<br />

⎛ 3 km ⎞<br />

= ⎜ ⎟tp<br />

+ 255 km)<br />

⎝ s ⎠<br />

⎛ 2 km ⎞<br />

⎜ ⎟tp<br />

⎝ s ⎠<br />

= 255 km<br />

t =<br />

128 s<br />

d p , d s<br />

p


Page 5 of 5<br />

Once you have determined the distance to the epicenter for three stations in kilometers, you can use a<br />

map to locate the epicenter. The steps are as follows:<br />

Step 1: Determine the radius of a circle around each seismographic station on a map. The radius will be<br />

proportional to distance from the epicenter. Use the formula below to convert the distances in kilometers to<br />

distances in centimeters. For this situation, we will assume that 200 kilometers = 1 centimeter. Record the scale<br />

distances in centimeters in the last row of Table 3.<br />

-----------------<br />

1 cm<br />

200 km<br />

Step 2: Draw circles around each seismic station. Use a geometric compass to make circles around each station.<br />

Remember that the radius of each circle is proportional to the distance to the epicenter.<br />

Step 3: The location where the three circles intersect is the location of the epicenter.<br />

=<br />

--------------------------------------------------------------x<br />

distance to epicenter in km<br />

20.1


Name: Date:<br />

20.2 Volcano Parts<br />

20.2


Name: Date:<br />

20.3 Basalt and Granite<br />

As you read Section 20.3 of your student text, you will learn how basalt and granite form. You’ll learn about<br />

ways they are alike and ways that they are different. The Venn diagram below can help you organize this<br />

information. As you learn about these types of rock, place facts that apply to both in the space where the circles<br />

intersect. Place facts that apply to only one type of rock in its individual space. Use this diagram as a study aid.<br />

20.3


Name: Date:<br />

21.2 Calculating Concentration of Solutions<br />

What’s the difference between regular and extra-strength cough syrup? Is the rubbing alcohol in your parents’<br />

medicine cabinet 70% isopropyl alcohol, or is it 90% isopropyl alcohol? The differences in these and many other<br />

pharmaceuticals is dependent upon the concentration of the solution. Chemists, pharmacists, and consumers<br />

often find it necessary to distinguish between different concentrations of solutions. Concentration is commonly<br />

expressed as of solute per mass of solution, known as mass percent.<br />

Mass of solute<br />

Mass percent = 100<br />

Total mass of solution ×<br />

Remember that a solution is defined as a mixture of two or more substances that is homogenous at the molecular<br />

level. The solvent is the substance that is present in the greatest amount. All other substances in the solution are<br />

known as solutes.<br />

• What is the mass percent concentration of a solution made up of 12 grams of sugar and 300. grams of water?<br />

Solution:<br />

In this case, the solute is sugar (12 g), and the total mass of the solution is the mass of the sugar plus the<br />

mass of the water, (12 g + 300. g).<br />

Substituting into the formula, where c = the percent concentration, we have:<br />

12 g 12 g<br />

c = × 100 = × 100 = 3.8%<br />

12 g + 300 g 312 g<br />

The concentration of a solution of 12 grams of sugar and 300. grams of water is 3.8%.<br />

• How many grams of salt and water are needed to make 150 grams of a solution with a concentration of<br />

15% salt?<br />

Solution:<br />

Here, we are given the concentration (15%) and the total mass of the solution (150 g). We are trying to find<br />

the mass of the solute (salt). Substituting into the same formula, where m is the mass of the salt, we have:<br />

m m<br />

15% = × 100, so 0.15 = , and 0.15 × (150 g) = m = 22.5 g<br />

150 g 150 g<br />

Since the total mass of the solution is 150 grams, and we now know that 22.5 grams are salt, that leaves:<br />

150 grams solution - 22.5 grams salt = 127.5 grams of water<br />

To make 150 grams of a solution with a concentration of 15% salt, you would need 22.5 grams of salt<br />

and 127.5 grams of water.<br />

21.2


Page 2 of 2<br />

Find the mass percent concentration of each solution or mixture.<br />

1. 5 grams of salt in 75 grams of water<br />

2. 40 grams of cinnamon in 2,000 grams of flour<br />

3. 1.5 grams of chocolate milk mix in 250 grams of 1% milk<br />

Find the mass of the solute in each situation.<br />

4. 1,000 grams of a 40% salt water solution<br />

5. 30 grams of a 12.5% sugar water solution<br />

6. 555 grams of a 25% sand and soil “solution”<br />

Carefully read and answer each of the following questions.<br />

7. Dawn is mixing 450 grams of dishwashing liquid with 600 grams of water to make a solution for her little<br />

brother to blow bubbles. What is the concentration of the dishwashing liquid?<br />

8. How many grams of glucose are needed to prepare 250 grams of a 5% glucose and water solution?<br />

9. Jill mixes 4 grams of vanilla extract into the 800 grams of cake batter she has prepared. What is the<br />

concentration of vanilla in her “solution” of cake batter?<br />

10. Challenge: Find the amount of red food coloring (in grams) necessary to add to 50 grams of water to prepare<br />

a 15% solution of red food coloring in water.<br />

21.2


Name: Date:<br />

21.2 Solubility<br />

In this skill sheet you will practice solving problems about solubility. You will use solubility values to identify<br />

solutions that are saturated, unsaturated, or supersaturated. Finally, you will practice your skills in interpreting<br />

temperature-solubility graphs.<br />

What is solubility?<br />

A solution is defined as a mixture of two or more substances that is homogenous at the molecular level. The<br />

substance present in the greatest amount is called the solvent. The other substances are known as solutes.The<br />

degree to which a solute dissolved is described by its solubility value. This value is the mass in grams of the<br />

solute that can dissolve in a given volume of solvent under certain conditions.<br />

For example, the solubility of table salt (NaCl) is 1 gram per 2.7 milliliters of water at 25 °C. Another way to<br />

state this solubility value is to say that 0.37 grams of salt will dissolve in one milliliter of water at 25 °C. Do you<br />

see that these values mean the same thing?<br />

1 gram NaCL 0.37 gram NaCL<br />

=<br />

2.7 ml H 0 i 25 ° C 2.7 ml H 0 i<br />

25 ° C<br />

2 2<br />

Information about the solubility of table salt and other substances is presented in the table below. Use these<br />

values to complete the questions that follow.<br />

Substance Solubility Value (grams/100 mL water 25°C)<br />

Table salt (NaCl) 37<br />

Sugar (C 12H 22O 11) 200<br />

Baking soda (NaHCO 3) 10<br />

Chalk (CaCO 3) insoluble<br />

Talc (Mg silicates) insoluble<br />

1. Chalk and talc are listed as “insoluble” in the table. What do you think this term means? In your response,<br />

come up with a reason to explain why chalk and talc cannot dissolve in water.<br />

2. Come up with a reason to explain why table salt, sugar, and baking soda dissolve in different amount for the<br />

same set of conditions (same volume and temperature).<br />

3. How much table salt would dissolve in 540 mL of water if the water was 25 °C?<br />

4. What volume of water would you need to dissolve 72 grams of salt at 25 °C?<br />

5. What volume of water at 25 °C would you need to dissolve 50 grams of sugar?<br />

6. How much baking soda will dissolve in 10 milliliters of water at 25 °C?<br />

21.2


Page 2 of 3<br />

Saturated, unsaturated, and supersaturated solutions<br />

The solubility value for a substance indicates how much solute is present in a saturated solution. When the<br />

amount of solute is less than the solubility value for a certain volume of water, we say the solution is unsaturated.<br />

When the amount of solute is more than the solubility value for a certain volume of water, we say the solution is<br />

supersaturated.<br />

Use the table on the previous page to help you fill in the table below. In each situation, is the solution saturated,<br />

unsaturated, or supersaturated?<br />

Substance Amount of substance in<br />

200 mL of water at 25°C<br />

Table salt (NaCl) 37 grams<br />

Sugar (C 12H 22O 11) 500 grams<br />

Baking soda (NaHCO 3) 20 grams<br />

Table salt (NaCl) 100 grams<br />

Sugar (C 12H 22O 11) 210 grams<br />

Baking soda (NaHCO 3) 25 grams<br />

Saturated, unsaturated, or<br />

supersaturated?<br />

21.2


Page 3 of 3<br />

The influence of temperature on solubility<br />

Have you noticed that sugar dissolves much easier in hot tea than in iced tea? The solubility of some substances<br />

increases greatly as the temperature of the solvent increases. For other substances, the dissolving rate changes<br />

very little. A solubility graph (sometimes called a solubility curve) can be used to show how temperature affects<br />

solubility.<br />

Below is a table of some imaginary substances dissolved in water at different temperatures. Study the table and<br />

then answer the questions.<br />

Substance dissolved in<br />

100 mL of water<br />

Solubility values (grams per 100 mL of water)<br />

at different temperatures<br />

10 °C 30 °C 50 °C 70 °C 90 °C<br />

gas A 0.2 0.2 0.1 0.08 0.05<br />

gas B 0.1 0.05 0.02 0.01 0.005<br />

solid A 30 32 40 55 74<br />

solid B 40 43 39 41 45<br />

1. Use graph paper to make two solubility graphs of the data in the table. On one graph, plot the data for gases<br />

A and B. On the other graph, plot the data for solids A and B. Use two different colors to plot the data for A<br />

and for B. Label the x-axis, “Temperature (°C).” Label the y-axis, “Solubility value (grams/100 mL H 2O).”<br />

2. How does the solubility of gases A and B differ from the solubility of solids A and B in water? Explain your<br />

response.<br />

3. For which substance does temperature seem to have the greatest influence on solubility?<br />

4. For which substance does temperature seem to have the least influence?<br />

5. If you had 500 mL of water at 70°C and you made a saturated solution by adding 205 grams of a substance,<br />

which of the substances above would you be adding?<br />

6. Organisms that live in ponds and lakes depend on dissolved oxygen to survive. Explain how the amount of<br />

dissolved oxygen in a pond or lake might vary with the seasons (winter, spring, summer, and fall). Justify<br />

your ideas.<br />

21.2


Name: Date:<br />

21.2 Salinity and Concentration Problems<br />

Bodies of water like ponds, lakes, and oceans contain solutions of dissolved substances. Often these substances<br />

are in small quantities, measured in parts per thousand (ppt), parts per million (ppm), and parts per billion (ppb).<br />

This skill sheet will provide you with practice in using these quantities and in doing calculations with them.<br />

Unit conversions<br />

Table 1 below provides unit conversions that will be helpful to you as you complete this skill sheet.<br />

Review: working with small concentrations<br />

When working with small concentrations, remember that the units of the numerator and denominator must<br />

match, as shown in the examples below.<br />

A. Parts per thousand (ppt)<br />

Example: 0.009 grams of phosphate in about 1000 grams of oxygenated water makes a solution that has an<br />

phosphate concentration of 0.009 ppt.<br />

B. Parts per million (ppm)<br />

Example: A good level of oxygen in a pond is 9 ppm. This means that there are 9 milligrams of oxygen for every<br />

one liter (1000 grams) of oxygenated water.<br />

C. Parts per billion (ppb)<br />

Table 1: Unit Conversions<br />

Milligrams = Grams = Kilograms = Liters of water<br />

1 0.001 0.000 001 0.000 001<br />

10 0.01 0.000 01 0.000 01<br />

1,000 1 0.001 0.001<br />

1,000,000 1,000 1 1<br />

1,000,000,000 1,000,000 1,000 1,000<br />

9<br />

----------------------------milligrams<br />

1 liter<br />

0.009 grams<br />

---------------------------- = 0.009 ppt<br />

1,000 grams<br />

=<br />

9<br />

----------------------------milligrams<br />

= --------------------------------------------------<br />

9 milligrams<br />

= 9 ppm<br />

1,000 grams 1,000,000 milligrams<br />

Example: The concentration of trace elements in seawater is very low. For example, the concentration of iron in<br />

seawater is 0.06 ppb. This means that there are 0.06 mg of iron in 1,000 liters of water. One thousand liters is<br />

equal to 1,000 times 1,000 grams of seawater.<br />

0.06<br />

------------------------------------milligrams<br />

1,000 liters<br />

------------------------------------------------<br />

0.06 milligrams 0.06 milligrams 0.06 milligrams<br />

= = --------------------------------------- =<br />

------------------------------------------------------------- = 0.06 ppb<br />

1,000 × 1,000 grams 1,000,000 grams 1,000,000,000 milligrams<br />

21.2


Page 2 of 3<br />

Work through these example problems and check your answers. Then you will be ready to try the practice<br />

problems on your own.<br />

• There are 16 grams of salt in 984 grams of water. What is the salinity of this solution?<br />

Solution:<br />

• A liter of solution has a salinity of 40 ppt. How many grams of salt are in the solution? How many grams of<br />

pure water are in the solution?<br />

Solution:<br />

• You measure the salinity of a seawater sample to be 34 ppt. How many grams of salt are in this sample if the<br />

mass is 2 kilograms?<br />

Solution: First, remember that there are 1,000 grams per kilogram. If a solution is given in parts per<br />

thousand, you can think of it as “grams per 1,000 grams” or “grams per kilogram.” Therefore, you can set up<br />

a proportion like this:<br />

Next, solve for x.<br />

16 grams salt 16 grams salt<br />

salinity = = = 16 ppt<br />

984 grams water + 16 grams salt 1,000 grams solution<br />

40 grams salt 40 grams salt<br />

40 ppt = =<br />

1,000 grams solution 40 grams salt + x grams water<br />

1,000 grams solution = 40 grams salt + x grams water<br />

1,000 grams solution - 40 grams salt = 960 grams water<br />

34 grams salt x grams salt<br />

=<br />

1 kilogram solution 2 kilograms solution<br />

34 grams salt ×<br />

2 kilograms solution<br />

x =<br />

1 kilogram solution<br />

x = 68 grams salt<br />

21.2


Page 3 of 3<br />

For each problem, show your work.<br />

1. Complete Table 2 below:<br />

Place Salinity<br />

(ppt)<br />

Salton Sea<br />

44<br />

California<br />

Great Salt Lake 280<br />

Utah<br />

Mono Lake 210<br />

California<br />

Pacific Ocean 87<br />

Table 2: Salinity of Famous Places<br />

Amount of salt in 1 liter<br />

(grams)<br />

2. How many grams of salt are in 2 liters of seawater that has a salinity of 36 ppt?<br />

Amount of pure water in 1 liter<br />

(grams)<br />

3. A one-liter sample of seawater contains 10 grams of salt. What is the salinity of this sample?<br />

4. You want to make a salty solution that has the same salinity as the Dead Sea. The salinity of the Dead Sea is<br />

210 ppt. Write a recipe for how you would make 2 liters of this solution.<br />

5. Five kilograms of seawater contains 30 grams of salt. What is the salinity of the volume of seawater?<br />

6. You measure the salinity of a seawater sample to be 30 ppt. How many grams of salt are in this sample if the<br />

mass is 1.5 kilograms?<br />

7. A solution has 2 grams of a substance in 1,000,000 grams of solution. Would you describe the concentration<br />

of the substance in solution as 2 parts per million or parts per billion?<br />

8. A solution has 5 grams of a substance in 1,000,000,000 grams of solution. Would you describe the<br />

concentration of the substance as 5 ppb or 5 ppm?<br />

9. Menthol is a substance that tastes sweet and minty and causes a cooling effect on your tongue. The taste<br />

threshold for menthol is 400 ppb. Could you taste menthol if there were 400 milligrams in 1,000,000 grams<br />

of menthol solution? Could you taste menthol if there were 400 milligrams in 1000 liters of menthol<br />

solution?<br />

10. Above-ground pipelines are used to transport natural gas, an important energy source. Gas leaks are<br />

potential problems with the pipelines. German Shepherd dogs can be trained to detect the gas leaks. The<br />

dogs sniff along the pipeline and then indicate a leak by perking up their ears or pawing the ground. The<br />

most sensitive electronic devices can detect gas leaks as low as 50 ppm. A German Shepherd can detect a<br />

gas leak as low as 1 ppb. How many times more sensitive is the dog as compared to the electronic device?<br />

21.2


Name: Date:<br />

21.3 Calculating pH<br />

The pH of a solution is a measure of the concentration of hydrogen ions (H+) in the solution. The pH scale,<br />

which ranges from 0 to 14, provides a tool to assess the degree to which a solution is acidic or basic. As you may<br />

remember, solutions with low pH values are very acidic and contain high concentrations of hydrogen ions. Why<br />

does a low pH value mean a high concentration of H+? The answer has to do with what pH means<br />

mathematically. In this skill sheet, we will examine how pH values are calculated.<br />

How do you calculate pH?<br />

The pH value for any solution is equal to the negative logarithm of the hydrogen ion (H+) concentration in that<br />

solution. The formula is written this way:<br />

pH – log H +<br />

= [ ]<br />

Concentration of hydrogen ions is implied by placing brackets (“[ ]”) around H+.<br />

A term used by scientists to describe the concentration of a substance in a solution is molarity. Molarity (M)<br />

means how many moles of a substance are present in a given volume of solution.<br />

For hydrogen ions in solutions, the concentration generally ranges from 10 to 10 –14 M. The larger the molarity,<br />

the greater the concentration of H+ in the solution. If a solution had a H + concentration of 10 –3 M, the<br />

corresponding pH value would be:<br />

pH – log 10 3 –<br />

= [ ]<br />

10 pH<br />

10 3 –<br />

– [ ]<br />

For a solution with an H + concentration of 10 –5 M, the corresponding pH value would be:<br />

pH<br />

=<br />

=<br />

pH = 3<br />

– [ – 3]<br />

pH – log 10 5 –<br />

= [ ]<br />

10 pH<br />

pH<br />

=<br />

=<br />

pH =<br />

5<br />

10 5 –<br />

– [ ]<br />

– [ – 5]<br />

The first solution has a higher H + concentration than the second solution (10 –3 M versus 10 –5 M); however, its<br />

pH value is a smaller number. Strong acids have small pH values. Larger pH values (like 14) have lower<br />

concentrations of H + , and the solutions represent weaker acids.<br />

21.3


Page 2 of 2<br />

1. Practice working with numbers that have exponents. In the blank provided, write greater than, less<br />

than, or equals.<br />

a. 10 –2 ____________________ 10 –3<br />

b. 10 –14 ____________________ 101 c. 10 –7 ____________________ 0.0000001<br />

d. 10 0 ____________________ 101 2. Solutions that range in pH from 0 to 7 are acidic. Solutions that range in pH from 7 to 14 are basic. Solutions<br />

that have pH of 7 are neutral. The hydrogen ion concentrations for some solutions are given below. Use the<br />

pH formula to determine which is an acid, which is a base, and which is neutral.<br />

a. Solution A: The hydrogen ion concentration is equal to 10 -1 M.<br />

b. Solution B: The hydrogen ion concentration is equal to 0.0000001 M.<br />

c. Solution C: The hydrogen ion concentration is equal to 10 –13M. 3. Orange juice has a hydrogen ion concentration of approximately 10 –4 M. What is the pH of orange juice?<br />

4. Black coffee has a hydrogen ion concentration of roughly 10 –5 M. Is black coffee a stronger or weaker acid<br />

than orange juice? Justify your answer and provide all relevant calculations for supporting evidence.<br />

5. Pure water has a hydrogen ion concentration of 10 –7 M. What is the pH of water? Would you say water is an<br />

acid or a base? Explain your answer.<br />

6. A solution has a pH of 11. What is the H + concentration of the solution? Is this solution an acid or a base?<br />

7. A solution has a pH of 8.4. What is the H + concentration of this solution?<br />

8. Acids are very good at removing hard water deposits from bathtubs, sinks, and glassware. Your father goes<br />

to the store to buy a cleaner to remove such deposits from your bathtub. He has a choice between a product<br />

containing lemon juice (H + =10 –2.5 M) and one containing vinegar (H + =10 –3.3 M). Which product would<br />

you recommend he purchase? Explain your answer.<br />

21.3


Name: Date:<br />

22.1 Groundwater and Wells Project<br />

When it rains, some of the water that falls on Earth seeps into the ground, while some water flows over the<br />

surface into local streams or lakes. Some water is absorbed by plants and some evaporates back into the<br />

atmosphere. The water that seeps into the ground flows downward, moving through empty spaces between soil,<br />

sand, or rocks. It continues its journey until it reaches rock through which it cannot easily move. Then, it starts to<br />

fill the spaces between the rock and soil above. The top of this wedge of water is called the water table.<br />

The water that fills the empty spaces is called groundwater. Areas that groundwater easily moves through are<br />

called aquifers. Aquitards are bodies of rock where water can move through—but very slowly. If the aquitard<br />

does not allow any water to pass, it is called an aquiclude. Groundwater comes from precipitation (rain and snow<br />

melt), from lakes or rivers that leak water, and even from extra water not used by agricultural crops when they are<br />

irrigated.<br />

Groundwater is a very important source of drinking water. According to the US Geological Survey, 51% of<br />

Americans get their drinking water from groundwater. 99% of the rural population in the US uses groundwater<br />

for drinking. 37% of agricultural water, which is mostly used for irrigation comes from groundwater.<br />

Groundwater is obtained by digging wells. The water fills the well underground and a pump inside pumps it up to<br />

the surface where it travels through pipes to bring it to our homes and businesses.<br />

This project will help you learn more about groundwater movement and wells.<br />

Materials:<br />

• GeoBox • ½”to ¾” white<br />

stone; rounded is<br />

better<br />

(approximately<br />

1,800 mL total)<br />

• Plastic wrap • Food dye - dark<br />

colors<br />

• Wooden skewer or<br />

dowel with diameter<br />

less than ½”<br />

• 3 wells (½” inside<br />

diameter PVC pipe<br />

with caps; 4 well<br />

holes near cap<br />

drilled with 13/64<br />

drill bit)<br />

• ¼” plastic foam; one<br />

piece 7 ¾” x 13 ¾”;<br />

second piece 7 ¾” x<br />

9"<br />

• 8-10 cotton swabs • Tape<br />

• Watering can or<br />

beaker<br />

• Caulk or plumbers<br />

putty (something<br />

that can be molded<br />

around the PVC<br />

wells for<br />

waterproofing)<br />

22.1


Page 2 of 3<br />

Constructing the model:<br />

1. Line the inside of the GeoBox with plastic wrap so that it comes up and over the edges of the box.<br />

2. Hold well #2 in the middle of the GeoBox, with the cap end sitting directly on the bottom of the GeoBox.<br />

Add approximately 1,800 mL of the rock, surrounding the well. The rock should just cover the holes of the<br />

well and the well should stand on its own.<br />

3. The larger plastic foam sheet will be layered next on top of the rock. In order to put it down, carefully poke<br />

the well through it so it fits over the well. Now place on top of it the plastic wrap that will come up and over<br />

the edges. Because you need to also make a hole in the plastic wrap through which to fit the well, use the<br />

caulk or putty to mold around the well and onto the plastic wrap to keep it water proof.<br />

4. Once this is set, hold well #3 in place on the right side<br />

(diagram A) and add approximately 2,000 mL of stone<br />

down on the surface, so that it surrounds well #3 and holds<br />

it upright.<br />

5. Now add approximately 1,300 mL of stone to the left side<br />

of the GeoBox to create a diagonal plane of stone that runs<br />

highest from the left edge to level just right of well #2.<br />

6. Place well #1 in the built up area of stone on the left side of<br />

the GeoBox, just above, but not touching the first plastic<br />

foam layer (as well #3 is). Make sure that the stone is<br />

covering the holes in the well.<br />

7. Place the smaller plastic foam sheet over well #1 and well<br />

#2, again poking holes in the plastic foam so that the sheet<br />

can sit on the rock layers below. This sheet will be slanted<br />

down towards the middle.<br />

8. Again you will cover just the sheet with plastic wrap which will come up<br />

and over the edge on three sides. Caulk the two wells that poke through<br />

this sheet.<br />

9. Use the remaining 3,000 mL of stone to fill the tray up to the top so that<br />

what is visible is just stone and three well tops. See photo at right.<br />

10. Tape one cotton swab to the end of the skewer or wooden rod so that the<br />

cotton swab reaches out from the end of the wood. See diagram B at right.<br />

11. Dye the water that you will be using for precipitation a dark color, such as<br />

blue, red, or green.<br />

Making predictions:<br />

a. Which well/s would you expect to collect water when it rains?<br />

b. If contamination entered from the surface, what well would you expect to first show contaminated<br />

water?<br />

c. Will well #2 get contaminated from surface contamination? Why?<br />

22.1


Page 3 of 3<br />

Testing the model:<br />

1. Watch the water flow closely as you do this experiment.<br />

2. Sprinkle or pour the dyed water into the top layer of rock to simulate precipitation, without allowing the<br />

water to precipitate into the wells. The dye will make it easier to see the water as it travels.<br />

3. Regularly check the wells with the cotton swab/dowel rods to see if water has entered the wells. In this way,<br />

you can also see which well collected water the quickest.<br />

4. For a demonstration of the movement of surface pollution—dye water another color and allow this<br />

contaminated water to percolate through the layers. Use new cotton swabs attached to the wooden rods to<br />

visualize if and when the wells will get contaminated. The cotton swab should change color as the two dyes<br />

mix.<br />

Thinking about what you observed:<br />

a. Which wells collected water when it rained? Was your hypothesis correct?<br />

b. Which well was first to be contaminated? Was your hypothesis correct?<br />

c. What does the plastic wrap/plastic foam layer represent? Label diagram C appropriately.<br />

d. What do the rock layers represent? Label diagram C appropriately.<br />

e. Did well #2 get contaminated from surface contamination? Why? Was your hypothesis correct?<br />

f. What effect would pumping from well #1 have on movement of surface contamination? Pumping from<br />

well #2?<br />

g. What would happen if there was a dry spell and the water table and thus the groundwater was lowered to<br />

below well #1? Would any well be able to pump water?<br />

h. If well #3 were located near the coast, what effect might pumping freshwater too quickly have on the<br />

water in the well?<br />

i. When you dig a well, how might you decide how deep to dig it?<br />

22.1


Name: Date:<br />

22.2 The Water Cycle<br />

As you study Section 22.2 in your student text, you will learn about the processes that move water around our<br />

planet. Together, these processes form the water cycle. Use the word box to help you label the water cycle<br />

diagram below. Some words may be used more than once.<br />

• condensation • groundwater flow • evaporation • water vapor<br />

transport<br />

• percolation • transpiration • precipitation<br />

Answer the following questions. Use the diagram above and Section 22.2 of your text to help you.<br />

1. Name two water cycle processes that are driven by the Sun. Explain the Sun’s role in each.<br />

2. How is wind involved in the water cycle?<br />

3. How does gravity affect the water cycle?<br />

22.2


Name: Date:<br />

24.1 Period and Frequency<br />

The period of a pendulum is the time it takes to move through one<br />

cycle. As the ball on the string is pulled to one side and then let go, the<br />

ball moves to the side opposite the starting place and then returns to<br />

the start. This entire motion equals one cycle.<br />

Frequency is a term that refers to how many cycles can occur in one<br />

second. For example, the frequency of the sound wave that<br />

corresponds to the musical note “A” is 440 cycles per second or 440 hertz. The unit hertz (Hz) is defined as the<br />

number of cycles per second.<br />

The terms period and frequency are related by the following equation:<br />

1. A string vibrates at a frequency of 20 Hz. What is its period?<br />

2. A speaker vibrates at a frequency of 200 Hz. What is its period?<br />

3. A swing has a period of 10 seconds. What is its frequency?<br />

4. A pendulum has a period of 0.3 second. What is its frequency?<br />

5. You want to describe the harmonic motion of a swing. You find out that it take 2 seconds for the swing to<br />

complete one cycle. What is the swing’s period and frequency?<br />

6. An oscillator makes four vibrations in one second. What is its period and frequency?<br />

7. A pendulum takes 0.5 second to complete one cycle. What is the pendulum’s period and frequency?<br />

8. A pendulum takes 10 seconds to swing through 2 complete cycles.<br />

a. How long does it take to complete one cycle?<br />

b. What is its period?<br />

c. What is its frequency?<br />

9. An oscillator makes 360 vibrations in 3 minutes.<br />

a. How many vibrations does it make in one minute?<br />

b. How many vibrations does it make in one second?<br />

c. What is its period in seconds?<br />

d. What is its frequency in hertz?<br />

24.1


Name: Date:<br />

24.1 Harmonic Motion Graphs<br />

A graph can be used to show the amplitude and period of an object in harmonic motion. An example of a graph<br />

of a pendulum’s motion is shown below.<br />

The distance to which the pendulum moves away from its center point is call the amplitude. The amplitude of a<br />

pendulum can be measured in units of length (centimeters or meters) or in degrees. On a graph, the amplitude is<br />

the distance from the x-axis to the highest point of the graph. The pendulum shown above moves 20 centimeters<br />

to each side of its center position, so its amplitude is 20 centimeters.<br />

The period is the time for the pendulum to make one complete cycle. It is the time from one peak to the next on<br />

the graph. On the graph above, one peak occurs at 1.5 seconds, and the next peak occurs at 3.0 seconds. The<br />

period is 3.0 – 1.5 = 1.5 seconds.<br />

1. Use the graphs to answer the following questions<br />

a. What is the amplitude of each vibration?<br />

b. What is the period of each vibration?<br />

24.1


Page 2 of 2<br />

2. Use the grids below to draw the following harmonic motion graphs. Be sure to label the y-axis to<br />

indicate the measurement scale.<br />

a. A pendulum with an amplitude of 2 centimeters and a period of 1 second.<br />

b. A pendulum with an amplitude of 5 degrees and a period of 4 seconds.<br />

24.1


Name: Date:<br />

24.2 Waves<br />

A wave is a traveling oscillator that carries energy from one place to another. A high point of a wave is called a<br />

crest. A low point is called a trough. The amplitude of a wave is half the distance from a crest to a trough. The<br />

distance from one crest to the next is called the wavelength. Wavelength can also be measured from trough to<br />

trough or from any point on the wave to the next place where that point occurs.<br />

• The frequency of a wave is 40 Hz and its speed is 100 meters per second. What is the wavelength of this<br />

wave?<br />

Solution:<br />

1. On the graphic at right label the following<br />

parts of a wave: one wavelength, half of a<br />

wavelength, the amplitude, a crest, and a<br />

trough.<br />

a. How many wavelengths are represented<br />

in the wave above?<br />

b. What is the amplitude of the wave shown<br />

above?<br />

100 m/s 100 m/s<br />

= =<br />

2.5 meters per cycle<br />

40 Hz 40 cycles/s<br />

The wavelength is 2.5 meters.<br />

24.2


Page 2 of 2<br />

2. Use the grids below to draw the following waves. Be sure to label the y-axis to indicate the<br />

measurement scale.<br />

a. A wave with an amplitude of<br />

1 cm and a wavelength of 2 cm<br />

b. A wave with an amplitude of<br />

1.5 cm and a wavelength of<br />

3cm<br />

3. A water wave has a frequency of 2 hertz and a wavelength of 5 meters. Calculate its speed.<br />

4. A wave has a speed of 50 m/s and a frequency of 10 Hz. Calculate its wavelength.<br />

5. A wave has a speed of 30 m/s and a wavelength of 3 meters. Calculate its frequency.<br />

6. A wave has a period of 2 seconds and a wavelength of 4 meters.Calculate its frequency and speed.<br />

Note: Recall that the frequency of a wave equals 1/period and the period of a wave equals 1/frequency.<br />

7. A sound wave travels at 330 m/s and has a wavelength of 2 meters. Calculate its frequency and period.<br />

8. The frequency of wave A is 250 hertz and the wavelength is 30 centimeters. The frequency of wave B is<br />

260 hertz and the wavelength is 25 centimeters. Which is the faster wave?<br />

9. The period of a wave is equal to the time it takes for one wavelength to pass by a fixed point. You stand on a<br />

pier watching water waves and see 10 wavelengths pass by in a time of 40 seconds.<br />

a. What is the period of the water waves?<br />

b. What is the frequency of the water waves?<br />

c. If the wavelength is 3 meters, what is the wave speed?<br />

24.2


Name: Date:<br />

24.2 Wave Interference<br />

Interference occurs when two or more waves are at the same location at the same time. For example, the wind<br />

may create tiny ripples on top of larger waves in the ocean.The superposition principle states that the total<br />

vibration at any point is the sum of the vibrations produced by the individual waves.<br />

Constructive interference is when waves combine to make a larger wave. Destructive interference is when waves<br />

combine to make a wave that is smaller than either of the individual waves. Noise cancelling headphones work<br />

by producing a sound wave that perfectly cancels the sounds in the room.<br />

This worksheet will allow you to find the sum of two waves with different wavelengths and amplitudes. The<br />

table below (and continued on the next page) lists the coordinates of points on the two waves.<br />

1. Use coordinates on the table and the graph paper (see last page) to graph wave 1 and wave 2 individually.<br />

Connect each set of points with a smooth curve that looks like a wave. Then, answer questions 2–9.<br />

2. What is the amplitude of wave 1?<br />

3. What is the amplitude of wave 2?<br />

4. What is the wavelength of wave 1?<br />

5. What is the wavelength of wave 2?<br />

6. How many wavelengths of wave 1 did you draw?<br />

7. How many wavelength of wave 2 did you draw?<br />

8. Use the superposition principle to find the wave that results from the interference of the two waves.<br />

a. To do this, simply add the heights of wave 1 and wave 2 at each point and record the values in the last<br />

column. The first four points are done for you.<br />

b. Then use the points in last column to graph the new wave. Connect the points with a smooth curve. You<br />

should see a pattern that combines the two original waves.<br />

9. Describe the wave created by adding the two original waves.<br />

x-axis<br />

(blocks)<br />

Height of wave 1<br />

(y-axis blocks)<br />

Height of wave 2<br />

(y-axis blocks)<br />

Height of wave 1 + wave 2<br />

(y-axis blocks)<br />

0 0 0 0<br />

1 0.8 2 2.8<br />

2 1.5 0 1.5<br />

3 2.2 –2 0.2<br />

4 2.8 0<br />

24.2


Page 2 of 3<br />

x-axis<br />

(blocks)<br />

Height of wave 1<br />

(y-axis blocks)<br />

Height of wave 2<br />

(y-axis blocks)<br />

5 3.3 2<br />

6 3.7 0<br />

7 3.9 –2<br />

8 4 0<br />

9 3.9 2<br />

10 3.7 0<br />

11 3.3 –2<br />

12 2.8 0<br />

13 2.2 2<br />

14 1.5 0<br />

15 0.8 –2<br />

16 0 0<br />

17 –0.8 2<br />

18 –1.5 0<br />

19 –2.2 –2<br />

20 –2.8 0<br />

21 –3.3 2<br />

22 –3.7 0<br />

23 –3.9 –2<br />

24 –4 0<br />

25 –3.9 2<br />

26 –3.7 0<br />

27 –3.3 –2<br />

28 –2.8 0<br />

29 –2.2 2<br />

30 –1.5 0<br />

31 –0.8 –2<br />

32 0 0<br />

Height of wave 1 + wave 2<br />

(y-axis blocks)<br />

24.2


Page 3 of 3<br />

24.2


Name: Date:<br />

24.3 Decibel Scale<br />

The loudness of sound is measured in decibels (dB). Most sounds fall between zero and 100 on the decibel scale<br />

making it a very convenient scale to understand and use. Each increase of 20 decibels (dB) for a sound will be<br />

about twice as loud to your ears. Use the following table to help you answer the questions.<br />

10-15 dB A quiet whisper 3 feet away<br />

30-40 dB Background noise in a house<br />

65 dB Ordinary conversation 3 feet away<br />

70 dB City traffic<br />

90 dB A jackhammer cutting up the street 10 feet away<br />

100 dB Listening to headphones at maximum volume<br />

110 dB Front row at a rock concert<br />

120 dB The threshold of physical pain from loudness<br />

• How many decibels would a sound have if its loudness was twice that of city traffic?<br />

Solution:<br />

City traffic = 70 dB<br />

Adding 20 dB doubles the loudness.<br />

70 dB + 20 dB = 90 dB<br />

90 dB is twice as loud as city traffic.<br />

1. How many times louder than a jackhammer does the front row at a rock concert sound?<br />

2. How many decibels would you hear in a room that sounds twice as loud as an average (35 dB) house?<br />

3. You have your headphones turned all the way up (100 dB).<br />

a. If you want them to sound half as loud, to what decibel level must the music be set?<br />

b. If you want them to sound 1/4 as loud, to what decibel level must the music be set?<br />

4. How many times louder than city traffic does the front row at a rock concert sound?<br />

5. When you whisper, you produce a 10-dB sound.<br />

a. When you speak quietly, your voice sounds twice as loud as a whisper. How many decibels is this?<br />

b. When you speak normally, your voice sounds 4 times as loud as a whisper. How many decibels is this?<br />

c. When you yell, your voice sounds 8 times as loud as a whisper. How many decibels is this?<br />

24.3


Name: Date:<br />

24.3 The Human Ear<br />

Write the name of the part that corresponds to each letter in the diagram. Then write the function of each<br />

part in the spaces on the next page.<br />

24.3


Page 2 of 2<br />

a. ______________________________________________________________________________<br />

b. ______________________________________________________________________________<br />

c. ______________________________________________________________________________<br />

d. ______________________________________________________________________________<br />

e. ______________________________________________________________________________<br />

f. ______________________________________________________________________________<br />

g. ______________________________________________________________________________<br />

24.3


Name: Date:<br />

24.3 Standing Waves<br />

A wave that is confined in a space is called a standing wave.<br />

Standing waves on the vibrating strings of a guitar produce the<br />

sounds you hear. Standing waves are also present inside the chamber<br />

of a wind instrument.<br />

A string that contains a standing wave is an oscillator. Like any<br />

oscillator, it has natural frequencies. The lowest natural frequency is<br />

called the fundamental. Other natural frequencies are called<br />

harmonics. The first five harmonics of a standing wave on a string<br />

are shown to the right.<br />

There are two main parts of a standing wave. The nodes are the<br />

points where the string does not move at all. The antinodes are the<br />

places where the string moves with the greatest amplitude.<br />

The wavelength of a standing wave can be found by measuring the length of two of the “bumps” on the string.<br />

The first harmonic only contains one bump, so the wavelength is twice the length of the individual bump.<br />

1. Use the graphic below to answer these questions.<br />

a. Which harmonic is shown in each of the strings below?<br />

b. Label the nodes and antinodes on each of the standing waves shown below.<br />

c. How many wavelengths does each standing wave contain?<br />

d. Determine the wavelength of each standing wave.<br />

24.3


Page 2 of 2<br />

2. Two students want to use a 12-meter rope to create standing waves. They first measure the speed at<br />

which a single wave pulse moves from one end of the rope to another and find that it is 36 m/s. This<br />

information can be used to determine the frequency at which they must vibrate the rope to create<br />

each harmonic. Follow the steps below to calculate these frequencies.<br />

a. Draw the standing wave patterns for the first six harmonics.<br />

b. Determine the wavelength for each harmonic on the 12-meter rope. Record the values in the table<br />

below.<br />

c. Use the equation for wave speed (v = fλ) to calculate each frequency.<br />

Harmonic Speed (m/s) Wavelength<br />

(m)<br />

1 36<br />

2 36<br />

3 36<br />

4 36<br />

5 36<br />

6 36<br />

d. What happens to the frequency as the wavelength increases?<br />

e. Suppose the students cut the rope in half. The speed of the wave on the rope only depends on the<br />

material from which the rope is made and its tension, so it will not change. Determine the wavelength<br />

and frequency for each harmonic on the 6-meter rope.<br />

Harmonic Speed (m/s) Wavelength<br />

(m)<br />

1 36<br />

2 36<br />

3 36<br />

4 36<br />

5 36<br />

6 36<br />

f. What effect did using a shorter rope have on the wavelength and frequency?<br />

Frequency<br />

(Hz)<br />

Frequency<br />

(Hz)<br />

24.3


Name: Date:<br />

24.3 Waves and Energy<br />

A wave is an organized form of energy that travels. The amount of energy a wave has is proportional to its<br />

frequency and amplitude. Therefore, higher energy waves have a higher frequency and/or a higher amplitude.<br />

Remember that the frequency is measured in hertz. The frequency of 1 Hz equals one wave cycle per second.<br />

For each set of diagrams, identify which of the standing waves has the<br />

highest energy and which has the lowest energy.<br />

Answers for frequency and energy:<br />

Standing wave Frequency Energy<br />

A lowest lowest<br />

B medium medium<br />

C highest highest<br />

Answers for amplitude and energy:<br />

Standing wave Frequency Energy<br />

A lowest lowest<br />

B highest highest<br />

1. When you drop a stone into a pool, water waves spread out from where the stone landed. Why?<br />

2. Ian and Igor have opposite ends of a jump rope and perform the following demonstration. First they moved<br />

the rope up and down one time a second, then two times a second, and then three times a second. Describe<br />

the trend in frequency for the jump rope and the trend in energy used by Ian and Igor for this demonstration.<br />

3. One wave has a frequency of 30 Hz and another has a frequency of 100 Hz. Both waves have the same<br />

amplitude. Which wave has more energy?<br />

4. On a calm day ocean waves are about 0.1 meter high. However, during a hurricane, ocean waves might be as<br />

much as 14 meters high. If both waves have the same frequency, during which set of conditions do the waves<br />

have more energy?<br />

5. Which wave has more energy: a wave that has an amplitude of 3 centimeters and a frequency of 2 Hz or a<br />

wave with an amplitude of 3 meters and a frequency of 2 Hz?<br />

6. The loudness of sound is related to its amplitude. Which sound wave would have the least energy: a lowvolume<br />

wave at 1,000 Hz or a high-volume (loud) wave at 1,000 Hz?<br />

7. The frequency of microwaves is less than that of visible light waves. Which type of wave is likely to have<br />

greater energy?<br />

8. Standing wave C in the first graphic above represents 1.5 wavelengths. Draw a standing wave that represents<br />

2 wavelengths. Compare the energy and frequency of this standing wave to A, B, and C.<br />

24.3


Name: Date:<br />

24.3 Palm Pipes Project<br />

A palm pipe is a musical instrument made from a simple material—PVC pipe. To play a palm pipe, you hit an<br />

open end of the pipe on the palm of your hand, causing the air molecules in the pipe to vibrate. These vibrations<br />

create the sounds that you hear.<br />

Materials:<br />

• 1 standard 10-foot length of 1/2 inch PVC pipe for 180°F water.<br />

• Flexible meter stick<br />

• PVC pipe cutter or a hacksaw<br />

• Sandpaper<br />

• Seven different colors of permanent markers for labeling pipes<br />

• Simple calculator<br />

Directions:<br />

1. Cut the PVC pipe into the lengths listed in the chart below. It works best if you measure one length, cut it,<br />

then make the next measurement. You may want to cut each piece a little longer than the given measurement<br />

so that you can sand out any rough spots and level the pipe without making it too short.<br />

Number Note Length of pipe (cm) Frequency (Hertz)<br />

1 F 23.60 349<br />

2 G 21.00 392<br />

3 A 18.75 440<br />

4 B flat 17.50 446<br />

5 C 15.80 523<br />

6 D 14.00 587<br />

7 E 12.50 659<br />

8 F 11.80 698<br />

9 G 10.50 748<br />

10 A 9.40 880<br />

11 B flat 9.20 892<br />

12 C 7.90 1049<br />

13 D 7.00 1174<br />

14 E 6.25 1318<br />

15 F 5.90 1397<br />

2. Lightly sand the cut ends to smooth any rough spots.<br />

3. Label each pipe with the number, note, and frequency using a different color permanent marker.<br />

4. Hit one open end of the pipe on the palm of your hand in order to make a sound.<br />

24.3


Page 2 of 3<br />

Activities:<br />

1. Try blowing across the top of a pipe as if you were playing a flute. Does the pipe sound the same as<br />

when you tap it on your palm? Why or why not?<br />

24.3<br />

Safety note: Wash the pipes with rubbing alcohol or a solution of 2 teaspoons household bleach per gallon<br />

of water before and after blowing across them.<br />

2. Take one of the longer pipes and place it in a bottle of water so that the top of the pipe extends above the top<br />

of the bottle. Blow across it like a flute. What happens to the tone as you raise or lower the pipe in the bottle?<br />

3. Try making another set of palm pipes out of 1/2-inch copper tubing. What happens when you strike these<br />

pipes against your palm? What happens when you blow across the top? How does the sound compare with<br />

the plastic pipes?<br />

4. At a hardware store, purchase two rubber rings for each copper pipe. These rings should fit snugly around<br />

the pipes. Place one ring on each end of each pipe, then lay them on a table. Try tapping the side of each pipe<br />

with different objects—wooden and stainless steel serving spoons, for example. How does this sound<br />

compare with the other sounds you have made with the pipes?<br />

5. Try playing some palm pipe music with your classmates. Here are two tunes to get you started:<br />

Happy Birthday<br />

Melody C C D C F E C C D C G F<br />

Harmony A A B b B b B b A<br />

Melody C C C A F E D B b B b A F G F<br />

Harmony F C B b C A<br />

Twinkle Twinkle Little Star<br />

Melody F F C C D D C B b B b A A G G F<br />

Harmony C C A A B b B b A G G F F E E C<br />

Melody C C B b B b A A G C C B b B b A A G<br />

Harmony A A G G F F C A A G G F F C<br />

Melody F F C C D D C B b B b A A G G F<br />

Harmony C C A A B b B b A G G F F E E C


Page 3 of 3<br />

6. Challenge:<br />

You can figure out the length of pipe needed to make other notes, too. All you need is a simple<br />

formula and your understanding of the way sound travels in waves.<br />

To figure out the length of the pipe needed to create sound of a certain frequency, we start with the formula<br />

frequency = velocity of sound in air ÷ wavelength, or f = v /λ. Next, we solve the equation for λ: λ = v/f.<br />

The fundamental frequency is the one that determines which note is heard. You can use the chart below to<br />

find the fundamental frequency of a chromatic scale in two octaves. Notice that for each note, the frequency<br />

doubles every time you go up an octave.<br />

Once you choose the frequency of the note you want to play, you need to know what portion of the<br />

fundamental frequency’s wavelength (S-shape) will fit inside the palm pipe.<br />

To help you visualize the wave inside the palm pipe, hold the center of a flexible meter stick in front of you.<br />

Wiggle the meter stick to create a standing wave. This mimics a column of vibrating air in a pipe with two<br />

open ends. How much of a full wave do you see? If you answered one half, you are correct.<br />

When a palm pipe is played, your hand closes one end of the pipe. Now use your meter stick to mimic this<br />

situation. Place the meter stick on a table top and use one hand to hold down one end of the stick. This<br />

represents the closed end of the pipe. Flick the other end of the meter stick to set it in motion. How much of<br />

a full wavelength do you see? Now do you know what portion of the wavelength will fit into the palm pipe?<br />

One-fourth of the wavelength of the fundamental frequency will fit inside the palm pipe. As a result the<br />

length of the pipe should be equal to 1/4λ, which is equal to 1/4(v/f).<br />

In practice, we find that the length of pipe needed to make a certain frequency is actually a bit shorter than<br />

this. Subtracting a length equal to 1/4 of the pipe’s inner diameter is necessary. The final equation, therefore,<br />

is: Length of pipe<br />

v 1<br />

=<br />

---- – --D where D represents the inner diameter of the pipe.<br />

4f 4<br />

Given that the speed of sound in air (at 20 °C) is 343 m/s and the inner diameter of the pipe is 0.0017 m,<br />

what is the length of pipe you would need to make the note B, with a frequency of 494 hertz? How about the<br />

same note one octave higher (frequency 988 hertz)? Make these two pipes so that you can play a C major<br />

scale.<br />

Chromatic scale in two octaves (frequencies rounded to nearest whole number)<br />

Note A A# B C C# D D# E F F# G G# A<br />

Frequency<br />

(Hertz)<br />

220 233 247 262 277 294 311 330 349 370 392 415 440<br />

Frequency<br />

(Hertz)<br />

440 466 494 523 554 587 622 659 698 740 784 831 880<br />

7. What is the lowest note you could make with a palm pipe? What is the highest? Explain these limits using<br />

what you know about the human ear and the way sound is created by the palm pipe.<br />

24.3


Name: Date:<br />

25.1 The Electromagnetic Spectrum<br />

Radio waves, microwaves, visible light, and x-rays are familiar kinds of electromagnetic waves. All of these<br />

waves have characteristic wavelengths and frequencies. Wavelength is measured in meters. It describes the length<br />

of one complete oscillation. Frequency describes the number of complete oscillations per second. It is measured<br />

in hertz, which is another way of saying “cycles per second.” The higher the wave’s frequency, the more energy<br />

it carries.<br />

Frequency, wavelength, and speed<br />

In a vacuum, all electromagnetic waves travel at the same speed: 3.0 × 10 8 m/s. This quantity is often called “the<br />

speed of light” but it really refers to the speed of all electromagnetic waves, not just visible light. It is such an<br />

important quantity in physics that it has its own symbol, c.<br />

The speed of light is related to<br />

frequency f and wavelength λ<br />

by the formula to the right.<br />

The different colors of light<br />

that we see correspond to<br />

different frequencies. The<br />

frequency of red light is lower<br />

than the frequency of blue<br />

light. Because the speed of both kinds of light is the same, a lower frequency wave has a longer wavelength. A<br />

higher frequency wave has a shorter wavelength. Therefore, red light’s wavelength is longer than blue light’s.<br />

When we know the frequency of light, the wavelength is given by: λ =<br />

When we know the wavelength of light, the frequency is given by: f =<br />

c<br />

-<br />

f<br />

--<br />

c<br />

λ<br />

25.1


Page 2 of 2<br />

Answer the following problems. Don’t forget to show your work.<br />

1. Yellow light has a longer wavelength than green light. Which color of light has the higher frequency?<br />

2. Green light has a lower frequency than blue light. Which color of light has a longer wavelength?<br />

3. Calculate the wavelength of violet light with a frequency of 750 × 10 12 Hz.<br />

4. Calculate the frequency of yellow light with a wavelength of 580 × 10 –9 m.<br />

5. Calculate the wavelength of red light with a frequency of 460 × 10 12 Hz.<br />

6. Calculate the frequency of green light with a wavelength of 530 × 10 –9 m.<br />

7. One light beam has wavelength, λ 1, and frequency, f 1. Another light beam has wavelength, λ 2, and<br />

frequency, f 2. Write a proportion that shows how the ratio of the wavelengths of these two light beams is<br />

related to the ratio of their frequencies.<br />

8. The waves used by a microwave oven to cook food have a frequency of 2.45 gigahertz (2.45 × 10 9 Hz).<br />

Calculate the wavelength of this type of wave.<br />

9. A radio station has a frequency of 90.9 megahertz (9.09 × 10 7 Hz). What is the wavelength of the radio<br />

waves the station emits from its radio tower?<br />

10. An x-ray has a wavelength of 5 nanometers (5.0 × 10 –9 m). What is the frequency of x-rays?<br />

11. The ultraviolet rays that cause sunburn are called UV-B rays. They have a wavelength of approximately 300<br />

nanometers (3.0 × 10 –7 m). What is the frequency of a UV-B ray?<br />

12. Infrared waves from the sun are what make our skin feel warm on a sunny day. If an infrared wave has a<br />

frequency of 3.0 × 10 12 Hz, what is its wavelength?<br />

13. Electromagnetic waves with the highest amount of energy are called gamma rays. Gamma rays have<br />

wavelengths of less than 10-trillionths of a meter (1.0 × 10 –11 m).<br />

a. Determine the frequency that corresponds with this wavelength.<br />

b. Is this the minimum or maximum frequency of a gamma ray?<br />

14. Use the information from this sheet to order the following types of waves from lowest to highest frequency:<br />

visible light, gamma rays, x-rays, infrared waves, ultraviolet rays, microwaves, and radio waves.<br />

15. Use the information from this sheet to order the following types of waves from shortest to longest<br />

wavelength: visible light, gamma rays, x-rays, infrared waves, ultraviolet rays, microwaves, and radio<br />

waves.<br />

25.1


Name: Date:<br />

25.2 Color Mixing with Additive and Subtractive Processes<br />

The way that color appears on a piece of paper and how your eyes interpret color involve two different color<br />

mixing processes. Your eyes see color using an additive color process. The RGB color model is the basis for how<br />

the additive process works and involves mixing colors of light. The CMYK color model is the basis for how the<br />

subtractive color process works and involves pigments of color which absorb colors of light.<br />

RGB color model CMYK color model<br />

Primary colors Mixed colors New color Primary colors Mixed colors New color<br />

red red + green yellow magenta magenta + yellow red<br />

green green + blue cyan yellow yellow + cyan green<br />

blue blue + red magenta cyan cyan + magenta blue<br />

How black is<br />

made<br />

How white is<br />

made<br />

• The human eye has photoreceptors for red, green, and blue light. Which of these photoreceptors are<br />

stimulated when looking at white paint?<br />

Solution: All three of these photoreceptors are stimulated equally.<br />

• A laser printer prints a piece of paper that includes black lettering and a blue border. How are these colors<br />

made using the CMYK color model?<br />

Solution: Pure black pigment is used to make the black lettering. If you were to mix the other colors<br />

(magenta, yellow, and cyan), you would only get a muddy gray. The blue border was made by mixing cyan<br />

and magenta pigments.<br />

1. A friend asks you to describe the difference between the RGB color model and the CMYK color model.<br />

Give him three differences between these color models.<br />

2. How would you see the following combinations of light colors?<br />

a. red only<br />

b. blue + red, both at equal intensity<br />

c. green only<br />

d. green + blue, both at equal intensity<br />

Absence of light How black is<br />

made<br />

red + green + blue How white is<br />

made<br />

Pure black pigment<br />

Absence of pigment or use of pure<br />

white pigment<br />

3. The color orange is perceived by the eyes when both the red and green photoreceptors are stimulated and the<br />

red signal is stronger than the green. Given this information, what kind of signals would be received by the<br />

eye for the color purple?<br />

25.2


Page 2 of 2<br />

4. You see a chair that is painted yellow. Most likely, pure yellow pigment was used to make the paint.<br />

However, explain how your eyes interpret the color yellow.<br />

5. White paint purchased at a store is often made of a pure white pigment called titanium dioxide. This<br />

white paint reflects about 97% of the light that strikes it. Why might this property of the paint mean that you<br />

interpret its color as white?<br />

6. The image you see on a color TV screen is made using the RGB color model. The image is made of<br />

thousands of pixels or dots of color. Describe how you could make the following pixels using the RGB color<br />

model.<br />

a. A white pixel<br />

b. A black pixel<br />

c. A cyan pixel<br />

d. A yellow pixel<br />

7. What colors of light are reflected and/or absorbed by a red apple when:<br />

a. white light shines on it?<br />

b. only red light shines on it?<br />

c. only blue light shines on it?<br />

8. The CMYK color model works because the combination of pigments absorb and reflect light. Imagine that<br />

white light containing a mixture of red, green, and blue light shines on the combination of CMYK pigments<br />

in the table below. Copy the table on your own paper. Indicate in the blank spaces which colors of light the<br />

pigments absorb and which color is reflected. Some parts of the table are filled in for you.<br />

Mixed colors Reflected color<br />

CMYK color model<br />

Which colors of light are absorbed?<br />

magenta + yellow red<br />

yellow + cyan<br />

cyan + magenta<br />

blue is absorbed by yellow<br />

red is absorbed by cyan<br />

9. If you mix magenta paint and cyan paint, what color will you achieve?<br />

10. A laser printer’s ink only includes the colors cyan, magenta, yellow, and black.<br />

a. Explain how it makes the color green using these pigments.<br />

b. Then, explain what happens for your eye to interpret this color as<br />

green.<br />

c. This Venn diagram illustrates color mixing for the CMYK color<br />

model. Now, make a Venn diagram for the RGB color model. Use<br />

color when you make your diagram. Be sure to label the difference<br />

between the primary colors and the new colors made by mixing.<br />

25.2


Name: Date:<br />

25.2 The Human Eye<br />

Write the name of the part that corresponds to each letter in the diagram. Then write the function of<br />

each part in the spaces on the next page.<br />

25.2


Page 2 of 2<br />

a. ______________________________________________________________________________<br />

b. ______________________________________________________________________________<br />

c. ______________________________________________________________________________<br />

d. ______________________________________________________________________________<br />

e. ______________________________________________________________________________<br />

f. ______________________________________________________________________________<br />

g. ______________________________________________________________________________<br />

h. ______________________________________________________________________________<br />

i. ______________________________________________________________________________<br />

j. ______________________________________________________________________________<br />

k. ______________________________________________________________________________<br />

25.2


Name: Date:<br />

25.3 Measuring Angles with a Protractor<br />

Measure each of these angles (A–Q) with a protractor. Record the angle measurements in the table<br />

below.<br />

Letter Angle Letter Angle<br />

A J<br />

B K<br />

C L<br />

D M<br />

E N<br />

F O<br />

G P<br />

H Q<br />

I<br />

25.3


Name: Date:<br />

25.3 Using Ray Diagrams<br />

This skill sheet gives you some practice using ray diagrams. A ray diagram helps you determine where an image<br />

produced by a lens will form and shows whether the image is upside down or right side up.<br />

1. Of the diagrams below, which one correctly illustrates how light rays come off an object? Explain your<br />

answer.<br />

2. Of the diagrams below, which one correctly illustrates how a light ray enters and exits a piece of thick glass?<br />

Explain your answer.<br />

In your own words, explain what happens to light as it enters glass from the air. Why does this happen? Use<br />

the term refraction in your answer.<br />

3. Of the diagrams below, which one correctly illustrates how parallel light rays enter and exit a converging<br />

lens? Explain your answer.<br />

4. Draw a diagram of a converging lens that has a focal length of 10 centimeters. In your diagram, show three<br />

parallel lines entering the lens and exiting the lens. Show the light rays passing through the focal point of the<br />

lens. Be detailed in your diagram and provide labels.<br />

25.3


Name: Date:<br />

25.3 Reflection<br />

You have seen the law of reflection at work using light and the smooth surface of a mirror. Did you know you<br />

can apply this law to other situations? It can help you win a game of pool or pass a basketball to a friend on the<br />

court.<br />

In this skill sheet you will review the law of reflection and work on practice problems that utilize this law. Use a<br />

protractor to make your angles correct in your diagrams.<br />

The law of reflection states that when an object hits a surface, its angle<br />

of incidence will equal the angle of reflection. This is true when the<br />

object is light and the surface is a flat, smooth mirror. When the object<br />

and the surface are larger and lack smooth surfaces (like a basketball<br />

and a gym floor), the angles of incidence and reflection are nearly but<br />

not always exactly equal. The angles are close enough that<br />

understanding the law of reflection can help you improve your game.<br />

A light ray strikes a flat mirror with a 30-degree angle of incidence. Draw a ray diagram to show how the light<br />

ray interacts with the mirror. Label the normal line, the incident ray, and the reflected ray.<br />

Solution:<br />

1. When we talk about angles of incidence and reflection, we often talk about the normal. The normal to a<br />

surface is an imaginary line that is perpendicular to the surface. The normal line starts where the incident ray<br />

strikes the mirror. A normal line is drawn for you in the sample problem above.<br />

a. Draw a diagram that shows a mirror with a normal line and a ray of light hitting the mirror at an angle of<br />

incidence of 60 degrees.<br />

b. In the diagram above, label the angle of reflection. How many degrees is this angle of reflection?<br />

25.3


Page 2 of 2<br />

2. Light strikes a mirror’s surface at 20 degrees to the normal. What will the angle of reflection be?<br />

3. A ray of light strikes a mirror. The angle formed by the incident ray and the reflected ray measures<br />

90 degrees. What are the measurements of the angle of incidence and the angle of reflection?<br />

4. In a game of basketball, the ball is bounced (with no spin) toward a player at an angle of 40 degrees to the<br />

normal. What will the angle of reflection be? Draw a diagram that shows this play. Label the angles of<br />

incidence and reflection and the normal.<br />

Challenge Questions:<br />

Use a protractor to figure out the angles of incidence and reflection for the following problems.<br />

5. Because a lot of her opponent’s balls are in the way for<br />

a straight shot, Amy is planning to hit the cue ball off<br />

the side of the pool table so that it will hit the 8-ball<br />

into the corner pocket. In the diagram, show the angles<br />

of incidence and reflection for the path of the cue ball.<br />

How many degrees does each angle measure?<br />

6. You and a friend are playing pool. You are<br />

playing solids and he is playing stripes. You<br />

have one ball left before you can try for the<br />

eight ball. Stripe balls are in the way. You<br />

plan on hitting the cue ball behind one of the<br />

stripe balls so that it will hit the solid ball<br />

and force it to follow the pathway shown in<br />

the diagram. Use your protractor to figure<br />

out what angles of incidence and reflection<br />

are needed at points A and B to get the solid<br />

ball into the far side pocket.<br />

25.3


Name: Date:<br />

25.3 Refraction<br />

25.3<br />

When light rays cross from one material to another they bend. This bending is called refraction.<br />

Refraction is a useful phenomenon. All kinds of optics, from glasses to camera lenses to binoculars depend on<br />

refraction.<br />

If you are standing on the shore looking at a fish in a stream, the fish appears<br />

to be in a slightly different place than it really is. That’s because light rays<br />

that bounce off the fish are refracted at the boundary between water and air. If<br />

you are a hunter trying to spear this fish, you better know about this<br />

phenomenon or the fish will get away.<br />

Why do the light rays bend as they cross from water into air?<br />

A light ray bends because light travels at different speeds in different<br />

materials. In a vacuum, light travels at a speed of 3 × 10 8 m/s. But when light<br />

travels through a material, it is absorbed and re-emitted by each atom or<br />

molecule it hits. This process of absorption and emission slows the light ray’s<br />

speed. We experience this slowdown as a bend in the light ray. The greater<br />

the difference in the light ray’s speed through two different materials, the greater the bend in the path of the ray.<br />

The index of refraction (n) for a material is the ratio of the speed of light in a vacuum to the speed of light in the<br />

material.<br />

speed of light in a vacuum<br />

Index of refraction = speed of light in a material<br />

The index of refraction for some common materials is given below:<br />

Material Index of refraction (n)<br />

Vacuum 1.0<br />

Air 1.0001<br />

Water 1.33<br />

Glass 1.5<br />

Diamond 2.42<br />

1. Could the index of refraction for a material ever be less than 1.0? Explain.<br />

2. Explain why the index of refraction for air (a gas) is smaller than the index of refraction for a solid like glass.<br />

3. Calculate the speed of light in water, glass, and diamond using the index of refraction and the speed of light<br />

in a vacuum (3 × 10 8 m/s).<br />

4. When a light ray moves from water into air, does it slow down or speed up?<br />

5. When a light ray moves from water into glass, does it slow down or speed up?


Page 2 of 2<br />

Which way does the light ray bend?<br />

Now let’s look at some ray diagrams showing refraction. To make a refraction ray<br />

diagram, draw a solid line to show the boundary between the two materials (water and<br />

air in this case). Arrows are used to represent the incident and refracted light rays. The<br />

normal is a dashed line drawn perpendicular to the boundary between the surfaces. It<br />

starts at the point where the incident ray hits the boundary.<br />

As you can see, the light ray is bent toward the normal as it crosses from air into water.<br />

Light rays always bend toward the normal when they move from a low-n to a high-n<br />

material. The opposite occurs when light rays travel from a high-n to a low-n material.<br />

These light rays bend away from the normal.<br />

The amount of bending that occurs depends on the difference in the index of refraction of the two materials. A<br />

large difference in n causes a greater bend than a small difference.<br />

1. A light ray moves from water (n = 1.33) to a transparent plastic (polystyrene n = 1.59). Will the light ray<br />

bend toward or away from the normal?<br />

2. A light ray moves from sapphire (n = 1.77) to air (n = 1.0001). Does the light ray bend toward or away from<br />

the normal?<br />

3. Which light ray will be bent more, one moving from diamond (n = 2.42) to water (n = 1.33), or a ray moving<br />

from sapphire (n = 1.77) to air (n = 1.0001)?<br />

4.<br />

Material Index of refraction (n)<br />

Helium 1.00004<br />

Water 1.33<br />

Emerald 1.58<br />

Cubic Zirconia 2.17<br />

The diagrams below show light traveling from water (A) into another material (B). Using the chart above,<br />

label material B for each diagram as helium, water, emerald, or cubic zirconia.<br />

25.3


Name: Date:<br />

25.3 Drawing Ray Diagrams<br />

A ray diagram helps you see where the image produced by a lens appears. The components of the diagram<br />

include the lens, the principal axis, the focal point, the object, and three lines drawn from the tip of the object and<br />

through the lens. These light rays meet at a point and intersect on the other side of the lens. Where the light rays<br />

meet indicates where the image of the object appears.<br />

A lens has a focal length of 2 centimeters. An object is placed 4 centimeters to the left of the lens. Follow the<br />

steps to make a ray diagram using this information. Trace the rays and predict where the image will form.<br />

Steps:<br />

• Draw a lens and show the principal axis.<br />

• Draw a line that shows the plane of the lens.<br />

• Make a dot at the focal point of the lens on the right and left sides of the lens.<br />

• Place an arrow (pointing upward and perpendicular to the principle axis) at 4 centimeters on the left side of<br />

the lens.<br />

• Line 1: Draw a line from the tip of the arrow that is parallel to the principal axis on the left, and that goes<br />

through the focal point on the right of the lens.<br />

• Line 2: Draw a line from the tip of the arrow that goes through the center of the lens (where the plane and<br />

the principal axis cross).<br />

• Line 3: Draw a line from the tip of the arrow that goes through the focal point on the left side of the lens,<br />

through the lens, and parallel to the principal axis on the right side of the lens.<br />

• Lines 1, 2, and 3 converge on the right side of the lens where the tip of the image of the arrow appears.<br />

• The image is upside down compared with the object.<br />

25.3


Page 2 of 2<br />

25.3<br />

1. A lens has a focal length of 4 centimeters. An object is placed 8 centimeters to the left of the lens.<br />

Trace the rays and predict where the image will form. Is the image bigger, smaller, or inverted as compared<br />

with the object?<br />

2. Challenge question: An arrow is placed at 3 centimeters to the left of a converging lens. The image appears<br />

at 3 centimeters to the right of the lens. What is the focal length of this lens? (HINT: Place a dot to the right<br />

of the lens where the image of the tip of the arrow will appear. You will only be able to draw lines 1 and 2.<br />

Where does line 1 cross the principal axis if the image appears at 3 centimeters?)<br />

3. What happens when an object is placed at a distance from the lens that is less than the focal length? Use the<br />

term virtual image in your answer.


Name: Date:<br />

26.1 Astronomical Units<br />

Talking and writing about distances in our solar system can be cumbersome. The Sun and Neptune are on<br />

average 4,500,000,000 (or four billion, five hundred million) kilometers apart. Earth’s average distance from the<br />

Sun is 150,000,000 (one hundred fifty million) kilometers. It can be difficult to keep track of all the zeroes in<br />

such large numbers. And it’s not easy to compare numbers that large.<br />

Astronomers often switch to astronomical units (abbreviated AU) when describing distances in our solar system.<br />

One astronomical unit is 150,000,000 km—the same as the distance from Earth to the Sun.<br />

Neptune is 30 AU from the Sun. Not only is 30 an easier number to work with than 4,500,000,000; but using<br />

astronomical units allows us to see immediately that Neptune is 30 times as far from the Sun as Earth.<br />

In this skill sheet, you will practice working with astronomical units.<br />

• Jupiter is 778 million kilometers from the Sun, on average. Find this distance in astronomical units.<br />

Solution: Divide 778 million km by 150 million km: 778,000,000 ÷ 150,000,000 = 5.19 AU<br />

• The average distance from Mars to the Sun is 1.52 AU. Find this distance in kilometers.<br />

Solution: Multiply 1.52 AU by 150 million km: 1.52 × 150,000,000 = 228,000,000 km<br />

1. The average distance from Saturn to the Sun is 1.43 billion kilometers. Find this distance in astronomical<br />

units.<br />

2. The average distance from Venus to the Sun is 108 million kilometers. Find this distance in astronomical<br />

units.<br />

3. Mercury’s average distance from the Sun is 0.387 astronomical units. How far is this in kilometers?<br />

4. The average distance from Uranus to the Sun is 19.13 astronomical units. How far is this in kilometers?<br />

5. Is the distance from Earth to the moon more or less than one astronomical unit? How do you know?<br />

6. Which planet is almost 20 times as far away from the Sun as Earth?<br />

7. Which planet is less than half as far away from the Sun as Earth?<br />

8. Which planet is almost twice as far from the Sun as Jupiter?<br />

9. An unmanned spacecraft launched from Earth has traveled 10 astronomical units in the direction away from<br />

the Sun. It most recently passed through the orbit of which planet?<br />

10. An unmanned spacecraft launched from Earth has traveled 0.5 astronomical units toward the Sun. Has it<br />

passed through the orbit of Venus yet?<br />

26.1


Name: Date:<br />

26.1 Gravity Problems<br />

In this skill sheet, you will practice using proportions as you learn more about the strength of gravity on different<br />

planets.<br />

Comparing the strength of gravity on the planets<br />

Table 1 lists the strength of gravity on each planet in our solar system. We can see more clearly how these values<br />

compare to each other using proportions. First, we assume that Earth’s gravitational strength is equal to “1.”<br />

Next, we set up the proportion where x equals the strength of gravity on another planet (in this case, Mercury) as<br />

compared to Earth.<br />

1<br />

x<br />

=<br />

Earth gravitational strength Mercury gravitational strength<br />

1 x<br />

=<br />

9.8 N/kg 3.7 N/kg<br />

(1× 3.7 N/kg) = (9.8 N/kg × x)<br />

3.7 N/kg<br />

= x<br />

9.8 N/kg<br />

0.38 =<br />

x<br />

Note that the units cancel. The result tells us that Mercury’s gravitational strength is a little more than a third of<br />

Earth’s. Or, we could say that Mercury’s gravitational strength is 38% as strong as Earth’s.<br />

Now, calculate the proportions for the remaining planets.<br />

Table 1: The strength of gravity on planets in our solar system<br />

Planet Strength of gravity<br />

(N/kg)<br />

Value compared to Earth’s<br />

gravitational strength<br />

Mercury 3.7 0.38<br />

Venus 8.9<br />

Earth 9.8 1<br />

Mars 3.7<br />

Jupiter 23.1<br />

Saturn 9.0<br />

Uranus 8.7<br />

Neptune 11.0<br />

Pluto 0.6<br />

26.1


Page 2 of 2<br />

How much does it weigh on another planet?<br />

Use your completed Table 1 to solve the following problems.<br />

Example:<br />

• A bowling ball weighs 15 pounds on Earth. How much would this bowling ball weigh on Mercury?<br />

Weight on Earth 1<br />

=<br />

Weight on Mercury 0.38<br />

1 15 pounds<br />

=<br />

0.38 x<br />

0.38× 15 pounds = x<br />

x = 5.7 pounds<br />

1. A cat weighs 8.5 pounds on Earth. How much would this cat weigh on Neptune?<br />

2. A baby elephant weighs 250 pounds on Earth. How much would this elephant weigh on Saturn? Give your<br />

answer in newtons (4.48 newtons = 1 pound).<br />

3. On Pluto, a baby would weigh 2.7 newtons. How much does this baby weigh on Earth? Give your answer in<br />

newtons and pounds.<br />

4. Imagine that it is possible to travel to each planet in our solar system. After a space “cruise,” a tourist returns<br />

to Earth. One of the ways he recorded his travels was to weigh himself on each planet he visited. Use the list<br />

of these weights on each planet to figure out the order of the planets he visited. On Earth he weighs<br />

720 newtons. List of weights: 655 N; 1,699 N; 806 N; 43 N; and 662 N.<br />

Challenge: Using the Universal Law of Gravitation<br />

Here is an example problem that is solved using<br />

the equation for Universal Gravitation.<br />

Example<br />

What is the force of gravity between Pluto and<br />

Earth? The mass of Earth is 6.0 × 10 24 kg. The<br />

mass of Pluto is 1.3 × 10 22 kg. The distance<br />

between these two planets is 5.76 × 10 12 meters.<br />

Force of gravity between Earth and Pluto<br />

Now use the equation for Universal Gravitation to solve this problem:<br />

6.67 10 -11<br />

× N-m 2<br />

kg 2<br />

⎛ ⎞<br />

⎜------------------------------------------ 6.0 10<br />

⎟<br />

⎝ ⎠<br />

24<br />

( × kg)<br />

1.3 10 22<br />

× ( × kg)<br />

5.76 10 12<br />

( × m)<br />

2<br />

=<br />

-----------------------------------------------------------------------------<br />

Force of gravity<br />

52.0 10 35<br />

×<br />

33.2 10 24<br />

-------------------------- 1.57 10<br />

×<br />

11 = =<br />

× N<br />

5. What is the force of gravity between Jupiter and Saturn? The mass of Jupiter is 6.4 × 10 24 kg. The mass of<br />

Saturn is 5.7 × 10 26 kg. The distance between Jupiter and Saturn is 6.52 × 10 11 m.<br />

26.1


Name: Date:<br />

26.1 Universal Gravitation<br />

The law of universal gravitation allows you to calculate the gravitational force between two objects from their<br />

masses and the distance between them. The law includes a value called the gravitational constant, or “G.” This<br />

value is the same everywhere in the universe. Calculating the force between small objects like grapefruits or huge<br />

objects like planets, moons, and stars is possible using this law.<br />

What is the law of universal gravitation?<br />

The force between two masses m 1 and m 2 that are separated by a distance r is given by:<br />

So, when the masses m 1 and m 2 are given in kilograms and the distance r is given in meters, the force has the<br />

unit of newtons. Remember that the distance r corresponds to the distance between the center of gravity of<br />

the two objects.<br />

For example, the gravitational force between two spheres that are touching each other, each with a radius of<br />

0.300 meter and a mass of 1,000. kilograms, is given by:<br />

F 6.67 10 11<br />

× 1,000. kg 1,000. kg<br />

=<br />

– N-m 2 kg 2<br />

⁄<br />

×<br />

( 0.300 m + 0.300 m)<br />

2<br />

---------------------------------------------------- = 0.000185 N<br />

Note: A small car has a mass of approximately 1,000. kilograms. Try to visualize this much mass<br />

compressed into a sphere with a diameter of 0.300 meters (30.0 centimeters). If two such spheres were<br />

touching one another, the gravitational force between them would be only 0.000185 newtons. On Earth, this<br />

corresponds to the weight of a mass equal to only 18.9 milligrams. The gravitational force is not very strong!<br />

26.1


Page 2 of 2<br />

Answer the following problems. Write your answers using scientific notation.<br />

1. Calculate the force between two objects that have masses of 70. kilograms and 2,000. kilograms. Their<br />

centers of gravity are separated by a distance of 1.00 meter.<br />

2. Calculate the force between two touching grapefruits each with a radius of 0.080 meters and a mass of<br />

0.45 kilograms.<br />

3. Calculate the force between one grapefruit as described above and Earth. Earth has a mass of<br />

5.9742 × 10 24 kilograms and a radius of 6.3710 × 10 6 meters. Assume the grapefruit is resting on Earth’s<br />

surface.<br />

4. A man on the moon with a mass of 90. kilograms weighs 146 newtons. The radius of the moon is<br />

1.74 × 106 meters. Find the mass of the moon.<br />

5. For m = 5.9742 × 1024 kilograms and r = 6.3710 × 106 meters, what is the value given by: G ?<br />

a. Write down your answer and simplify the units.<br />

b. What does this number remind you of?<br />

c. What real-life values do m and r correspond to?<br />

m<br />

r 2<br />

----<br />

6. The distance between the centers of Earth and its moon is 3.84 × 10 8 meters. Earth’s mass is<br />

5.9742 × 10 24 kilograms and the mass of the moon is 7.36 × 10 22 kilograms. What is the force between Earth<br />

and the moon?<br />

7. A satellite is orbiting Earth at a distance of 35.0 kilometers. The satellite has a mass of 500. kilograms. What<br />

is the force between the planet and the satellite? Hint: Recall Earth’s mass and radius from earlier problems.<br />

8. The mass of the sun is 1.99 × 10 30 kilograms and its distance from Earth is 150. million kilometers<br />

(150. × 10 9 meters). What is the gravitational force between the sun and Earth?<br />

26.1


Page 1 of 2<br />

26.1 Nicolaus Copernicus<br />

Nicolaus Copernicus was a church official, mathematician, and influential astronomer. His<br />

revolutionary theory of a heliocentric (sun-centered) universe became the foundation of<br />

modern-day astronomy.<br />

Wealth, education, and religion<br />

Nicolaus Copernicus was<br />

born on February 19, 1473<br />

in Torun, Poland.<br />

Copernicus’ father was a<br />

successful copper merchant.<br />

His mother also came from<br />

wealth. Being from a<br />

privileged family, young<br />

Copernicus had the luxury<br />

of learning about art,<br />

literature, and science.<br />

When Copernicus was only<br />

10 years old, his father died. Copernicus went to live<br />

his uncle, Lucas Watzenrode, a prominent Catholic<br />

Church official who became bishop of Varmia (now<br />

part of modern-day Poland) in 1489. The bishop was<br />

generous with his money and provided Copernicus<br />

with an education from the best universities.<br />

From church official to astronomer<br />

Copernicus lived during the height of the Renaissance<br />

period when men from a higher social class were<br />

expected to receive well-rounded educations. In 1491,<br />

Copernicus attended the University of Krakow where<br />

he studied mathematics and astronomy. After four<br />

years of study, his uncle appointed Copernicus a<br />

church administrator. Copernicus used his church<br />

wages to help pay for additional education.<br />

In January 1497, Copernicus left for Italy to study<br />

medicine and law at the University of Bologna.<br />

Copernicus’ passion for astronomy grew under the<br />

influence of his mathematics professor, Domenico<br />

Maria de Novara. Copernicus lived in his professor’s<br />

home where they spent hours discussing astronomy.<br />

In 1500, Copernicus lectured on astronomy in Rome.<br />

A year later, he studied medicine at the University of<br />

Padua. In 1503, Copernicus received a doctorate in<br />

canon (church) law from the University of Ferrara.<br />

Observations with his bare eyes<br />

After his studies in Italy, Copernicus returned to<br />

Poland to live in his uncle’s palace. He resumed his<br />

church duties, practiced medicine, and studied<br />

astronomy. Copernicus examined the sky from a<br />

palace tower. He made his observations without any<br />

equipment. In the late 1500s, the astronomer Galileo<br />

used a telescope and confirmed Copernicus’ ideas.<br />

26.1<br />

A heliocentric universe<br />

In the 1500s, most astronomers believed that Earth<br />

was motionless and the center of the universe. They<br />

also thought that all celestial bodies moved around<br />

Earth in complicated patterns. The Greek astronomer<br />

Ptolomy proposed this geocentric theory more than<br />

1000 years earlier.<br />

However, Copernicus believed that the universe was<br />

heliocentric (sun-centered), with all of the planets<br />

revolving around the sun. He explained that Earth<br />

rotates daily on its axis and revolves yearly around the<br />

sun. He also suggested that Earth wobbles like a top as<br />

it rotates.<br />

Copernicus’ theory led to a new ordering of the planets.<br />

In addition, it explained why the planets farther from<br />

the sun sometimes appear to move backward<br />

(retrograde motion), while the planets closest to the sun<br />

always seem to move in one direction. This retrograde<br />

motion is due to Earth moving faster around the sun<br />

than the planets farther away.<br />

Copernicus was reluctant to publish his theory and<br />

spent years rechecking his data. Between 1507 and<br />

1515, Copernicus circulated his heliocentric principles<br />

to only a few astronomers. A young German<br />

mathematics professor, George Rheticus, was<br />

fascinated with Copernicus’ theory. The professor<br />

encouraged Copernicus to publish his ideas. Finally,<br />

Copernicus published The Revolutions of the<br />

Heavenly Orbs near his death in 1543.<br />

Years later, several astronomers (including Galileo)<br />

embraced Copernicus’ sun-centered theory. However,<br />

they suffered much persecution by the church for<br />

believing such ideas. At the time, church law held<br />

great influence over science and dictated a geocentric<br />

universe. It wasn’t until the eighteenth century that<br />

Copernicus’ heliocentric principles were completely<br />

accepted.


Page 2 of 2<br />

Reading reflection<br />

1. How did Copernicus’ privileged background help him become knowledgeable in so many areas of<br />

study?<br />

2. Which people influenced Copernicus in his work as a church official and an astronomer?<br />

3. How did Copernicus make his observations of the sky?<br />

4. What did astronomers believe of the universe prior to the sixteenth century?<br />

5. Describe Copernicus’ revolutionary heliocentric theory of the universe.<br />

6. Why did so many astronomers face persecution by the church for their beliefs in a heliocentric universe?<br />

7. Research: Using the library or Internet, find out which organizations developed the Copernicus Satellite<br />

(OAO-3) and why it was used.<br />

26.1


Page 1 of 2<br />

26.1 Galileo Galilei<br />

Galileo Galilei was a mathematician, scientist, inventor, and astronomer. His observations led to<br />

advances in our understanding of pendulum motion and free fall. He invented a thermometer, water pump,<br />

military compass, and microscope. He refined a Dutch invention, the telescope, and used it to revolutionize<br />

our understanding of the solar system.<br />

An incurable mathematician<br />

Galileo Galilei was born in<br />

Pisa, Italy, on February 15,<br />

1564. His father, a musician<br />

and wool trader, hoped his son<br />

would find a more profitable<br />

career. He sent Galileo to a<br />

monastery school at age 11 to<br />

prepare for medical school.<br />

After four years there, Galileo<br />

decided to become a monk.<br />

The eldest of seven children,<br />

he had sisters who would need dowries in order to<br />

marry, and his father had planned on Galileo’s<br />

support. His father decided to take Galileo out of the<br />

monastery school.<br />

Two years later, he enrolled as a medical student at the<br />

University of Pisa, though his interests were<br />

mathematics and natural philosophy. Galileo did not<br />

really want to pursue medical studies. Eventually, his<br />

father agreed to let him study mathematics.<br />

Seeing through the ordinary<br />

Galileo was extremely curious. At 20, he found<br />

himself watching a lamp swinging from a cathedral<br />

ceiling. He used his pulse like a stopwatch and<br />

discovered that the lamp’s long and short swings took<br />

the same amount of time. He wrote about this in an<br />

early paper titled “On Motion.” Years later, he drew<br />

up plans for an invention, a pendulum clock, based on<br />

this discovery.<br />

Inventions and experiments<br />

Galileo started teaching at the University of Padua in<br />

1592 and stayed for 18 years. He invented a simple<br />

thermometer, a water pump, and a compass for<br />

accurately aiming cannonballs. He also performed<br />

experiments with falling objects, using an inclined<br />

plane to slow the object’s motion so it could be more<br />

accurately timed. Through these experiments, he<br />

realized that all objects fall at the same rate unless<br />

acted on by another force.<br />

26.1<br />

Crafting better telescopes<br />

In 1609, Galileo heard that a Dutch eyeglass maker<br />

had invented an instrument that made things appear<br />

larger. Soon he had created his own 10-powered<br />

telescope. The senate in Venice was impressed with its<br />

potential military uses, and in a year, Galileo had<br />

refined his invention to a 30-powered telescope.<br />

Star gazing<br />

Using his powerful telescope, Galileo’s curiosity now<br />

turned skyward. He discovered craters on the moon,<br />

sunspots, Jupiter’s four largest moons, and the phases<br />

of Venus. His observations led him to conclude that<br />

Earth could not possibly be the center of the universe,<br />

as had been commonly accepted since the time of the<br />

Greek astronomer Ptolemy in the second century.<br />

Instead, Galileo was convinced that Polish astronomer<br />

Nicolaus Copernicus (1473-1543) must have been<br />

right: The Sun is at the center of the universe and the<br />

planets revolve around it.<br />

House arrest<br />

The Roman Catholic Church held that Ptolemy’s<br />

theory was truth and Copernican theory was heresy.<br />

Galileo had been told by the Inquisition in 1616 to<br />

abandon Copernican theory and stop pursuing these<br />

ideas.<br />

Despite these threats, in February 1632, he published<br />

his ideas in the form of a conversation between two<br />

characters. He made the one representing Ptolemy’s<br />

view seem foolish, while the other, who argued<br />

Copernicus’s theory, seemed wise.<br />

This angered the church, whose permission was<br />

needed for publishing <strong>book</strong>s. Galileo was called to<br />

Rome before the Inquisition. He was given a formal<br />

threat of torture and so he abandoned his ideas that<br />

promoted Copernican theory. He was sentenced to<br />

house arrest, and lived until his death in 1642 watched<br />

over by Inquisition guards.


Page 2 of 2<br />

Reading reflection<br />

1. What scientific information was presented in Galileo’s paper “On Motion”?<br />

2. Research one of Galileo’s inventions and draw a diagram showing how it worked.<br />

3. How were Galileo’s views about the position of Earth in the universe supportive of Copernicus’s ideas?<br />

4. Imagine you could travel back in time to January 1632 to meet with Galileo just before he publishes his<br />

“Dialogue Concerning the Two Chief World Systems.” What would you say to him?<br />

5. In your opinion, which of Galileo’s ideas or inventions had the biggest impact on history? Why?<br />

26.1


Page 1 of 2<br />

26.1 Johannes Kepler<br />

26.1<br />

Johannes Kepler was a mathematician who studied astronomy. He lived at the same time as two other<br />

famous astronomers, Tycho Brahe and Galileo Galilei. Kepler is recognized today for his use of mathematics to<br />

solve problems in astronomy. Kepler explained that the orbit of Mars and other planets is an ellipse. In his most<br />

famous <strong>book</strong>s he defended the sun-centered universe and his three laws of planetary motion.<br />

Early years in Germany<br />

Johannes Kepler was born<br />

December 27, 1571, in Weil<br />

der Stadt, Wurttemburg,<br />

Germany, now called the<br />

“Gate to the Black Forest.”<br />

He was the oldest of six<br />

children in a poor family.<br />

As a child he lived and<br />

worked in an inn run by his<br />

mother’s family. He was<br />

sickly, nearsighted, and<br />

suffered from smallpox at a<br />

young age. Despite his physical condition, he was a<br />

bright student.<br />

The first school Kepler attended was a convent school<br />

in Adelberg monastery. Kepler’s original plan was to<br />

study to become a Lutheran minister. In 1589, Kepler<br />

received a scholarship to attend the University of<br />

Tubingen. There he spent three years studying<br />

mathematics, philosophy, and theology. His interest in<br />

math led him to take a mathematics teaching position<br />

at the Academy in Graz. There he began teaching and<br />

studying astronomy.<br />

Influenced by Copernicus<br />

At Tubingen, Kepler’s professor, Michael Mastlin,<br />

introduced Kepler to Copernican astronomy.<br />

Nicholaus Copernicus (1473-1543), had published a<br />

revolutionary theory in, “On the Revolutions of<br />

Heavenly Bodies.” Copernicus’ theory stated that the<br />

sun was the center of the solar system. Earth and the<br />

planets rotated around the sun in circular orbits. At the<br />

time most people believed that Earth was the center of<br />

the universe.<br />

Copernican theory intrigued Kepler and he wrote a<br />

defense of it in 1596, Mysterium Cosmographicum.<br />

Although Kepler’s original defense was flawed, it was<br />

read by several other famous European astronomers of<br />

the time, Tycho Brahe (1546–1601) and Galileo<br />

Galilei (1546–1642).<br />

Kepler published many <strong>book</strong>s in which he explained<br />

how vision, optics, and telescopes work. His most<br />

famous work, though, dealt with planetary motion.<br />

Working with Tyco Brahe<br />

In 1600, Brahe invited Kepler to join him. Brahe,<br />

a Danish astronomer, was studying in Prague,<br />

Czechoslovakia. Every night for years Brahe recorded<br />

planetary motion without a telescope from his<br />

observatory. Brahe asked Kepler to figure out a<br />

scientific explanation for the motion of Mars. Less<br />

than two years later, Brahe died. Kepler was awarded<br />

Brahe’s position as Imperial Mathematician. He<br />

inherited Brahe’s collection of planetary observations<br />

to use to write mathematical descriptions of planetary<br />

motion.<br />

Kepler’s Laws of Planetary Motion<br />

Kepler discovered that Mars’ orbit was an ellipse, not<br />

a circle, as Copernicus had thought. Kepler published<br />

his first two laws of planetary motion in Astronomia<br />

Nova in 1609. The first law of planetary motion stated<br />

that planets orbit the sun in an elliptical orbit with the<br />

sun in one of the foci. The second law, the law of<br />

areas, said that planets speed up as their orbit is closest<br />

to the sun, and slow down as planets move away from<br />

the sun. Kepler published a third law, called the<br />

harmonic law, in 1619. The third law shows how a<br />

planet’s distance from the sun is related to the amount<br />

of time it takes to revolve around the sun. His work<br />

influenced Isaac Newton’s later work on gravity.<br />

Kepler’s calculations were done before calculus was<br />

invented!<br />

Other scientific discoveries<br />

Kepler sent his <strong>book</strong> in 1609 to Galileo. Galileo’s<br />

theories did not agree with Kepler’s ideas and the two<br />

scientists never worked together. Despite his<br />

accomplishments, when Kepler died at age 59, he was<br />

poor and on his way to collect an old debt. It would<br />

take close to a century for his work to gain the<br />

recognition it deserved.


Page 2 of 2<br />

Reading reflection<br />

1. Why was Copernicus’ idea of the sun at the center of the solar system considered revolutionary?<br />

2. Explain how Brahe helped Kepler make important discoveries in astronomy.<br />

3. How was Kepler’s approach to astronomy different than Brahe’s and Galileo’s?<br />

4. Kepler discovered that Mars and other planets traveled in an ellipse around the sun. Does this agree with<br />

Copernicus’ theory?<br />

5. Describe Kepler’s three laws of planetary motion.<br />

6. Kepler observed a supernova in 1604. It challenged the way people at the time thought about the universe<br />

because people did not know the universe could change. When people have to change their beliefs about<br />

something because scientific evidence says otherwise, that is called a “paradigm shift.” Find three examples<br />

in the text of scientific discoveries that led to a “paradigm shift.”<br />

26.1


Name: Date:<br />

26.1 Measuring the Moon’s Diameter<br />

In this skill sheet you will explore how to measure the moon’s diameter using simple tools and calculations.<br />

Materials<br />

Here are the materials you will need to measure the moon’s diameter:<br />

• A 3-meter piece of string • Scissors<br />

• A metric tape measure • Marker<br />

• A small metric ruler • One-centimeter semi-circle card (Cut out from the<br />

bottom of the last page)<br />

• Tape<br />

Proportions and geometry<br />

The method you will use to measure the moon’s diameter depends on the properties of similar triangles. The<br />

following exercise demonstrates these properties.<br />

Below is a large triangle. A line drawn from one side to the other of the large triangle results in a smaller triangle<br />

inside the larger one. The ends of each line are labeled with letters.<br />

1. Make the following measurements of the lines on the triangle:<br />

Distance AC: ____________________ cm<br />

Distance AD: ____________________ cm<br />

Distance AB: ____________________ cm<br />

Distance AE: ____________________ cm<br />

Distance BE: ____________________ cm<br />

Distance CD: ____________________ cm<br />

26.1


Page 2 of 3<br />

2. How is the distance from AB related to AC?<br />

3. How is the distance from BE related to CD?<br />

4. Based on your measurements and your answers to questions (2) and (3), come up with a statement<br />

that explains the properties of similar triangles.<br />

Finding the diameter of the moon<br />

Now, you are ready to use your supplies to find the diameter of the moon. Follow these steps carefully and<br />

answer the questions as you go.<br />

1. Locate a place where you can see the moon from a window. This is possible at night or early in the morning.<br />

2. Use scissors to carefully cut out the semi-circle card found on the next page.<br />

3. Tape this card to the window when you can see the full (or gibbous) moon through the window.<br />

4. Tape one end of the 3-meter piece of string to the card directly underneath the semi-circle.<br />

5. Now, slowly move backward from the window while holding on to the string. Watch your step! As you<br />

move backward, pay attention to the moon. You want to move back far enough so that the bottom half of the<br />

moon “sits” in the semi-circle cutout. You want the semi-circle to be the same size as the lower half to the<br />

moon.<br />

6. When the lower half of the moon is the same size as the semi-circle cut out, stop moving backward and hold<br />

the string up to the side of one of your eyes. Have a friend carefully mark the string at this distance.<br />

7. Now, measure the distance from the window to the mark on the string to the nearest millimeter. Convert this<br />

distance to meters. Write the string distance in Table 1.<br />

Table 1:Finding the moon’s diameter data<br />

Semi-circle diameter 0.01 meter<br />

String distance<br />

Diameter of the moon ???<br />

Distance from Earth to the moon 384, 400, 000 meters<br />

Finding the moon’s diameter<br />

1. You have three out of four measurements in Table 1. The only measurement you need is the moon’s<br />

diameter. You can find the moon’s diameter using proportions. Which calculation will help you?<br />

a. b.<br />

semi-circle<br />

----------------------------------------------diameter<br />

moon diameter<br />

c. d.<br />

----------------------------------------------moon<br />

diameter<br />

semi-circle diameter<br />

=<br />

=<br />

---------------------------------------------------------------------------distance<br />

to semi-circle<br />

distance from Earth to the moon<br />

---------------------------------------------------------------------------distance<br />

to semi-circle<br />

distance from Earth to the moon<br />

---------------------------------------------------semi-circle<br />

diameter<br />

distance to semi-circle<br />

----------------------------------------------moon<br />

diameter<br />

semi-circle diameter<br />

2. Use the proportion that you selected in question (1) to calculate the moon’s diameter.<br />

3. How is performing this calculation like the exercise you did in part 2?<br />

=<br />

=<br />

---------------------------------------------------------------------------moon<br />

diameter<br />

distance from Earth to the moon<br />

distance---------------------------------------------------------------------------from<br />

Earth to the moon<br />

distance to semi-circle<br />

26.1


Page 3 of 3<br />

Semi-Circle Card:<br />

26.1


Page 1 of 2<br />

26.2 Benjamin Banneker<br />

Benjamin Banneker was a farmer, naturalist, civil rights advocate, self-taught mathematician,<br />

astronomer and surveyor who published his detailed astronomical calculations in popular almanacs.<br />

He was appointed by President George Washington as one of three surveyors of the territory that<br />

became Washington D.C.<br />

Early times<br />

Benjamin Banneker was born<br />

in rural Maryland in 1731. His<br />

family was part of a<br />

population of about two<br />

hundred free black men and<br />

women in Baltimore county.<br />

They owned a small farm<br />

where they grew tobacco and<br />

vegetables, earning a<br />

comfortable living.<br />

A mathematician builds a clock<br />

Benjamin’s grandmother taught him to read, and he<br />

briefly attended a Quaker school near his home.<br />

Benjamin enjoyed school and was especially fond of<br />

solving mathematical riddles and puzzles. When he<br />

was 22, Benjamin borrowed a pocket watch, took it<br />

apart, and made detailed sketches of its inner<br />

workings. Then he carved a large-scale wooden model<br />

of each piece, fashioned a homemade spring, and built<br />

his own clock that kept accurate time for over<br />

50 years.<br />

A keen observer of the night sky<br />

As a young adult, Benjamin designed an irrigation<br />

system that kept his family farm prosperous even in<br />

dry years. The Bannekers sold their produce at a<br />

nearby store owned by a Quaker family, the Ellicotts.<br />

There, Benjamin became friends with George Ellicott,<br />

who loaned him <strong>book</strong>s about astronomy and<br />

mathematics.<br />

Banneker was soon recording detailed observations of<br />

the night sky. He performed complicated calculations<br />

to predict the positions of planets and the timing of<br />

eclipses. From 1791 to 1797, Banneker published his<br />

astronomical calculations along with weather and tide<br />

predictions, literature, and commentaries in six<br />

almanacs. The almanacs were widely read in<br />

Maryland, Delaware, Pennsylvania, and Virginia,<br />

bringing Banneker a measure of fame.<br />

26.2<br />

A keen observer of nature<br />

Banneker was also a keen observer of the natural<br />

world and is believed to be the first person to<br />

document the cycle of the 17-year cicada, an insect<br />

that exists in the larval stage underground for 17 years,<br />

and then emerges to live for just a few weeks as a loud<br />

buzzing adult.<br />

Banneker writes Thomas Jefferson<br />

Banneker sent a copy of his first almanac to then-<br />

Secretary of State Thomas Jefferson, along with a<br />

letter challenging Jefferson’s ownership of slaves as<br />

inconsistent with his assertion in the Declaration of<br />

Independence that “all men are created equal.”<br />

Jefferson sent a letter thanking Banneker for the<br />

almanac, saying that he sent it onto the Academy of<br />

Sciences of Paris as proof of the intellectual<br />

capabilities of Banneker’s race. Although Jefferson’s<br />

letter stated that he “ardently wishes to see a good<br />

system commenced for raising the condition both of<br />

[our black brethren’s] body and mind,” regrettably, he<br />

never freed his own slaves.<br />

Designing Washington D.C.<br />

In 1791, George Ellicott’s cousin Andrew Ellicott<br />

asked him to serve as an astronomer in a large<br />

surveying project. George Ellicott suggested that he<br />

hire Benjamin Banneker instead.<br />

Banneker left his farm in the care of relatives and<br />

traveled to Washington, where he became one of three<br />

surveyors appointed by President George Washington<br />

to assist in the layout of the District of Columbia.<br />

After his role in the project was complete, Banneker<br />

returned to his Maryland farm, where he died in 1806.<br />

Banneker Overlook Park, in Washington D.C.,<br />

commemorates his role in the surveying project. In<br />

1980, the U.S. Postal Service issued a stamp in<br />

Banneker’s honor.


Page 2 of 2<br />

Reading reflection<br />

1. Benjamin Banneker built a working clock that lasted 50 years. Why would his understanding of<br />

mathematics have been helpful in building the clock?<br />

2. Identify one of Banneker’s personal strengths. Justify your answer with examples from the reading.<br />

3. Benjamin Banneker lived from 1731 to 1806. During his lifetime, he advocated equal rights for all people.<br />

Find out the date for each of the following “equal rights” events: (a) the Emacipation Proclamation, (b) the<br />

end of the Civil War, (c) women gain the right to vote, and (d) the desegregation of public schools (due to<br />

the landmark Supreme Court case, Brown versus the Board of Education).<br />

4. Name three of Benjamin Banneker’s lifetime accomplishments.<br />

5. What do you think motivated Banneker during his lifetime? What are some possible reasons that he was<br />

persistent in his scientific work?<br />

6. Research: Find a mathematical puzzle written by Banneker. Try to solve it with your class.<br />

26.2


Name: Date:<br />

26.3 Touring the Solar System<br />

What would a tour of our solar system be like? How long would it take? How much food would you have to<br />

bring? In this skill sheet, you will calculate the travel times for an imaginary tour of the solar system. For our<br />

purposes, we will pretend that the planets form one straight line away from the Sun. Your mode of transportation<br />

will be a space vehicle travelling at 250 meters per second or 570 miles per hour.<br />

Part 1: Planets on the tour<br />

Starting from Earth, the tour itinerary is: Earth to Mars to Saturn to Neptune to Venus and then back to Earth.<br />

The distances between each planet of the tour are provided in Table 1. The space vehicle travels at 250 meters per<br />

second or 900 kilometers per hour. Using this rate and the speed formula, find out how long it will take to travel<br />

each leg of the itinerary. An example is provided below. For the table, also calculate the time in days and years.<br />

• How many days will it take to travel from Earth to Mars if the distance between the planets is 78 million<br />

kilometers?<br />

Solution:<br />

Legs of the trip Distance<br />

traveled for<br />

each leg (km)<br />

Earth to Mars 78,000,000<br />

Mars to Saturn 1,202,000,000<br />

Saturn to Neptune 3,070,000,000<br />

Neptune to Venus 4,392,000,000<br />

Venus to Earth 42,000,000<br />

distance<br />

time =<br />

speed<br />

78 million km<br />

time to travel from Earth to Mars =<br />

km<br />

900<br />

hour<br />

time to travel from Earth to Mars = 86,666 hours<br />

1 day<br />

86,666 hours× =<br />

3,611 days<br />

24 hours<br />

Table 1: Solar System Trip<br />

Hours<br />

traveled<br />

Days<br />

traveled<br />

Years<br />

traveled<br />

26.3


Page 2 of 2<br />

Part 2: Provisions for the trip<br />

A trip through the solar system is a science fiction fantasy. Answer the following questions as if such a<br />

journey were possible.<br />

1. It is recommended that a person drink eight glasses of water each day. To keep yourself hydrated on your<br />

trip. How many glasses of water would you need to drink on the leg from Earth to Mars?<br />

2. An average person needs 2,000 food calories per day. How many food calories will you need for the leg of<br />

the journey from Neptune to Venus?<br />

3. Proteins and carbohydrates provide 4 calories per gram. Fat provides 9 calories per gram. Given this<br />

information, would it be more efficient to pack the plane full of foods that are high in fat or high in protein<br />

for the journey? Explain your answer.<br />

4. You decide that you want to celebrate Thanksgiving each year of your travel. How many frozen turkeys will<br />

you need for the entire journey?<br />

Part 3: Planning a trip to all eight planets<br />

Section 26.3 of your student text presents a table that lists the properties of the eight planets. Use this table to<br />

answer the following questions.<br />

1. On which planet would there be the most opportunities to visit a moon?<br />

2. Which planets would require high-tech clothing to endure high temperatures? Which planets would require<br />

high-tech clothing to endure cold temperatures?<br />

3. Which planet has the longest day?<br />

4. Which has the shortest day?<br />

5. On which planet would you have the most weight? How much would you weigh in newtons?<br />

6. Which planet would take the longest time to travel around?<br />

7. Which planet would require your spaceship to orbit with the fastest orbital speed? Explain your answer.<br />

26.3


Name: Date:<br />

27.1 The Sun: A Cross-Section<br />

27.1


Page 1 of 2<br />

27.1 Arthur Walker<br />

27.1<br />

Arthur Walker pioneered several new space-based research tools that brought about significant<br />

changes in our understanding of the sun and its corona. He was instrumental in the recruitment and retention of<br />

minority students at Stanford University, and he advised the United States Congress on physical science policy<br />

issues.<br />

Not to be discouraged<br />

Arthur Walker was born in<br />

Cleveland in 1936. His father<br />

was a lawyer and his mother a<br />

social worker. When he was 5,<br />

the family moved to New<br />

York. Arthur was an excellent<br />

student and his mother<br />

encouraged him to take the<br />

entrance exam for the Bronx<br />

<strong>High</strong> <strong>School</strong> of Science.<br />

Arthur passed the exam, but when he entered school a<br />

faculty member told him that the prospects for a black<br />

scientist in the United States were bleak.<br />

Rather than allow him to become dissuaded from his<br />

aspirations, Arthur’s mother visited the school and<br />

told them that her son would pursue whatever course<br />

of study he wished.<br />

Making his mark in space<br />

Walker went on to earn a bachelor’s degree in physics,<br />

with honors, from Case Institute of Technology in<br />

Cleveland and, by 1962, his master’s and doctorate<br />

from the University of Illinois.<br />

Afterward, he spent three years’ active duty with the<br />

Air Force, where he designed a rocket probe and<br />

satellite experiment to measure radiation that affects<br />

satellite operation. This work sparked Walker’s<br />

lifelong interest in developing new space-based<br />

research tools.<br />

After completing his military service, Walker worked<br />

with other scientists to develop the first X-ray<br />

spectrometer used aboard a satellite. This device<br />

helped determine the temperature and composition of<br />

the sun’s corona and provided new information about<br />

how matter and radiation interact in plasma.<br />

Snapshots of the sun<br />

In 1974, Walker joined the<br />

faculty at Stanford<br />

University. There he<br />

pioneered the use of a new<br />

multilayer mirror<br />

technology in space<br />

observations. The mirrors<br />

selectively reflected X rays<br />

of certain wavelengths, and<br />

enabled Walker to obtain<br />

the first high-resolution images showing different<br />

temperature regions of the solar atmosphere. He then<br />

worked to develop telescopes using the multilayer<br />

mirror technology, and launched them into space on<br />

rockets. The telescopes produced detailed photos of<br />

the sun and its corona. One of the pictures was<br />

featured on the cover of the journal Science in<br />

September 1988.<br />

A model for student scientists<br />

Walker was a mentor to many graduate students,<br />

including Sally Ride, who went on to become the first<br />

American woman in space. He worked to recruit and<br />

retain minority applicants to Stanford’s natural and<br />

mathematical science programs. Walker was<br />

instrumental in helping Stanford University graduate<br />

more black doctoral physicists than any university in<br />

the United States.<br />

At work in other orbits<br />

Public service was important to Walker, who served<br />

on several committees of the National Aeronautics<br />

and Space Administration (NASA), National Science<br />

Foundation, and National Academy of Science,<br />

working to develop policy recommendations for<br />

Congress. He was also appointed to the presidential<br />

commission that investigated the 1986 space shuttle<br />

Challenger accident.<br />

Walker died of cancer in April 2001.


Page 2 of 2<br />

Reading reflection<br />

1. Use your text<strong>book</strong>, an Internet search engine, or a dictionary to find the definition of each word in<br />

bold type. Write down the meaning of each word. Be sure to credit your source.<br />

2. What have you learned about pursuing goals from Arthur Walker’s biography?<br />

3. Why is a spectrometer a useful device for measuring the temperature and composition of something like the<br />

sun’s corona?<br />

4. Research: Use a library or the Internet to find one of Walker’s revolutionary photos of the sun and its<br />

corona. Present the image to your class.<br />

5. Research: Use a library or the Internet to find more about the commission that investigated the explosion of<br />

the space shuttle Challenger in 1986. Summarize the commission’s findings and recommendations in two or<br />

three paragraphs.<br />

27.1


Name: Date:<br />

27.2 The Inverse Square Law<br />

If you stand one meter away from a portable stereo blaring your favorite music, the intensity of the sound may<br />

hurt your ears. As you back away from the stereo, the sound’s intensity decreases. When you are two meters<br />

away, the sound intensity is one-fourth its original value. When you are ten meters away, the sound intensity is<br />

one-one hundredth its original value.<br />

The sound intensity decreases according to the inverse square law. This means that the intensity decreases as the<br />

square of the distance. If you triple your distance from the stereo, the sound intensity decreases by nine times its<br />

original value.<br />

Many fields follow an inverse square law, including sound, light from a small source (like a match or light bulb),<br />

gravity, and electricity. Magnetic fields are the exception. They decrease much faster because there are two<br />

magnetic poles.<br />

Example 1: The light intensity one meter from a bulb is 2 W/m 2 . What is the light intensity measured from a<br />

distance of four meters from the bulb?<br />

Solution: The distance has increased to four times its original value. The light intensity will decrease by 42 or 16, times.<br />

1 1<br />

2× =<br />

or 0.125 W/m<br />

16 8<br />

Example 2: Mercury has a gravitational force of 3.7 N/kg. Its radius is 2,439 kilometers. How far away from the<br />

surface of Mercury would you need to move in order to experience a gravitational force of 0.925 N/kg?<br />

Solution: For the gravitational force to be reduced to one-fourth its original value, the distance from Mercury’s center<br />

must be doubled. Therefore you would have to move to a spot 2,439 kilometers away from Mercury’s surface or 4,878<br />

meters from its center.<br />

1. You stand 4 meters away from a light and measure the intensity to be 1 W/m 2 . What will be the intensity if<br />

you move to a position 8 meters away from the bulb?<br />

2. You are standing 1 meter from a squawking parrot. If you move to a distance three meters away, the sound<br />

intensity will be what fraction of its original value?<br />

3. Venus has a gravitational force of 8.9 N/kg. Its radius is 6,051 kilometers. How far away from the surface of<br />

Mercury would you need to move in order to experience a gravitational force of 0.556 N/kg?<br />

4. Earth’s radius is 6,378 kilometers. If you weigh 500 newtons on Earth’s surface, what would you weigh at a<br />

distance of 19,134 kilometers from Earth?<br />

5. Compare the intensity of light 2 meters away from a lit match to the intensity 6 meters away from the match.<br />

6. How does the strength of a sound field 1 meter from its source compare with its strength 4 meters away?<br />

2<br />

27.2


Name: Date:<br />

28.1 Scientific Notation<br />

28.1<br />

A number like 43,200,000,000,000,000,000 (43 quintillion, 200 quadrillion) can take a long time to<br />

write, and an even longer time to read. Because scientists frequently encounter very large numbers like this one<br />

(and also very small numbers, such as 0.000000012, or twelve trillionths), they developed a shorthand method<br />

for writing these types of numbers. This method is called scientific notation. A number is written in scientific<br />

notation when it is written as the product of two factors, where the first factor is a number that is greater than or<br />

equal to 1, but less than 10, and the second factor is an integer power of 10. Some examples of numbers written<br />

in scientific notation are given in the table below:<br />

Scientific Notation Standard Form<br />

4.32 × 10 19 43,200,000,000,000,000,000<br />

1.2 × 10 –8 0.000000012<br />

5.2777 × 10 7<br />

Rewrite numbers given in scientific notation in standard form.<br />

52,777,000<br />

6.99 10 -5 0.0000699<br />

• Express 4.25 × 10 6 in standard form: 4.25 × 10 6 = 4,250,000<br />

Move the decimal point (in 4.25) six places to the right. The exponent of the “10” is 6, giving us the number<br />

of places to move the decimal. We know to move it to the right since the exponent is a positive number.<br />

• Express 4.033 × 10 –3 in standard form: 4.033 × 10 –3 = 0.004033<br />

Move the decimal point (in 4.033) three places to the left. The exponent of the “10” is negative 3, giving the<br />

number of places to move the decimal. We know to move it to the left since the exponent is negative.<br />

Rewrite numbers given in standard form in scientific notation.<br />

• Express 26,040,000,000 in scientific notation: 26,040,000,000 = 2.604 × 10 10<br />

Place the decimal point in 2 6 0 4 so that the number is greater than or equal to one (but less than ten). This<br />

gives the first factor (2.604). To get from 2.604 to 26,040,000,000 the decimal point has to move 10 places to<br />

the right, so the power of ten is positive 10.<br />

• Express 0.0001009 in scientific notation: 0.0001009 = 1.009 × 10 –4<br />

Place the decimal point in 1 0 0 9 so that the number is greater than or equal to one (but less than ten). This<br />

gives the first factor (1.009). To get from 1.009 to 0.0001009 the decimal point has to move four places to<br />

the left, so the power of ten is negative 4.


Page 2 of 3<br />

1. Fill in the missing numbers. Some will require converting scientific notation to standard form,<br />

while others will require converting standard form to scientific notation.<br />

a. 6.03 × 10 –2<br />

b. 9.11 × 10 5<br />

c. 5.570 × 10 –7<br />

2. Explain why the numbers below are not written in scientific notation, then give the correct way to write the<br />

number in scientific notation.<br />

Example: 0.06 × 10 5 is not written in scientific notation because the first factor (0.06) is not greater than or<br />

equal to 1. The correct way to write this number in scientific notation is 6.0×10 3 .<br />

a. 2.004 × 1 11<br />

b. 56 × 10 –4<br />

c. 2 × 100 2<br />

d. 10 × 10 –6<br />

Scientific Notation Standard Form<br />

d. 999.0<br />

e. 264,000<br />

f. 761,000,000<br />

g. 7.13 × 10 7<br />

h. 0.00320<br />

i. 0.000040<br />

j. 1.2 × 10 –12<br />

k. 42,000,000,000,000<br />

l. 12,004,000,000<br />

m. 9.906 × 10 –2<br />

28.1


Page 3 of 3<br />

3. Write the numbers in the following statements in scientific notation:<br />

a. The national debt in 2005 was about $7,935,000,000,000.<br />

b. In 2005, the U.S. population was about 297,000,000<br />

c. Earth's crust contains approximately 120 trillion (120,000,000,000,000) metric tons of gold.<br />

d. The mass of an electron is 0.000 000 000 000 000 000 000 000 000 000 91 kilograms.<br />

e. The usual growth of hair is 0.00033 meters per day.<br />

f. The population of Iraq in 2005 was approximately 26,000,000.<br />

g. The population of California in 2005 was approximately 33,900,000.<br />

h. The approximate area of California is 164,000 square miles.<br />

i. The approximate area of Iraq in 2005 was 169,000 square miles.<br />

j. In 2005, one right-fielder made a salary of $12,500,000 playing professional baseball.<br />

28.1


Name: Date:<br />

28.1 Understanding Light Years<br />

28.1<br />

How far is it from Los Angeles to New York? Pretty far, but it can still be measured in miles or<br />

kilometers. How far is it from Earth to the Sun? It’s about one hundred forty-nine million, six hundred thousand<br />

kilometers (149,600,000, or 1.496 × 10 8 km). Because this number is so large, and many other distances in space<br />

are even larger, scientists developed bigger units in order to measure them. An Astronomical Unit (AU) is<br />

1.496 × 10 8 km (the distance from Earth to the sun). This unit is usually used to measure distances within our<br />

solar system. To measure longer distances (like the distance between Earth and stars and other galaxies), the light<br />

year (ly) is used. A light year is the distance that light travels through space in one year, or 9.468 × 10 12 km.<br />

1. Converting light years (ly) to kilometers (km)<br />

Earth’s closest star (Proxima Centauri) is about 4.22 light years away. How far is this in kilometers?<br />

Explanation/Answer: Multiply the number of kilometers in one light year (9.468 × 10 12 km/ly) by the<br />

number of light years given (in this case, 4.22 ly).<br />

2. Converting kilometers to light years<br />

Polaris (the North Star) is about 4.07124 × 10 15 km from the earth. How far is this in light years?<br />

Explanation/Answer: Divide the number of kilometers (in this case, 4.07124 × 10 15 km) by the number of<br />

kilometers in one light year (9.468 × 10 12 km/ly).<br />

Convert each number of light years to kilometers.<br />

1. 6 light years<br />

2. 4.5 × 10 6 light years<br />

3. 4 × 10 –fio3 light years<br />

Convert each number of kilometers to light years.<br />

4. 5.06 × 10 16 km<br />

5. 11 km<br />

4.07124 10 15 × km<br />

6. 11,003,000,000,000 km<br />

9.468 10 12<br />

(<br />

--------------------------------------------<br />

× ) km<br />

× 4.22 ly 3.995 10<br />

1 ly<br />

13 ≈ × km<br />

9.468 10 12 × km 4.07124 10<br />

÷ ---------------------------------------<br />

1 ly<br />

15 × km 1 ly<br />

---------------------------------------------<br />

1 9.468 10 12 = × --------------------------------------- ≈<br />

430 light years<br />

× km


Page 2 of 2<br />

Solve each problem using what you have learned.<br />

7. The second brightest star in the sky (after Sirius) is Canopus. This yellow-white supergiant is about<br />

1.13616 × 10 16 kilometers away. How far away is it in light years?<br />

8. Regulus (one of the stars in the constellation Leo the Lion) is about 350 times brighter than the sun. It is 85<br />

light years away from the earth. How far is this in kilometers?<br />

9. The distance from earth to Pluto is about 28.61 AU from the earth. Remember that an AU = 1.496 × 108 km.<br />

How many kilometers is it from Pluto to the earth?<br />

10. If you were to travel in a straight line from Los Angeles to New York City, you would travel 3,940<br />

kilometers. How far is this in AU’s?<br />

11. Challenge: How many AU’s are equivalent to one light year?<br />

28.1


Name: Date:<br />

28.1 Parsecs<br />

You have already learned about two units of measurement commonly used in astronomy:<br />

• The astronomical unit (AU), which is the average distance between Earth and the Sun: 1.46 × 108 km<br />

• The light year (ly), which is the distance light travels through space in one year: 9.468 × 1012 km<br />

Chapter 28 of your text introduces a third unit of distance, the parsec (pc). The word parsec stands for “parallax<br />

of one arcsecond” which refers to the geometric method used by astronomers to figure out distances between<br />

objects in space. For our purposes, we will define the parsec as equal to 3.26 light years, or 206,265 astronomical<br />

units. This means that you would have to make 206,265 trips from Earth to the Sun (or 103,132.5 round trips) in<br />

order to travel 1 parsec!<br />

1 parsec = 3.26 light years or 206,265 astronomical units<br />

In this skill sheet, you will practice converting between parsecs, light years, astronomical units, and kilometers.<br />

1. Converting light years (ly) to parsecs (pc)<br />

Earth’s closest star (Proxima Centauri) is about 4.22 light years away. How far is this in parsecs?<br />

Explanation/Answer: Divide the number of light years given (in this case, 4.22 ly) by the number of light<br />

years in one parsec (3.26 ly).<br />

3.26 ly 1 pc<br />

4.22 ly ÷ = 4.22 ly× = 1.29 ly<br />

1 pc 3.26 ly<br />

2. Converting astronomical units (AU) to parsecs (pc)<br />

Polaris (the North Star) is about 2.789 × 107 astronomical units from Earth. How far is this in parsecs?<br />

Explanation/Answer: Divide the number of astronomical units (in this case, 2.789 × 10 7 AU) by the<br />

number of astronomical units in one parsec (206,265 AU).<br />

7 206,265 AU 7 1 pc<br />

2<br />

2.789× 10 AU ÷ = 2.789× 10 AU × = 1.352× 10 or 135.2 pc<br />

5<br />

1 pc 2.06265× 10 AU<br />

28.1


Page 2 of 2<br />

Convert each number of light years to parsecs.<br />

1. 6.00 light years<br />

2. 4.50 × 10 6 light years<br />

3. 4.00 × 10 -3 light years<br />

Convert each number of astronomical units to parsecs.<br />

4. 5.25 × 10 6 AU<br />

5. 100. AU<br />

6. 11,300,000 AU<br />

Solve each problem using what you have learned.<br />

7. The Milky Way galaxy is about 100,000 light years across. How large is this in parsecs?<br />

8. The Andromeda galaxy is approximately 2,500,000 light years from Earth. How far is this in parsecs?<br />

9. Regulus (one of the stars in the constellation Leo the Lion) is about 85 light years from Earth. How far is this<br />

in parsecs?<br />

10. The average distance from the Sun to Pluto is approximately 29.6 astronomical units. How far is this in<br />

parsecs?<br />

28.1


Page 1 of 2<br />

28.1 Edwin Hubble<br />

Edwin Hubble was an accomplished academic that many astronomers credit with<br />

“discovering the universe.”<br />

A good student and even better athlete<br />

Edwin Hubble was born on<br />

November 29, 1889, in<br />

Marshfield, Missouri. His<br />

family moved to Chicago<br />

when he was ten years old.<br />

Hubble was an active,<br />

imaginative boy. He was an<br />

avid reader of science<br />

fiction. Jules Verne’s<br />

adventure novels were<br />

among his favorite stories.<br />

Science fascinated Hubble, and he loved the way<br />

Verne wove futuristic inventions and scientific<br />

content into stories that took the reader on voyages to<br />

some strange and exotic destinations.<br />

Hubble was a very good student and also an excellent<br />

athlete. In 1906 he set an Illinois state record for the<br />

high jump, and in that same season he took seven first<br />

place medals and one third place medal in a single<br />

high school track meet.<br />

Focus turns to academics<br />

Hubble continued his athletic success by participating<br />

in basketball and boxing at the University of Chicago.<br />

Eventually though, his studies became his primary<br />

focus. Hubble graduated with a bachelors degree in<br />

Mathematics and Astronomy in 1910.<br />

Hubble was selected as a Rhodes Scholar and spent<br />

the next three years at the University of Oxford, in<br />

England. Instead of continuing his studies in math and<br />

science, he decided to pursue a law degree. He<br />

completed the degree in 1913 and returned to the<br />

United States. He set up a law practice in Louisville,<br />

Kentucky. However, it was a short lived law career.<br />

Returning to Astronomy<br />

It took Hubble less than a year to become bored with<br />

his law practice, and he returned to the University of<br />

Chicago to study astronomy. He did much of his work<br />

at the Yerkes Observatory, and received his Ph.D. in<br />

astronomy in 1917.<br />

Hubble joined the army at this time and served a tour<br />

of duty in World War I. He attained the rank of Major.<br />

When he returned in 1919, he was offered a job by<br />

George Ellery Hale, the founder and director of<br />

Carnegie Institution's Mount Wilson Observatory,<br />

near Pasadena, California.<br />

28.1<br />

The best tool for the job<br />

The timing could not have been better. The 100-inch<br />

Hooker telescope, the world’s most powerful<br />

telescope at the time, had just been constructed. This<br />

telescope could easily focus images that were fuzzy,<br />

too dim, or too small to be seen clearly through other<br />

large telescopes.<br />

The Hooker telescope enabled Hubble to make some<br />

astounding discoveries. Astronomers had believed that<br />

the many large fuzzy patches they saw through their<br />

powerful telescopes were huge gas clouds within our<br />

own Milky Way galaxy. They called these fuzzy<br />

patches “nebulae,” a Greek word meaning “cloud.”<br />

Hubble’s observations in 1923 and 1924 proved that<br />

while a few of these fuzzy objects were inside our<br />

galaxy, most were in fact entire galaxies themselves,<br />

not only separate from the Milky Way but millions of<br />

light years away. This greatly enlarged the accepted<br />

size of the universe, which many scientists at the time<br />

believed was limited to the Milky Way alone.<br />

Another landmark discovery<br />

Hubble also used spectroscopy to study galaxies. He<br />

observed that galaxies’ spectral lines were shifting<br />

toward the red end of the spectrum, which meant they<br />

were moving away from each other. He showed that<br />

the farther away a galaxy was, the faster it was moving<br />

away from Earth. In 1929, Hubble and fellow<br />

astronomer Milton Humason announced that all<br />

observed galaxies are moving away from each other<br />

with a speed proportional to the distance between<br />

them. This became known as Hubble’s Law, and it<br />

proved that the universe was expanding.<br />

Albert Einstein visited Hubble and personally thanked<br />

him for this discovery, as it matched with Einstein’s<br />

calculations, providing observable evidence<br />

confirming his predictions.<br />

Hubble worked at the Wilson Observatory until his death<br />

in 1953. He is considered the father of modern<br />

cosmology. To honor him, scientists have named a space<br />

telescope, a crater on the moon, and an asteroid after him.


Page 2 of 2<br />

Reading reflection<br />

1. Look up the definition of each boldface word in the article. Write down the definitions and be sure<br />

to credit your source.<br />

2. Research: What is a Rhodes Scholarship?<br />

3. Research: Why does a larger telescope allow astronomers to see more?<br />

4. Imagine you knew Edwin Hubble. Describe how you think he may have felt when Albert Einstein came to<br />

visit and thank him for his discoveries.<br />

5. Research: Before Hubble’s discovery, people thought that the universe had always been about the same<br />

size. How did Hubble’s discovery that the universe is currently expanding change scientific thought about<br />

the size of the universe in the past?<br />

28.1


Name: Date:<br />

28.2 Light Intensity Problems<br />

Light is a form of energy. Light intensity describes the amount of energy per second falling on a surface, using<br />

units of watts per meter squared (W/m 2 ). Light intensity follows an inverse square law. This means that the<br />

intensity decreases as the square of the distance from the source. For example, if you double the distance from<br />

the source, the light intensity is one-fourth its original value. If you triple the distance, the light intensity is oneninth<br />

its original value.<br />

Most light sources distribute their light equally in all directions,<br />

producing a spherical pattern.The area of a sphere is 4πr 2 , where r is the<br />

radius or the distance from the light source. For a light source, the<br />

intensity is the power per area. The light intensity equation is:<br />

I<br />

= --<br />

P<br />

=<br />

A<br />

P<br />

4πr 2<br />

-----------<br />

Remember that the power in this equation is the amount of light emitted<br />

by the light source. When you think of a “100 watt” light bulb, the<br />

number of watts represents how much energy the light bulb uses, not how much light it emits. Most of the energy<br />

in an incandescent light bulb is emitted as heat, not light. That 100-watt light bulb may emit less than 1 watt of<br />

light energy with the rest being lost as heat.<br />

Solve the following problems using the intensity equation. The first problem is done for you.<br />

1. For a light source of 60 watts, what is the intensity of light 1 meter away from the source?<br />

P P<br />

I = = 2 A 4πr 60 W<br />

2<br />

= =<br />

4.8 W/m<br />

2<br />

4 π(1m)<br />

2. For a light source of 60 watts, what is the intensity of light 10 meters away from the source?<br />

3. For a light source of 60 watts, what is the intensity of light 20 meters away from the source?<br />

4. If the distance from a light source doubles, how does light intensity change?<br />

5. Answer the following problems for a distance of 4 meters from the different light sources.<br />

a. What is the intensity of light 4 meters away from a 1-watt light source?<br />

b. What is the intensity of light 4 meters away from a 10-watt light source?<br />

c. What is the intensity of light 4 meters away from a 100-watt light source?<br />

d. What is the intensity of light 4 meters away from a 1,000-watt light source?<br />

6. What is the relationship between the watts of a light source and light intensity?<br />

28.2


Page 1 of 2<br />

28.2 Henrietta Leavitt<br />

Leavitt, although deaf, had a keen eye for observing the stars. Her ability to identify the magnitude<br />

of stars set the standard for determining a star's distance—hundreds or even millions of light years away.<br />

Star struck<br />

Henrietta Swan Leavitt was<br />

born on July 4, 1868 in<br />

Lancaster, Massachusetts.<br />

Henrietta's family lived in<br />

Cambridge, Massachusetts<br />

and later moved to<br />

Cleveland, Ohio. In Ohio,<br />

Leavitt attended Oberlin<br />

College for two years and<br />

was enrolled in the school's<br />

conservatory of music. She<br />

then moved back to<br />

Cambridge and attended Radcliffe College, which was<br />

then known as the Society for the Collegiate<br />

Instruction of Women.<br />

In her last year of college, Henrietta took an<br />

astronomy course—then began her fascination with<br />

and love for the stars. She graduated in 1892 and later<br />

became a volunteer research assistant working at the<br />

Harvard College Observatory.<br />

Computing the stars<br />

In the 1880s, Harvard College established a goal to<br />

catalog the stars. Edward Pickering, a former<br />

professor at the Massachusetts Institute of<br />

Technology, became director of the Harvard<br />

Observatory. Pickering was an authority in<br />

photographic photometry—determining a star's<br />

magnitude from a photograph. He wanted to gather<br />

information about the brightness and color of stars.<br />

In order to complete this work, Pickering needed<br />

people to perform the tedious task of examining<br />

photographs of stars. Men typically did not perform<br />

this type of work, but women were hired and known as<br />

“computers.” Leavitt was hired at a rate of $.30 per<br />

hour to complete this painstakingly detailed work.<br />

Leavitt was not a healthy woman, struggling with<br />

complete hearing loss and other illnesses during<br />

college and throughout her career. Despite these<br />

setbacks, she became a super computer, devoting her<br />

life to studying the stars. She eventually became the<br />

head of the photographic stellar photometry<br />

department.<br />

28.2<br />

Leavitt catalogued variable stars that altered in<br />

brightness over the course of a few days, weeks, or<br />

even months. She studied Cepheid variable stars in the<br />

Magellanic Clouds, two galaxies near the Milky Way.<br />

Leavitt examined photographic plates, comparing the<br />

same regions on several plates taken at different times.<br />

Stars that had changed in brightness would look<br />

different in size. Leavitt continued to examine plates,<br />

discovering nearly 2,000 new variable stars in the<br />

Magellenic Clouds.<br />

Leavitt found an inverse relationship between a star's<br />

brightness cycle and its magnitude. A stronger star<br />

took longer to cycle between brightness and dimness.<br />

Therefore, brighter Cepheid stars took longer to rotate<br />

between brightness and dimness. In 1912, Leavitt had<br />

established the Period-Luminosity relation.<br />

These stars, all located within the Magellenic Clouds,<br />

were roughly the same distance from the Earth. This<br />

rule provided astronomers with the ability to measure<br />

distances within and beyond our galaxy.<br />

Astronomical findings<br />

Leavitt's discovery had a tremendous impact on future<br />

research in the field of astronomy. Astronomers could<br />

now determine distances to galaxies and within the<br />

universe overall. Ejnar Hertzsprung was able to plot<br />

the distance of Cepheid stars. Harlow Shapley, using<br />

Leavitt's findings, was able to map the Milky Way and<br />

determine its size. Edwin Hubble applied her rule to<br />

establish the age of the Universe.<br />

An unsung heroine<br />

In 1925, the Swedish Academy of Sciences wished to<br />

nominate Leavitt for a Nobel Prize. However, she had<br />

died of cancer nearly four years earlier at the age of<br />

53. The Nobel Prize must be given during a recipient’s<br />

lifetime.<br />

In addition to her discovery of numerous variable<br />

stars, Leavitt discovered four novas and developed the<br />

standard method for determining the magnitude of<br />

stars. An asteroid and crater on the moon are named in<br />

her honor.


Page 2 of 2<br />

Reading reflection<br />

1. How did Henrietta Leavitt discover new variable stars?<br />

2. What were Leavitt's two significant contributions to the field of astronomy?<br />

3. Research: What is the name of the asteroid named in Leavitt's honor?<br />

4. Research: When was the Harvard Observatory established and what does It do now?<br />

5. Research: State three facts about each of these astronomers: Ejnar Hertzsprung, Harlow Shapley, and Edwin<br />

Hubble.<br />

6. Research: What is the Nobel Prize?<br />

28.2


Name: Date:<br />

28.2 Calculating Luminosity<br />

You have learned that in order to understand stars, astronomers want to know their luminosity. Luminosity<br />

describes how much light is coming from the star each second. Luminosity can be measured in watts (W).<br />

Measuring the luminosity of something as far away as a star is difficult to do. However, we can measure its<br />

brightness. Brightness describes the amount of the star’s light that reaches a square meter of Earth each second.<br />

Brightness is measured in watts/square meter (W/m 2 ).<br />

The brightness of a star depends on its luminosity and its distance from Earth. A star, like a light bulb, radiates<br />

light in all directions. Imagine that you are standing one meter away from an ordinary 100-watt incandescent<br />

light bulb. These light bulbs are about ten percent efficient. That means only ten percent of the 100 watts of<br />

electrical power is used to produce light. The rest is wasted as heat. So the luminosity of the bulb is about ten<br />

percent of 100 watts, or around 10 watts.<br />

The brightness of this bulb is the same at all points one meter away from the bulb. All those points together form<br />

a sphere with a radius of one meter, surrounding the bulb.<br />

If you want to find the brightness of that bulb, you take the luminosity (10 watts) and divide it by the amount of<br />

surface area it has to cover—the surface area of the sphere. So, the formula for brightness is:<br />

brightness<br />

The brightness of the bulb at a distance of one meter is:<br />

10 watts<br />

4π(1 meter) 2<br />

------------------------------<br />

--------------------------------------------------luminosityluminosity<br />

surface area of sphere<br />

4π( radius)<br />

2<br />

=<br />

= ----------------------------<br />

10 watts<br />

12.6 meter 2<br />

--------------------------- 0.79 W/m 2<br />

=<br />

=<br />

Notice that the radius in the equation is the same as the distance from the bulb to the point at which we’re<br />

measuring brightness. If you were standing 10 meters away from the bulb, you would use 10 for the radius in the<br />

equation. The surface area of your sphere would be 4π(100) or 1, 256 square meters! The same 10 watts of light<br />

energy is now spread over a much larger surface. Each square meter receives just 0.008 watts of light energy. Can<br />

you see why distance has such a huge impact on brightness?<br />

28.2


Page 2 of 2<br />

If we know the brightness and the distance, we can calculate luminosity by rearranging the equation:<br />

luminosity brightness × surface area of sphere brightness 4π(distance) 2<br />

=<br />

= ×<br />

This is the same formula that astronomers use to calculate the luminosity of stars.<br />

You are standing 5 meters away from another incandescent light bulb. Using a light-meter, you measure its<br />

brightness at that distance to be 0.019 watts/meter 2 . Calculate the luminosity of the bulb. Assuming this bulb is<br />

also about ten percent efficient, estimate how much electric power it uses (this is the wattage printed on the bulb).<br />

Step 1: Plug the known values into the formula:<br />

0.019 watts<br />

luminosity<br />

meter 2<br />

--------------------------<br />

4π( 5 meters)<br />

2<br />

=<br />

× ----------------------------------<br />

1<br />

Step 2: Solve for luminosity:<br />

Step 3: If the bulb is only about ten percent efficient, the electric power used must be about ten times the<br />

luminosity. The bulb must use about 10 × 6 watts, or 60 watts, of electric power.<br />

1. Ten meters away from a flood lamp, you measure its brightness to be 0.024 W/m 2 . What is the luminosity of<br />

the flood lamp? What is the electrical power rating listed on the bulb, assuming it is ten percent efficient?<br />

2. You hold your light-meter a distance of one meter from the light bulb in your refrigerator. You measure the<br />

brightness to be 0.079 W/m 2 . What is the luminosity of this light bulb? What is its power rating, assuming it<br />

is ten percent efficient?<br />

3. Challenge: Finding the luminosity of the sun.<br />

You can use the same formula to calculate the luminosity of the sun.<br />

Astronomers have measured the average brightness of the sun at the top of Earth’s atmosphere to be 1,370 W/m 2 .<br />

This quantity is known as the solar constant.<br />

We also know that the distance from Earth to the sun is 150 billion meters (or 1.5 × 10 11 meters).<br />

What is the luminosity of the sun?<br />

luminosity =<br />

0.019 × 100π<br />

watts= 6 watts<br />

Hints:<br />

1. You may wish to rewrite the solar constant as 1.370 × 103 W/m2 .<br />

2. (10 11 ) 2 is the same as 1011 × 1011 . To find the product, add the exponents.<br />

3. Don’t forget to find the square of 1.5!<br />

28.2


Name: Date:<br />

28.3 Doppler Shift<br />

Doppler shift is an important tool used by astronomers to study the motion of objects, such as stars and galaxies,<br />

in space. For example, if an object is moving toward Earth, the light waves it emits are compressed, shifting them<br />

toward the blue end (shorter wavelengths, higher frequencies) of the visible spectrum. If an object is moving<br />

away from Earth, the light waves it emits are stretched, shifting them toward the red end (longer wavelengths,<br />

lower frequencies) of the visible spectrum. In this skill sheet, you will practice solving problems that involve<br />

doppler shift.<br />

Understanding Doppler Shift<br />

28.3<br />

You have learned that astronomers use a spectrometer to determine which<br />

elements are found in stars and other objects in space. When burned, each<br />

element on the periodic table produces a characteristic set of spectral lines.<br />

When an object in space is moving very fast, its spectral lines show the<br />

characteristic patterns for the elements it contains. However, these lines<br />

are shifted.<br />

If the object is moving away from Earth, its spectral lines are shifted<br />

toward the red end of the spectrum. If the object is moving toward Earth,<br />

its spectral lines are shifted toward the blue end of the spectrum.<br />

1. The graphic to the right shows two spectral lines from an object that is not<br />

moving. Use an arrow to indicate the direction that the spectrum would appear to<br />

shift if the object was moving toward you.<br />

2. The graphic to the right shows the spectral<br />

lines emitted by four moving objects. The<br />

spectral lines for when the object is<br />

stationary are shown as dotted lines on each<br />

spectrum. The faster a star is moving, the<br />

greater the shift in wavelength. Use the<br />

graphic to help you answer the following<br />

questions.<br />

a. Which of the spectra show an object<br />

that is moving toward you?<br />

b. Which of the spectra show an object<br />

that is moving away from you?<br />

c. Which of the spectra show an object that is moving the fastest away from you?<br />

d. Which of the spectra show an object that is moving the fastest toward you?


Page 2 of 2<br />

Solving Doppler Shift Problems<br />

28.3<br />

By analyzing the shift in wavelength, you can also determine the speed at which a star is moving. The<br />

faster a star is moving, the larger the shift in wavelength. The following proportion is used to help you calculate<br />

the speed of a moving star. Remember that the speed of light is a constant value equal to 3 × 10 8 m/s.<br />

The<br />

-------------------------------------------speed<br />

of a star<br />

The speed of light<br />

=<br />

-------------------------------------------------------------------------------------<br />

The difference in wavelength<br />

The stationary value for wavelength<br />

The spectral lines emitted by a distant galaxy are analyzed. One of the lines for hydrogen has shifted from 450<br />

nm to 498 nm. Is this galaxy moving away from or toward Earth? What is the speed of galaxy?<br />

The speed of a star<br />

3 10 8 --------------------------------------------<br />

× m/s<br />

=<br />

498 nm – 450 nm<br />

-----------------------------------------<br />

450 nm<br />

The speed of a star<br />

48 nm<br />

----------------- 3 10<br />

450 nm<br />

8 × × m/s 0.11 3 10 8 × × m/s 3.3 10 7 = = =<br />

× m/s<br />

The galaxy is moving away from Earth at a speed of 33 million meters per second.<br />

1. One of the spectral lines for a star has shifted from 535 nm to 545 nm. What is the speed of this star? Is the<br />

star moving away from or toward Earth?<br />

2. One of the spectral lines for a star has shifted from 560 nm to 544 nm. What is the speed of the star? Is it<br />

moving away from or toward Earth?<br />

3. An astronomer has determined that two galaxies are moving away from Earth. A spectral line for galaxy A is<br />

red shifted from 501 nm to 510 nm. The same line for galaxy B is red shifted from 525 nm to 540 nm. Which<br />

galaxy is moving the fastest? Justify your answer.<br />

4. Does the fact that both galaxies in the question above are moving away from Earth support or refute the Big<br />

Bang theory? Explain your answer.


Answer Keys<br />

<strong>Skill</strong> Sheet 1.1: Lab Safety<br />

Safety quiz answers:<br />

1. Answers will vary.<br />

2. Answers will vary.<br />

3. Answers will vary. Example: (1) Be quite and listen, (2)<br />

Locate your safety buddy, (3) In a safe and orderly manner,<br />

follow your teacher out of the lab to a designated safe<br />

location.<br />

4. In many cases, investigations require the use of chemicals<br />

that may cause harm to your eyes or clothing if these are not<br />

protected. Gloves are also important when working with<br />

chemicals. For investigations that require heat, using a hot<br />

pad is very important.<br />

5. Teamwork helps you to complete the lab efficiently, but<br />

keeping safe also requires teamwork. Sometimes, you may<br />

need someone to help you pour a chemical or perform a<br />

procedure that would be unsafe if you tried it by yourself. A<br />

team of people can also work together to keep everyone on<br />

the team safe. On your own, it is more difficult to be aware of<br />

all the possible dangers in a laboratory setting.<br />

6. Cleaning up after an investigation prepares the work space for<br />

the next day’s investigation. A clean work space is safer<br />

because all chemicals and any sharp or dangerous objects are<br />

removed. Clean up also involves turning off any appliances<br />

that could heat up and cause a fire.<br />

7. (1) Immediately tell my teacher, (2) Listen carefully and<br />

follow any safety instructions provided by the teacher, and (3)<br />

Follow the appropriate safety guidelines.<br />

8. Answers are:<br />

a. First, I would make sure that my classmates know to stay<br />

away from the broken beaker and I would tell my teacher<br />

what had happened. Then, I would clean up the glass with<br />

<strong>Skill</strong> Sheet 1.1: Using Your Text<strong>book</strong><br />

Part 1 answers:<br />

1. Green<br />

2. Correct answers include any four of the following vocabulary<br />

words:<br />

relief, elevation, sea level, topographic map, contour lines,<br />

slope<br />

3. blue<br />

4. Hypotheses must be testable to be scientific.<br />

5. Section reviews are found at the end of each section of each<br />

chapter.<br />

6. Why does chocolate melt in your hand?<br />

7. Answers are:<br />

1. Looking for:<br />

2. Given<br />

3. Relationships<br />

4. Solution<br />

8. The four parts are vocabulary, concepts, problems, and<br />

applying your knowledge.<br />

<strong>Skill</strong> Sheet 1.1: SI Units<br />

1. 10 g<br />

2. 1000 mm<br />

3. 600 mm<br />

4. 420 g<br />

5. 5 L<br />

6. 0.1 m<br />

Page 1 of 57<br />

a dust pan and a brush. I would not use my hands to clean<br />

up the glass. I would place the broken glass in a cardboard<br />

box, seal the box, and label it “sharps.”<br />

b. I would make sure my classmates know about the water so<br />

they don’t slip. I would tell my teacher as soon as<br />

possible. I would begin placing paper towels on the wet<br />

spot as soon as possible. Carefully, I would work with my<br />

classmates to clean up the spill. It would be best to use<br />

gloves during the clean up in case any chemicals are<br />

mixed in with the water.<br />

c. I would tell my teacher about the smell and follow any<br />

safety instructions given to me by the teacher. I would<br />

help to make sure that the lab is well ventilated (by<br />

helping to open windows and doors, for example). I would<br />

ask to leave the lab to get some fresh air if I needed to do<br />

so.<br />

d. I would stop talking and listen to any safety instructions<br />

from my teacher. I would follow the classroom plan for<br />

exiting the lab as soon as possible. I would not worry<br />

about removing my lab apron. I may remove my goggles<br />

if it seems unsafe to keep them on.<br />

e. I would take her hand and lead her to the eye wash station<br />

as soon as possible. I would tell a classmate to tell our<br />

teacher as soon as possible. When the teacher arrives, I<br />

would let her/him help my lab partner.<br />

f. I would stop talking and listen to any safety instructions<br />

from my teacher. I would follow the classroom plan for<br />

exiting the lab as soon as possible. I may need to use the<br />

nearest classroom fire extinguisher if my teacher is unable<br />

to do so.<br />

Part 2 answers:<br />

1. There are eight units:<br />

Science <strong>Skill</strong>s<br />

Motion, Force, and Energy<br />

Matter, Energy, and Earth<br />

Matter and Its Changes<br />

Electricity and Magnetism<br />

Earth’s Structure<br />

Waves<br />

Matter and Motion in the Universe<br />

2. Answers will vary according to student interest.<br />

3. The glossary and index are found at the back of the <strong>book</strong>,<br />

after Unit 8. The glossary contains definitions; the index tells<br />

where to find information on specific topics.<br />

Part 3 answers:<br />

1. Velocity is a variable that tells you both speed and direction.<br />

2. Pages 250 and 251<br />

3. Page 347<br />

7. 1,500,000 mg<br />

8. 300 L<br />

9. 6,500,000 cm<br />

10. 120,000 mg<br />

11. 7.2 L<br />

12. 5.3 kL


13. A decimeter is 100 times larger than a millimeter.<br />

14. A dekagram is 1000 times larger than a centigram<br />

<strong>Skill</strong> Sheet 1.1: Scientific Notation<br />

1. Answers are:<br />

a. 122,200<br />

b. 90,100,000<br />

c. 3,600<br />

d. 700.3<br />

e. 52,722<br />

<strong>Skill</strong> Sheet 1.2: Measuring Length<br />

Stop and Think:<br />

f. 10 millimeters = 1 centimeter<br />

g. It is better to measure with the English system when you<br />

want to buy fabric for making curtains. Fabric is sold in<br />

yards and inches in the U.S. It is better to measure with<br />

the metric system when you are describing the length of<br />

something to your pen pal in Germany.<br />

Example 1:<br />

1. 63.0 mm<br />

2. 6.30 cm<br />

3. 0.063 m<br />

Example 2:<br />

1. 189.0 mm<br />

2. 3 blocks<br />

3. 7.44 inches<br />

Example 3:<br />

1. 42.5 mm<br />

2. 7 dominoes<br />

<strong>Skill</strong> Sheet 1.2: Averaging<br />

1. Four gloves per household<br />

2. On average, each person spent about $8.33.<br />

3. $3.95; $31.60<br />

<strong>Skill</strong> Sheet 1.2: SQ3R Reading and Study Method<br />

No student responses are required.<br />

<strong>Skill</strong> Sheet 1.2: Stopwatch Math<br />

1. Answers are:<br />

a. 5, 5.05, 5.15, 5.2, 5.5<br />

b. 6:06, 6:06.004, 6:06.04, 6:06.4<br />

2. Answers are:<br />

Time 9.88w 9.88w 9.91 9.95w<br />

Year 2002 1998 2004 2001<br />

Time 9.97w 10.01 10.08 10.11<br />

Year 1999 2000 2005 2003<br />

<strong>Skill</strong> Sheet 1.2: Understanding Light Years<br />

1. 5.7 × 10 13 km<br />

2. 4.3 × 10 19 km<br />

3. 3.8 × 10 10 km<br />

4. 5,344 ly<br />

5. 1.16 × 10 –12 ly<br />

6. 1.16 ly<br />

Page 2 of 57<br />

15. A millimeter is 10 times smaller than a centimeter.<br />

2. Answers are:<br />

a. 4.051 × 10 6<br />

b. 1.3 × 10 9<br />

c. 1.003 × 10 6<br />

d. 1.602 × 10 4<br />

e. 9.9999 × 10 12<br />

Practice with converting units for length:<br />

Prefix Your multiplication factor Your domino length in:<br />

pico- 10 -9<br />

42.5 × 10 9 pm<br />

nano- 10 -6 42.5 × 10 6 nm<br />

micro- 10-3 42.5 × 103 µm<br />

milli 10 0<br />

42.5 × 10 0 mm<br />

centi- 10 1 42.5 × 10 –1 cm<br />

deci- 10242.5 × 10 –2 dm<br />

deka- 10 4<br />

42.5 × 10 –4 dam<br />

hecto- 10 5 42.5 × 10 –5 hm<br />

kilo- 10 6 42.5 × 10 –6 km<br />

4. ≈ 6 points each<br />

5. ≈ 5 slices each (5 1 / 3 slices each)<br />

3. Answers are:<br />

a. 1:22.04, 1:22.4, 1:23.117, 1:23.2, 1:24, 1:24.007, 1:25,<br />

1:33<br />

b. 1:17.99, 1:18.22, 1:18.3, 1:20, 1:20.22, 1:21.003, 1:21.2<br />

c. 1:24.099, 1:24.9899, 1:24.99, 1:24.9901, 1:25, 1:25.001<br />

4. Infinitely many solutions possible.<br />

Example: 26:15.21, 26:15.215, 26:15.22, 26:15.225, 26:15.23<br />

7. 1,200 ly<br />

8. 8.0478 × 10 14 km<br />

9. 4,280,056,000 km, or 4.28 × 10 9 km<br />

10. 0.000026336 AU, or 2.6 × 10 –5 AU<br />

11. 63,288 AU


<strong>Skill</strong> Sheet 1.2: Indirect Measurement<br />

1. The tree is 3 meters tall.<br />

2. The flagpole’s shadow is 12.8 meters.<br />

3. Answers are:<br />

a. 2,200 feet<br />

b. 670 meters<br />

4. The average mass is 0.1 kilogram or 100 grams.<br />

5. Each staple is 0.0324 gram or 3.24 milligrams.<br />

6. One business card is 0.034 centimeter thick.<br />

7. The thickness of a CD is approximately 0.13 centimeter or<br />

1.3 millimeters.<br />

8. Answers are:<br />

a. 4.8 millimeters<br />

b. 9.6 millimeters<br />

c. 48 millimeters<br />

d. 9,600 millimeters or 9.6 meters<br />

9. Answers are:<br />

a. Each cheesecake takes 0.85 hour or 51 minutes to make.<br />

b. Yvonne earns $12 per cheesecake.<br />

c. Yvonne earns $14.12 per hour.<br />

<strong>Skill</strong> Sheet 1.3: Dimensional Analysis<br />

1. Answers are:<br />

a. $72/day<br />

b. 210 lbs/week<br />

2. Answers are:<br />

a. 2.75 gal<br />

b. 2.20 m<br />

c. ≈ 0.0947 mile<br />

d. 64 cups<br />

<strong>Skill</strong> Sheet 1.3: Fractions Review<br />

Part 1 answers:<br />

1. 13 / 12<br />

2. 89 / 56<br />

3. 19 / 6<br />

4. 1.1, 1.6, 3.2<br />

Part 2 answers:<br />

1. –5 / 12<br />

2. 9 / 56<br />

3. –5 / 3<br />

4. –0.42, 0.16, –1.67<br />

Part 3 answers:<br />

1. 1 / 4<br />

2. 5 / 8<br />

Page 3 of 57<br />

10. The mass of the block of marble is 324,000 grams or<br />

324 kilograms.<br />

11. Sample answer:<br />

First fill the dropper with water from the glass.<br />

Then place drops of water one-by-one into the graduated<br />

cylinder. Count the number of drops it takes to reach the<br />

5.0 mL mark on the graduated cylinder.<br />

To find the volume of one drop, divide the value 5.0 mL by<br />

the number of drops.<br />

12. Sample answer:<br />

(1) Remove the newspaper from the recycling bin.<br />

(2) Unfold each sheet and smooth the paper.<br />

(3) Neatly stack the sheets of paper.<br />

(4) Place the newspaper on a flat surface.<br />

(5) Place something heavy, like a hardbound <strong>book</strong>, on the<br />

newspaper to remove excess space between the sheets of paper.<br />

(6) Measure the height of the stack of newspaper.<br />

(7) Divide the stack of paper by the number of sheets.<br />

Note to teacher: This question is designed to prompt students to<br />

think about sources of experimental error. You may wish to ask<br />

the students what would happen if they divided the height of the<br />

recycle bin by the number of sheets of newsprint multiplied by<br />

four. Why wouldn’t this method yield an accurate result?<br />

3. Answers are:<br />

a. 126,144,000 s<br />

b. ≈ 71.0 ft<br />

c. 4.5 qt<br />

d. 14 2 / 3 fields<br />

e. 48. km/gal<br />

f. ≈ 13 km/l<br />

g. 95 ft/s<br />

3. 15 / 16<br />

4. 0.25, 0.63, 0.94<br />

Part 4 answers:<br />

1. 4 / 9<br />

2. 49 / 40<br />

3. 2<br />

4. 2 / 3<br />

5. 0.44, 1.23, 2, 0.67<br />

Part 5 answers:<br />

1. 1<br />

2. 27 / 70<br />

3. 1<br />

4. 5 / 6<br />

5. 1, 0.39, 1, 0.83


<strong>Skill</strong> Sheet 1.3: Significant Digits<br />

Part 1 answers:<br />

a. 4<br />

b. 1<br />

c. 4<br />

d. 2<br />

e. 2<br />

f. 3<br />

g. infinitely many (# of students is counted)<br />

<strong>Skill</strong> Sheet 1.3: Study Notes<br />

This skill sheet provides a note-taking grid for students. It can be<br />

used with reading assignments throughout the school year.<br />

<strong>Skill</strong> Sheet 1.3: Science Vocabulary<br />

Prefix is in bold and suffix is underlined:<br />

thermometer electrolyte monoatomic<br />

volumetric endothermic spectroscope<br />

prototype convex supersaturated<br />

Student definitions:<br />

Answers may vary. Correct answers include:<br />

1. The study of water<br />

2. Many units<br />

3. The same kind<br />

4. Different kinds<br />

5. Existing light<br />

6. An instrument for measuring the full range of something<br />

Dictionary definitions:<br />

1. The science dealing with the properties, distribution, and<br />

circulation of water<br />

2. A chemical compound formed by the union of small<br />

molecules, usually consisting of repeating units<br />

3. Of the same kind, having uniform structure<br />

<strong>Skill</strong> Sheet 1.3: SI Units Extra Practice<br />

1. 12,756,000 m<br />

2. 347,600,000 cm<br />

3. 384,000 km<br />

4. 200,000,000 m<br />

5. 16,000,000 cm<br />

6. 3,600,000 mm<br />

7. 125,000 m long, 400 m deep, 1,500 m wide<br />

8. 11.18 km/sec<br />

9. 5,400,000 g<br />

10. 2 g<br />

<strong>Skill</strong> Sheet 1.3: SI-English Conversions<br />

1. ≈ 4.3 mi<br />

2. ≈ 4.11 oz<br />

3. ≈ 896 kg<br />

4. ≈ 2.1 qt<br />

5. ≈ 2,400 g<br />

6. ≈ 33 mi<br />

7. 2830 in; 78.7 yd<br />

8. ≈ 1.74 mi<br />

9. ≈ 3.77 L<br />

10. ≈ 0.379 lb<br />

Page 4 of 57<br />

Part 2 answers:<br />

1. 34,000 cm 2<br />

2. 0.9 liters<br />

3. 12.8 m 2<br />

4. 24.2°C<br />

5. 40:32<br />

6. Answers will vary.<br />

4. Consisting of dissimilar ingredients<br />

5. The emission of light (as by a chemical or physiological<br />

process)<br />

6. An instrument for measuring spectra<br />

Definitions based on prefixes and suffixes:<br />

1. thermometer<br />

2. sonogram (or sonograph)<br />

3. monoatomic<br />

4. telescope<br />

Word Dictionary Definition<br />

thermometer An instrument for measuring temperature<br />

sonogram A graph that shows the loudness of sound at<br />

different frequencies<br />

monoatomic Containing only one type of atom<br />

telescope A cylindrical instrument for viewing distant<br />

objects<br />

11. 1,200 mg to 2,700 mg<br />

12. 158,000 kg<br />

13. 450,000,000 mg<br />

14. 23,000 g to 90,000 g<br />

15. 40,000 mL<br />

16. 1,000 mL<br />

17. 26,600 kL<br />

18. 1,558,000 L<br />

19. 60 mL<br />

20. 0.947 L


<strong>Skill</strong> Sheet 1.4: Creating Scatterplots<br />

1. Answers are:<br />

Data pair<br />

not necessarily<br />

in order<br />

Temp. Hours of<br />

heating<br />

Stopping<br />

distance<br />

Number of<br />

people<br />

in family<br />

Stream<br />

flow<br />

Tree<br />

age<br />

2. Answers are:<br />

3. Answers are:<br />

Hours of<br />

heating<br />

Page 5 of 57<br />

a. Table answers:<br />

b. 60 minutes<br />

c. 27.0 kilometers<br />

d. Adjusted scale for the x-axis: 3 per line or 5 per line;<br />

adjusted scale for the y-axis: 1.5 per line or 2 per line<br />

e. & f.<br />

Graph of position (km) vs. time (min):<br />

Time (min)<br />

g. After 45 minutes, the position would be about<br />

25.25 kilometers.<br />

<strong>Skill</strong> Sheet 1.4: What’s the Scale?<br />

1. Answers are: 2. The range is 30 and the scale is 1 per line.<br />

Range<br />

from 0<br />

14<br />

# of<br />

Lines<br />

10<br />

Range ≥ # of Lines<br />

14 ≥ 10 = ≥<br />

Calculated<br />

scale<br />

1.4<br />

Adj. scale<br />

(whole #)<br />

2<br />

3. The range is 25 and the scale is 3 per line.<br />

4. Answers are:<br />

Independent variable: Days; Dependent variable: Average<br />

Temperature (°F)<br />

8<br />

1000<br />

5<br />

20<br />

8 ≥ 5 =≥<br />

1000 ≥ 20 = ≥<br />

1.6<br />

50<br />

2<br />

50<br />

Range for x-axis = 11; Range for y-axis = 73<br />

Scale for x-axis = 1 day/box; Scale for y-axis = 4 °F/box<br />

547 15 547 ≥ 15 = ≥ 36.5 37<br />

99 30 99 ≥ 30 = ≥ 3.3 4<br />

35 12 35 ≥ 12 = ≥ 2.9 3<br />

<strong>Skill</strong> Sheet 1.4: Interpreting Graphs<br />

Independent Dependent<br />

Temp.<br />

Speed of a car Speed of a car Stopping<br />

distance<br />

Cost<br />

per week<br />

for groceries<br />

Number of<br />

people<br />

in family<br />

Cost<br />

per week<br />

for groceries<br />

Rainfall Amount of rainfall Rate of<br />

stream flow<br />

Average tree<br />

height<br />

Test score Number of<br />

hours studying<br />

for a test<br />

Population of<br />

a city<br />

Range Number<br />

of lines<br />

Number of<br />

schools needed<br />

Tree age Average tree<br />

height<br />

Number of hours Test score<br />

studying<br />

Population of a city Number of<br />

schools needed<br />

Range ≥ No.<br />

of lines<br />

Calculated<br />

scale<br />

(per line)<br />

Adj.<br />

scale<br />

(per line)<br />

13 24 13 ≥ 24 0.54 1<br />

83 43 83 ≥ 43 1.9 2<br />

31 35 31 ≥ 35 0.88 1<br />

100 33 100 ≥ 33 3.03 5<br />

300 20 300 ≥ 20 15 15<br />

900 15 900 ≥ 15 60 60<br />

Practice set 1: Scatterplot<br />

1. Graph title: “Money in cash box vs. hours washing cars.”<br />

2. The two variables are number of hours washing cars and the<br />

amount of money in the cash box.<br />

3. Hours<br />

Independent variable Dependent variable<br />

0 5.0<br />

10 9.5<br />

20 14.0<br />

30 18.5<br />

40 23.0<br />

50 27.5<br />

60 32.0<br />

4. Dollars<br />

5. 0 to 6<br />

6. The data would be concentrated toward the bottom quarter of<br />

the graph. All the data would appear within first three grid<br />

boxes of the y-axis.


7. Yes, there is a relationship between the variables.<br />

8. As the time spent washing cars increases, the money in the<br />

cash box increases.<br />

9. If the theater club worked for five hours a Saturday for at least<br />

14 Saturdays, they could earn $1050. This amount is based on<br />

earning $75 during the five hour period (assuming $20 is the<br />

starting amount of money in the cash box). Between April<br />

and the fall, there would the Saturdays in May, June, July, and<br />

August for doing the car wash; a total of about 16 Saturdays.<br />

This would be enough time to earn $1000.<br />

Practice set 2: Bar Graph<br />

1. Graph title: “Percentage of teenagers that are employed in<br />

four cities.”<br />

2. The two variables represented on the x-axis are cities (four are<br />

represented) and gender (boys and girls). The variable<br />

represented on the y-axis is the percentage of teenagers that<br />

are employed. The range of values is from 0 to 80.<br />

3. The highest percentages of boys and girls employed is in city<br />

C. The lowest percentages is in city D. The percentages of<br />

boys and girls employed is about the same in city A which has<br />

the second highest percentage of teenagers employed. Girls<br />

employed outnumber boys employed in cities B and D.<br />

4. In cities A and C, the percentage of boys employed is greater<br />

than the percentage of girls employed. In cities B and D, the<br />

percentage of girls employed is greater than the percentage of<br />

boys employed.<br />

5. Answers will vary. Sample Answer:<br />

The type of businesses in city C are suited to hiring workers<br />

that can only work in the afternoons or evenings for a pay rate<br />

that is suitable to teenagers. The type of jobs in city D are<br />

more suited to people who can work full time.<br />

6. Answers will vary. Sample Answer:<br />

In city C, the kinds of jobs that are available to teenagers may<br />

be more suited for boys. The opposite is true for city B; there,<br />

the jobs may be more suitable and appealing to girls. By<br />

doing a survey of the teens in city C, this hypothesis could be<br />

tested.<br />

Practice set 3: Pie graph<br />

1. Graph title: “Percent distribution of jobs held by teenagers.”<br />

<strong>Skill</strong> Sheet 1.4: Recognizing Patterns on Graphs<br />

1. A, E, F<br />

2. B, C<br />

3. A, B, F<br />

<strong>Skill</strong> Sheet 2.1: Scientific Processes<br />

1. Maria and Elena’s question is: Does hot water in an ice cube<br />

tray freeze faster than cold water in an ice cube tray?<br />

2. Maria’s hypothesis: Hot water will take longer to freeze into<br />

solid ice cubes than cold water, because the hot water<br />

molecules have to slow down more than cold water molecules<br />

to enter the solid state and become ice.<br />

3. Examples of variables include:<br />

Amount of water in each ice cube tray “slot” must be<br />

uniform.<br />

Each ice cube tray must be made of same material, “slots” in<br />

all trays must be identical.<br />

Placement of trays in freezer must provide equal cooling.<br />

All “hot” water must be at the same initial temperature.<br />

All “cold” water must be at the same initial temperature.<br />

4. Examples of measurements include:<br />

Initial temperature of hot water.<br />

Initial temperature of cold water.<br />

Volume of water to fill each ice cube tray “slot.”<br />

Page 6 of 57<br />

2. Types of jobs held by teenagers and the percentages.<br />

3. No units are used in this graph. Instead, the graph is showing<br />

how categories (jobs in this case) are related to each other.<br />

4. The majority of jobs held by teenagers are in the retail<br />

industry (28%). Working teenagers are next likely to work in<br />

the food service industry (23%) and administrative support<br />

(21%). Other kinds of jobs held by teenagers include freight<br />

and stock handling (15%) and farm work (10%). Three<br />

percent of working teenagers participate in jobs that are not<br />

included in these categories.<br />

5. Answers will vary. A sample hypothesis based on this data is:<br />

The numbers of teenage girls and boys working in each job<br />

category is equal. I could test this hypothesis by interviewing<br />

employed teenagers that represent each job category. I would<br />

compare the numbers of girls and boys working in each<br />

category to see if my hypothesis is correct.<br />

6. Answers will vary.<br />

Practice set 4: Line graph<br />

1. Graph title: “Springfield <strong>High</strong> <strong>School</strong> Population 1970-2005”<br />

2. The two variables are the year and the number of students.<br />

3. The range of x-axis values is the 35 years between 1970 and<br />

2005. The range of y-axis values is 900-1,400 students, a<br />

population difference of 500 students.<br />

4. The student population rose sharply between 1970 and 1980,<br />

from just over 1,000 students to almost 1,400 students. Then<br />

the population plummeted to about 1,000 students between<br />

1980 and 1985.<br />

5. a. Student answers will vary. Possible reasons include an<br />

economic decline in the district (perhaps a major industry<br />

closed), a government home buyout (this can happen in<br />

conjunction with a major airport expansion, for example),<br />

or the school district may have built a new high school to<br />

ease overcrowding.<br />

b. Student answers will vary. Students could call the district<br />

office, ask a school staff member who was there during<br />

that time period, or ask a friend or relative who lived in<br />

the district.<br />

6. This is a line graph, not a scatterplot, because the x-values<br />

have no cause-and-effect relationship with the y-values.<br />

4. C, E<br />

5. D<br />

6. A<br />

Time taken for water to freeze solid.<br />

5. Sample procedure in 9 steps:<br />

(1) Place 1 liter of water in a refrigerator to chill for 1 hour.<br />

(2) Boil water in pot on a stove (water will be 100°C).<br />

(3) Using pot holders, a kitchen funnel, and a medicinemeasuring<br />

cup, carefully measure out 15 mL of boiling water<br />

into each slot in two labeled ice cube trays.<br />

(4) Remove chilled water from refrigerator, measure<br />

temperature.<br />

(5) Carefully measure 15 mL chilled water into each slot in<br />

two labeled ice cube trays.<br />

(6) Place trays on bottom shelf of freezer, along the back wall.<br />

(7) Start timer.<br />

(8) After 1/2 hour, begin checking trays every 15 minutes to<br />

see if solid ice has formed in any tray.<br />

(9) Stop timing when at least one tray has solid ice cubes in it.<br />

6. The average time was 3 hours and 15 minutes.


7. Repeating experiments ensures the accuracy of your results.<br />

Each time you are able to repeat your results, you reduce the<br />

effect of sources of error in the experiment that may come<br />

from following a certain procedure, human error, or from the<br />

conditions in which the experiment is taking place.<br />

8. The only valid conclusion that can be drawn is (d).<br />

9. Maria and Elena could ask a few of their friends to repeat<br />

their experiment. This would mean that the experiment would<br />

be repeated in other places with other freezers. If their friends<br />

<strong>Skill</strong> Sheet 2.1: What’s Your Hypothesis?<br />

1. Sample hypothesis: The water level in the cup is lower<br />

because the Sun heated the water in the cup and that caused<br />

evaporation of the water.<br />

2. Sample hypothesis: The candle heats up the air above it.<br />

Warm air is less dense so it rises. The effect causes the air in<br />

the box to move in the area above the candle. When the<br />

smoke from moves above the candle, it gets heated and rises<br />

out of the chimney above the candle.<br />

3. Sample hypothesis: Increasing the temperature of water will<br />

increase the rate at which evaporation occurs.<br />

4. Sample hypothesis: If the river is flowing down a mountain, it<br />

will flow faster than if it is flowing along flat land. In other<br />

words, the force of gravity causes river water to flow faster if<br />

the water is moving from a high to a lower place.<br />

<strong>Skill</strong> Sheet 2.2: Recording Observations in the Lab<br />

Exercise 1:<br />

1. c<br />

2. a<br />

3. c<br />

<strong>Skill</strong> Sheet 2.2: Lab Report Format<br />

This skill sheet can be used throughout the school year as a guide<br />

to writing a formal lab report.<br />

<strong>Skill</strong> Sheet 2.2: Using Computer Spreadsheets<br />

Example graph:<br />

1. Time is the independent variable; temperature is the<br />

dependent variable.<br />

2. The independent variable goes in the first column.<br />

3. The temperature increases slowly for the first 90 seconds and<br />

then increases much more rapidly from 90-300 seconds.<br />

Page 7 of 57<br />

are able to repeat the girls’ results, then the kind of freezer<br />

used can be eliminated as a factor that influenced the results.<br />

10. A new question could be: Do dissolved minerals in water<br />

affect how fast water freezes?<br />

For further study: Ask student to come up with a plan to test<br />

the validity of statements b and c. Encourage your students to<br />

research methods for measuring dissolved minerals and<br />

oxygen in water.<br />

5. Sample hypothesis: I think the flower bulbs have been dug up<br />

and eaten by squirrels.<br />

6. Sample hypothesis: Since kelp is a food source for the sea<br />

urchins, the urchin population might die out. Without a sea<br />

urchin population as a food source, the sea otter population<br />

might die out.<br />

7. Sample hypothesis: Snowshoe hares turn white in the winter<br />

so that they can blend in with the snow and avoid being<br />

caught by lynx. In the summertime, the brown coat of the hare<br />

blends in with the color of the ground.<br />

8. Sample hypothesis: Yes, I think there would be animals like<br />

coyotes in other deserts. [Example: The jackal in the Kalahari<br />

Desert in Southwest Africa plays a similar ecological role as<br />

the coyote.]<br />

Exercise 2:<br />

a. Disappearance of copper color on pennies<br />

b. Mass by year<br />

c. Data/observations<br />

d. answers vary.<br />

4. The slope for the first 90 seconds is 0.02 degrees per second,<br />

and then it increases to 0.08 degrees per second for the period<br />

from 180 to 300 seconds.<br />

5.


6.<br />

The slope is the same (2.0) until you get to the final segment,<br />

when it increases to 2.6.<br />

<strong>Skill</strong> Sheet 2.2: Identifying Control and Experimental Variables<br />

1. Experimental variable: antibacterial cleaner (antibacterial<br />

cleaner vs. no antibacterial cleaner). Control Variables: Petri<br />

dish, cotton swab, source of bacteria, length of experiment,<br />

incubation temperature, incubation light exposure<br />

2. Experimental variable: amount of water each plant gets.<br />

Control variables: plant type, plant size, pot, soil, duration of<br />

experiment, amount of light exposure.<br />

<strong>Skill</strong> Sheet 3.1: Position on the Coordinate Plane<br />

1.<br />

2.<br />

Page 8 of 57<br />

3. Experimental variable: bread type (preservatives vs. no<br />

preservatives). Control variables: plastic bag, damp paper<br />

towel, dark environment, duration of experiment.<br />

4. Experimental variable: fertilizer (fertilizer vs. no fertilizer).<br />

Control variables: amount of water, algae sample, location of<br />

beakers (sunlight), duration of experiment.<br />

3. Yes the order does matter. The coordinate (2, 3) shows a point<br />

that is 2 to the right and 3 up, while the coordinate (3, 2)<br />

shows a point that is 3 to the right and 2 up.


<strong>Skill</strong> Sheet 3.1: Latitude and Longitude<br />

Part 1 answers:<br />

1. Answers are:<br />

a. Iceland<br />

b. Algeria<br />

c. Argentina<br />

d. Australia<br />

e. New Zealand<br />

2. Answers are:<br />

a. Bay of Bengal<br />

b. Aegean Sea<br />

c. Red Sea<br />

d. Gulf of Mexico<br />

<strong>Skill</strong> Sheet 3.1: Map Scales<br />

1. Answers are:<br />

a. Andora<br />

b. No. I need to know the scale to answer the question.<br />

c. Yes. They are both 1.5 cm.<br />

d. No. One centimeter could represent different distances on<br />

each island.<br />

e. Calypso is much bigger.<br />

2. Answers are:<br />

(Note: Allow answers that are + or – 3 kilometers.)<br />

a. 40 km<br />

<strong>Skill</strong> Sheet 3.1: Vectors on a Map<br />

1. (+60 km, –10 km)<br />

2. (+1 km, +2 km)<br />

3. (+1 km, +1 km)<br />

<strong>Skill</strong> Sheet 3.1: Navigation<br />

Note to teacher: It is highly recommended that you do a trial run<br />

yourself before doing this activity with your students. This<br />

will enable you to help the students more efficiently during<br />

the activity, especially when it comes to finding locations on<br />

the maps.<br />

Making predictions:<br />

a. I expect to find coral reefs.<br />

b. We will need to watch where we navigate our boat<br />

because coral reefs exist close to the surface and could be<br />

an obstruction for our boat.<br />

It’s time to go!<br />

1. No student response required.<br />

2. No student response required.<br />

3. 1:326,856<br />

Page 9 of 57<br />

e. Baffin Bay<br />

Part 2 answers:<br />

1. Answers are:<br />

a. 30.33°N<br />

b. 45.75°N<br />

c. 20.61°S<br />

d. 60.33°S<br />

2. Answers are:<br />

a. 25.92°E<br />

b. 145.25°E<br />

c. 130.62°W<br />

d. 85.44°W<br />

b. 128 km<br />

c. 50 km<br />

d. 110 km<br />

e. 80 km<br />

f. 185 km<br />

g. 125 km<br />

h. 170 km (50 + 120)<br />

i. 118 km (78 + 40)<br />

j. 298 km (120 + 50 + 128)<br />

Bonus: Give credit for estimates between 850 – 950 km.<br />

4.<br />

5. Answers to student-designed problems will vary.<br />

4. It means that one unit on the map represents 326,856 of those<br />

units in real life.<br />

5. There are six feet in a fathom.<br />

6. There is a lighthouse with an occulting light, which means the<br />

period of darkness when the light is covered or obscured is<br />

less than the time period when the light is showing.<br />

7. Two fathoms<br />

8. Dump site of dredged material<br />

9. Dredged material is material that was dug by machine<br />

(usually from another channel or river bed) and has been<br />

dumped here.<br />

10. Yes<br />

11. No<br />

12. Yes


13. S Sh—Sand and shells on the bottom, Co Sh—coral shells on<br />

bottom, h S—hard sand on bottom, Co S—coral sand on<br />

bottom, bk SH—broken shells on bottom, bk Co—broken<br />

coral on bottom, Sh Co—shells and coral on bottom<br />

14. It is a flashing lighthouse. The light is obscured for longer<br />

than it is showing (the opposite is true with an occulting<br />

light).<br />

15. No student response required.<br />

16. Explosive Dumping Area<br />

17. A bad idea!<br />

18. No. Lower the anchor and use the rowboat to get ashore.<br />

19. Four<br />

20. The fourth is aeronautical, which means that it displays<br />

flashes, in this case, of white and green, to indicate the<br />

location of an airport, a heliport, a landmark, a certain point<br />

of a federal airway in mountainous terrain, or an obstruction.<br />

21. Seven nautical miles<br />

22. Cables<br />

23. Drop anchor<br />

24. No<br />

25. It means that the rocks on the isle are covered and uncovered<br />

by water and that they reach a height of 269 feet.<br />

26. Fathoms<br />

27. No student response required.<br />

28. 1:100,000<br />

29. You can see more detail on the second map.<br />

30. National Response Center or the nearest US Coast Guard<br />

facility<br />

<strong>Skill</strong> Sheet 3.2: Topographic Maps<br />

1. Answer graphic: 2. Answer graphic (at<br />

right):<br />

3. Answers are:<br />

a. 0 or sea level<br />

b. 400-499 feet<br />

c. 100 feet<br />

d. 300 feet<br />

e. between 0 and<br />

100 feet.<br />

4. Answers are:<br />

a. The island<br />

becomes two<br />

islands.<br />

<strong>Skill</strong> Sheet 3.3: Bathymetric Maps<br />

1. Example answers:<br />

a. Mid-Atlantic Ridge<br />

b. East Pacific Rise<br />

c. Middle America Trench or Mariana Trench<br />

d. Falkland Plateau<br />

e. Mendocino Fracture Zone<br />

2. Two tectonic plates move apart.<br />

3. Answers are:<br />

a. East Pacific Rise<br />

b. Chile Rise; this rise looks like the Mid-Atlantic Ridge<br />

c. Chatham Rise; this feature seems to be a plateau on the sea<br />

floor<br />

4. Subduction<br />

Page 10 of 57<br />

31. WXM-96; 162.475 Hz<br />

32. 2.5 fathoms<br />

33. Going right up to the shore gives you three feet of water<br />

below the ship—that’s not very much. It is recommended to<br />

anchor further out and row or perhaps swim to shore.<br />

34. 46 feet<br />

35. Coral shells<br />

36. A cay is a small, low island or reef made mostly of sand or<br />

coral.<br />

37. Cables<br />

38. feet<br />

39. 1:15,000<br />

40. More detail is evident in this map than the other two.<br />

41. The lines represent areas that have been swept clear, called<br />

wire-dragged areas. The depth is noted on the map to 42 feet<br />

offshore of the solid line and to 36 feet between the solid and<br />

dashed lines.<br />

42. We know that our boat is clear in this area because our boat is<br />

12 feet deep.<br />

43. Yes<br />

44. Row in—it is a very shallow channel with many areas even<br />

less than five feet deep.<br />

45. No student response required.<br />

46. Yes<br />

47. No student response required.<br />

48. No student response required.<br />

49. Answers will vary.<br />

50. No student response required.<br />

b. The original island is now three islands.<br />

c. No, the storm wave would wash over the island.<br />

5. A combination of two diverging plates at the East Pacific Rise<br />

and subduction zones in the northern part of the North Pacific<br />

Ocean. The plate movement associated with these features<br />

may have caused the fracture zone.<br />

6. The ridge has a lot of faults. There is a thin, dark-blue line in<br />

the middle of the ridge. The white areas around the ridge are<br />

not very prominent.<br />

7. This rise is less jagged. The white area near the rise is more<br />

prominent here than at the Mid-Atlantic Ridge.<br />

8. Mid-Atlantic Ridge; the dark line indicates a valley in the<br />

middle of the ridge<br />

9. Cross-sections of the Mid-Atlantic Ridge and the East Pacific<br />

Rise (Based on viewing the bathymetric map):


Mid-Atlantic Ridge cross-section East Pacific Rise cross-section<br />

<strong>Skill</strong> Sheet 3.3: Tanya Atwater<br />

1. Tanya Atwater came from a family of scientists—her father<br />

was an engineer and her mother a botanist. Atwater recalls<br />

many dinner discussions about science and she eventually<br />

shared in her parents’ passion. Atwater and her family went<br />

on many vacations. The family often found the most remote<br />

places to explore. This explains Atwater’s deep love for the<br />

outdoors.<br />

2. In 1967, Atwater began graduate school at the Scripps<br />

Oceanographic Institution in La Jolla, California. During this<br />

time, many exciting geological discoveries were being made.<br />

The concept of sea floor spreading was emerging, leading to<br />

the current theory of plate tectonics.<br />

3. While at Scripps, Atwater joined a research group that used<br />

sophisticated equipment to study the sea floor off of northern<br />

California. It was her first close look at sea floor spreading.<br />

Atwater also took twelve trips down to the ocean floor in the<br />

tiny submarine Alvin. She collected samples nearly 2 miles<br />

down on the ocean floor using mechanical arms. Atwater’s<br />

firsthand view through Alvin’s portholes gave her a better<br />

understanding of the pictures and sonar records she had<br />

previously studied.<br />

<strong>Skill</strong> Sheet 4.1: Solving Equations With One Variable<br />

Part 1 answers:<br />

1. w = 4.0 mm<br />

2. l = 0.8 m<br />

3. h = 8.00 cm<br />

4. d = 7.5 m<br />

5. s = 4.0 m/s<br />

6. t = 30 s<br />

7. t = 31.3 s<br />

8. D = 7.8 g/cm 3<br />

<strong>Skill</strong> Sheet 4.1: Problem Solving Boxes<br />

This skill sheet can be used throughout the school year to help<br />

students organize their work for questions or problems<br />

involving formulas and computation.<br />

<strong>Skill</strong> Sheet 4.1: Problem Solving with Rates<br />

1.<br />

2.<br />

3.<br />

4.<br />

365 days<br />

--------------------<br />

1 year<br />

----------------------<br />

1 foot<br />

12 inches<br />

$10.00<br />

---------------------------------<br />

3 small pizzas<br />

3 boxes<br />

-----------------------<br />

36 pencils<br />

Page 11 of 57<br />

10. The Mid-Atlantic Ridge has a slow spreading rate, while the<br />

spreading rate of the East Pacific Rise is fast. Because the<br />

Mid-Atlantic Ridge is so slow, a valley has developed<br />

between the two separating plates.<br />

4. Propagating rifts are created when sea floor spreading centers<br />

realign themselves. This realignment is in response to<br />

changes in plate motion or uneven magma supplies. In the<br />

1980s, Atwater was part of a team that researched<br />

propagating rifts near the Galapagos Islands off the coast of<br />

Ecuador. She has also discovered many propagating rifts on<br />

the sea floor off the northeast Pacific Ocean and evidence of<br />

propagating rifts in ancient sea floor records worldwide.<br />

5. Atwater has been a geology professor at the University of<br />

California, Santa Barbara for over 25 years. Atwater also<br />

works with the media, museums, and teachers to educate<br />

them about the Earth. She has created presentations and an<br />

animated teaching film, “Continental Drift and Plate<br />

Tectonics,” that has been used by educators from the<br />

elementary school level through college.<br />

6. Answers may vary. Some of Alvin’s noteworthy trips include<br />

locating a hydrogen bomb accidentally dropped in the<br />

Mediterranean Sea (1966), several trips to the Mid-Atlantic<br />

Ridge and Galapagos Rift, surveying the sunken ocean liner<br />

Titanic (1986), and IMAX filming off of the San Diego coast<br />

for a deep sea feature production (2002).<br />

9. m = 1.1 g<br />

10. m = 4.5 g<br />

11. V = 113 cm 3<br />

12. V = 2.3 cm 3<br />

Part 2 answers:<br />

1. Force = 8 N<br />

2. p = 20 Pa<br />

3. p = 216 Pa<br />

4. p = 15,000 Pa<br />

5.<br />

----------------------------------------------------<br />

360 miles<br />

18 gallons of gasoline<br />

6. 2,100 calories<br />

7. 1095<br />

-------------------------sodas<br />

year<br />

8. 725, 760 heatbeats<br />

-------------------------------------------week<br />

9. $27.48


10. 410 miles<br />

11. 6.6 miles<br />

--------------------hour<br />

12. 22.2 lbs<br />

13. 55 kg<br />

14. 5.4977 miles<br />

15. $280<br />

16. About 10 years<br />

17. 53 grams<br />

18. 0.11 miles<br />

-----------------------hour<br />

19. 270 pills<br />

20. 95 feet/sec<br />

<strong>Skill</strong> Sheet 4.1: Percent Error<br />

Table answers: 1. ≈ 0.43%<br />

Distance from A to B<br />

(cm)<br />

<strong>Skill</strong> Sheet 4.1: Speed<br />

1. 17 km/hr<br />

2. 55 mph<br />

3. 4.5 seconds<br />

4. 5.9 hours; 490 mph<br />

5. 4.0 km<br />

6. 2.5 miles<br />

7. 4.5 meters<br />

8. Answers are:<br />

a. 2.54 cm/inch<br />

b. 12 inches/min<br />

9. 6 km/hr<br />

10. Answers are:<br />

a. 600 seconds<br />

b. 10 minutes<br />

11. 1,200 meters<br />

12. Answers are:<br />

a. 42 km<br />

b. 9.3 km/hr<br />

Time from A to B<br />

(s)<br />

10 1.0050<br />

20 1.8877<br />

30 2.8000<br />

40 3.7850<br />

50 4.7707<br />

60 5.6101<br />

70 6.9078<br />

80 7.9648<br />

90 9.0140<br />

Page 12 of 57<br />

2. ≈ 3.30%<br />

3. ≈ 1.34%<br />

4. ≈ 0.16%<br />

5. ≈ 0.16%<br />

6. Answers are:<br />

a. Avg. ≈ 17.18 s; percent error ≈ 5.06%<br />

b. Avg. = 38.39 s; percent error ≈ 9.61%<br />

c. Avg. = 67.91 s; percent error ≈ 0.68%<br />

13. Answers are:<br />

a. 0.2 km/min<br />

b. 0.5 km/min<br />

c. 0.3 km/min faster by bicycle<br />

14. 12.5 km<br />

15. 40 minutes<br />

16. 90. km/hr<br />

17. 820 km/hr<br />

18. 32.5 km or 33 km<br />

19. 8 hours<br />

20. 633 km/hr<br />

21. 731 km/hr<br />

22. 1,680 km<br />

23. 3.84 × 10 5 km<br />

24. 2.03 × 10 4 seconds<br />

25. Answers for 25 (a)–(c) will vary. Having students write their<br />

own problems will further develop their understanding of<br />

how to solve speed problems.


<strong>Skill</strong> Sheet 4.1: Velocity<br />

1. 420. km/hr, north<br />

2. 0.30 seconds<br />

3. 224 minutes<br />

4. Answers are:<br />

a. 1.62 m/s, west<br />

b. 1.62 m/s, east<br />

5. 16.0 hours<br />

6. 3.0 m/s, west<br />

<strong>Skill</strong> Sheet 4.2: Calculating Slope from a Graph<br />

Numbers correlate to graph numbers:<br />

1.<br />

2.<br />

3. m = -- Calculated using points (0,3) and (9,8)<br />

4.<br />

5.<br />

– 3 × 3 – 3<br />

m = -------------- = -----<br />

6 × 2 4<br />

– 2 × 3 – 3<br />

m = -------------- = -----<br />

4 × 2 4<br />

5<br />

9<br />

4<br />

m = -- = 2<br />

2<br />

3<br />

m = -- = 1<br />

3<br />

Page 13 of 57<br />

7. Answers are:<br />

a. 4.5 m/s, south<br />

b. 4.5 m/s, north<br />

8. 22 seconds<br />

9. 0.36 miles/min, southwest or 22 mph, southwest<br />

10. 116 kilometers<br />

11. 3.9 km/hr, southeast<br />

12. 9.39 kilometers<br />

6.<br />

7. m =0<br />

8.<br />

9. m =2<br />

<strong>Skill</strong> Sheet 4.2: Analyzing Graphs of Motion with Numbers<br />

1. Answers are:<br />

a. The bicycle trip through hilly country.<br />

b. A walk in the park.<br />

c. Up and down the supermarket aisles.<br />

10.<br />

– 2<br />

m = ----- = – 1<br />

2<br />

m<br />

3<br />

= --<br />

4<br />

– 1<br />

m =<br />

-----<br />

2<br />

2. Answers are:<br />

a. The honey bee among the flowers.<br />

b. Rover runs the street.<br />

c. The amoeba.


<strong>Skill</strong> Sheet 4.2: Analyzing Graphs of Motion without Numbers<br />

1. Little Red Riding Hood. Graph Little Red Riding Hood:<br />

2. The Tortoise and the Hare. Use two lines to graph both the<br />

tortoise and the hare:<br />

<strong>Skill</strong> Sheet 4.3: Acceleration<br />

1. –0.75 m/s 2<br />

2. –8.9 m/s 2<br />

3. Answers are:<br />

Time (seconds) Speed (km/h)<br />

0 (start) 0 (start)<br />

2 3<br />

4 6<br />

6 9<br />

8 12<br />

10 15<br />

The acceleration of the ball is 1.5 km/hr/s.<br />

<strong>Skill</strong> Sheet 4.3: Acceleration and Speed-Time Graphs<br />

1. Acceleration = 5 miles/hour/hour or 5 miles/hour 2<br />

2. Acceleration = –2 meters/minute/minute or –2<br />

meters/minute 2<br />

3. Acceleration = 0 feet/minute/minute or 0 feet/minute 2 or no<br />

acceleration<br />

4. Answers are:<br />

<strong>Skill</strong> Sheet 4.3: Acceleration due to Gravity<br />

1. velocity = –14.7 m/s<br />

2. velocity = 11.3 m/s<br />

3. velocity = –76.4 m/s<br />

4. velocity = –16 m/s; depth = 86 meters<br />

Page 14 of 57<br />

3. The Skyrocket. Graph the altitude of the rocket:<br />

4. Each student story will include elements that are controlled<br />

by the graphs and creative elements that facilitate the story.<br />

Only the graph-controlled elements are described here.<br />

a. The line begins and ends on the baseline, therefore Tim<br />

must start from and return to his house.<br />

b. The line rises toward the first peak as a downward curved<br />

line that becomes horizontal. This indicates that Tim’s<br />

pace toward Caroline’s house slowed to a stop.<br />

c. Then the line rises steeply to the first peak. This indicates<br />

that after his stop, Tim continues toward Caroline’s house<br />

faster than before.<br />

d. The first peak is sharp, indicating that Tim did not spend<br />

much time at Caroline’s house on first arrival.<br />

e. The line then falls briefly, turns to the horizontal, and then<br />

rises to a second peak. This indicates that Tim left,<br />

paused, and then returned quickly to Caroline’s house.<br />

f. The line then remains at the second peak for a long time,<br />

then drops steeply to the baseline. This indicates that after<br />

spending a long time at Caroline’s house, Tim probably<br />

ran home.<br />

4. 7.5 seconds<br />

5. 88 mph<br />

6. 22 m/s<br />

7. 7 m/s 2<br />

8. –1.9 mph/s<br />

9. 67 m/s<br />

10. 32 m/s<br />

11. 1.7 m/s 2<br />

12. 2.6 seconds<br />

13. –2.3 m/s 2<br />

a. Segment 1: Acceleration = 2 feet/second/second,<br />

or 2 feet/second 2<br />

b. Segment 2: Acceleration = 0.67 feet/second/second,<br />

or 0.67 feet/second 2<br />

5. Distance = 1,400 meters<br />

6. Distance = 700 feet<br />

7. Distance = 75 kilometers<br />

5. height = 11 meters; yes<br />

6. time = 5.6 seconds<br />

7. time = 7.0 seconds


<strong>Skill</strong> Sheet 5.1: Ratios and Proportions<br />

1. 6 tablespoons; 2 eggs<br />

2. 3 / 4 cup; 1 / 3 teaspoon<br />

3. 1 / 4 teaspoon; 3 / 4 cup<br />

<strong>Skill</strong> Sheet 5.1: Internet Research<br />

Part 1 answers:<br />

1. Example answer: “science museums” + “South Carolina” not<br />

“Columbia”<br />

2. “dog breeds” + “inexpensive”<br />

3. “producing electricity” not “coal” not “natural gas”<br />

Part 2 answers:<br />

1. Answers will vary. Sites that may be authoritative include<br />

non-profit sites (recognizable by having “org” as the<br />

extension in the web address) or government sites such as<br />

www.nasa.gov (recognizable by the “gov” extension address)<br />

or college/university websites (recognizable by the “edu”<br />

extension address). These sites often provide information to<br />

large, diverse groups and are not typically supported by<br />

advertising. Sites that are supported by advertising can be<br />

<strong>Skill</strong> Sheet 5.1: Bibliographies<br />

No student responses are required.<br />

<strong>Skill</strong> Sheet 5.1: Mass vs. Weight<br />

1. 15 pounds<br />

2. 2.6 pounds<br />

3. 7.0 kilograms<br />

4. Yes, a balance would function correctly on the moon. The<br />

unknown mass would tip the balance one-sixth as far as it<br />

would on Earth, but the masses of known quantity would tip<br />

the balance one-sixth as far in the opposite direction as they<br />

did on Earth. The net result is that it would take the same<br />

amount of mass to equalize the balance on the moon as it did<br />

on Earth. (In the free fall environment of the space shuttle,<br />

however, the masses wouldn’t stay on the balance, so the<br />

balance would not work).<br />

5. Answers are:<br />

a. As the elevator begins to accelerate upward, the scale<br />

reading is greater than the normal weight. As the elevator<br />

accelerates downward, the scale reads less than the<br />

normal weight.<br />

Page 15 of 57<br />

4. Table answers:<br />

Sugar<br />

3 /8 cup<br />

Butter 3 tablespoons<br />

Milk 1 tablespoon<br />

Chocolate chips 1 cup<br />

Eggs<br />

Vanilla extract<br />

1 egg<br />

Baking soda<br />

Salt<br />

1 /2 teaspoon<br />

1 /6 teaspoon<br />

1 /8 teaspoon<br />

Confectioner’s sugar 1 tablespoon<br />

5. To make 16 brownies, you need two eggs and 2 tablespoons<br />

of sugar. Therefore, to make 8 brownies, you only need 1 of<br />

each unit for each ingredient: 1 egg and 1 tablespoon of sugar.<br />

6. Since 8 brownies requires 1 cup of chocolate chips, 3 cups of<br />

chocolate chips will make 24 brownies.<br />

7. 1.5 teaspoons vanilla are needed to make 24 brownies.<br />

authoritative, but may be biased in the information presented.<br />

Another characteristic of authoritative sites are that they are<br />

actively updated on a regular basis.<br />

2. Answers will vary. Reasons for why a source may not seem to<br />

be authoritative include: the author of the site is not affiliated<br />

with an organization and does not have obvious credentials,<br />

and the information seems to be one-sided. Many science<br />

topic searches will lead to student papers published on the<br />

Internet. These may contain mistakes, or they may have been<br />

written by a younger student.<br />

3. Answers will vary. Intended audiences can be young children,<br />

pre-teens, teenagers, adults, or select groups of people<br />

(women, men, people who like dogs, etc.).<br />

4. Answers will vary.<br />

b. When the elevator is at rest, the scale reads the normal<br />

weight.<br />

c. The weight appears to change because the spring is being<br />

squeezed between the top and the bottom of the scale.<br />

When the elevator accelerates upward, it is as if the<br />

bottom of the scale is being pushed up while the top is<br />

being pushed down. The upward force is what causes the<br />

spring to be compressed more than it is normally. When<br />

the elevator accelerates downward, the bottom of the scale<br />

provides less of a supporting force for the feet to push<br />

against. Therefore, the spring is not compressed as much<br />

and the scale reads less than the normal weight.


<strong>Skill</strong> Sheet 5.1: Mass, Weight, and Gravity<br />

1. Answers are:<br />

a. 22 newtons<br />

b. 8.1 newtons<br />

c. 8.9 N/kg<br />

2. Answers are:<br />

a. 65 kilograms<br />

b. 640 newtons<br />

c. 240 newtons<br />

Page 16 of 57<br />

3. Answers are:<br />

a. 23.10 N/kg<br />

b. 0.6 N/kg<br />

c. 4.9 newtons<br />

4. Answers are:<br />

a. 195,700 newtons<br />

b. 19,970 kilograms<br />

c. 146,800 newtons<br />

d. weight of toy-filled boxes = 48,900 newtons.<br />

mass of toy-filled boxes = 4,990. kg<br />

⎛ ⎞ i<br />

⎜ ⎟<br />

⎝ ⎠<br />

<strong>Skill</strong> Sheet 5.1: Gravity Problems<br />

Table 1 answers: 1. 9.5 pounds on Neptune<br />

Planet Force of gravity in Value compared to 2. 1,030 newtons on Saturn<br />

Newtons (N) Earth’s gravity 3. The baby weighs 45 newtons on Earth which is equal to 10.04<br />

Mercury<br />

Venus<br />

Earth<br />

Mars<br />

Jupiter<br />

3.7<br />

8.9<br />

9.8<br />

3.7<br />

23.1<br />

0.38<br />

0.91<br />

1<br />

0.38<br />

2.36<br />

pounds.<br />

4. Venus, Jupiter, Neptune, Pluto, then Saturn<br />

5. Answer:<br />

−11<br />

2 24 26<br />

6.67 × 10 Nm (6.4× 10 )(5.7 × 10 )<br />

Gravity = 2 11 2<br />

kg<br />

(6.52× 10 )<br />

Saturn 9.0 0.92<br />

17<br />

= 5.72× 10 N<br />

Uranus 8.7 0.89<br />

Neptune 11.0 1.12<br />

Pluto 0.6 0.06<br />

<strong>Skill</strong> Sheet 5.1: Universal Gravitation<br />

1. F =9.34× 10 –6 N. This is basically the force between you<br />

and your car when you are at the door.<br />

2. 5.27 × 10 –10 N<br />

3. 4.42 N<br />

4. 7.36 × 10 22 kilograms<br />

5. Answers are:<br />

<strong>Skill</strong> Sheet 5.2: Friction<br />

1. Answers are:<br />

a. rolling friction<br />

b. Sliding friction is generally greater than rolling friction,<br />

so it would probably take more force to transport the<br />

blocks in the sled.<br />

c. The friction force would increase, because more blocks<br />

would mean more weight force squeezing the two<br />

surfaces together.<br />

d. static friction<br />

2. Answers are:<br />

a. viscous friction<br />

b. The friction force would increase because the boat would<br />

sit lower in the water.<br />

3. Answers are:<br />

a. rolling friction and air friction<br />

<strong>Skill</strong> Sheet 5.3: Equilibrium<br />

1. 142 N<br />

2. A is 40 N; B is 8 N<br />

3. 340 N<br />

a. 9.8 N/kg = 9.8 kg-m/sec 2 -kg = 9.8 m/sec 2<br />

b. Acceleration due to the force of gravity of Earth.<br />

c. Earth’s mass and radius.<br />

6. 1.99 ×10 20 N<br />

7. 4,848 N<br />

8. 3.52 × 10 22 N<br />

b. rolling friction<br />

4. Answers are:<br />

a. Student responses will vary. Encourage students to look<br />

for a sports car rather than a professional racing car.<br />

Racing car spoilers may serve a different purpose.<br />

b. Sports car spoilers are generally designed to increase<br />

down force on the rear of the car, causing greater friction<br />

between the rear tires and the road.<br />

c. Spoilers on hybrid cars and sport utility vehicles are<br />

usually designed to create a smoother, less turbulent<br />

airflow over the rear of the vehicle. This reduces drag (air<br />

friction). Sports car spoilers are most often designed to<br />

increase rolling friction, not to decrease air friction.<br />

Spoilers on different types of cars serve different<br />

purposes.<br />

4. From the outside of a balloon, two forces act inward. The<br />

elastic membrane of the balloon and the pressure of Earth’s<br />

atmosphere work together to balance the outward force of the<br />

helium compressed inside. Together with the elastic force,


atmospheric pressure near Earth’s surface applies enough<br />

force to maintain this equilibrium, but as the balloon rises,<br />

atmospheric pressure decreases. Although the inward force<br />

supplied by the elastic membrane remains unchanged, the<br />

decreasing atmospheric pressure force causes an imbalance<br />

with the outward force of the contained helium and the<br />

balloon expands. At some point, the membrane of the balloon<br />

reaches its elastic limit and bursts.<br />

<strong>Skill</strong> Sheet 6.1: Net Force and Newton’s First Law<br />

1. When at rest, the cart experiences a normal force of 105 N<br />

and its weight of –105 N.<br />

2. The net force on the cart is +20 N. While the cart is on the<br />

slippery margarine, it is not moving at constant velocity since<br />

it is experiencing a net force (acceleration).<br />

3. The normal force on the cart after it is loaded with groceries is<br />

+180 N.<br />

<strong>Skill</strong> Sheet 6.1: Isaac Newton<br />

1. The isolation due to the Plague allowed Newton to focus on<br />

his scientific work, free from the distractions of university<br />

life. However, most scientists learn a great deal from<br />

discussing their ideas with peers. Collaboration also enables<br />

experimental scientists to test a greater number of hypotheses.<br />

2. Newton was an active member of the scientific community at<br />

Cambridge for just under 30 years. In that time, he made great<br />

strides in understanding light and optics, planetary motion,<br />

universal gravitation, and calculus. He made extraordinary<br />

contributions to many scientific fields during those years.<br />

3. Example answer: Newton’s first law says that unless you<br />

apply an unbalanced force to an object, the object will keep<br />

on doing what is was doing in the first place. So a rolling ball<br />

will keep on rolling until an unbalanced force changes its<br />

motion, while a ball that is not moving will stay still unless<br />

acted on by an unbalanced force.<br />

<strong>Skill</strong> Sheet 6.2: Newton’s Second Law<br />

1. 2.100 m/s2 2. 83 m/s2 3. 82 N<br />

4. 6 kg<br />

5. 9800 N<br />

6. 900 kg<br />

7. 1.9 m/s2 Page 17 of 57<br />

4. Gravity accelerates the cart down the ramp.<br />

5. The friction force is greater on the rough blacktop than on the<br />

smooth tile.<br />

6. The line of twenty empty carts has twenty times as much<br />

inertia, so it takes a much greater force to get it moving.<br />

4. Example answer: The law of universal gravitation says that<br />

the force of attraction between two objects is directly related<br />

to the masses of the objects and inversely related to the<br />

distance between them.<br />

5. Newton’s law of universal gravitation.<br />

6. Newton claimed that 20 years earlier, he had invented the<br />

material that Leibnitz published. Newton accused Leibnitz of<br />

plagiarism. Most historians today agree that the two<br />

developed the material independently, and therefore they are<br />

known as co-discoverers.<br />

Extra information: The famous legend of Newton’s apple tells<br />

of Newton sitting in his garden in Linconshire in 1666,<br />

watching an apple fall from a tree. He later noted that “In the<br />

same year, I began to think of gravity extending to the orb of<br />

the moon.” However, he did not make public his musings<br />

about gravity until the 1680’s, when he formulated his law of<br />

universal gravitation.


<strong>Skill</strong> Sheet 6.3: Applying Newton’s Laws<br />

Table answers are:<br />

Newton’s laws<br />

of motion<br />

The<br />

first<br />

law<br />

The<br />

second<br />

law<br />

The<br />

third<br />

law<br />

Write the law here in your<br />

own words<br />

An object will continue<br />

moving in a straight<br />

line at constant speed<br />

unless acted upon by an<br />

outside force.<br />

The acceleration (a) of<br />

an object is directly<br />

proportional to the<br />

force (F) on an object<br />

and inversely<br />

proportional to its mass<br />

(m). The formula that<br />

represents this law is<br />

For every action force<br />

there is an equal and<br />

opposite reaction force.<br />

<strong>Skill</strong> Sheet 6.3: Momentum<br />

1.<br />

2.<br />

3.<br />

4.<br />

5.<br />

a<br />

=<br />

---<br />

F<br />

m<br />

35 m<br />

momentum 4,000 kg × ----------- 140,000 kg<br />

s<br />

m<br />

= =<br />

⋅ --s<br />

35 m<br />

momentum 1,000 kg × ----------- 35,000 kg<br />

s<br />

m<br />

= = ⋅ --s<br />

8 kg × speed 16 kg m<br />

= ⋅ --s<br />

2 m<br />

speed = -------s<br />

0.5 m<br />

mass × ------------ 0.25 kg<br />

s<br />

m<br />

= ⋅ --s<br />

mass = 0.5 kg<br />

45,000 kg m<br />

⋅ --s<br />

Example of the law<br />

A seat belt in a car<br />

prevents you from<br />

continuing to move<br />

forward when your car<br />

suddenly stops. The seat<br />

belt provides the “outside<br />

force.”<br />

A bowling ball and a<br />

basketball, if dropped from<br />

the same height at the<br />

same time, will fall to<br />

Earth in the same amount<br />

of time. The resistance of<br />

the heavier ball to being<br />

moved due to its inertia is<br />

balanced by the greater<br />

gravitational force on this<br />

ball.<br />

When you push on a wall,<br />

it pushes back on you.<br />

<strong>Skill</strong> Sheet 6.3: Momentum Conservation<br />

1. m = p/v; m = (10.0 kg·m/s) / (1.5 m/s); m = 6.7 kg<br />

2. v = p/m; v = (1000 kg·m/s) / (2.5 kg); v = 400 m/s<br />

3. p = mv<br />

(mass is conventionally expressed in kilograms)<br />

p = (0.045 kg)(75.0 m/s)<br />

p = 3.38 kg·m/s<br />

4. p (before firing) = p (after firing)<br />

m 1 v 1 + m 2 v 2 = m 3 v 3 + m 4 v 4<br />

400 kg(0 m/s) + 10 kg(0 m/s) = 400 kg(v 2 ) + 10 kg(20 m/s)<br />

0 = 400 kg(v2) + 200 kg·m/s<br />

(v 2 ) = (–200 kg·m/s)/400 kg<br />

(v 2 ) = –0.5 m/s<br />

Page 18 of 57<br />

1. The purse continues to move forward and fall off of the seat<br />

whenever the car comes to a stop. This is due to Newton’s first law<br />

of motion which states that objects will continue their motion unless<br />

acted upon by an outside force. In this case, the floor of the car is<br />

the stopping force for the purse.<br />

2. Newton’s third law of motion states that forces come in action and<br />

reaction pairs. When a diver exerts a force down on the diving<br />

board, the board exerts an equal and opposite force upward on the<br />

diver. The diver can use this force to allow himself to be catapulted<br />

into the air for a really dramatic dive or cannonball.<br />

3. Newton’s second law<br />

4. The correct answer is b. One newton of force equals 1 kilogrammeter/second<br />

2 . These units are combined in Newton’s second law<br />

of motion: F = mass × acceleration.<br />

5.<br />

6.<br />

0.30 m<br />

s 2<br />

---------------<br />

F<br />

= ------------<br />

65 kg<br />

F<br />

0.30 m<br />

s 2<br />

--------------- × 65 kg 20. kg m<br />

s 2<br />

⎞<br />

⎛<br />

⎠<br />

⎝<br />

=<br />

= ⋅ ----<br />

2 kg ⋅ m<br />

2<br />

2 N m<br />

a = =<br />

sec<br />

= 0.2<br />

10 kg 10 kg sec<br />

7. The hand pushing on the ball is an action force. The ball provides a<br />

push back as a reaction force. The ball then provides an action force<br />

on the floor and the floor pushes back in reaction. Another pair of<br />

forces occurs between your feet and the floor.<br />

8. A force<br />

6.<br />

-----------<br />

30.m<br />

s<br />

7. The 4.0-kilogram ball requires more<br />

force to stop.<br />

8. 980 kg<br />

9.<br />

4.2 kg<br />

10.<br />

11.<br />

m<br />

⋅ --s<br />

15 m<br />

--s<br />

0.01 kg m<br />

⋅<br />

--s<br />

5. p (before throwing) = p (after throwing)<br />

m 1 v 1 + m 2 v 2 = m 3 v 3 + m 4 v 4<br />

0 = m 1 (0.05 m/s) + 0.5 kg(10.0 m/s)<br />

m 1 = (–0.5 kg)(10.0 m/s)/ (0.05 m/s)<br />

Eli’s mass + the skateboard (m 1 ) = 100 kg<br />

6. Answers are:<br />

a. p = mv + Δp; p = mv + FΔt<br />

p = (80 kg)(3.0 m/s) + (800 N)(0.30 s)<br />

p = 480 kg·m/s<br />

b. v = p/m; v = (480 kg·m/s)/80 kg; v = 6.0 m/s<br />

7. Answers are:<br />

a. p = mv; p = (2000 kg)(30 m/s); p = 60,000kg·m/s<br />

2


. FΔt = mΔv<br />

F = (mΔv)/(Δt); F = (60,000 kg·m/s)/(0.72 s)<br />

F = 83,000 N<br />

8. Answers are:<br />

a. P = (# of people)(mv); p =(2.0 × 10 9 )(60 kg)(7.0 m/s);<br />

p =8.4× 10 11 kg·m/s<br />

b. p (before jumping) = p (after jumping)<br />

m 1 v 1 + m 2 v 2 = m 3 v 3 + m 4 v 4<br />

0 = 8.4 × 10 11 kg·m/s + 5.98 × 10 24 (v 4 );<br />

(v 4 ) = (–8.4 × 10 11 kg·m/s)/ 5.98 × 10 24<br />

Earth moves beneath their feet at the speed<br />

v 4 = –1.4 × 10 –13 m/s<br />

<strong>Skill</strong> Sheet 6.3: Collisions and Momentum Conservation<br />

1. p = mv; p = (100.kg)(3.5m/s); p = 350kg·m/s<br />

2. p = mv; p = (75.0kg)(5.00m/s); p = 375kg·m/s<br />

3. Answer:<br />

p (before coupling) = p (after coupling)<br />

m 1 v 1 + m 2 v 2 = (m 1 + m 2 )(v 3+4 )<br />

(2000 kg)(5m/s) + (6000 kg)(–3m/s) = (8000 kg)(v 3+4 )<br />

v 3+4 = –1m/s or 1m/s west<br />

4. Answer:<br />

p (before collision) = p (after collision)<br />

m 1 v 1 + m 2 v 2 = m 1 v 3 + m 2 v 4<br />

(4 kg)(8m/s) + (1 kg)(0m/s) = (4 kg)(4.8m/s) + (1kg)v 4<br />

v 4 = (32kgm/s – 19.2kg-m/s)/(1kg); v 4 = 12.8m/s or 13m/s<br />

5. Answer:<br />

p (before shooting) = p (after shooting)<br />

m 1 v 1 + m 2 v 2 = (m 1 + m 2 )(v 3+4 )<br />

(0.0010 kg)(50.0 m/s) + (0.35 kg)(0 m/s) = (0.351kg)(v 3+4)<br />

(v 3+4 ) = (0.050kg-m/s)/ (0.351kg); (v 3+4 ) = 0.14 m/s<br />

6. Answer:<br />

p (before tackle) = p (after tackle)<br />

m 1 v 1 + m 2 v 2 = (m 1 + m 2 )(v 3+4 )<br />

(70 kg)(7.0 m/s) + (100 kg)(–6.0 m/s) = (170 kg)(v 3+4 )<br />

(v 3+4) = (490kg·m/s – 600kg·m/s)/ (170kg)<br />

(v 3+4 ) = –0.65 m/s<br />

Terry is moved backwards at a speed of 0.65 m/s while Jared<br />

holds on.<br />

7. Answer:<br />

p (before hand) = p (after hand)<br />

m 1v 1 + m 2v 2 = (m 1+ m 2)(v 3+4)<br />

(50.0 kg)(7.00 m/s) + (100. kg)(16.0m/s) = (150 kg)(v 3+4 )<br />

(v 3+4 ) = (350 kg·m/s + 1,600 kg·m/s)/150 kg<br />

(v 3+4) = 13 m/s<br />

<strong>Skill</strong> Sheet 6.3: Rate of Change of Momentum<br />

1. Force = 200,000 N. At this level of force, after a couple of<br />

hits with a wrecking ball, any impressive-looking wall<br />

crumbles to pieces.<br />

2. 3 seconds<br />

3. 250 N<br />

4. 750 N<br />

5. 75 million N<br />

6. The answers are:<br />

a. The force created on the egg is about:<br />

0.05 kg × 10 m/s<br />

---------------------------------------- = 500 N<br />

0.001 sec<br />

Page 19 of 57<br />

9. Answers are:<br />

a. p = mv; p = (60 kg)(6.00 m/s); p = 360 kg·m/s<br />

b. v av = (v i + v f )/2; v av = (6.00 m/s + 0 m/s)/2 = 3.00 m/s<br />

Δt = d/ΔV; Δt = 0.10 m/3.00 m/s; Δt = 0.033 s<br />

FΔt = mΔV; F = (mΔV)/Δt;<br />

F = (60 kg)(6.00 m/s)/(0.033 s); F = 11,000 N<br />

10. Since the gun and bullet are stationary before being fired, the<br />

momentum of the system is zero. The “kick” of the gun is the<br />

momentum of the gun that is equal but opposite to that of the<br />

bullet maintaining the “zero” momentum of the system.<br />

11. It means that momentum is transferred without loss.<br />

8. Answer:<br />

p (before jump) = p (after jump)<br />

m 1 v 1 + m 2 v 2 = (m 1 + m 2 )(v 3+4 )<br />

(520 kg)(13.0 m/s) + (85.0 kg)(3.00 m/s) = (605 kg)(v 3+4);<br />

v 3+4 = (7,015 kg·m/s)/(605 kg)<br />

v 3+4 = 11.6 m/s<br />

9. Answer:<br />

p (before jumping) = p (after jumping)<br />

m 1 v 1 + m 2 v 2 + m 3 v 3 = m 1 v 4 + m 2 v 5 + m 3 v 6<br />

(45 kg)(1.00 m/s) + (45 kg)(1.00 m/s) + (70 kg)(1.00 m/s) =<br />

(45 kg)(3.00 m/s) – (45 kg)(4.00 m/s) + (70 kg)(v 6 )<br />

160 kg·m/s = (–45 kg·m/s) + (70 kg)(v 6 )<br />

(v 6) = (205 kg·m/s)(70 kg)<br />

(v 6 ) = 2.9 m/s<br />

10. Answers are:<br />

a. Answer:<br />

p (before toss) = p (after toss)<br />

m 1 v 1 + m 2 v 2 = (m 3 + m 4 )(v 3+4 )<br />

(0.10 kg)(v 1) + (0.10 kg)(0 m/s) = (0.20 kg)(15 m/s);<br />

(v 1 ) = (0.30 kg·m/s)(0.10 kg)<br />

(v 1 ) = 30. m/s<br />

b. Answer:<br />

p (before collision) = p (after collision)<br />

m 1 v 1 + m 2 v 2 = m 1 v 3 + m 2 v 4<br />

(0.10 kg)(30 m/s) + (0.10 kg)(0 m/s) = (0.10 kg)(v 3) +<br />

(0.10 kg)(–30 m/s)<br />

(v 3 ) = (3.0 kg·m/s + 3.0 kg·m/s)/(0.10 kg)<br />

(v 3) = 60 m/s<br />

The block slides away at a much higher speed of v 3 = 60<br />

m/s. The “bouncy ball, by rebounding, has experienced a<br />

greater change in momentum. The block will experience<br />

this change in momentum as well.<br />

b. The force created on the egg by the person is:<br />

50 kg 9.8 m/s<br />

c. The force created by the person is close to the amount of<br />

force that broke the egg. Therefore, if the person fell on<br />

the egg, it would probably break.<br />

d. As a result the force will be 500 times smaller:<br />

e. The egg would probably not break if it fell on the pillow<br />

because the force is 500 times smaller than if it fell on the<br />

hard floor.<br />

2<br />

× = 490 N<br />

0.05 kg × 10 m/s<br />

---------------------------------------- =<br />

1 N<br />

0.5 s


<strong>Skill</strong> Sheet 7.1: Mechanical Advantage<br />

1. 4<br />

2. 0.4<br />

3. 100 newtons<br />

4. 25 newtons<br />

5. 300 newtons<br />

6. 26 newtons<br />

<strong>Skill</strong> Sheet 7.1: Mechanical Advantage of Simple Machines<br />

1. 5<br />

2. 1.5<br />

3. 0.5 meters<br />

4. 4.8 meters<br />

5. 0.4<br />

6. 0.8 meters<br />

7. 0.25 meters<br />

<strong>Skill</strong> Sheet 7.1: Work<br />

1. Work is force acting upon an object to move it a certain<br />

distance. In scientific terms, work occurs ONLY when the<br />

force is applied in the same direction as the movement.<br />

2. Work is equal to force multiplied by distance.<br />

3. Work can be represented in joules or newton·meters.<br />

4. Answers are:<br />

a. No work done<br />

b. Work done<br />

c. No work done<br />

d. Work done<br />

e. Work done<br />

5. 100 N·m or 100 joules<br />

6. 180 N·m or 180 joules<br />

7. 100,000 N·m or 100,000 joules<br />

8. 50 N·m to lift the sled; no work is done to carry the sled<br />

9. No work was done by the mouse. The force on the ant was<br />

upward, but the distance was horizontal.<br />

<strong>Skill</strong> Sheet 7.1: Types of Levers<br />

Part 1 and 2 answers:<br />

1. 2nd class lever<br />

Page 20 of 57<br />

7. 3<br />

8. 150 newtons<br />

9. 1.5<br />

10. Answers are:<br />

a. 1,500 newtons<br />

b. 2 meters<br />

8. 6.7<br />

9. 2 meters<br />

10. 12 meters<br />

11. 2.4<br />

12. 6 newtons<br />

13. 560 newtons<br />

14. 4 meters<br />

10. 10,000 joules<br />

11. Answers are:<br />

a. 1.25 meters<br />

b. 27 pounds<br />

12. 2,500 N or 562 pounds<br />

13. 1,500 N<br />

14. 54 N·m or 54 joules<br />

15. 225 N·m or 225 joules<br />

16. 0.50 meters<br />

17. Answers are:<br />

a. No work was done.<br />

b. 100 N·m or 100 joules<br />

18. Answers are:<br />

a. No work is done<br />

b. 11 N·m or 11 joules<br />

c. 400 N·m or 400 joules (Henry did the most work.)<br />

2. 1st class lever<br />

3. 3rd class lever<br />

Answers for (4) include answers for (1)–(4) in Part 2:


4. Two examples:<br />

<strong>Skill</strong> Sheet 7.1: Gear Ratios<br />

1. 9 turns<br />

7. Answers for the table are:<br />

2. 1 turn<br />

Table 2: Setup for three gears<br />

3. 4 turns<br />

Set Gears Number Ratio 1 Ratio 2<br />

4. 10 turns<br />

5. 6 turns<br />

6. Answers for the table are:<br />

Table 1: Using the gear ratio to calculate number of turns<br />

up<br />

1 Top gear<br />

of<br />

teeth<br />

12<br />

(top gear:<br />

middle<br />

gear)<br />

(middle<br />

gear: bottom<br />

gear)<br />

Input<br />

Gear<br />

(# of teeth)<br />

Output<br />

Gear<br />

(# of teeth)<br />

Gear<br />

ratio<br />

(Input Gear:<br />

Output Gear)<br />

How many<br />

turns does<br />

the output<br />

gear make if<br />

the input<br />

gear turns 3<br />

times?<br />

How many<br />

turns does the<br />

input gear<br />

make if the<br />

output gear<br />

turns 2 times?<br />

2<br />

Middle<br />

gear<br />

Bottom<br />

gear<br />

Top gear<br />

24<br />

36<br />

24<br />

1<br />

--<br />

2<br />

2<br />

--<br />

3<br />

24<br />

36<br />

24<br />

12<br />

1<br />

3<br />

3<br />

9<br />

2<br />

0.67, or 2/3<br />

of a turn<br />

Middle<br />

gear<br />

Bottom<br />

36<br />

12<br />

2<br />

--<br />

3<br />

3<br />

--<br />

1<br />

24 36 0.67, or 2/3 2 3<br />

gear<br />

48 36 1.33, or 4/3 4 1.5<br />

3 Top gear 12<br />

24 48 0.5, or 1/2 1.5 4<br />

Middle<br />

gear<br />

Bottom<br />

gear<br />

48<br />

24<br />

1<br />

--<br />

4<br />

4<br />

--<br />

2<br />

4 Top gear 24<br />

Middle<br />

gear<br />

Bottom<br />

gear<br />

48<br />

36<br />

1<br />

--<br />

2<br />

4<br />

--<br />

3<br />

Page 21 of 57<br />

Total gear ratio<br />

(Ratio 1 × Ratio 2)<br />

1<br />

--<br />

3<br />

2<br />

--<br />

1<br />

1<br />

--<br />

2<br />

2<br />

--<br />

3


8. The middle gear turns left. The bottom gear turns right.<br />

9. 3 times<br />

10. 4 times<br />

<strong>Skill</strong> Sheet 7.1: Levers in the Human Body<br />

Lever A:<br />

1. Type of lever: Third-class lever.<br />

2. This lever is used to lift objects.<br />

Lever B:<br />

<strong>Skill</strong> Sheet 7.1: Bicycle Gear Ratios Project<br />

Students evaluate the gear ratios of their own bicycles to<br />

complete this project. Answers will vary.<br />

<strong>Skill</strong> Sheet 7.2: Potential and Kinetic Energy<br />

1. First shelf: 5.0 newton-meters<br />

Second shelf: 7.5 newton-meters<br />

Third shelf: 10. newton-meters<br />

2. Answers are:<br />

a. 588 newtons<br />

b. 1.7 meters<br />

c. 5.8 m/s<br />

<strong>Skill</strong> Sheet 7.2: Identifying Energy Transformations<br />

1. (1) Chemical energy from food to kinetic energy to elastic<br />

energy; (2) chemical energy from food to kinetic energy; (3)<br />

elastic energy to kinetic energy.<br />

2. Electrical energy to radiant (microwave) energy to kinetic<br />

energy (increased movement of molecules in the soup, which<br />

increases the soup’s temperature).<br />

3. Chemical energy from food to kinetic energy (as Dmitri<br />

operates the pump) to pressure energy.<br />

Page 22 of 57<br />

11. 1/2 time<br />

12. 6 times<br />

3. Type of lever: First-class lever.<br />

4. This lever is used to chew food.<br />

Lever C:<br />

5. Type of lever: Second-class lever.<br />

6. This lever is used to raise and lower the heel of the foot while<br />

standing.<br />

3. Answers are:<br />

a. 450 joules<br />

b. 450 joules<br />

c. 46 meters<br />

4. 25,000 joules<br />

5. 4 m/s<br />

6. 75 kilograms<br />

4. Chemical energy from the battery in controller to radiant<br />

energy (radio waves). Radio waves to electrical signal in car;<br />

chemical energy from car battery to kinetic energy as car<br />

moves.<br />

5. Chemical energy from food is transformed to kinetic energy<br />

which is transformed to potential energy as Adeline moves<br />

toward the top of the hill. As she coasts down the other side,<br />

potential energy is transformed to kinetic energy.


<strong>Skill</strong> Sheet 7.2: Energy Transformations—Extra Practice<br />

Part 1 answers:<br />

1. The potential energy of the stretched bungee cord is changed<br />

into the kinetic energy of the person bouncing back up.<br />

2. The potential energy of the football from its high point is<br />

changed into kinetic energy as it spirals down.<br />

3. In this case, radiant energy is converted to chemical energy in<br />

the battery, which is then converted to the electrical energy<br />

needed to run the calculator. Mechanical energy (kinetic or<br />

potential, is not being used in this case.<br />

Part 2 answers:<br />

1. The chemical potential energy of the wood is changed to<br />

radiant energy in the form of heat. Radiant energy is changed<br />

<strong>Skill</strong> Sheet 7.2: Conservation of Energy<br />

1. 0.20 meters<br />

2. 3.5 m/s<br />

3. 39.6 m/s<br />

4. 196,000 joules<br />

<strong>Skill</strong> Sheet 7.2: James Joule<br />

1. Perhaps because he thought that the pursuit of science was<br />

worthwhile.<br />

2. His father hired one of the most famous scientists of his time<br />

to tutor his sons.<br />

3. His interest was based upon his desire to improve the<br />

brewery. He wanted to make a more efficient electric motor to<br />

replace the old steam engines that they had at the time.<br />

4. His goal had been to replace the old steam engines with more<br />

efficient electric motors. He was not able to do that, however,<br />

he learned a great deal about electromagnets, magnetism,<br />

heat, motion, electricity, and work.<br />

5. Electricity produces heat when it travels through a wire<br />

because of the resistance of the wire. Joule’s Law also<br />

provided a formula so that scientists could calculate the exact<br />

amount of heat produced.<br />

<strong>Skill</strong> Sheet 7.3: Efficiency<br />

1. 27.1 percent<br />

2. 92 joules<br />

3. 100,000 joules<br />

4. 94.2 kilojoules<br />

<strong>Skill</strong> Sheet 7.3: Power<br />

1. 250 watts<br />

2. 50 watts<br />

3. 1,200 watts<br />

4. 1,500 watts<br />

5. 741 watts<br />

6. 720 watts<br />

7. work = 500 joules; power = 33 watts<br />

8. 1,800 seconds or 30 minutes<br />

Page 23 of 57<br />

into mechanical energy as the water boils, changes to steam,<br />

and makes the whistle vibrate, which causes vibrations in air<br />

molecules that we experience as sound.<br />

2. Nuclear energy is changed into electrical energy, which is<br />

changed into radiant energy (light to make the television<br />

picture) and mechanical energy (vibration of speakers) that<br />

causes vibration of air molecules that we experience as sound.<br />

3. The chemical potential energy of food is changed into<br />

mechanical energy of the bicyclist, which is changed into<br />

electrical energy of the generator, which is changed into<br />

radiant energy from the light.<br />

5. 10. meters<br />

6. 30. meters<br />

7. Answers will vary.<br />

6. Joule believed that heat was a state of vibration caused by the<br />

collision of molecules. This contradicted the beliefs of his<br />

peers who thought that heat was a fluid.<br />

7. Joule knew that the temperature of the water at the bottom of<br />

the waterfall was warmer than the water at the top of the<br />

waterfall. He thought that this was true because the energy<br />

produced by the falling water was converted into heat energy.<br />

He wanted to measure how far water had to fall in order to<br />

raise the temperature of the water by one degree. Joule used a<br />

large thermometer to measure the temperature at the top of<br />

the waterfall and the temperature at the bottom of the<br />

waterfall. The experiment failed because the water did not fall<br />

the right distance for his calculations and there was too much<br />

spray from the waterfall to read the instruments accurately.<br />

8. Refrigeration<br />

9. The joule is the international measurement for a unit of<br />

energy.<br />

10. Answers will vary.<br />

5. Answers are:<br />

a. 2,025 million watts<br />

b. b. 39.5 percent<br />

6. 59 percent<br />

9. 2,160,000 joules<br />

10. 2,500 watts<br />

11. 90,000 joules<br />

12. work = 1,500 joules; time = 60 seconds<br />

13. force =25 newtons; power = 250 watts<br />

14. distance = 100 meters; power = 1,000 watts<br />

15. force = 333 newtons, work = 5,000 joules


<strong>Skill</strong> Sheet 7.3: Power in Flowing Energy<br />

1. Answers are given in table below: 2. Answers are:<br />

a. 1,800 J<br />

Force (N) Distance<br />

(m)<br />

Time<br />

(sec)<br />

<strong>Skill</strong> Sheet 7.3: Efficiency and Energy<br />

1. 55%<br />

2. 12%<br />

3. Answers are:<br />

a. 91%<br />

b. Energy is lost due to friction with the track (which creates<br />

heat), air resistance, and the sound made by the track and<br />

wheels.<br />

<strong>Skill</strong> Sheet 8.2: Measuring Temperature<br />

Stop and Think:<br />

a. 15°C<br />

b. 21°C<br />

c. 23.0°C, 30.0°C, 30.0°C, 31.5°C, 31.5°C<br />

Work (J) Power<br />

(W)<br />

100 2 5 200 40<br />

100 2 10 200 20<br />

100 4 10 400 40<br />

100 5 25 500 20<br />

50 20 20 1000 50<br />

20 30 10 600 60<br />

9 20 3 180 60<br />

3 25 15 75 5<br />

<strong>Skill</strong> Sheet 8.2: Temperature Scales<br />

1. Answers are:<br />

a. 100°C<br />

b. 37°C<br />

c. 4.4°C<br />

d. –12.2°C<br />

e. 32°F<br />

f. 77°F<br />

g. 167°F<br />

2. 7.2°C<br />

3. 177°C<br />

4. 107°C<br />

5. 374°F<br />

6. 450°F<br />

7. The table shows that the friend in Europe thinks that the<br />

temperature is on the Celsius scale because 15°C is equal to<br />

59°F, a relatively warm air temperature. However, 15°F is a<br />

relatively cold air temperature, equivalent to –9.4°C.<br />

°F °C<br />

15°F = –9.4°C<br />

59°F = 15°C<br />

<strong>Skill</strong> Sheet 8.2: Reading a Heating/Cooling Curve<br />

1. The iron changed from liquid to gas between points D and E.<br />

2. The heat added to the iron was used to break the<br />

intermolecular forces between the iron atoms.<br />

Page 24 of 57<br />

b. 450 watts<br />

3. Answers are:<br />

a. 60,000 J<br />

b. 2,000 W; 2.68 hp<br />

4. Answers are:<br />

a. 36,750 J (37,000 J with correct significant figures)<br />

b. 20.4 W (20 W with correct significant figures)<br />

5. 200 W<br />

6. Answers are:<br />

a. 5,000 seconds or 1.4 hours<br />

b. 8,640,000 J<br />

c. 17.3 apples<br />

7. Answers are:<br />

a. 98,000 J<br />

b. 98,000 W; 131 hp<br />

8. 3.3 W<br />

c. The first hill is the tallest because a roller coaster loses<br />

energy as it moves along the track. No roller coaster is<br />

100% efficient. Unless there is a motor to give it<br />

additional energy, it will never be able to make it back up<br />

to a height as high as the first hill.<br />

4. 80%<br />

5. 278 m<br />

d. At 0°C, water changes from liquid to solid. At 100°C,<br />

water changes from liquid to gas.<br />

e. The liquid will expand so the level in the tube will<br />

increase.<br />

Answers to the “Reading the temperature” sections will vary.<br />

8. Answers are:<br />

a. –283°F<br />

b. The melting point for this liquid is 35°F which is equal to<br />

1.7°C. The melting point for mercury is –38.9°C<br />

(–38.0°F). The unknown substance is not mercury, since<br />

its boiling point is not the same as that of mercury.<br />

Extension Answers:<br />

1. 184K to 242K<br />

2. –108°C<br />

3. –139°C<br />

4. –223°C<br />

5. 5,273K to 8,273K<br />

6. 10,273K<br />

7. 1,000,273K<br />

8. 15,000,273K<br />

9. 622K<br />

10. 900°F<br />

3. The melting temperature of iron is about 1,500°C.


4. The freezing temperature of iron is about 1,500°C. The<br />

melting and freezing temperatures of a substance are the<br />

same.<br />

5. The boiling temperature of iron is about 2,800°C.<br />

6. The boiling temperature of iron is about 2,700°C higher than<br />

the boiling temperature of water. That means it takes a lot<br />

more heat energy to break the intermolecular forces between<br />

iron atoms than those between water molecules. Iron’s<br />

intermolecular forces are much stronger than water’s.<br />

7. Freezing occurred between points B and C.<br />

8. The freezing and melting temperatures are the same—69°C.<br />

9. The melting temperature of stearic acid is higher than water’s<br />

melting temperature. The intermolecular forces between<br />

<strong>Skill</strong> Sheet 9.1: Specific Heat<br />

1. Gold would heat up the quickest because it has the lowest<br />

specific heat.<br />

2. Pure water is the best insulator because it has the highest<br />

specific heat.<br />

3. Silver is a better conductor of heat than wood because its<br />

specific heat is lower than that of wood.<br />

<strong>Skill</strong> Sheet 9.1: Using the Heat Equation<br />

1. 323 J<br />

2. 588 J<br />

3. 2,243 J<br />

4. The gold would cool down fastest. It has to release only 323 J<br />

of energy to return to its original temperature.<br />

<strong>Skill</strong> Sheet 9.2: Heat Transfer<br />

Definitions:<br />

Heat conduction: The transfer of heat by the direct contact of<br />

particles of matter.<br />

Convection: The transfer of heat by the motion of matter,<br />

such as by moving air or water.<br />

Thermal radiation: Heat transfer by electromagnetic waves,<br />

including light.<br />

1. Conduction. The water molecules collide with the frozen<br />

shrimp, transferring thermal energy by direct contact.<br />

2. Radiation and heat conduction. Heat from the Sun is radiated<br />

to Earth. The black asphalt absorbs more of this radiation than<br />

the light-colored sidewalk. Heat is transferred from the<br />

sidewalk and the asphalt to Juan’s feet by the process of heat<br />

conduction. Since the sidewalk absorbed more heat, it can<br />

transfer more heat to Juan’s feet.<br />

3. Convection. A thermal is a convection current in the<br />

atmosphere.<br />

4. Radiation and convection. the hot space heater emits thermal<br />

radiation, and convection currents distribute the heat<br />

throughout the room.<br />

Page 25 of 57<br />

stearic acid molecules are stronger than those between water<br />

molecules. That’s why it would take more heat energy to melt<br />

stearic acid.<br />

10. Yes, a substance can definitely be cooled below its freezing<br />

temperature. The ice in the first graph started at –20°C. The<br />

iron started 1,500°C below its freezing temperature, and the<br />

stearic acid continued to cool well below its freezing<br />

temperature. The molecules in a solid have some kinetic<br />

energy at their freezing temperatures. Their kinetic energy<br />

slowly decreases as they cool down further. Absolute zero<br />

(–273°C) is the point at which molecules have the minimum<br />

possible kinetic energy.<br />

4. Aluminum, because it has the higher specific heat.<br />

5. 5°C × 4,184J/kg °C = 20,920 J<br />

6. At the same temperature, the larger mass of water contains<br />

more thermal energy.<br />

5. 711,280 J<br />

6. 440,000 J<br />

7. 5.6 °C<br />

8. 4,393,200 J<br />

<strong>Skill</strong> Sheet 10.1: Measuring Mass With a Triple-Beam Balance<br />

Answers will vary.<br />

<strong>Skill</strong> Sheet 10.1: Measuring Volume<br />

Stop and Think:<br />

a. Sample student answer: The water level will rise so that it<br />

spills out of the spout.<br />

b. A fist-sized rock will displace more water because it is<br />

bigger and will sink (an acorn may float).<br />

collected. The volume of water that spills equals the volume of the object.<br />

5. Conduction. The mother duck is in direct contact with the<br />

eggs so the heat is transferred from her body directly to the<br />

eggs.<br />

6. Radiation. Thermal energy from the Sun is absorbed by the<br />

car.<br />

7. Conduction. The molecules of hot coffee collide with the<br />

molecules of cold milk. The average kinetic energy of the<br />

coffee molecules decreases, and the average kinetic energy of<br />

the milk molecules increases until thermal equilibrium is<br />

reached. The equilibrium temperature is lower than the initial<br />

temperature of the coffee.<br />

8. Convection. A sea breeze is a convection current in the<br />

atmosphere, created when air over the land is heated and<br />

rises. Then cool air from over the water rushes in to take its<br />

place, creating the sea breeze.<br />

9. Conduction. The heat from the water is transferred directly to<br />

the pipes, then to the marble floor, then to the feet.<br />

10. Conduction and convection. First the heat from the water is<br />

transferred to the pipes and then to the floor. Then convection<br />

currents circulate the heat from the floor to all parts of the<br />

room.<br />

c. The volume of spilled water equals the amount of water<br />

that is displaced by the object.<br />

d. Water is added to the displacement tank until it can hold<br />

no more without the water spilling out of the spout. Then<br />

an object is placed in the tank and the spilled water is


<strong>Skill</strong> Sheet 10.1: Calculating volume<br />

Stop and Think:<br />

a. cm 3 or centimeters cubed<br />

b. 64 in 3<br />

c. Area is calculated using two dimensions (length and<br />

width). Volume is calculated using three dimensions<br />

(length, width, and height).<br />

d. 64 cubes each 1 cm 3 would fit<br />

Calculating volume of a rectangular prism:<br />

1. 24 cm 2<br />

2. 48 cm 3<br />

3. 90 cm 3<br />

Calculating volume of a triangular prism:<br />

1. 3 cm 2<br />

2. 18 cm 3<br />

3. 100 cm 3<br />

Calculating volume of a cylinder:<br />

1. 28.3 cm 2<br />

2. 170 cm 3<br />

3. 402 cm 3<br />

Calculating volume of a cone:<br />

<strong>Skill</strong> Sheet 10.1: Density<br />

1. 1.10 g/cm3 2. 0.870 g/cm3 3. 2.7 g/cm3 4. 920,000 grams or 920 kilograms<br />

5. 2,420 grams or 2.42 kilograms<br />

6. 1,025 grams or 1.025 kilograms<br />

7. 1,200 cm3 8. 29.8 cm3 9. 11.4 mL<br />

<strong>Skill</strong> Sheet 10.3: Pressure in Fluids<br />

1. 50,000 Pa<br />

2. 157,000 N<br />

3. 0.0015 m2 4. 5 m2 5. If the area of the input piston is doubled, the pressure<br />

transmitted by the system is cut in half.<br />

6. If the area of the input piston is doubled (and no other<br />

variables are changed), the output force is cut in half.<br />

<strong>Skill</strong> Sheet 10.3: Boyle’s Law<br />

1. 3.25 atm<br />

2. 36 m 3<br />

3. 563 kPa<br />

4. 570 liters<br />

5. 25 liters<br />

<strong>Skill</strong> Sheet 10.4: Buoyancy<br />

1. Sink<br />

2. Float<br />

3. 0.12 N<br />

4. 0.10 N<br />

5. The light corn syrup has greater buoyant force than the<br />

vegetable oil.<br />

6. 0.13 N<br />

Page 26 of 57<br />

1. 78.5 cm 2<br />

2. 628 cm 3<br />

3. 134 cm 3 ; this volume is one-third the value of the volume of<br />

the cylinder with similar dimensions.<br />

Calculating volume of a rectangular pyramid:<br />

1. 20 cm 2<br />

2. 40 cm 3<br />

3. 66.7 cm 3<br />

4. About 6.25 cm<br />

Calculating volume of a triangular pyramid:<br />

1. 6 cm 2<br />

2. 14 cm 3<br />

3. 50 cm 3<br />

4. 12.6 cm<br />

Calculating volume of a sphere:<br />

1. 268 cm 3<br />

2. 33.5 cm 3<br />

3. 523 cm 3 (assume 3.14 for pi)<br />

10. Answers are:<br />

a. density = 960. kg/m 3 , HDPE<br />

b. 76,000 grams or 76 kilograms<br />

c. The volume needed is 0.11 m 3 ; 11 10-liter containers would<br />

be needed to hold the plastic<br />

d. HDPE, LDPE, PP (PS would probably be suspended in<br />

seawater)<br />

7. No, the output distance must be less than the input distance.<br />

Output work (output force × output distance) can never be<br />

greater than input work (input force × input distance).<br />

8. The woman’s force (540 N) remains constant whether she’s<br />

wearing high heels or snowshoes. But the area over which the<br />

force is applied is much greater with snowshoes than high<br />

heels. Since pressure = force ÷ area, the pressure applied to<br />

the floor by high heeled shoes is much greater than the<br />

pressure applied to the snow by the snowshoes.<br />

7. The buoyant force would be smaller if the gold cube were<br />

suspended in water. Student explanations may vary. A simple<br />

observation, such as “The water is thinner than the molasses”<br />

is acceptable, as well as the more sophisticated “The<br />

displaced water would weigh less than the displaced<br />

molasses” or “The water is less dense than the molasses.”


<strong>Skill</strong> Sheet 10.4: Charles’s Law<br />

1. 25.4 liters<br />

2. 22.8 liters<br />

<strong>Skill</strong> Sheet 13.2: Pressure-Temperature Relationship<br />

1. 0.27 atmospheres<br />

2. 1,000 K<br />

<strong>Skill</strong> Sheet 10.4: Archimedes<br />

1. Density: a property that describes the relationship between a<br />

material’s mass and volume. Buoyancy: A measure of the<br />

upward force a fluid exerts on an object.<br />

2. Sample answer: I, Archimedes, have a wide variety of skills<br />

to offer. First, I am an inventor of problem-solving devices,<br />

including a device for transporting water upward. I have also<br />

worked as a crime scene investigator for the king, uncovering<br />

fraud through scientific testing of materials. Furthermore, I<br />

am a writer with several treatises already published. I also<br />

have advanced skills in mathematics and can even estimate<br />

for you the number of grains of sand needed to fill the entire<br />

universe.<br />

3. In the treatise entitled “The Sand Reckoner,” Archimedes<br />

devised a system of exponents that allowed him to represent<br />

large numbers on paper—up to 8 × 10 63 in modern scientific<br />

notation. This was large enough, he said, to count the grains<br />

<strong>Skill</strong> Sheet 10.4: Narcís Monturiol<br />

1. Monturiol was motivated by the suffering of the coral divers<br />

he saw along the coast of Spain. The divers performed<br />

dangerous work to retrieve pieces of coral. They could drown,<br />

hurt themselves on rocks and coral, or be attacked by sharks.<br />

A submarine would provide a safer means of gathering coral.<br />

2. Ictineo I relied on human power to turn the propellers. It had a<br />

spherical inner chamber to withstand water pressure and an<br />

egg-shaped outer chamber for ease of movement. It also had a<br />

ventilator, two sets of propellers, and several portholes.<br />

Monturiol even created a backup system to ensure that the<br />

submarine could be raised to the surface in case of<br />

emergency. Ictineo I could dive to 20 meters and stayed<br />

underwater for nearly two hours.<br />

Ictineo II used steam power instead of human power to turn<br />

the propellers. Monturiol developed a chemical reaction to<br />

power the engine. This reaction also added oxygen to the<br />

<strong>Skill</strong> Sheet 10.4: Archimedes’ Principle<br />

1. If they are both submerged, then they both displace the same<br />

amount of water and have the same buoyant force acting on<br />

them.<br />

Page 27 of 57<br />

3. Answers are:<br />

V1 T1 V2 T2 a. 6,114 mL 838 K 1,070 mL 147 K<br />

b. 3,250 mL 475°C 1,403 mL 50°C<br />

(748 K)<br />

(323 K)<br />

c. 10 L – 58°C 15 L 50°C<br />

(215 K)<br />

(323 K)<br />

3. Answers are:<br />

P1 T1 P2 T2 a. 30.0 atm –100 °C 134 atm 500 °C<br />

(173 K)<br />

(773 K)<br />

b. 15.0 atm 25.0 °C 18.0 atm 85 °C<br />

(298 K)<br />

(360 K)<br />

c. 5.00 atm 488 K 3.00 atm 293 K<br />

of sand that would be needed to fill the universe. His<br />

assessment of the universe’s size was an underestimate, but<br />

he was the first to think of the universe being so large.<br />

4. A helium balloon floats in air because the air it displaces<br />

weighs more than the filled balloon. A balloon filled with air<br />

from someone’s lungs sinks because the combined weight of<br />

the latex balloon and the air inside is greater than the weight<br />

of the air it displaces.<br />

5. Inventions attributed to Archimedes include war machines<br />

(such as a lever used to turn enemy boats upside down), the<br />

Archimedes screw, compound pulley systems, and a<br />

planetarium. There is some debate about whether he invented<br />

a water organ and a system of mirrors and/or lenses to focus<br />

intense, burning light on enemy ships. Students can use the<br />

Internet or library to find more information and diagrams of<br />

the inventions.<br />

submarine. Ictineo II was longer than Ictineo I, had two<br />

engines, dove to 30 meters, and remained underwater for<br />

nearly seven hours.<br />

3. A replica of Ictineo I is located at the Marine Museum in<br />

Barcelona. A replica of Ictineo II can be found at Barcelona<br />

harbor.<br />

4. In 1862, Ictineo I was destroyed by a freight ship while<br />

anchored in Barcelona Harbor.<br />

5. Monturiol developed the following: A process to speed up the<br />

manufacturing of adhesive paper, a machine to copy letters, a<br />

stone cutter, and a meat preservative.<br />

6. Spain created a postage stamp in Monturiol’s honor.<br />

7. The Narcís Monturiol medal is an award given for<br />

“distinction in science and technology and for contributions<br />

to the scientific development of Catalonia.”<br />

2. Answers are:<br />

a. 100 cm 3<br />

b. 0.98 N<br />

c. 0.98 N<br />

d. sink


3. Answers are:<br />

a. 100 cm 3<br />

b. 13 N<br />

c. 13 N<br />

d. float<br />

4. In both cases, a material sinks in a fluid if it is more dense<br />

than the fluid. A material floats in a fluid if it is less dense<br />

than the fluid.<br />

<strong>Skill</strong> Sheet 11.1: Layers of the Atmosphere<br />

<strong>Skill</strong> Sheet 11.2: Gustave-Gaspard Coriolis<br />

1. Coriolis attended one of the best-known engineering schools<br />

in France. His exceptional ability coupled with great<br />

schooling provided him with a solid foundation for his<br />

thoughts, research, and studies.<br />

2. His first <strong>book</strong> presented mechanics in a way that could easily<br />

be applied. It was the foundation and establishment of<br />

applying the concept of work to the field of mechanics.<br />

Coriolis was intent on using and applying proper terms. He<br />

was the first to derive formulas expressing kinetic energy and<br />

mechanical work.<br />

3. As you are flying overhead, Earth is rotating from west to east<br />

beneath you. By the time you are ready to land, Earth has<br />

rotated far enough that Little Rock is east of your current<br />

position.<br />

4. In the northern hemisphere, the Coriolis effect bends winds to<br />

the right. In the southern hemisphere it bends winds to the<br />

left.<br />

5. Answers will vary, but should include the following:<br />

Trade winds are surface wind currents that move between 30<br />

degrees North latitude and the equator. The Coriolis effect<br />

bends the trade winds moving across the surface so they flow<br />

from northeast to southwest in the northern hemisphere and<br />

from southwest to northeast in the southern hemisphere.<br />

<strong>Skill</strong> Sheet 11.2: Degree Days<br />

Part 1 answers:<br />

1. Cooling degree day value = 88 – 65 = 23.<br />

2. Heating degree day value = 65 – 14 = 51.<br />

Page 28 of 57<br />

5. Answers are:<br />

a. Floats<br />

b. Sinks<br />

c. Sinks<br />

d. Sinks<br />

Layer Distance from Earth’s surface Thickness Facts<br />

Troposphere 0–11 km 11 km Most of Earth’s water vapor, carbon dioxide, dust,<br />

airborne pollutants, and terrestrial life forms are<br />

found in this layer.<br />

The temperature drops as you go higher into the<br />

troposphere.<br />

Stratosphere 11–50 km 39 km The ozone layer is located here.<br />

The temperature increases as you go higher into<br />

the stratosphere.<br />

Mesosphere 50–80 km 30 km “Shooting stars” occur when meteors burn up in<br />

this layer.<br />

The mesosphere is the coldest layer of the<br />

atmosphere.<br />

Thermosphere 80—approx. 500 km 420 km Very low density of air molecules in this layer.<br />

Very high temperatures because sun’s rays hit here<br />

first.<br />

Exosphere 500 km—no specific<br />

outer limit<br />

undefined Lightweight atoms and molecules escape into<br />

space.<br />

Many man-made satellites orbit in this region,<br />

about 36,000 km above the equator.<br />

Polar easterlies form when the air over the poles cools and<br />

sinks, and spreads along the surface to about the 60 degree<br />

latitude. The polar wind is bent by the Coriolis effect and the<br />

air flows from northeast to southwest in the northern<br />

hemisphere, and from southeast to northwest in the southern<br />

hemisphere. Bands of cold air move away from the poles.<br />

Prevailing westerlies are created when air bends to the right<br />

due to the Coriolis effect. These winds blow towards the poles<br />

from the west and are bent to the right in the northern<br />

hemisphere and to the left in the southern hemisphere.<br />

6. His work was not accepted outside the field of mechanics<br />

until 1859 when the French Academy of Science arranged for<br />

a discussion on Earth’s rotation and its effects on water<br />

currents. His work on Earth’s rotation was discussed and<br />

linked to the field of meteorology in the late 1800’s early<br />

1900’s.<br />

7. Coriolis was able to connect theory with application in each<br />

of these <strong>book</strong>s. The billiard <strong>book</strong> was published in 1835 and<br />

provided the mathematical theory of spin, friction, and<br />

collision in the game of billiards. Coriolis discussed how<br />

physics determines and explains the game of billiards. The<br />

Treatise <strong>book</strong> was published after his death in 1944.<br />

3. On July 22, 2002 the heating degree day value was zero. On<br />

January 22, 2003 the cooling degree day value was zero.


Part 2 answers:<br />

1. Answers:<br />

Day <strong>High</strong> Low Average Heating Cooling<br />

temp temp temp 2 degree day degree day<br />

1 73 61 67 0 2<br />

2 63 52 58 7 0<br />

3 70 44 57 8 0<br />

4 65 52 59 6 0<br />

5 83 58 71 0 6<br />

6 79 59 69 0 4<br />

7 74 60 67 0 2<br />

8 71 53 62 3 0<br />

9 90 70 80 0 15<br />

10 82 62 72 0 7<br />

11 65 52 59 6 0<br />

12 71 52 62 3 0<br />

13 74 56 65 0 0<br />

14 75 60 68 0 3<br />

Two week totals: 33 39<br />

2. St. Louis residents were more likely to use heating systems on<br />

six days and more likely to cool their homes on seven days.<br />

However, many of these days had such small degree day<br />

values that residents may have used either system only rarely.<br />

The most likely day to use energy for heating or cooling was<br />

May 9, with a cooling degree day value of 15.<br />

Part 3 answers:<br />

1. Total heating degree day value: 33<br />

2. Total cooling degree day value: 39<br />

3. The monthly total heating degree day value for May 2003 was<br />

33+31, or 64. The monthly total cooling degree day value was<br />

39+32, or 71.<br />

4. May 2003’s total heating degree day value was 15 less than<br />

normal, and its total cooling degree day value was 43 less<br />

than normal. With average temperatures closer to 65°F, St.<br />

Louis residents probably used less energy for heating and<br />

cooling in May than is usually needed.<br />

<strong>Skill</strong> Sheet 11.3: Joanne Simpson<br />

1. Simpson is the first woman to earn a Ph.D. in meteorology,<br />

the first person to create a computerized cloud model, and the<br />

first woman to have served as president of the American<br />

Meteorological Society.<br />

2. Simpson faced discrimination based on the fact that she was a<br />

woman. At the end of the war, most women returned home after<br />

temporarily filling the roles of men away during the war.<br />

Simpson was not one of those women. She continued on with<br />

her studies after teaching meteorology to aviation cadets. She<br />

earned a master’s degree and was so interested in meteorology<br />

that she wanted to go on for a Ph.D. Her advisor and the all-male<br />

faculty at her university did not support a woman going on for<br />

an advanced degree. They felt that women were unable to do the<br />

work which included shifts and flying planes. Simpson did<br />

finally find an advisor to support her Ph.D., but even he had<br />

negative comments about her topic. Simpson did have difficulty<br />

finding a job, but eventually landed a position as an assistant<br />

professor of physics. She continued to move into numerous<br />

positions and did not let others’ opinions and comments stop her<br />

career pursuits.<br />

3. Students should comment on Simpson’s determination<br />

despite the obstacles she faced along her journey to complete<br />

her degree and to work in the field of meteorology. Students<br />

will understand that if you really want to achieve a goal, you<br />

need to stay focused, work hard, and not be discouraged by<br />

negative opinions along the way.<br />

Page 29 of 57<br />

Part 4 answers:<br />

1. Graph:<br />

2. January<br />

3. July<br />

4. Answers are:<br />

a. I chose May because the total heating and cooling degree<br />

day values are significantly less than any other month, and<br />

because many days with low degree day values probably<br />

won’t require any actual heating or cooling.<br />

b. You would need to know how much energy it takes to run<br />

each system per hour, the type of fuel used to run it, and<br />

the cost of the fuel. Natural gas and heating oil are<br />

commonly used for home heating systems, while<br />

electricity (most commonly generated by coal or natural<br />

gas) is commonly used for air conditioning systems.<br />

Electricity is often more expensive than natural gas or<br />

heating oil. As a result, cooling a home is often more<br />

expensive than heating it.<br />

4. A slide rule is a mechanical tool used to calculate complicated<br />

mathematical problems involving multiplication, divisions,<br />

square roots, cube roots, and trigonometry. The invention of<br />

the slide rule dates back to the 16th century. The slide rule, a<br />

handheld tool, was used commonly in science and<br />

engineering. The scientific calculator and computers made<br />

the slide rule obsolete.<br />

5. This is the highest award given by the American<br />

Meteorological Society for atmospheric science including<br />

meteorology, climatology, atmospheric physics, and<br />

atmospheric chemistry. It is named after Carl-Gustaf Rossby,<br />

a leader in the fields of oceanography and meteorology. He<br />

was also the 2nd recipient of this award.<br />

6. Woods Hole Oceanographic Institute (WHOI) is located in<br />

Woods Hole, Massachusetts on Cape Cod. Scientists at<br />

WHOI study oceans, their function, and their interaction with<br />

the Earth. WHOI provides opportunities for research and<br />

higher education. Students can visit the WHOI website on the<br />

Internet for more information.<br />

7. Students can locate hot tower clouds in scientific articles or<br />

websites. Hot towers are commonly associated with<br />

hurricanes. The NASA website provides some specific<br />

information about hot towers.


<strong>Skill</strong> Sheet 11.3: Weather Maps<br />

Sample table with answers:<br />

City <strong>High</strong> Low Temp difference Sky cover Pressure<br />

Seattle 72 54 18 Partly<br />

cloudy<br />

<strong>High</strong><br />

Los Angeles 85 68 17 Sunny <strong>High</strong><br />

Las Vegas 108 81 27 Pcldy <strong>High</strong><br />

Phoenix 108 86 22 Pcldy <strong>High</strong><br />

Atlanta 89 66 23 Pcldy Low<br />

Tampa 88 74 14 T-storms Low<br />

San Francisco 76 56 20 Sunny Low<br />

Oklahoma City 101 74 27 Pcldy <strong>High</strong><br />

New Orleans 94 76 18 T-storms Low<br />

Kansas City 91 70 21 Pcldy Low<br />

Tucson 103 76 27 Pcldy <strong>High</strong><br />

Denver 94 62 32 Pcldy Low<br />

Dallas 105 78 27 Sunny <strong>High</strong><br />

Houston 98 76 22 Pcldy <strong>High</strong><br />

Minneapolis 86 64 22 Sunny <strong>High</strong><br />

Memphis 94 73 21 Sunny <strong>High</strong><br />

Chicago 85 66 19 T-storms Low<br />

Miami 93 75 18 T-storms Low<br />

New York 73 63 20 T-storms Low<br />

Baltimore 78 70 8 T-storms Low<br />

1. The highest temperatures (daily high over 95°F) are in the<br />

lower latitudes such as Las Vegas, Phoenix, Tucson, Dallas,<br />

and Houston. The coolest temperatures (daily high under<br />

82°F) are found in Seattle, Chicago, and New York. These<br />

cities are at higher latitudes. The Sun is more directly<br />

overhead in lower latitude regions, and it is lower on the<br />

horizon and therefore less intense at midday in the higher<br />

latitude regions.<br />

<strong>Skill</strong> Sheet 11.3: Tracking a Hurricane<br />

Map answers:<br />

Part 2 answers:<br />

2. Students may mention Florida or Cuba as likely hurricane<br />

watch areas. The Bahamas should be mentioned as a hurricane<br />

Page 30 of 57<br />

2. Even though Los Angeles is the southern part of the<br />

continent, its high temperature was only 85°F. This is due to<br />

the cooling effect of the Pacific Ocean. Chicago is farther<br />

south than Minneapolis, but it is cooler because it sits on the<br />

shore of Lake Michigan. Denver, with its Rocky Mountain<br />

location, cools down to a nighttime low of 62°F in the<br />

summer.<br />

3. On the map, the thickest cloud cover is in the Northeast,<br />

where it cools Baltimore and New York. Baltimore and<br />

Kansas City are near the same latitude, but Kansas City (with<br />

less cloud cover) was 13°F warmer. During the day, the<br />

clouds reflect some of the sun’s heat away.<br />

4. Sample answers: The high-pressure regions center on<br />

Oregon, New Mexico, and Quebec. The low-pressure regions<br />

center on Wyoming, North Dakota, and North Carolina.<br />

5. See the table for answers.<br />

6. <strong>High</strong> pressure regions tend to have sunny weather, since less<br />

air is rising, cooling, and condensing. Humidity is much<br />

lower in these regions.<br />

7. Low-pressure regions tend to be overcast and/or stormy. The<br />

humidity is higher.<br />

8. Fronts are associated with low pressure regions. Fronts tend<br />

to bring precipitation.<br />

9. Cold fronts are associated with stormy areas. Warm fronts<br />

tend to be accompanied by bands of light precipitation.<br />

10. The air in a low-pressure region rises. The air in a high<br />

pressure region sinks.<br />

11. In a low pressure region, warm, moist air can be carried<br />

upward by convection. As the air cools, water condenses into<br />

clouds and precipitation.<br />

12. A low-pressure region is a good place for a volume of air to<br />

reach the dew point temperature because the warm, moist air<br />

in this region rises. As this air rises, it cools to the dew point<br />

temperature. The result is that the water in the air mass<br />

condenses, clouds form, and eventually precipitation occurs.<br />

warning area. Here are the actual watches and warnings issued<br />

by the Tropical Prediction Center for this time period:<br />

Date Time (GMT) Action Region<br />

8/22/1992 1500 Hurricane watch Northwest Bahamas<br />

8/22/1992 2100 Hurricane warning Northwest Bahamas<br />

8/22/1992 2100 Hurricane watch Florida east coast from<br />

Titusville through the<br />

Florida keys<br />

8/23/1992 0600 Hurricane warning Central Bahamas<br />

8/23/1992 1200 Hurricane warning Florida east coast from<br />

Vero Beach southward<br />

through the Florida keys<br />

8/23/1992 1200 Hurricane watch Florida west coast south<br />

of Bayport including<br />

greater Tampa area to<br />

north of Flamingo<br />

Parts 3, 4, and 5 answers:<br />

3.2 The Bahama islands. Note: The National Hurricane Center<br />

reported that landfall occurred at the northern Eleuthera<br />

Island, Bahamas.<br />

4.2 Southern Florida. Note: The National Hurricane Center<br />

reported that landfall occurred at Homestead Air Force Base,<br />

Florida.<br />

5.2 Louisiana. Note: The National Hurricane Center reported that<br />

landfall occurred at Point Chevreuil, Louisiana.


<strong>Skill</strong> Sheet 12.1: Structure of the Atom<br />

1. Answers are: 2. Answers are:<br />

a. hydrogen-2: 1 proton, 1 neutron<br />

What is this element? How many electrons does<br />

the neutral atom have?<br />

<strong>Skill</strong> Sheet 12.1: Atoms and Isotopes<br />

Part 1 answers:<br />

1. protium has 0 neutrons; deuterium has 1 neutron; tritium has<br />

2 neutrons<br />

2. Answers are:<br />

a. 3<br />

b. Lithium<br />

c. 7<br />

d.<br />

What is the mass<br />

number?<br />

lithium 3 7<br />

carbon 6 12<br />

hydrogen 1 1<br />

hydrogen<br />

(a radioactive isotope,<br />

3H, called tritium)<br />

1 3<br />

beryllium 4 9<br />

Li<br />

7<br />

3<br />

<strong>Skill</strong> Sheet 12.1: Ernest Rutherford<br />

1. Alpha particle: a particle that has two protons and two<br />

neutrons (also known as a helium nucleus). Beta particle: An<br />

electron emitted by an atom when a neutron splits into a<br />

proton and an electron.<br />

2. For one atom to turn into another kind of atom, the number of<br />

protons in the nucleus must change. This can happen when an<br />

alpha particle is ejected (two protons are lost then) or when a<br />

neutron splits into a proton and an electron (in that case the<br />

number of protons increases by one).<br />

3. Diagram:<br />

<strong>Skill</strong> Sheet 12.2: Electrons and Energy Levels<br />

1. Danish physicist Neils Bohr<br />

2. Energy levels can be thought of as similar to steps on a<br />

staircase. Electrons can exist only in one energy level or<br />

another and cannot remain between energy levels, just as a<br />

Page 31 of 57<br />

b. scandium-45: 21 protons, 24 neutrons<br />

c. aluminum-27: 13 protons, 14 neutrons<br />

d. uranium-235: 92 protons, 143 neutrons<br />

e. carbon-12: 6 protons, 6 neutrons<br />

3. Most of an atom’s mass is concentrated in the nucleus. The<br />

number of electrons and protons is the same but electrons are<br />

so light they contribute very little mass. The mass of the<br />

proton is 1,835 times the mass of the electron. Neutrons have<br />

a bit more mass than protons, but the two are so close in size<br />

that we usually assume their masses are the same.<br />

4. Yes, it has a proton (+1) and no electrons to balance charge.<br />

Therefore, the overall charge of this atom (now called an ion)<br />

is +1.<br />

5. This sodium atom has 10 electrons, 11 protons, and<br />

12 neutrons.<br />

Part 2 answers:<br />

1. Bromine-80<br />

2. Potassium-39 has 20 neutrons.<br />

3. Lithium-7<br />

4. Neon-20 has 10 neutrons.<br />

4. Rutherford’s planetary model suggested that an atom consists<br />

of a tiny nucleus surrounded by a lot of empty space in which<br />

electrons orbit in fixed paths. Subsequent research has shown<br />

that electrons don’t exist in fixed orbitals. The Heisenberg<br />

uncertainty principle tells us that it is impossible to know both<br />

an electron’s position and its momentum at the same time.<br />

Scientists now discuss the probability that an electron will<br />

exist in a certain position. Computer models predict where an<br />

electron is most likely to exist, and three-dimensional shapes<br />

can be drawn to show the most likely positions. The sum of<br />

these shapes produces the charge-cloud model of the electron.<br />

5. In the game of marbles, players “shoot” one marble at a group<br />

of marbles and then watch the deflection as collisions occur.<br />

This is a lot like what Rutherford was doing on a much, much<br />

smaller scale. Rutherford’s comment is reflective of his<br />

typical self-deprecating humor. While “playing with<br />

marbles,” he discovered the proton.<br />

6. Answers will vary. Students may wish to write about one of<br />

the following discoveries: Rutherford first described two<br />

different kinds of particles emitted from radioactive atoms,<br />

calling them alpha and beta particles. He also proved that<br />

radioactive decay is possible. He developed the planetary<br />

model of the atom, and was the first to split an atom.<br />

person can be on one step or another but not between steps<br />

except in passing.<br />

3. Energy levels are filled from the lowest (innermost) energy<br />

level outward.<br />

4. answers are:


a. b. c.<br />

d. e f.<br />

<strong>Skill</strong> Sheet 12.2: Neils Bohr<br />

1. Both Rutherford and Bohr described atoms as having a tiny<br />

dense core (the nucleus) surrounded by electrons in orbit.<br />

Bohr described the nature of the electrons’ orbits in much<br />

greater detail.<br />

2. Niels Bohr described<br />

electrons as existing in<br />

specific orbital pathways,<br />

and explained how atoms<br />

emit light.<br />

3. In Bohr’s model of the<br />

atom, the electrons are in<br />

different energy levels.<br />

Bohr’s model of the atom<br />

at right:<br />

4. An electron absorbs<br />

energy as it jumps from an<br />

inner orbit to an outer one.<br />

When the electron falls<br />

<strong>Skill</strong> Sheet 12.3: The Periodic Table<br />

1. Fluorine<br />

2. Argon<br />

3. Manganese<br />

4. Phosphorous<br />

5. Technetium<br />

6. The atomic number tells the number of protons in an atom of<br />

the element.<br />

7. Iron, 55.8 amu<br />

8. Cesium, 132.9 amu<br />

9. Silicon, 28.1 amu<br />

10. Sodium, 23.0 amu<br />

11. Bismuth, 209.0 amu<br />

12. The atomic mass tells the average mass of all known isotopes<br />

of an element, expressed in amu.<br />

13. The atomic mass isn’t always a whole number because it is an<br />

average mass of all known isotopes.<br />

14. The mass of an electron is too small to be significant.<br />

15. Alkali metals<br />

Page 32 of 57<br />

back to the inner orbit, it releases the absorbed energy in the<br />

form of visible light.<br />

5. Answers will vary. You may wish to ask students to research<br />

world events from the end of World War II to Bohr’s death in<br />

1962. Students should look for events that may have raised<br />

concerns in Bohr’s mind about the potential use/misuse of<br />

nuclear weapons. They might also choose to research Bohr’s<br />

own comments on the subject.<br />

16. Any two of the following:<br />

soft, silvery, highly reactive, combines in 2:1 ratio with<br />

oxygen<br />

17. Any three of the following:<br />

F, Cl, Br, I, At<br />

18. They are toxic gases or liquids in pure form, highly reactive,<br />

and form salts with alkali metals.<br />

19. In the far right column<br />

20. They rarely form chemical bonds with other atoms.<br />

21. See figure 12.20, student text.<br />

22. as above<br />

23. as above<br />

24. as above<br />

25. as above<br />

26. Hydrogen<br />

27. Fluorine<br />

28. Carbon<br />

29. Sodium<br />

30. Chlorine


<strong>Skill</strong> Sheet 13.1: Dot Diagrams<br />

Part 1 answers: Part 2 answers:<br />

Element Chemical Total No. of Valence Dot Diagram Elements Dot Diagram for Each Dot Diagram for<br />

Symbol Electrons Electrons<br />

Element Compound Formed<br />

Potassium K 19 1<br />

Chemical<br />

Formula<br />

Na and F NaF<br />

Nitrogen N 7 5<br />

Carbon C 6 4<br />

Beryllium Be 4 2<br />

<strong>Skill</strong> Sheet 13.2: Finding the Least Common Multiple<br />

1. 21<br />

2. 24<br />

3. 45<br />

4. 50<br />

5. 80<br />

Neon Ne 10 8<br />

Sulfur S 16 6<br />

6. 147<br />

7. 108<br />

8. 315<br />

9. 880<br />

10. 192<br />

<strong>Skill</strong> Sheet 13.2: Chemical Formulas<br />

Answers:<br />

Element Oxidation No. Element Oxidation No. Chemical Formula for Compound<br />

Potassium (K) 1+ Chlorine (Cl) 1– KCl<br />

<strong>Skill</strong> Sheet 13.2: Naming Compounds<br />

Page 33 of 57<br />

Br and Br Br 2<br />

Mg and O MgO<br />

Calcium (Ca) 2+ Chlorine (Cl) 1– CaCl2 Sodium (Na) 1+ Oxygen (O) 2– Na2O Boron (B) 3+ Phosphorus (P) 3– BP<br />

Lithium (Li) 1+ Sulfur (S) 2– Li2S Aluminum (Al) 3+ Oxygen (O) 2– Al 2 O 3<br />

Beryllium (Be) 2+ Iodine (I) 1– BeI 2<br />

Calcium (Ca) 2+ Nitrogen (N) 3– Ca3N2 Sodium (Na) 1+ Bromine (Br) 1– NaBr<br />

Combination Compound Name Compound Name Chemical Family<br />

Al + Br aluminum bromide 1–<br />

Si + C2H3O2 silicon acetate<br />

Be + O beryllium oxide Lipase enzymes<br />

K + N potassium nitride Methanol alcohols<br />

2–<br />

Ba+CrO4 barium chromate Formic Acid organic acids<br />

Cs + F cesium fluoride Butane alkanes<br />

1+<br />

NH3 +S ammonium sulfide Sucrose sugars<br />

Mg + Cl magnesium chloride Acetone ketones<br />

B + I boron iodide Acetic Acid organic acids<br />

2–<br />

Na+SO4 sodium sulfate Compound Name Chemical Family


<strong>Skill</strong> Sheet 14.1: Chemical Equations<br />

Part 1 answers: Part 2 answers:<br />

Reactants<br />

Hydrochloric acid<br />

HCl and<br />

Sodium hydroxide<br />

NaOH<br />

Calcium carbonate<br />

CaCO3 and<br />

Potassium iodide<br />

KI<br />

Aluminum<br />

fluoride<br />

AlF3 and<br />

Magnesium nitrate<br />

Mg(NO3) 2<br />

Products<br />

Water<br />

H2O and<br />

Sodium chloride<br />

NaCl<br />

Potassium<br />

carbonate<br />

K2CO3 Calcium iodide<br />

CaI2 Aluminum nitrate<br />

Al(NO3 ) 3<br />

and Magnesium<br />

fluoride<br />

MgF2 Unbalanced Chemical Equation<br />

HCl + NaOH → NaCl + H20 CaCO3 + KI → K2CO3 + CaI2 AlF3 + Mg(NO3 ) 2 → Al(NO3 ) 3<br />

+ MgF2 1. 4Al + 3O2 → 2Al2O3 2. CO + 3H2 → H2O + CH4 3. 2HgO → 2Hg + O2 4. CaCO3 → CaO + CO2 5. 3C + 2Fe2O3 → 4Fe + 3CO2 6. N2 + 3H2 → 2NH3 7. 2K+ 2H2O → 2KOH + H2 8. 4P + 5O2 → 2P2O5 9. Ba(OH) 2 + H2SO4 → 2H2O + BaSO4 10. CaF2 + H2SO4 → CaSO4 + 2HF<br />

11. 4KClO3 → 3KClO4 + KCl<br />

<strong>Skill</strong> Sheet 14.1: The Avogadro Number<br />

Substance Elements in<br />

substance<br />

Atomic<br />

masses of<br />

elements<br />

(amu)<br />

<strong>Skill</strong> Sheet 14.1: Formula Mass<br />

Part 1 answers:<br />

1. First ion: Ca +2 ; Second ion: PO 4 3–<br />

2. Ca 3 (PO 4 ) 2<br />

3.<br />

No. of<br />

atoms of<br />

each<br />

element<br />

<strong>Skill</strong> Sheet 14.2: Classifying Reactions<br />

Formula<br />

mass<br />

(amu)<br />

Molar<br />

mass<br />

(g)<br />

Sr Sr 87.6 1 87.6 87.6<br />

Ne Ne 20.2 1 20.2 20.2<br />

Ca(OH) 2 Ca, O, H Ca, 40.1<br />

0, 16.0<br />

1<br />

2<br />

74.1 74.1<br />

H, 1.01 2<br />

NaCl Na, Cl Na, 23.0 1 58.5 58.5<br />

Cl, 35.5 1<br />

O3 O 16.0 3 48.0 48.0<br />

C6H12O C, H, O C, 12.0<br />

H, 1.01<br />

6<br />

12<br />

100 100<br />

O; 16.0 1<br />

Atom Quantity Atomic mass<br />

Total mass<br />

(from periodic table) (number × atomic mass)<br />

Ca 3 40.08 120.24<br />

P 2 30.97 61.94<br />

O 8 16.00 128.00<br />

1. Synthesis. Two substances combine to make a new compound.<br />

2. Single displacement. Chlorine replaces potassium in the<br />

compound.<br />

3. Decomposition. A single compound is broken into two<br />

substances.<br />

4. Decomposition. A single compound is broken into two<br />

substances.<br />

5. Combustion. A carbon compound reacts with oxygen to<br />

produce carbon dioxide and water.<br />

6. Double displacement. Two solutions are mixed and a solid is<br />

one of the products.<br />

7. Double displacement. Two solutions are mixed and a solid is<br />

one of the products.<br />

8. Single displacement. Sodium replaces calcium in the compound.<br />

Page 34 of 57<br />

Substance Molar mass<br />

(g)<br />

Mass of sample<br />

(g)<br />

Number of particles<br />

present<br />

MgCO3 84.32 12.75 9.10 × 1022 H2O 18.02 8.85 × 1029 296 × 1050 N2 28.02 3.30 × 10-14 7.1 × 108 Yb 173.04 0.00038 1.3 × 10 18<br />

Al2 (SO3 ) 3 294.17 4657 9.53 × 1024 K2CrO4 194.20 0.00074 0.23 × 10 19<br />

4. The formula mass for Ca3 (PO4 ) 2 : 120.24 g + 61.94 g +<br />

128.00 g = 310.18 g<br />

Part 2 answers:<br />

1.<br />

2.<br />

3.<br />

4.<br />

BaCl2 NaHCO3 Mg(OH) 2<br />

NH4NO3 formula mass:<br />

formula mass:<br />

formula mass:<br />

formula mass:<br />

208.23<br />

84.01<br />

58.33<br />

80.06<br />

5. Sr3(PO4) 2 formula mass: 452.80<br />

9. Decomposition. A single compound is broken into two<br />

substances.<br />

10. Combustion. A carbon compound reacts with oxygen to<br />

produce carbon dioxide and water.<br />

11. This is a double displacement reaction because a solid forms<br />

from two solutions.<br />

12. A combustion reaction.<br />

13. Synthesis reaction because two substances form a single<br />

compound.<br />

14. The reaction is similar to a combustion reaction because a<br />

substance (hydrogen) reacts with oxygen and energy is<br />

produced. So is water. It is different because carbon is not<br />

involved in the reaction.


<strong>Skill</strong> Sheet 14.2: Predicting Chemical Equations<br />

1. Ca<br />

2. K<br />

3. Al<br />

4. Al + LiCl<br />

5. Ca + K 2 O<br />

<strong>Skill</strong> Sheet 14.2: Percent Yield<br />

1. 110.98 g<br />

2. 87.9%<br />

3. 104.3 g<br />

4. 63.55 g<br />

5. 88.0%<br />

6. 61.0 g<br />

7. 119.06 g<br />

<strong>Skill</strong> Sheet 14.4: Lise Meitner<br />

1. Ludwig Boltzmann was a pioneer of statistical mechanics. He<br />

used probability to describe how properties of atoms (like<br />

mass, charge, and structure) determine visible properties of<br />

matter (like viscosity and thermal conductivity).<br />

2. They discovered protactinium. Its atomic number is 91 and<br />

atomic mass is 231.03588. It has 20 isotopes. All are<br />

radioactive.<br />

3. The graphic at right<br />

illustrates fission<br />

(n = a neutron):<br />

4. Some topics students<br />

may research and<br />

describe include<br />

nuclear power<br />

plants, nuclear<br />

weapons, nuclear-<br />

<strong>Skill</strong> Sheet 14.4: Marie and Pierre Curie<br />

1. Sample answer: Marie (or Marya, as she was called) had a<br />

strong desire to learn and had completed all of the schooling<br />

available to young women in Poland. She was part of an<br />

illegal “underground university” that helped young women<br />

prepare for higher education. Perhaps her own thirst for<br />

knowledge fueled her empathy for the peasant children, who<br />

were also denied the right to an education.<br />

2. Marie Curie proposed that uranium rays were an intrinsic part<br />

of uranium atoms, which encouraged physicists to explore the<br />

possibility that atoms might have an internal structure.<br />

3. Marie and Pierre worked with uranium ores, separating them<br />

into individual chemicals. They discovered two substances<br />

that increased the conductivity of the air. They named the new<br />

substances polonium and radium.<br />

<strong>Skill</strong> Sheet 14.4: Rosalyn Yalow<br />

1. There are some striking similarities in the lives of Rosalyn<br />

Yalow and Marie Curie. As young women, both were<br />

outstanding math and science students. Even though Yalow<br />

was 54 years younger than Marie Curie, both faced limited<br />

higher education opportunities because they were women.<br />

Undaunted, each earned a doctorate degree in physics. Both<br />

Yalow and Curie’s research focused on radioactive materials.<br />

Curie’s work was at the forefront of discovery of how<br />

radiation works, while Yalow’s work was to develop a new<br />

application of radiation. Both women were particularly<br />

Page 35 of 57<br />

6. I 2 + KF<br />

7. Ca + K 2 S → 2K + CaS<br />

8. 3Mg + Fe 2 O 3 → 3MgO + 2Fe<br />

9. Li + NaCl → Na + LiCl<br />

8. 84%<br />

9. 107.2 g<br />

10. Experimental error, such as not measuring reactants or<br />

products carefully, spilling a reactant or product, or<br />

introducing a contaminant can affect the actual yield. Also, it<br />

may be impossible to measure every last bit of a reactant or<br />

product.<br />

powered submarines or aircraft carriers.<br />

5. Meitner’s honors included the Enrico Fermi award, and<br />

element 109, meitnerium, named in her honor.<br />

6. Students should include the following pieces of evidence in<br />

their letters:<br />

Meitner suggested tests to perform on the product of uranium<br />

bombardment.<br />

Meitner proved that splitting the uranium atom was<br />

energetically possible.<br />

Meitner explained how neutron bombardment caused the<br />

uranium nucleus to elongate and eventually split.<br />

4. Answers include nuclear physics, nuclear medicine, and<br />

radioactive dating.<br />

5. Marie Curie thought carefully about how to balance her<br />

scientific career and the needs of her children. When the<br />

children were young, Pierre’s father lived with the family and<br />

took care of the children while their parents were working.<br />

Marie spent a great deal of time finding schools that best fit<br />

the individual needs of her children and at one point set up an<br />

alternative school where she and several friends took turns<br />

tutoring their children. When her daughters were in their<br />

teens, Marie included them in her professional activities when<br />

possible. Irene, for example, helped her mother set up mobile<br />

x-ray units for wounded soldiers during the war.<br />

interested in the medical uses of radiation. Each was<br />

committed to using their scientific discoveries to promote<br />

humanitarian causes. Both women won Nobel Prizes for their<br />

work (Marie Curie won two!).<br />

2. RIA is a technique that uses radioactive molecules to measure<br />

tiny amounts of biological substances (like hormones) or<br />

certain drugs in blood or other body fluids.<br />

3. Using RIA, they showed that adult diabetics did not always<br />

lack insulin in their blood, and that, therefore, something


must be blocking their insulin’s normal action. They also<br />

studied the body’s immune system response to insulin<br />

injected into the bloodstream.<br />

4. The issue of patents in medical research remains a hotly<br />

debated issue in our society. Proponents of patents, especially<br />

for new drugs, claim that because very few new drugs make it<br />

through the extensive safety and effectiveness trials required<br />

for FDA approval, research costs are very high. Patents, they<br />

claim, are the only means of recouping these research costs.<br />

<strong>Skill</strong> Sheet 14.4: Chien-Shiung Wu<br />

1. Weak nuclear force: One of the fundamental forces in the<br />

atom that governs certain processes of radioactive decay. It is<br />

weaker than both the electric force and the strong nuclear<br />

force. If you leave a solitary neutron outside the nucleus, the<br />

weak force eventually causes it to break into a proton and an<br />

electron. The weak force does not play an important role in a<br />

stable atom, but comes into action in certain cases when<br />

atoms break apart.<br />

Beta decay: a radioactive transformation in which a neutron<br />

splits into a proton and an electron. The electron is emitted as<br />

a beta particle and the proton stays in the nucleus, increasing<br />

the atomic number by one.<br />

Isotope: Forms of the same element that have different<br />

numbers of neutrons and different mass numbers.<br />

2. When Enrico Fermi was having difficulty with a fission<br />

experiment, he turned to Wu for assistance. She recognized<br />

the cause of the problem: a rare gas she had studied in<br />

graduate school. Because she was familiar with the behavior<br />

of the gas she was able to help Fermi get on with his work.<br />

3. The law of conservation of parity stated that in nuclear<br />

reactions, there should be no favoring of left or right. In beta<br />

decay, for example, electrons should be ejected to the left and<br />

to the right in equal numbers.<br />

4. Cobalt-60 has 27 protons and 33 neutrons. There is only one<br />

stable isotope of cobalt, cobalt-59.<br />

5. Wu cooled cobalt-60 to less than one degree above absolute<br />

zero, then placed the material in a strong magnetic field so<br />

that all the cobalt nuclei lined up and spun along the same<br />

axis. As the radioactive cobalt broke down and gave off<br />

<strong>Skill</strong> Sheet 14.4: Radioactivity<br />

1. In the answers below, “a” is alpha decay and “b” is beta<br />

decay.<br />

a. Answers are:<br />

238<br />

92 a→ b→ b→ a→ a→<br />

U<br />

234<br />

90 Th<br />

234<br />

91 Pa 234<br />

92 U 230<br />

90 Th<br />

226<br />

88 a→ a→ a→ b→ b→<br />

a→ b→ b→ a→<br />

b. Answers are:<br />

b→ a→ a→ b→ a→<br />

Ra<br />

222<br />

86 Rn 218<br />

84 Po 214<br />

82 Pb 214<br />

83 Bi<br />

214<br />

84 Po<br />

210<br />

82 Pb 210<br />

83 Bi 210<br />

84 Po 206<br />

82 Pb<br />

240<br />

94 Pu<br />

240<br />

95 Am<br />

236<br />

93 Np 232<br />

91 Pa 232<br />

92 U<br />

228<br />

90 Bi<br />

212<br />

83 Bi<br />

224<br />

88 Ra<br />

a→ b→ a→ a→ a→<br />

212<br />

84 Po<br />

224<br />

89 Ac<br />

208<br />

82 Pb<br />

b→ a→ b→<br />

220<br />

87 Fr<br />

<strong>Skill</strong> Sheet 15.2: Svante Arrhenius<br />

208<br />

83 Bi<br />

216<br />

85 At<br />

1. Arrhenius studied what happens when electricity is passed<br />

through solutions. He proposed that molecules in solutions<br />

could break up into electrically charged fragments called ions.<br />

Page 36 of 57<br />

On the other side of the issue, critics say that the profit motive<br />

drives research into certain types of medicines—tending to be<br />

drugs for chronic illnesses, so that patients will take the drugs<br />

for a long time. Research into drugs (like new antibiotics) that<br />

are generally taken only for a short period of time tends to be<br />

less of a priority. You may wish to have students research the<br />

pros and cons of the patent system and write a position paper<br />

or hold a class discussion or debate on the topic.<br />

electrons, Wu observed that far more electrons flew off in the<br />

direction opposite the spin of the nuclei. She proved that the<br />

law of conservation of parity does not hold true in all cases.<br />

6. Margaret Burbridge, professor of astronomy, UCLA-awarded<br />

the National Medal of Science in the physical sciences in<br />

1983. Citation: “For leadership in observational astronomy.<br />

Her spectroscopic investigations have provided crucial<br />

information about the chemical composition of stars and the<br />

nature of quasi-stellar objects.”<br />

Vera C. Rubin, staff member, astronomy, Carnegie Institute of<br />

Washington-awarded the National Medal of Science in the<br />

physical sciences in 1993. Citation: “For her pioneering<br />

research programs in observational cosmology which<br />

demonstrated that much of the matter in the universe is dark<br />

and for significant contributions to the realization that the<br />

universe is more complex and more mysterious than had been<br />

imagined.”<br />

7. Answers may vary. Some interesting questions to research<br />

include:<br />

What are some other experiments or projects Wu undertook<br />

during the course of her career?<br />

Did she ever return to her home country?<br />

What advice would she give young people who wish to<br />

become scientists?<br />

8. Unfortunately, the contributions of women to science were<br />

often overlooked. When the prize was awarded to Lee and<br />

Yang, playwright Clare Booth Luce commented, “When Dr.<br />

Wu knocked out that principle of parity, she established the<br />

principle of parity between men and women.”<br />

2. Answers are:<br />

a. During 11 minutes, fluorine-18 would experience 6 halflives.<br />

b. 0.16 gram would be left after 11 minutes (660 seconds).<br />

3. The amount after 28,650 years would be 0.0313m (or 1/32m)<br />

where m is the mass of the sample.<br />

4. For one-fourth of the original mass to be left, there must have<br />

been time for two half-lives. Therefore, the half-life for this<br />

radioactive isotope is 9 months.<br />

5. Answers are:<br />

a. 0.8 W/m 2<br />

b. 3.6 × 10 13 reactions per second<br />

2. Arrhenius had always been a strong student. However, his<br />

doctoral thesis was not well understood and he was given a


arely passing grade. This made it difficult for him to find<br />

work in a university after graduation.<br />

3. Arrhenius was interested in physical chemistry, biology,<br />

astronomy, and meteorology.<br />

4. Arrhenius suggested that life may have spread from one part<br />

of the universe to another when living spores from one planet<br />

escaped their atmosphere and were then pushed along by<br />

<strong>Skill</strong> Sheet 16.1: Open and Closed Circuits<br />

1. Answers are:<br />

a. A, B<br />

b. A, C, D<br />

c. A, B, C<br />

d. no current<br />

e. A, B, D<br />

f. B, C, D<br />

g. A, C<br />

h. A<br />

i. A<br />

j. no current<br />

2. Answers are:<br />

a. A, B, C, D, E<br />

b. A, B, C, D, E, F, G<br />

c. A, B, C, D, E<br />

d. A, B, C, D, F, G<br />

e. A, B, C<br />

f. B, C, D, E, F, G<br />

g. no current<br />

h. B, C, D, F, G<br />

i. A, B, C, D, F, G<br />

j. A, B, C<br />

k. A, B, C, D, E<br />

l. B, C<br />

m. A, B, C<br />

<strong>Skill</strong> Sheet 16.1: Benjamin Franklin<br />

1. Franklin learned through reading, writing, discussing, and<br />

experimenting.<br />

2. Franklin’s hypothesis was that lightning is an example of a<br />

large-scale discharge of static electricity.<br />

3. Franklin’s reported results were that the loose threads of the<br />

hemp stood up and that touching the key resulted in a static<br />

electric shock. He concluded that the results were consistent<br />

with other demonstrations of static electricity; therefore,<br />

lightning was a large-scale example of the same phenomenon.<br />

4. If the kite had been struck by lightning, the amount of charge<br />

coming down the hemp string would most likely have<br />

electrocuted Franklin.<br />

<strong>Skill</strong> Sheet 16.2: Using an Electrical Meter<br />

Page 37 of 57<br />

radiation pressure to other places in the universe until they<br />

landed on another hospitable planet.<br />

5. Arrhenius was one of the first to explain the role of<br />

“greenhouse gases” in warming the surface of the planet. He<br />

also proposed that humans could change Earth’s average<br />

temperature by adding carbon dioxide to the atmosphere.<br />

n. no current<br />

o. B, C<br />

3. Diagrams:<br />

4. diagram #1: 4 paths; diagram #2: 8 paths<br />

5. Student drawings will vary. If time permits, allow students to<br />

build and test their circuits<br />

5. The diagram shows<br />

electrons moving from<br />

the glass rod to the silk<br />

so that the silk becomes<br />

negatively charged and<br />

the glass becomes<br />

positively charged.<br />

6. A lightning rod is a<br />

metal rod attached to the<br />

roof of a building. A<br />

thick cable stretches<br />

from the rod to a metal<br />

1. Sample diagram: 2. First battery: 1.553 volts; second battery: 1.557 volts<br />

3. First bulb: 1.514 volts; second bulb: 1.586 volts<br />

4. 3.113 volts<br />

5. 3.108 volts<br />

6. The two voltages are approximately equal.<br />

7. post #1: 0.0980 amps<br />

post #2: 0.0981 amps<br />

post #3: 0.0978 amps<br />

post #4: 0.0980 amps<br />

post #5: 0.0980 amps


8. The current is approximately the same at all points.<br />

9. First bulb: 15.4 ohms; second bulb: 16.2 ohms<br />

10. Measuring resistance: First, set the meter dial to measure<br />

resistance. Remove the bulb from its holder. Then place one<br />

lead on the side of the metal portion of the light bulb (where<br />

the bulb is threaded to fit into the socket). Place the other lead<br />

on the “bump” at the base of the light bulb. The meter will<br />

display the bulb’s resistance.<br />

Measuring voltage: First, set the meter dial to measure DC<br />

voltage. Locate the device (battery, bulb, etc.) that you wish<br />

to measure the voltage across. Then place one meter lead on<br />

<strong>Skill</strong> Sheet 16.3: Voltage, Current, and Resistance<br />

Reading section answers:<br />

1. A battery with a larger voltage can create a greater energy<br />

difference. A 9-volt battery gives a greater “push” to the<br />

charges and creates a larger current.<br />

2. You could increase the number of batteries, which would<br />

increase the total voltage in the circuit. You could replace the<br />

light bulb with a bulb of lower resistance. You could also use<br />

thicker wires, shorter wires, or wires made from a material<br />

with a higher conductivity. These changes to the wires would<br />

decrease the total amount of resistance in the circuit.<br />

3. If the circuit used a 9-volt battery, you could try replacing it<br />

with five (or less than five) 1.5-volt batteries to lower the<br />

voltage in the circuit. You could replace the bulb with a bulb<br />

of higher resistance. Or, you could use thinner wires, longer<br />

wires, or wires made from a material with a lower<br />

conductivity. These changes to the wires would increase the<br />

<strong>Skill</strong> Sheet 16.3: Ohm’s Law<br />

1. 3 amps<br />

2. 0.75 amp<br />

3. 0.5 amp<br />

4. 1 amp<br />

5. 120 volts<br />

6. 8 volts<br />

7. 50 volts<br />

8. 12 ohms<br />

9. 240 ohms<br />

10. 1.5 ohms<br />

11. 3 ohms<br />

12. Answers are:<br />

a. Circuit A: 6 V; Circuit B: 12 V<br />

b. Circuit A: 1 A; Circuit B: 2 A<br />

c. Circuit A: 0.5 A; Circuit B: 1 A<br />

<strong>Skill</strong> Sheet 16.4: Series Circuits<br />

1. Answers are:<br />

a. 6 volts c. 3 amps<br />

b. 2 ohms d. 3 volts<br />

c. Diagram:<br />

Page 38 of 57<br />

one of the posts next to the device. Put the other meter lead on<br />

the post on the other side of the device. The meter will display<br />

the device’s voltage. If it shows a negative voltage, switch the<br />

two leads.<br />

Measuring current: First, set the meter to measure DC current.<br />

Then break the circuit at the location where you wish to<br />

measure the current. Connect one of the meter leads at one<br />

side of the break. Connect the other lead at the other side of<br />

the break. The meter will display the current. If it shows a<br />

negative current, switch the two leads.<br />

total resistance in the circuit. To stop the current completely,<br />

you could simply open the switch.<br />

4. You could simply cut each piece of wire in the circuit as short<br />

as possible. A shorter wire has less resistance than a longer<br />

wire. To make a more significant decrease in resistance, you<br />

would need to replace the wire with a thicker gauge wire or a<br />

wire made from a material with greater conductivity.<br />

5. Ohm’s law states that, in a circuit, the amount of current is<br />

directly related to voltage, and inversely related to the<br />

resistance in the circuit.<br />

Problem section answers:<br />

1. 1.5 amps<br />

2. 0.75 amps<br />

3. 50 volts<br />

4. 12 ohms<br />

d. It is brighter in circuit B because there is a greater voltage<br />

and greater current (and more power is consumed since<br />

power equals current times voltage).<br />

13. The current becomes 4 times as great.<br />

14. If resistance increases, the current decreases. The two are<br />

inversely proportional.<br />

15. If voltage increases, current increases. The two are directly<br />

proportional.<br />

16. Remove one of the light bulbs. This decreases the resistance<br />

and increases the current.<br />

17. Remove one of the batteries. This decreases the voltage and<br />

decreases the current.<br />

18. Answers are:<br />

a. 2 batteries and a 3 ohm bulb (or 4 batteries and all 3 bulbs)<br />

b. 4 batteries and a 3 ohm bulb<br />

c. 2 batteries and a 1 ohm bulb (or 4 batteries and a 2 ohm<br />

bulb)<br />

d. 4 batteries and a 1 ohm bulb<br />

2. Answers are:<br />

a. 6 volts c. 2 amps<br />

b. 3 ohms d. 2 volts<br />

c. Diagram:


3. The current decreases because the resistance increases.<br />

4. The brightness decreases because the voltage across each<br />

bulb decreases and the current decreases. Since power equals<br />

voltage times current, the power consumed also decreases.<br />

5. Answers are:<br />

a. 3 ohms<br />

b. 2 amps<br />

c. 1 ohm bulb: 2 volts; 2 ohm bulb: 4 volts<br />

6. Answers are:<br />

a. 12 volts<br />

b. 4 ohms<br />

c. 3 amps<br />

d. 6 volts<br />

e. Diagram:<br />

<strong>Skill</strong> Sheet 16.4: Parallel Circuits<br />

Practice set 1:<br />

1. Answers are:<br />

a. 12 volts<br />

b. 6 amps<br />

c. 12 amps<br />

d. 1 ohm<br />

2. Answers are:<br />

a. 12 volts<br />

b. 4 amps<br />

c. 8 amps<br />

d. 1.5 ohms<br />

3. Answers are:<br />

a. 12 volts<br />

b. 2 ohm branch: 6 amps; 3 ohm branch: 4 amps<br />

c. 10 amps<br />

d. 1.2 ohms<br />

<strong>Skill</strong> Sheet 16.4: Thomas Edison<br />

1. Edison’s education included one-on-one tutoring from his<br />

mother, reading lots of <strong>book</strong>s, and performing experiments in<br />

laboratories that he set up.<br />

2. Edison learned that in order to sell an invention, not only does<br />

it have to be a technical success, it also has to be something<br />

that people want to buy.<br />

3. Edison’s research facility at Menlo Park had workshops,<br />

laboratories, offices, and a library. Edison hired a team of<br />

assistants with various specialties to work there.<br />

Page 39 of 57<br />

7. Answers are:<br />

a. 2 ohms<br />

b. 1 volt<br />

c. Diagram:<br />

8. Answers are:<br />

a. 6 ohms<br />

b. 1.5 amps<br />

c. 2 ohm resistor: 3 volts; 3 ohm resistor: 4.5 volts; 1 ohm<br />

resistor: 1.5 volt<br />

d. The sum is 9 volts, the same as the battery voltage.<br />

9. Answers are:<br />

a. Diagram A: 0.5 amps; Diagram B: 1.0 amps<br />

b. Diagram A: 0.25 amps; Diagram B: 0.5 amps<br />

c. The amount of current increases.<br />

4. Answers are:<br />

a. 9 volts<br />

b. 2 ohm branch: 4.5 amps; 3 ohm branch: 3 amps; 1 ohm<br />

branch: 9 amps<br />

c. 16.5 amps<br />

Practice set 2:<br />

1. Answers are:<br />

a. 4 ohms<br />

b. 6 ohms<br />

c. 2.67 ohms<br />

d. 2.4 ohms<br />

2. Answers are:<br />

a. 2.67 ohms<br />

b. 1.2 ohms<br />

c. 0.545 ohms<br />

4. The tin foil phonograph and a practical, safe, and affordable<br />

incandescent light were developed at Menlo Park.<br />

5. Edison’s invention process was to brainstorm as many ideas<br />

as possible, try everything that seems even remotely<br />

workable, record everything, and use failed experiments to<br />

redirect the project.<br />

6. Edison was not easily discouraged by failure. Instead, he saw<br />

failed projects as providing useful information to narrow<br />

down the possibilities of what does work.


7. Students can find information about Edison’s tin foil<br />

phonograph using the Internet. Here’s a summary of how it<br />

worked:<br />

Edison set up a membrane that vibrated when exposed to<br />

sound waves. The membrane (called a diaphragm) had an<br />

embossing needle attached. When someone spoke, the<br />

diaphragm would vibrate and the embossing needle would<br />

make indentations on tin foil wrapped around a metal<br />

cylinder. The cylinder was turned by a hand-crank at around<br />

<strong>Skill</strong> Sheet 16.4: George Westinghouse<br />

1. Westinghouse first developed his talents as an inventor in his<br />

father’s agricultural machine shop.<br />

2. Westinghouse enabled trains to travel more safely at higher<br />

speeds in two ways: He invented an air brake which allowed<br />

the engineer to stop all the cars at once, and he developed<br />

signaling and switching systems which reduced the likelihood<br />

of collisions.<br />

3. Westinghouse promoted alternating current because it could<br />

be transmitted over longer distances.<br />

4. Westinghouse demonstrated the potential of alternating<br />

current by lighting the streets of Philadelphia and then the<br />

entire Chicago World’s Fair using this technology.<br />

5. Direct current occurs when charge flows in one direction.<br />

Batteries provide direct current. Alternating current, in<br />

contrast, switches directions. Household circuits in the United<br />

States run on alternating current that reverses direction 60<br />

times each second. The diagrams below compare direct and<br />

alternating current.<br />

<strong>Skill</strong> Sheet 16.4: Lewis Latimer<br />

1. Lewis Latimer did not attend college, but learned how to be a<br />

draftsman by studying the drawings of draftsmen while<br />

working as an office boy for a patent law firm. He was selfmotivated<br />

and used any available <strong>book</strong>s and tools to learn the<br />

skills needed to become an outstanding draftsman. He was<br />

also a self-taught electrical engineer. He learned all that he<br />

could about electricity while working for Hiram Maxim at<br />

U.S. Electric Lighting.<br />

2. Latimer invented the following:<br />

Mechanical improvements for railroad train water closets<br />

(also known as toilets)<br />

Carbon filaments to replace paper filaments in light bulbs<br />

An improved manufacturing process for carbon filaments<br />

An early version of the air conditioner<br />

A locking rack for hats, coats, and umbrellas<br />

A <strong>book</strong> support<br />

His most important inventions are the development of carbon<br />

fibers and improved manufacturing process to produce those<br />

filaments. The light bulb invented by Thomas Edison used a<br />

paper filament. As a result, it had a very short life span. With<br />

carbon filaments, light bulbs lasted longer, were more<br />

Page 40 of 57<br />

60 revolutions per minute. There was a second diaphragmand-needle<br />

apparatus for playback. When the needle followed<br />

the “tracks” made in the tin foil, it made the diaphragm<br />

vibrate which reproduced the recorded sounds.<br />

8. Answers will vary. Some of Edison’s interesting inventions<br />

include paraffin paper, an “electric pen” (the forerunner of the<br />

mimeograph machine) a carbon rheostat, a fluoroscope, and<br />

sockets, switches, and insulating tape.<br />

6. Westing house used hydroelectric generators at Niagara<br />

Falls to provide electricity to the city of Buffalo.<br />

7. Westinghouse’s Manhattan elevated trains were electrical<br />

powered using alternating current.<br />

8. Answers may vary. Some of Westinghouse’s other<br />

inventions include: Apparatus for safe transmission of<br />

natural gas, the transformer, a machine that placed<br />

derailed train cars back on the tracks, and a compressed<br />

air spring.<br />

affordable, and could be used in a variety of ways. Latimer<br />

improved the technology so that light bulbs would become<br />

commonplace in both industry and homes.<br />

3. A “Renaissance man” is a scholar with a depth of knowledge<br />

in a variety of areas. During the Renaissance, there were men<br />

who were accomplished in multiple disciplines including<br />

math, engineering, art, and music. Leonardo da Vinci was one<br />

of the great Renaissance men. Lewis Latimer can be called a<br />

Renaissance man because he, too, excelled in a variety of<br />

areas including science, literature, music, and art.<br />

4. Two of Latimer’s poems were titled Friends and Ebon Venus.<br />

5. The Edison Pioneers first met on February 11, 1918. This was<br />

Thomas Edison’s 71st birthday.<br />

Excerpt from obituary:<br />

“He was of the colored race, the only one in our organization,<br />

and was one of those to respond to the initial call that led to<br />

the formation of the Edison Pioneers, January 24, 1918.<br />

Broadmindedness, versatility in the accomplishment of things<br />

intellectual and cultural, a linguist, a devoted husband and<br />

father, all were characteristic of him, and his genial presence<br />

will be missed from our gatherings.”


<strong>Skill</strong> Sheet 17.1: Magnetic Earth<br />

1. Students learn in Chapter 17 that Earth’s magnetic field<br />

comes from its core. For those who desire a more detailed<br />

explanation, scientists believe that the motion of molten<br />

metals in Earth’s outer core create its magnetic field.<br />

2. Seven percent of 0.5 gauss is 0.035 gauss. In 100 years,<br />

Earth’s strength could be 0.465 gauss.<br />

3. Student answers will vary but may include responses such as:<br />

devices that depend on magnets would work differently or not<br />

at all. Earth science experts tell us that the poles could reverse<br />

within the next 2,000 years. During a reversal, the field would<br />

not completely disappear. The main magnetic field that we<br />

use for navigation would be replaced by several smaller fields<br />

with poles in different locations.<br />

4. Rock provides a good record because as the rock is made,<br />

atoms in the rock align with the magnetic field of Earth.<br />

(Actually, oceanic rock is made of a substance called<br />

magnetite!) Rock that was made 750,000 years ago would<br />

have a north-south orientation that is exactly opposite the<br />

north-south orientation of rock that is made today. Therefore,<br />

we can use the north-south orientation of bands of rock on the<br />

sea floor to understand how many times the poles have<br />

reversed over geologic time.<br />

5. NOTE: In many references, magnetic south pole is referred to<br />

as “magnetic north pole” because it is located at the<br />

geographic north pole. This terminology can be confusing to<br />

students who know that opposite poles attract. The north pole<br />

of a compass needle is in fact the north end of a bar magnet.<br />

This is why we think it is best to use the term magnetic south<br />

pole as the point to which the north end of a compass needle<br />

is attracted. For more information about Earth’s magnetism<br />

see http://www.ngdc.noaa.gov/.<br />

Answers are:<br />

a. Both the magnetic south pole and geographic north are<br />

located near the Arctic.<br />

<strong>Skill</strong> Sheet 17.2: Maglev Train Model Project<br />

Students build a model to complete this skill sheet. No written<br />

responses are required.<br />

<strong>Skill</strong> Sheet 17.4: Michael Faraday<br />

1. Faraday took careful notes during lectures given by Davy,<br />

then bound his notes and sent them to Davy along with a<br />

request for a job. Faraday had little formal training in science,<br />

so this was a means of proving his capabilities.<br />

2. Benzene is used in cleaning solvents, herbicides, insecticides,<br />

and varnishes. Note: Benzene is a known carcinogen and<br />

highly flammable. Safety precaution must be taken whenever<br />

it is used.<br />

3. Electromagnetic induction is the use of a moving magnet to<br />

create an electric current.<br />

4. Sample answer: When light is polarized (vibrating in one<br />

plane), a strong magnetic field can change the orientation of<br />

that plane.<br />

5. Faraday instituted the Friday Evening Discourses and the<br />

Christmas Lectures for Children, which gave non-scientists<br />

<strong>Skill</strong> Sheet 17.4: Transformers<br />

1. 25 turns<br />

2. 230 volts<br />

3. 220 volts<br />

4. 100 volts<br />

Page 41 of 57<br />

b. The magnetic south pole is about 1,000 kilometers from<br />

the geographic north pole.<br />

c. Magnetic south pole is the point on Earth’s surface that<br />

corresponds to Earth’s south pole if you think of Earth’s<br />

core as a bar magnet. Magnetic south pole is located at a<br />

distance from the geographic north pole or ‘true north.’<br />

True north is a point on Earth’s surface that we call north.<br />

‘True north’ and ‘true south’ follow Earth’s axis. If we<br />

want to go north, we need to head toward Earth’s<br />

geographic north. The tool we use to head north, however,<br />

points us toward the magnetic south pole. We use<br />

magnetic declination to correct for this.<br />

6. A compass needle is a bar magnet. Its north pole is attracted<br />

to Earth’s magnetic south pole (if you consider the interior of<br />

Earth is like a bar magnet). Using magnetism, we can find our<br />

way using north as a reference point. However, using<br />

magnetism actually points us a bit off course because the<br />

magnetic south pole is not located at the same position as true<br />

north.<br />

7. Answers are:<br />

a. Example problem: northeast.<br />

b. south<br />

c. west<br />

d. east<br />

e. southeast<br />

f. northwest<br />

8. If you didn’t correct your compass for magnetic declination,<br />

you would be off course and possibly get lost.<br />

9. Yes, magnetic declination equals zero on Earth’s surface<br />

along a line that goes from New Orleans through the eastern<br />

edge of Minnesota up through Churchill, Canada. However,<br />

the location of the zero-degree line is always changing. For<br />

more information about Earth’s magnetic declination and<br />

magnetism, see http://www.ngdc.noaa.gov/.<br />

the opportunity to learn about the scientific community and<br />

about recent advances in science.<br />

6. The electric motor, invented by Faraday, is used in all sorts of<br />

household appliances including electric fans, hair dryers, food<br />

processors, and vacuum cleaners. Electromagnetic induction<br />

is used by local power plants to generate the electricity that is<br />

used every day.<br />

7. Faraday had a reputation as an engaging speaker who used<br />

exciting demonstrations to catch his audience’s interest. He<br />

had a knack for communicating scientific knowledge in terms<br />

that non-scientists understood. It also would be interesting to<br />

see who else might be in attendance at the lecture!<br />

8. Iron filings are available from many science supply catalogs.<br />

Place some iron filings on a piece of clear plastic (such as an<br />

overhead transparency). Place the plastic over a magnet to<br />

observe the field lines.<br />

5. 440 turns<br />

6. 131 turns


<strong>Skill</strong> Sheet 17.4: Electrical Power<br />

1. Answers are:<br />

a. 5 kW<br />

b. 10 kWh<br />

c. $1.50<br />

2. Answers are:<br />

a. 300 minutes<br />

b. 5 hours<br />

c. 1.2 kW<br />

d. 6 kW<br />

e. $0.90<br />

3. 960 W<br />

4. 24 W<br />

5. Answers are:<br />

a. 60 W<br />

b. 0.06 kW<br />

c. 525.6 kWh<br />

d. $78.84<br />

6. 0.625 A<br />

7. Answers are:<br />

a. 3 V<br />

b. 1 A<br />

c. 3 W<br />

<strong>Skill</strong> Sheet 18.1: Andrew Douglass<br />

1. Douglass originally started as an astronomer. He helped to<br />

set-up observatories and also studied Mars with Percival<br />

Lowell. He noticed a possible relationship between sunspot<br />

cycles and the climate and wished to study this further. He<br />

noted that tree rings held information about weather patterns<br />

and hoped he could find a link between periods of drought<br />

and sunspot activity. This marked his move away from<br />

astronomy and toward tree ring analysis.<br />

2. Douglass created the science of dendrochronology or tree ring<br />

dating. He specifically developed cross-dating as a technique<br />

to match tree ring samples with ancient ruins.<br />

3. Douglass, in 1929, was able to date with accuracy Native<br />

American ruins in Arizona. He studied pine tree rings dating<br />

back to the time of Native American dwellings. He matched<br />

wooden beam samples against pine tree rings to determine a<br />

<strong>Skill</strong> Sheet 18.2: Relative Dating<br />

1. A thunderstorm began. A child ran through a mud puddle<br />

leaving footprints. Hail began to fall. Finally, the mud puddle<br />

dried and cracked.<br />

2. One April afternoon, a thunderstorm began. A child was<br />

outside playing. When the rain began to fall hard, the child<br />

ran home. Her footprints were left in a mud puddle.<br />

Fortunately, she made it home just in time because small<br />

hailstones suddenly began to fall. The hailstorm lasted for a<br />

few minutes and then the clouds cleared. The next morning<br />

was bright and sunny. The mud puddle dried and then cracked<br />

in many places.<br />

3. C<br />

4. E (This is the term that will be new to students.)<br />

5. A<br />

6. F<br />

7. B<br />

8. D<br />

9. There is no matching picture for this concept. The name of<br />

this concept is faunal succession.<br />

Page 42 of 57<br />

8. Answers are:<br />

a. 24 ohms<br />

b. 600 W<br />

c. 0.6 kW<br />

9. Answers are:<br />

a. 20.5 A<br />

b. 10.8 ohms<br />

c. 18 kWh<br />

d. $140.40<br />

10. Answers are:<br />

a. 6 ohms<br />

b. 2 A<br />

c. 12 W<br />

d. 24 W<br />

11. Answers are:<br />

a. 12 V<br />

b. 4 A<br />

c. 48 W<br />

d. 8 A<br />

e. 96 W<br />

precise date for the ancient ruins. This cross-dating technique<br />

provided a tool for all archaeologists to date prehistoric<br />

remains and ruins.<br />

4. The second asteroid is called Minor Planet or Asteroid<br />

(15420) Aedouglass.<br />

5. The Boyden Observatory is now located in Bloemfontein,<br />

South Africa.<br />

6. The Spacewatch Project is located at the University of<br />

Arizona’s Lunar and Planetary Laboratory. Scientists at<br />

Spacewatch study and explore small objects in the solar<br />

system including asteroids and comets. The Project was<br />

founded by Professor Tom Gehrels and Dr. Robert S.<br />

McMillan in 1980.<br />

10. The rock bodies formed in this order: H, G, F, D, C, B, E, and A.<br />

11. The fault formed after layer F and before both layer D and the<br />

intrusion.<br />

12. The stream formed after layer A. Like an intrusion, the stream<br />

cut across the rock bodies.<br />

Extension<br />

13. The set of clues<br />

includes three layers<br />

of colored sand in a<br />

clear tank. The layers<br />

were made while a<br />

small pencil was held<br />

upright. The sand<br />

filled in around the<br />

pencil. After the layers<br />

were created, a<br />

second, longer pencil<br />

was pushed through<br />

the layers.


14. The concepts of original horizontality and lateral continuity<br />

are demonstrated here. Pencil E represents cross-cutting<br />

relationships. Pencil D represents the idea of inclusions.<br />

<strong>Skill</strong> Sheet 18.2: Nicolas Steno<br />

1. Steno is responsible for the following three principles of<br />

geology:<br />

The principle of superposition states that layers of<br />

sedimentary rock settle on top of each other. The oldest<br />

layers are at the bottom and the younger layers are on top.<br />

The bottom layers are formed first with younger layers<br />

sitting above. Geologists use this principle to determine the<br />

relative ages of layers.<br />

The principle of original horizontality states that<br />

sedimentary rock layers form horizontally.<br />

The principle of lateral continuity states that rock layers<br />

spread out until they reach something that stops this<br />

spreading. The layers will continue to move out in all<br />

directions horizontally until they are stopped.<br />

2. People did not understand how fossils formed and what<br />

fossils truly were. Common misconceptions included the<br />

following: fossils grew inside rocks, fossils fell from the sky,<br />

and fossils fell from the moon. People did not consider<br />

extinction or have an understanding of geological principles<br />

to understand fossils and fossil formation.<br />

3. Steno identified tongue stones as ancient shark teeth. He<br />

understood that particles settled in sediment. The shark teeth<br />

had settled into soft sediment that eventually hardened.<br />

Sharks had once lived in the mountains that at one time had<br />

been covered by the sea. Shark teeth became buried in mud<br />

and rock layers formed around the teeth. These layers became<br />

buried under new layers of rock<br />

4. As an anatomist, Steno developed keen observation skills. He<br />

was comfortable examining something in depth and trying to<br />

<strong>Skill</strong> Sheet 18.3: The Rock Cycle<br />

<strong>Skill</strong> Sheet 19.1: Earth’s Interior<br />

A. Crust<br />

B. Upper mantle<br />

C. Asethenosphere<br />

D. Lower mantle<br />

E. Outer core<br />

F. Inner core<br />

Page 43 of 57<br />

15. The order of the events is D, A, B, C, and E. My classmates<br />

successfully figured out the correct order of the events.<br />

understand how something worked. He was not merely<br />

satisfied with viewing something. He liked to take things<br />

apart. His medical background and work in anatomy made<br />

him a hands-on researcher. He took his observation and<br />

interest in understanding structure and applied those skills to<br />

geology. Unlike many of his counterparts who simply read<br />

scholarly works, Steno was a true field scientist. He traveled,<br />

observed, and touched.<br />

5. Answers will vary. Students might suggest observations they<br />

have made in a science lab, at the beach, or on a field trip. In<br />

general, observation often teaches us that things are not what<br />

they may initially appear. Observation means paying attention<br />

to the details, even if minute or mundane.<br />

6. A goldsmith is a metalworker who often makes jewelry. A<br />

goldsmith will solder, file, and polish. A goldsmith does not<br />

work only with gold, but will handle a variety of metals. Most<br />

work by a goldsmith is done by hand. Steno’s father’s<br />

goldsmith shop was a laboratory providing him with the<br />

opportunity to use his hands to handle various tools and<br />

materials.<br />

Alchemy is the early ancestor of chemistry. This ancient form<br />

of chemistry included herbs and metals. Alchemists often<br />

looked for cures for illnesses. Goldsmith work and alchemy<br />

both took place in a laboratory-like setting providing Steno<br />

exposure to scientific concepts and materials. Alchemists<br />

liked to experiment and tried to understand the world around<br />

them. Steno used observation and his hands throughout his<br />

career as a scientist to make sense of the world around him.


<strong>Skill</strong> Sheet 19.1: Charles Richter<br />

1. Answers are:<br />

theoretical physics—a branch of physics that attempts to<br />

understand the world by making a model of reality, used<br />

for rationalizing, explaining, and predicting physical<br />

phenomena through a “physical theory.”<br />

seismology—The study of earthquakes and of the<br />

structure of the Earth by natural and artificial seismic<br />

waves.<br />

seismograms—A written record of an earthquake,<br />

recorded by a seismograph.<br />

magnitude—the property of relative size or extent<br />

(whether large or small)<br />

seismographs—An instrument for automatically detecting<br />

and recording the intensity, direction, and duration of a<br />

movement of the ground, especially of an earthquake.<br />

2. Sample answer: I would feel proud that a leader in the field<br />

considered me above anyone else. It would be disappointing<br />

to lose the opportunity to work with Dr. Millikan, but if he<br />

considered me ready for the job, maybe he felt I had learned<br />

all I could and was ready to move on.<br />

3. Richter responded by taking on routine tasks and making<br />

something extraordinary out of something ordinary.<br />

4. Dr. Beno Gutenburg<br />

<strong>Skill</strong> Sheet 19.1: Jules Verne<br />

1. Verne’s novels offered people an opportunity to go on<br />

voyages into unknown realms of the world, and even space.<br />

Tales like these are still popular today, but during Verne’s<br />

time, fewer people had the opportunity to travel. Verne’s<br />

novels pulled readers away from their everyday life and<br />

allowed their imaginations to consider futuristic inventions<br />

and machines that were far removed from life in the<br />

nineteenth century.<br />

2. From Earth to the Moon, 20,000 Leagues Under the Sea,<br />

Journey to the Center of the Earth, Around the World in 80<br />

Days, and The Mysterious Island have all been made into<br />

movies several times. Around the World in 80 Days won five<br />

academy awards in 1956 including best picture and best<br />

cinematography.<br />

3. The bar exam is a written test that must be passed in order to<br />

qualify a person to practice law.<br />

<strong>Skill</strong> Sheet 19.2: Alfred Wegener<br />

1. He developed an interest in Greenland when he was a young<br />

boy. As an adult scientist, he went there on several scientific<br />

expeditions to study the movement of air masses over the<br />

polar ice cap. He studied the movement of air masses long<br />

before the common acceptance of the jet stream. He died<br />

there during a blizzard on one of his expeditions just a few<br />

days after his fiftieth birthday.<br />

2. He and his brother set the world record for staying aloft in a<br />

hot air balloon for the longest period of time, 52 hours.<br />

3. Wegener studied and used several different fields of science<br />

in his work. His main areas of expertise were astronomy and<br />

meteorology, however, he also explored paleontology<br />

(fossils), geology, and climatology as he gathered evidence<br />

for his drifting continent theory.<br />

4. Fossils of the small reptile were found only on the eastern<br />

coast of Brazil and the western coast of Africa. Since there<br />

was no way that the reptile could have crossed the Atlantic<br />

Ocean, Wegener figured that those two continents must have<br />

been connected when that reptile was alive.<br />

Page 44 of 57<br />

5. Answers will vary. Correct answers include:<br />

a. Mercalli scale<br />

b. Moment magnitude scale<br />

c. JMA scale (Japanese Meteorological Agency)<br />

d. MSK scale (Medvedev, Sponheuer and Karnik)<br />

e. European Macroseismic scale<br />

f. Rossi-Forel scale<br />

g. Omori scale<br />

6. Some scales measure intensity (like the Mercalli scale), while<br />

others measure magnitude (like the Richter scale). Intensity<br />

scales measure how strongly a quake affects a specific place,<br />

while magnitude scales indicate how much total energy a<br />

quake expends. Also, many times different countries have<br />

different building codes or standards of construction. Some of<br />

the scales used to measure earthquakes are based on<br />

traditional construction materials and techniques, which can<br />

vary around the world. These scales may be used to define the<br />

quake resistant construction guidelines adopted by different<br />

countries or regions with different occurrences of<br />

earthquakes.<br />

7. Seismograph and seismometer are usually interchangeable, as<br />

they both describe devices designed to do the same thing.<br />

Seismometer seems to be the more modern term.<br />

4. Victor-Marie Hugo (February 26, 1802–May 22, 1885) is<br />

recognized as the most influential French Romantic writer of the<br />

19th century and is often identified as the greatest French poet.<br />

Two of his best known works are Les Misérables and The<br />

Hunchback of Notre-Dame. Verne must have been inspired to<br />

meet the most famous and well-respected author of his time.<br />

5. Airplanes, movies, guided missiles, submarines, the electric<br />

chair, air conditioning, the fax machine, gas-powered cars,<br />

and an elevated mass transit system are among his best<br />

known. One of his most eerily true-to-life ideas appears in<br />

both From Earth to the Moon (1865) and All Around the<br />

Moon (1870). In these stories an aluminum craft launched<br />

from central Florida achieves a speed of 24,500 miles per<br />

hour, circles the moon and splashes down in the Pacific. A<br />

century later Apollo 8, made of aluminum and traveling at<br />

24,500 miles an hour, took off from central Florida. It circled<br />

the moon and splashed down in the Pacific.<br />

5. Coal can only be formed under certain conditions. It can be<br />

formed only from plants that grow in warm, wet climates. Those<br />

type of plants could not grow in either England or Antarctica<br />

today. That means that at some time, England and Antarctica<br />

must have been located somewhere around the equator where<br />

those type of plants could survive, and they must have moved<br />

away from the equator to their present locations.<br />

6. Answers will vary.<br />

7. Wegener was a relatively unknown scientist at the time, and<br />

geology wasn’t even his field of expertise, yet he was proposing<br />

a theory that went against everything that scientists at the time<br />

believed about geology. The most famous scientists alive at that<br />

time attacked him viciously and called his theory utter rot! Also,<br />

even though he had gathered what appeared to be a lot of<br />

evidence to show that the continents had indeed moved over<br />

millions of years, he could never explain how or why that<br />

happened. He could never explain what driving force could be<br />

powerful enough to move continents.<br />

8. Answers will vary.


<strong>Skill</strong> Sheet 19.2: Harry Hess<br />

1. Hess used his time in the Navy to further his geological<br />

research. Between battles, Hess and his crew gathered data<br />

about the structure of the ocean floor using the ship’s<br />

sounding equipment. They recorded thousands of miles worth<br />

of depth recordings.<br />

2. While in the Navy, Hess measured the deepest point of the<br />

ocean ever recorded-nearly 7 miles deep. He also discovered<br />

hundreds of flat-topped mountains lining the Pacific Ocean<br />

floor. He named these unusual mountains “guyouts”.<br />

3. Hess explained that sea floor spreading occurs when molten<br />

rock (or magma) oozes up from inside the Earth along the<br />

mid-oceanic ridges. This magma creates new sea floor that<br />

spreads away from the ridge and then sinks into the deep<br />

oceanic trenches where it is destroyed.<br />

4. Hess explained that the ocean floor is continually being<br />

recycled and that sediment has been accumulating for no<br />

<strong>Skill</strong> Sheet 19.2: John Tuzo Wilson<br />

1. Wilson’s adventurous parents helped to expand Canada’s<br />

frontiers. Wilson’s mother, Henrietta Tuzo, was a famous<br />

mountaineer who had Mount Tuzo in western Canada named<br />

in her honor. Wilson’s father, also named John, helped plan<br />

the Canadian Arctic Expedition of 1913 to 1918. He also<br />

helped develop airfields throughout Canada.<br />

2. Wilson is sometimes called an adventurous scholar because<br />

he enjoyed traveling to unusual locations. He became the first<br />

person to scale Mount Hague in Montana. Wilson also led an<br />

expedition called Exercise Musk-Ox in which ten army<br />

vehicles traveled 3,400 miles through the Canadian Arctic.<br />

While a professor, Wilson mapped glaciers in Northern<br />

Canada and became the second Canadian to fly over the<br />

North Pole during his search for unknown Arctic islands.<br />

3. Volcanic islands, like the Hawaiian Islands, are found<br />

thousands of kilometers away from plate boundaries. In 1963,<br />

Wilson published a paper that explained how plates move<br />

<strong>Skill</strong> Sheet 19.3: Earth’s Largest Plates<br />

A. Pacific Plate<br />

B. North American Plate<br />

C. Eurasian Plate<br />

<strong>Skill</strong> Sheet 19.4: Continental U.S. Geology<br />

No student responses are required.<br />

<strong>Skill</strong> Sheet 20.1: Finding an Earthquake Epicenter<br />

Practice 1:<br />

Table 1 answers:<br />

Station<br />

name<br />

Arrival time<br />

difference between<br />

P- and S-waves<br />

Distance to<br />

epicenter in<br />

kilometers<br />

1 15 seconds 130 km 1.3 cm<br />

2 24 seconds 200 km 2.0 cm<br />

3 42 seconds 350 km 3.5 cm<br />

Scale distance<br />

to epicenter in<br />

centimeters<br />

Page 45 of 57<br />

more than 300 million years. This is the amount of time<br />

needed for the ocean floor to spread from the ridge crest to the<br />

trenches. Therefore, the oldest fossils found on the sea floor<br />

are no more than 180 million years old.<br />

5. In 1962, President John F. Kennedy appointed Hess as<br />

Chairman of the Space Science Board -an advisory group for<br />

the National Aeronautics and Space Administration (NASA).<br />

During the late 1960s, Hess helped plan the first landing of<br />

humans on the moon. He was part of a committee assigned to<br />

analyze rock samples brought back from the moon by the<br />

Apollo 11 crew.<br />

6. In 1984, the American Geophysical Union established the<br />

Harry H. Hess medal in recognition of “outstanding<br />

achievements in research in the constitution and evolution of<br />

Earth and sister planets.”<br />

over stationary “hotspots” in the earth’s mantle and form<br />

volcanic islands.<br />

4. In 1965, Wilson proposed that a type of plate boundary must<br />

connect ocean ridges and trenches. He suggested that a plate<br />

boundary ends abruptly and transforms into major faults that<br />

slip horizontally. Wilson called these boundaries “transform<br />

faults”.<br />

5. In 1967, Wilson published an article that described the<br />

repeated process of ocean basins opening and closing-a<br />

process later named the Wilson Cycle. Geologists believe that<br />

the Atlantic Ocean basin closed millions of years ago and<br />

caused the formation of the Appalachian and Caledonian<br />

mountain systems. The basin later re-opened to form today’s<br />

Atlantic Ocean.<br />

6. Antarctica<br />

D. African Plate<br />

E. Indo-Australian Plate<br />

F. Antarctic Plate


Practice 2:<br />

1. First problem is done for students.<br />

Station A: t p = 128 seconds<br />

2. Station B:<br />

5 km/sec × t p = 3 km/sec × (t p + 80 sec)<br />

(2 km/sec) t p = 240 km<br />

t p = 120 seconds<br />

3. Station C: t p = 180 seconds<br />

4. Station A: distance = 5 km/s × 128 sec = 640 km<br />

Station B: distance = 5 km/s × 120 sec = 600 km<br />

Station C: distance = 5 km/s × 180 sec = 900 km<br />

5. Answers are:<br />

(a) 5 km/s × 200 s = 3 km/s × (200 s + x)<br />

1000 km = 600 km + (3 km/s)x<br />

400 km = (3 km/s)x<br />

x = 133 s = time between the P-waves and S-waves<br />

(b) 133 s + 200 s = 333 s = t s<br />

<strong>Skill</strong> Sheet 20.2: Volcano Parts<br />

A. Vent<br />

B. Layers of lava and ash<br />

C. Volcano<br />

D. Magma<br />

<strong>Skill</strong> Sheet 20.3: Basalt and Granite<br />

Sample student answer:<br />

<strong>Skill</strong> Sheet 21.2: Concentration of Solutions<br />

1. 6.7%<br />

2. 2.0%<br />

3. 0.6%<br />

4. 400 g salt<br />

5. 3.75 g sugar<br />

6. 139 g sand<br />

Page 46 of 57<br />

6. Table 3 answers:<br />

Variables Station 1 Station 2 Station 3<br />

Speed of P-waves rp 5 km/s 5 km/s 5 km/s<br />

Speed of S-waves r s 3 km/s 3 km/s 3 km/s<br />

Time between the<br />

arrival of P- and<br />

S-waves<br />

Total travel time of<br />

P-waves<br />

Total travel time of<br />

S-waves<br />

E. Conduit<br />

F. Magma chamber<br />

G. Vent<br />

H. Lava<br />

7. 43%<br />

8. 12.5 g<br />

9. 0.5%<br />

10. 8.8 g red food coloring<br />

t s –t p<br />

t p<br />

t s<br />

70<br />

seconds<br />

105<br />

seconds<br />

175<br />

seconds<br />

115<br />

seconds<br />

173<br />

seconds<br />

288<br />

seconds<br />

92<br />

seconds<br />

138<br />

seconds<br />

230<br />

seconds<br />

Distance to epicenter d p , d s 525 km 865 km 690 km<br />

Scale distance to<br />

epicenter<br />

2.6 cm 4.3 cm 3.5 cm


<strong>Skill</strong> Sheet 21.2: Solubility<br />

Part 1 answers:<br />

1. Insoluble means that no amount of this substance will<br />

dissolve in water at this temperature under these conditions.<br />

Chalk and talc are substances that do no interact with water<br />

molecules. It is possible that the bonds in chalk and talc<br />

molecules are nonpolar.<br />

2. The degree to which a substance is soluble depends on the<br />

nature of the bonds and the size of the molecules in the<br />

substance. Molecules of sugar, salt, and baking soda are<br />

different with respect to the nature of the bonds and sizes of<br />

these molecules. Therefore, these molecules will each<br />

dissolve in water in different ways and to different degrees.<br />

3. 205 g<br />

4. 190 mL<br />

5. 25 mL<br />

6. 1 g<br />

Part 2 answers:<br />

2. 72 grams<br />

3. 10 ppt<br />

Substance Amount of<br />

substance in<br />

200 mL of water<br />

at 25°C<br />

Saturated,<br />

unsaturated,<br />

or supersaturated?<br />

Table salt (NaCl) 38 grams unsaturated<br />

Sugar (C12H22O11 ) 500 grams supersaturated<br />

Baking soda<br />

(NaHCO3 )<br />

20 grams saturated<br />

Table salt (NaCl) 100 grams supersaturated<br />

Sugar (C12H22O11 ) 210 grams unsaturated<br />

Baking soda<br />

(NaHCO3 )<br />

25 grams supersaturated<br />

Page 47 of 57<br />

Part 3 answers:<br />

1. Graphs:<br />

2. Gases A and B are less soluble in water as temperature<br />

increases. Solid A is more soluble as temperature increases.<br />

Solid B is slightly more soluble as temperature increases.<br />

3. This question may be challenging for students. Temperature<br />

affects gas B most dramatically. The range of solubility<br />

values is large, ranging from one-tenth to five-thousandths.<br />

4. Temperature affects the solubility of solid A more than it<br />

affects solid B.<br />

5. Solid B<br />

6. In fall and winter, when the weather turns cold, the water will<br />

have more dissolved oxygen. During the warmer months of<br />

the spring and summer the amount of dissolved oxygen in the<br />

water will decrease.<br />

<strong>Skill</strong> Sheet 21.2: Salinity and Concentration problems<br />

1. Answers are:<br />

4. Add 420 grams of salt to 1,580 grams of water. If the<br />

Place<br />

Salton Sea<br />

California<br />

Salinity<br />

44<br />

Salt in 1 L<br />

44<br />

Pure water in 1 L<br />

956<br />

dissolved salt does not bring the volume up to 2,000 mL,<br />

make a second batch of solution. Add the second batch to the<br />

first until you have two liters of solution.<br />

5. 6 ppt<br />

Great Salt<br />

Lake Utah<br />

Mono Lake<br />

California<br />

Pacific Ocean<br />

280<br />

210<br />

87<br />

280<br />

210<br />

87<br />

720<br />

790<br />

913<br />

6. 45 grams<br />

7. 2 ppm<br />

8. 5 ppb<br />

9. yes; yes<br />

10. 50,000 times more sensitive


<strong>Skill</strong> Sheet 21.3: Calculating pH<br />

1. Answers:<br />

a. 10 –2 > 10 –3<br />

b. 10 –14 < 10 1<br />

c. 10 –7 = 0.0000001<br />

d. 10 0 < 10 1<br />

2. Answers:<br />

a. acid<br />

b. neutral<br />

c. base<br />

<strong>Skill</strong> Sheet 22.1: Groundwater and Wells<br />

Making predictions:<br />

a. 1,3<br />

b. 3<br />

c. No, because water can’t pass through the cling wrap/foam<br />

layer.<br />

Thinking about what you observed:<br />

a. 1,3; yes<br />

b. 3; yes<br />

c. Aquiclude<br />

d. Aquifer<br />

e. No, because the well is below the aquiclude. Yes,<br />

hypothesis was correct.<br />

<strong>Skill</strong> Sheet 22.2: The Water Cycle<br />

Part 1 answers:<br />

A. condensation<br />

B. precipitation<br />

C. percolation<br />

D. groundwater flow<br />

E1. evaporation<br />

E2. evaporation<br />

F. transpiration<br />

G. water vapor transport.<br />

<strong>Skill</strong> Sheet 24.1: Period and Frequency<br />

1. 0.05 sec<br />

2. 0.005 sec<br />

3. 0.1 Hz<br />

4. 3.33 Hz<br />

5. period = 2 sec; frequency = 0.5 Hz<br />

6. period = 0.25 sec; frequency = 4 Hz<br />

7. period = 0.5 sec; frequency = 2 Hz<br />

Page 48 of 57<br />

3. pH = 4<br />

4. pH 5; weaker acid<br />

5. pH 7; Water is neutral and has an equal number of H+ and<br />

OH- ions.<br />

6. 10 –11 ; base<br />

7. 1 × 10 –8.4<br />

8. The product with lemon juice contains a greater concentration<br />

of acid which would mean that it might be a better cleaning<br />

solution than the cleaner with the weaker acid, vinegar.<br />

f. The surface contamination would move toward well #1. If<br />

well #2 was being pumped, it would not have an effect on<br />

the movement of surface contamination because it is<br />

located below the aquiclude.<br />

g. Well #3 would possibly be able to provide water. It<br />

depends on how low the water table became.<br />

h. It might pull in salt water from the ocean. That is a form<br />

of contamination.<br />

i. Answers will vary. Sample answer: Look up how low the<br />

water table got during the worst drought of the last 100<br />

years. Dig the well a little lower than that level.<br />

Part 2 answers:<br />

1. evaporation—The Sun’s heat provides energy to enable water<br />

molecules to enter the atmosphere in the gas phase;<br />

transpiration—The Sun’s energy makes photosynthesis<br />

possible, which in turn causes plants to release water into the<br />

atmosphere in a process known as transpiration.<br />

2. Wind pushes water in the atmosphere to new locations, so that<br />

the water doesn’t always fall back to Earth as precipitation in<br />

the same spot from which it evaporated.<br />

3. Gravity causes water to run down a mountain toward the<br />

coast, and causes water droplets to fall to Earth as<br />

precipitation. Gravity is also the primary force that moves<br />

water from Earth’s surface through the ground to become<br />

groundwater.<br />

8. Answers are:<br />

a. 5 sec<br />

b. 5 sec<br />

c. 0.2 Hz<br />

9. Answers are:<br />

a. 120 vibrations<br />

b. 2 vibrations<br />

c. 0.5 sec<br />

d. 2 Hz


<strong>Skill</strong> Sheet 24.1: Harmonic Motion Graphs<br />

1. Answers are:<br />

a. A = 5 degrees; B = 100 cm<br />

b. A = 1 second; B = 2 seconds<br />

2. Answers are:<br />

a. Diagram:<br />

<strong>Skill</strong> Sheet 24.2: Waves<br />

1. Diagram:<br />

a. Two wavelengths<br />

b. The amplitude of a wave is the distance that the wave<br />

moves beyond the average point of its motion. In the<br />

graphic, the amplitude of the wave is 5 centimeters.<br />

2. Answers are:<br />

a. Diagram:<br />

<strong>Skill</strong> Sheet 24.2: Wave Interference<br />

1. Diagram:<br />

2. 4 blocks<br />

3. 2 blocks<br />

4. 32 blocks<br />

5. 4 blocks<br />

6. 1 wavelength<br />

7. 8 wavelengths<br />

Page 49 of 57<br />

b. Diagram:<br />

b. Diagram:<br />

3. 10 m/s<br />

4. 5 m<br />

5. 10 Hz<br />

6. frequency = 0.5 Hz; speed = 2 m/s<br />

7. frequency = 165 Hz; period = 0.006 s<br />

8. A’s speed is 75 m/s, and B’s speed is 65 m/s, so A is faster.<br />

9. Answers are:<br />

a. 4 s<br />

b. 0.25 Hz<br />

c. 0.75 m/s<br />

8. A portion of the table and a graphic of the new wave are<br />

shown below. The values for the third column of the table are<br />

found by added the heights for wave 1 and wave 2.<br />

x<br />

(blocks)<br />

Height wave 1<br />

(blocks)<br />

Height wave 2<br />

(blocks)<br />

Height of wave 1<br />

+2 (blocks)<br />

0 0 0 0<br />

1 0.8 2 2.8<br />

2 1.5 0 1.5<br />

3 2.2 –2 0.2<br />

4 2.8 0 2.8<br />

5 3.3 2 5.3<br />

6 3.7 0 3.7<br />

7 3.9 –2 –1.9<br />

8 4 0 4


Question 8 (con’t) 9. The new wave looks like the second wave, but it vibrates<br />

about the position of the first wave, rather than about the zero<br />

line.<br />

<strong>Skill</strong> Sheet 24.3: Decibel Scale<br />

1. Twice as loud.<br />

2. 55 dB<br />

3. Answers are:<br />

a. 80 dB<br />

b. 60 dB<br />

4. Four times louder<br />

<strong>Skill</strong> Sheet 24.3: Human Ear<br />

a. Ear canal—leads to the middle ear.<br />

b. Eardrum—vibrates as the sound waves reach it.<br />

c. Semicircular canals—balance.<br />

d. Cochlea—a spiral-shaped, fluid-filled cavity that contains<br />

nerve endings and is essential to interpreting sound waves.<br />

<strong>Skill</strong> Sheet 24.3: Waves and Energy<br />

1. Most of the stone’s kinetic energy is converted into water<br />

waves and the waves carry that energy away from where the<br />

stone landed.<br />

2. The frequency of the jump rope increases and the energy<br />

expended by Ian and Igor increases for this demonstration.<br />

3. The 100-hertz wave has more energy.<br />

4. During a hurricane, waves have more energy as indicated by<br />

their higher amplitude.<br />

5. A wave that has a 3-meter amplitude has more energy than a<br />

wave that has a 0.03 meter (3 centimeter) amplitude.<br />

<strong>Skill</strong> Sheet 24.3: Standing Waves<br />

1. Answers are:<br />

a. A = 2nd; B = 3rd; C = 1st or fundamental; D = 4th (You<br />

can easily determine the harmonics of a vibrating string<br />

by counting the number of “bumps” on the string. The<br />

first harmonic (the fundamental) has one bump. The<br />

second harmonic has two bumps and so on.)<br />

b. Diagram:<br />

Page 50 of 57<br />

5. Answers are:<br />

a. 30 dB<br />

b. 50 dB<br />

c. 70 dB<br />

e. Malleus—transfers vibrations from the eardrum.<br />

f. Incus—between the malleus and stapes; transmits vibrations<br />

from malleus to stapes.<br />

g. Stapes—vibrates against the cochlea.<br />

6. The low-volume sound has the least amount of energy.<br />

7. Visible light waves are likely to have greater energy.<br />

8. This wave has more energy and a higher frequency than the<br />

other waves. Sketch:<br />

c. A = 1 wavelength, B = 1.5 wavelengths, C = half a<br />

wavelength, D = 2 wavelengths<br />

d. A = 3 meters, B = 10 meters, C = 6 meters, D = 7.5 meters<br />

2. Answers are:<br />

a. Diagram:<br />

b. See answer for (c).


c. See table below:<br />

Harmonic Speed (m/sec) Wavelength<br />

(m)<br />

Frequency (Hz)<br />

1 36 24 1.5<br />

2 36 12 3<br />

3 36 8 4.5<br />

4 36 6 6<br />

5 36 4.8 7.5<br />

6 36 4 9<br />

d. The frequency decreases as the wavelength increases.<br />

They are inversely proportional.<br />

<strong>Skill</strong> Sheet 25.1: The Electromagnetic Spectrum<br />

1. Green<br />

2. Green<br />

3. 400 × 10 –9 m or 4.0 × 10 –7 m<br />

4. 517 × 10 12 Hz or 5.17 × 10 14 Hz<br />

5. 652 × 10 –9 m or 6.52 × 10 –7 m<br />

6. 566 × 10 12 Hz or 5.66 × 10 14 Hz<br />

λ<br />

1<br />

f<br />

2<br />

7. The answer is: ----- = ----<br />

λ 2<br />

f 1<br />

8. λ = 0.122 meter or 1.22 × 10 –1 m<br />

9. λ = 3.3 meters<br />

<strong>Skill</strong> Sheet 25.2: Color Mixing<br />

1. The differences include: (1) for RGB the primary colors are<br />

red, green, and blue and for CMYK the primary colors are<br />

cyan, magenta, and yellow, (2) for RGB black is the absence<br />

of light but for CMYK black is an added pigment, and (3) to<br />

make white with RGB you mix the three primary colors and<br />

to make white in CMYK you omit pigment (or you need a<br />

special white pigment to make white, as in making white<br />

paint).<br />

2. (a) Your eye sees red. (b) Your eye sees magenta. (c) Your eye<br />

sees green. (d) Your eye sees cyan.<br />

3. The blue and red photoreceptors would be stimulated to see<br />

purple. For purple, the signal for blue would be stronger.<br />

Note: This can be tested for purple or any color by adjusting<br />

the RGB intensity for font color in a word processing<br />

program.<br />

4. To see yellow, the red and green photoreceptors of your eyes<br />

are stimulated.<br />

5. By reflecting most of the light that strikes it, all the colors of<br />

visible light reach your eyes and all three photoreceptors (red,<br />

green, and blue) are stimulated so that your brain interpret the<br />

color as being white.<br />

6. (a) All three colors of light would be mixed, (b) There would<br />

be an absence of light, (c) green and blue light would be<br />

mixed in equal amounts, (d) red and green light would be<br />

mixed in equal amounts.<br />

7. (a) red light is reflected and all the other colors are absorbed,<br />

(b) red light is reflected and no colors of light are absorbed (c)<br />

blue light is absorbed and no light is reflected so that the<br />

apple appears black.<br />

Page 51 of 57<br />

e. See table below.<br />

Harmonic Speed (m/sec) Wavelength<br />

(m)<br />

Frequency (Hz)<br />

1 36 12 3<br />

2 36 6 6<br />

3 36 4 9<br />

4 36 3 12<br />

5 36 2.4 15<br />

6 36 2 18<br />

f. The shorter rope resulted in harmonics with shorter<br />

wavelengths. The second harmonic on the short rope is<br />

equivalent in terms of wavelength and frequency to the<br />

4th harmonic on the longer rope.<br />

10. f = 6 × 10 16 Hz<br />

11. 1.0 × 10 15 Hz<br />

12. 1.0 × 10 –4 m<br />

13. Answers are:<br />

a. f = 3 × 10 19 hertz<br />

b. It is the minimum frequency.<br />

14. radio waves, microwaves, infrared, visible, ultraviolet,<br />

X rays, and gamma rays<br />

15. gamma rays, X rays, ultraviolet, visible, infrared,<br />

microwaves, and radio waves<br />

8. Table answers:<br />

CMYK color model<br />

Mixed colors Reflected color Which colors of light are absorbed?<br />

magenta + red green absorbed by magenta<br />

yellow<br />

blue absorbed by yellow<br />

yellow + green blue absorbed by yellow<br />

cyan<br />

red absorbed by cyan<br />

cyan +<br />

magenta<br />

blue red absorbed by cyan<br />

green absorbed by magenta<br />

9. If magenta and cyan paint are mixed, you get blue paint.<br />

10. (a) The color green is made by mixing yellow and cyan.<br />

(b) The green photoreceptors in your eyes are stimulated by<br />

this color combination and the brain interprets the color as<br />

green.<br />

(c) Sample diagram:


<strong>Skill</strong> Sheet 25.2: The Human Eye<br />

a. Cornea—refracts and focuses light.<br />

b. Iris—pigmented part of the eye that opens or closes to change<br />

the size of the pupil.<br />

c. Ciliary muscles—contract to change the shape of the lens.<br />

d. Sclera—outer protective covering.<br />

e. Vitreous humor—liquid inside of the eye.<br />

f. Optic nerve—transmits signals from the retina to the brain.<br />

<strong>Skill</strong> Sheet 25.3: Measuring Angles<br />

Answers are:<br />

Letter Angle Letter Angle<br />

A 56° J 153°<br />

<strong>Skill</strong> Sheet 25.3: Using Ray Diagrams<br />

1. A is the correct answer. Light travels in straight lines and<br />

reflects off objects in all directions. This is why you can see<br />

something from different angles.<br />

2. C is the correct answer. In this diagram, when light goes from<br />

air to glass it bends about 13 degrees from the path of the light<br />

ray in air. The light bends toward the normal to the air-glass<br />

surface because air has a lower index of refraction compared<br />

to glass. When the light re-enters the air, it bends about<br />

13 degrees away for the light path in the glass and away from<br />

the normal.<br />

As a ray of light approaches glass at an angle, it bends<br />

(refracts) toward the normal. As it leaves the glass, it bends<br />

away from the normal. However, if a ray of light enters a<br />

piece of glass perpendicular to the glass surface, the light ray<br />

will slow, but not bend because it is already in line with the<br />

normal. This happens because the index of refraction for air is<br />

lower than the index of refraction for glass. The index of<br />

refraction is a ratio that tells you how much light is slowed<br />

when it passes through a certain material.<br />

<strong>Skill</strong> Sheet 25.3: Reflection<br />

1. Diagram at right:<br />

2. The angle of reflection will<br />

be 20 degrees.<br />

3. Each angle will measure 45<br />

degrees.<br />

B 110° K 131°<br />

C 10° L 148°<br />

D 96° M 81°<br />

E 167° N 90°<br />

F 122° O 73°<br />

G 34° P 27°<br />

H 45° Q 139°<br />

I 19°<br />

Page 52 of 57<br />

g. Retina—thin layer of cells in the back of the eye that converts<br />

light into nerve signals.<br />

h. Choroid—provides oxygen and nutrients to the retina.<br />

i. Lens—refracts and focuses light.<br />

j. Aqueous humor—liquid in the front part of the eye.<br />

k. Pupil—opening in the iris that controls the amount of light<br />

entering the eye.<br />

3. A is the correct answer. Light rays that approach the lens that<br />

are in line with a normal to the surface pass right through,<br />

slowing but not bending. This is what happens at the principal<br />

axis. However, due to the curvature of the lens, the parallel<br />

light rays above and below the principal axis, hit the lens<br />

surface at an angle. These rays bend toward the normal (this<br />

bending occurs toward the fat part of the lens) and are focused<br />

at the lens’ focal point. The rays diverge (move apart) past the<br />

focal point.<br />

4. Diagram:<br />

4. Diagram at right:<br />

5. The angle is 72 degrees.<br />

Therefore, the angles of<br />

incidence and reflection<br />

will each be 36 degrees.<br />

6. The angles of<br />

incidence and<br />

reflection at point A<br />

are each 70 degrees;<br />

the angles of incidence<br />

and reflection at point<br />

B are each 21 degrees.


<strong>Skill</strong> Sheet 25.3: Refraction<br />

Part 1 answers:<br />

1. The index of refraction will never be less than one because<br />

that would require the speed of light in a material to be faster<br />

than the speed of light in a vacuum. Nothing in the universe<br />

travels faster than that.<br />

2. The index of refraction for air is less than that of glass<br />

because a gas like air is so much less dense than a solid like<br />

glass. The light rays are slowed each time they bump into an<br />

atom or molecule because they are absorbed and re-emitted<br />

by the particle. A light ray in a solid bumps into many more<br />

particles than a light ray traveling through a gas.<br />

3. water: 2.26 × 10 8 ; glass: 2.0 × 10 8 ; diamond; 1.24 × 10 8<br />

4. speed up<br />

<strong>Skill</strong> Sheet 25.3: Drawing Ray Diagrams<br />

1. Diagram:<br />

The image<br />

is inverted<br />

as<br />

compared<br />

with the<br />

object.<br />

<strong>Skill</strong> Sheet 26.1: Astronomical Units<br />

1. 9.53 AU<br />

2. 0.72 AU<br />

3. 58 million kilometers<br />

4. 2.87 billion kilometers<br />

5. Less. The moon is not nearly as far from Earth as the Sun.<br />

Page 53 of 57<br />

5. slow down<br />

Part 2 answers:<br />

1. The light ray is moving from low-n to high-n so it will bend<br />

toward the normal.<br />

2. The light ray is moving from high-n to low-n so it will bend<br />

away from the normal.<br />

3. The difference in n from diamond to water is 1.09 while the<br />

difference from sapphire to air is 0.770. The ray traveling<br />

from diamond to water experiences the greater change in n so<br />

it would bend more.<br />

4. From left to right, material B is water, emerald, helium, cubic<br />

zirconia.<br />

2. Diagram:<br />

3. A lens acts like<br />

a magnifying<br />

glass if an<br />

object is placed<br />

to the left of a<br />

converging lens<br />

at a distance<br />

less than the<br />

focal length.<br />

The lens bends the rays so that they appear to be coming from<br />

a larger, more distant object than the real object. These rays<br />

you see form a virtual image. The image is virtual because the<br />

rays appear to come from an image, but don’t actually meet.<br />

6. Uranus<br />

7. Mercury<br />

8. Saturn<br />

9. Saturn<br />

10. yes<br />

<strong>Skill</strong> Sheet 26.1: Gravity Problems<br />

Table 1 answers: 1. 9.5 pounds on Neptune<br />

Planet Force of gravity in Value compared to 2. 1,029 newtons on Saturn<br />

Newtons (N)<br />

Earth’s gravity 3. The baby weighs 44.1 Newtons on Earth which is equal to 9.8<br />

Mercury<br />

Venus<br />

Earth<br />

3.7<br />

8.9<br />

9.8<br />

0.38<br />

0.91<br />

1<br />

pounds.<br />

4. Venus, Jupiter, Neptune, Pluto, then Saturn<br />

5. Answer:<br />

Mars<br />

Jupiter<br />

Saturn<br />

3.7<br />

23.1<br />

9.0<br />

0.38<br />

2.36<br />

0.92<br />

−11<br />

2 24 26<br />

⎛6.67× 10 Nm i<br />

⎞(6.4×<br />

10 )(5.7× 10 )<br />

Gravity = ⎜ 2 ⎟<br />

11 2<br />

⎝ kg ⎠ (6.52× 10 )<br />

Uranus 8.7 0.89<br />

17<br />

= 5.72× 10 N<br />

Neptune 11.0 1.12<br />

Pluto 0.6 0.06<br />

<strong>Skill</strong> Sheet 26.1: Universal Gravitation<br />

1. F =9.34× 10 –6 N. This is basically the force between you<br />

and your car when you are at the door.<br />

2. 5.28 × 10 -10 N<br />

3. 4.42 N<br />

4. 7.33 × 10 22 kilograms<br />

5. Answers are:<br />

a. 9.8 N/kg = 9.8 kg-m/sec 2 -kg = 9.8 m/sec 2<br />

b. Acceleration due to the force of gravity of Earth.<br />

c. Earth’s mass and radius.<br />

6. 1.99 ×10 20 N<br />

7. 4,848 N<br />

8. 3.52 × 10 22 N


<strong>Skill</strong> Sheet 26.1: Nicolaus Copernicus<br />

1. Because he was from a privileged family, young Copernicus<br />

had the luxury of learning about art, literature, and science.<br />

When Copernicus was only 10 years old, his father died.<br />

Copernicus went to live his uncle who was generous with his<br />

money and provided Copernicus with an education from the<br />

best universities. Copernicus lived during the height of the<br />

Renaissance period when men from a higher social class were<br />

expected to receive well-rounded educations.<br />

2. Copernicus’ uncle, Lucas Watzenrode, was a prominent<br />

Catholic Church official who became bishop of Varmia in<br />

1489. After Copernicus finished four years of study at the<br />

University of Krakow, Watzenrode appointed Copernicus a<br />

church administrator. Copernicus used his church wages to<br />

help pay for additional education. While studying at the<br />

University of Bologna, Copernicus’ passion for astronomy<br />

grew under the influence of his mathematics professor,<br />

Domenico Maria de Novara. Copernicus lived in his<br />

professor’s home where they spent hours discussing<br />

astronomy.<br />

3. Copernicus examined the sky from a tower in his uncle’s<br />

palace. He made his observations without any equipment.<br />

4. Prior to the 1500s, most astronomers believed that Earth was<br />

motionless and the center of the universe. They also thought<br />

that all celestial bodies moved around Earth in complicated<br />

patterns. The Greek astronomer Ptolomy proposed this<br />

geocentric theory more than 1000 years earlier.<br />

<strong>Skill</strong> Sheet 26.1: Galileo Galilei<br />

1. “On Motion” described how a pendulum’s long and short<br />

swings take the same amount of time.<br />

2. Galileo’s many inventions include the thermometer, water<br />

pump, military compass, microscope, telescope, and<br />

pendulum clock. Information and illustrations of the<br />

inventions can be found using the Internet or library.<br />

3. Galileo observed the motion of Jupiter’s moons and realized<br />

that despite what Ptolemy said, heavenly bodies do not<br />

revolve exclusively around Earth. He also realized that his<br />

observation of the phases of Venus showed that Venus was<br />

revolving around the sun, not around Earth. Galileo therefore<br />

<strong>Skill</strong> Sheet 26.1: Johannes Kepler<br />

1. Copernicus’ idea that the sun was at the center of the solar<br />

system was revolutionary because people believed Earth was<br />

the center of the universe.<br />

2. Brahe helped Kepler make his important discoveries in<br />

several ways. Brahe invited Kepler to come and work with<br />

him. He asked Kepler to solve the problem of Mars’ orbit.<br />

When Brahe died, Kepler gained all of his observational<br />

records. Kepler also got Brahe’s job.<br />

3. Kepler used mathematics to solve problems in astronomy. For<br />

this reason, Kepler is considered a theoretical positional<br />

astronomer. Brahe was an observational astronomer. He made<br />

and recorded the motion of planets and the stars in the night<br />

sky without a telescope. Galileo was also an observational<br />

astronomer. He used and improved the telescope, but he was<br />

not a mathematician.<br />

4. Kepler’s discovery that Mars traveled in an elliptical orbit<br />

was different than Copernicus’ theory which said planets<br />

traveled in circular orbits.<br />

5. Kepler’s three laws of planetary motion are:<br />

Planets orbit the sun in an elliptical orbit with the sun in<br />

one of the foci.<br />

Page 54 of 57<br />

5. Copernicus believed that the universe was heliocentric (suncentered),<br />

with all of the planets revolving around the sun. He<br />

explained that Earth rotates daily on its axis and revolves<br />

yearly around the sun. He also suggested that Earth wobbles<br />

like a top as it rotates. Copernicus’ theory led to a new<br />

ordering of the planets. In addition, it explained why the<br />

planets farther from the sun sometimes appear to move<br />

backward (retrograde motion), while the planets closest to the<br />

sun always seem to move in one direction. This retrograde<br />

motion is due to Earth moving faster around the sun than the<br />

planets farther away.<br />

6. At the time, Church law held great influence over science and<br />

dictated a geocentric universe.<br />

7. The Copernicus Satellite, or Orbiting Astronomical<br />

Observatory 3 (OAO-3) was a collaborative project of both<br />

the United States’ National Aeronautics and Space<br />

Administration (NASA) and the United Kingdom’s Science<br />

and Engineering Research Council (SERC). The satellite<br />

operated from August 1972 to February 1981. The main<br />

experiment on the satellite was a Princeton University<br />

ultraviolet (UV) telescope. An x-ray astronomy experiment<br />

created by the University College London/Mullard Space<br />

Science Laboratory was also onboard. The Copernicus<br />

Satellite gathered a series of high-resolution ultraviolet<br />

spectral scans of over 500 objects, most of them being bright<br />

stars.<br />

concluded that Copernicus’ assertion that the sun, not Earth,<br />

was at the center must be correct.<br />

4. Answers will vary. Students might suggest that Galileo use a<br />

less abrasive approach to convince people that the Copernican<br />

view is correct.<br />

5. Galileo’s telescope is the most likely student response,<br />

because it so profoundly changed our understanding of the<br />

solar system. However, students may choose another<br />

invention as long as they provide valid reasons for their<br />

decision.<br />

The law of areas says that planets speed up as they travel<br />

in their orbit closer to the sun and they slow down as they<br />

travel in their orbit farther away from the sun.<br />

The harmonic law says that a planet’s distance from the<br />

sun is mathematically related to the amount of time it<br />

takes the planet to revolve around the sun.<br />

6. Three examples of a paradigm shift:<br />

Copernicus’ theory that the sun and not Earth was the<br />

center of the solar system.<br />

Kepler’s discovery that planets orbit the sun in an<br />

elliptical and not a circular path.<br />

Newton’s laws of gravitational attraction.


<strong>Skill</strong> Sheet 26.1: Measuring the Moon’s Diameter<br />

Part 1 answers:<br />

There are no questions to answer for Part 1.<br />

Part 2 answers:<br />

1. AC = 6 cm<br />

AD = 6 cm<br />

AB = 2 cm<br />

AE = 2 cm<br />

BE = 2 cm<br />

CD = 6 cm<br />

2. Distance AB is 1/3 of the distance AC<br />

3. Distance BE is 1/3 of the distance CD<br />

4. If a triangle is drawn inside a larger triangle so that they share<br />

the same vertex and have bases that are parallel, then the sides<br />

<strong>Skill</strong> Sheet 26.2: Benjamin Banneker<br />

1. An understanding of gear ratios was necessary for building<br />

the clock. He used geometry skills to figure out how to create<br />

a large-scale model of each tiny piece of the watch he<br />

examined.<br />

2. Personal strengths identified from the reading include strong<br />

spacial skills (building the clock), creativity and problem<br />

solving skills (irrigation system), curiosity and attention to<br />

detail (astronomical observations, cicada observations, and<br />

almanac), concern for others (letter to Jefferson).<br />

3. Dates are as follows:<br />

a. 1863<br />

b. 1865<br />

c. 1920<br />

d. 1954<br />

4. Any three of the following answers is correct. Banneker’s<br />

accomplishments include:<br />

a. Designed an irrigation system<br />

b. Documented cycle of 17-year cicada<br />

c. Published detailed astronomical calculations in popular<br />

almanacs<br />

<strong>Skill</strong> Sheet 26.3: Touring the Solar System<br />

Part 1 answers:<br />

Legs of the trip Distance Hours Days Years<br />

travelled for<br />

each leg (km)<br />

travelled travelled travelled<br />

Earth to Mars 78,000,000 86,666 3,611 9.9<br />

Mars to Saturn 1,202,000,000 1,335,600 55,648 152<br />

Saturn to Neptune 3,070,000,000 3,411,100 142,130 389<br />

Neptune to Venus 4,392,000,000 4,880,000 203,330 557<br />

Venus to Earth 42,000,000 46,667 1,944 5.3<br />

Part 2 answers:<br />

1.<br />

2.<br />

8 glasses 3,611 days = 28,888 glasses of water<br />

1 day ×<br />

2, 000 food calories<br />

×<br />

203, 330 days =<br />

1 day<br />

406,660,000 food calories<br />

<strong>Skill</strong> Sheet 27.1: The Sun: A Cross-Section<br />

A. Corona<br />

B. Chromosphere<br />

C. Photosphere<br />

Page 55 of 57<br />

and base of the small triangle will be proportional to the sides<br />

and base of the large triangle.<br />

Part 3 answers:<br />

Answers will vary. A string distance of about 1 meter will<br />

yield good results.<br />

Part 4 answers:<br />

1. A and D are the same, but D is most helpful because it is set<br />

up with the unknown in the numerator.<br />

2. Answers will vary. A string distance of about 1 meter should<br />

give a value close to the accepted moon diameter; 3,476,000<br />

meters.<br />

3. The semi-circle diameter is the base of the small triangle; the<br />

base of the large triangle is what we are solving for: the<br />

d. Served as surveyor of territory that became Washington<br />

D.C.<br />

5. Banneker evidently had a strong innate curiosity about the<br />

natural world. He was passionate about improving the welfare<br />

of the black men and women in the United States and his<br />

letter to Jefferson stated that he hoped his scientific work<br />

would be seen as proof that people of all races are created<br />

equal.<br />

6. Banneker’s puzzles can be found on several web sites. Using<br />

an Internet search engine, look for “Benjamin Banneker” +<br />

puzzle. Some of the sites publish the answers while others do<br />

not. Here is one of Banneker’s puzzles taken from the web<br />

site www.thefriendsofbanneker.org. Note that the puzzle was<br />

written in the 1700’s and is from Banneker’s personal<br />

journals.<br />

THE PUZZLE ABOUT TRIANGLES<br />

“Suppose ladder 60 feet long be placed in a Street so as to<br />

reach a window on the one side 37 feet high, and without<br />

moving it at bottom, will reach another window on the other<br />

side of the Street which is 23 feet high, requiring the breadth<br />

of the Street.” [No solution recorded in historic records.]<br />

3. Pack foods high in fat for the journey because you get more<br />

calories per gram than from proteins and carbohydrates and<br />

you want a payload minimum.<br />

4. Your entire trip will take 1,113 years, so you will need that<br />

many turkeys.<br />

Part 3 answers:<br />

1. Jupiter; it has 39 moons.<br />

2. Venus has the hottest surface temperature; Neptune has the<br />

coldest surface temperature.<br />

3. Venus; it takes 243 Earth days to rotate once around its axis.<br />

4. Jupiter has the shorter day; it takes 0.41 Earth days to rotate.<br />

5. Jupiter; it has the strongest gravitational force. You will<br />

weigh 2.36 times your Earth weight in Newtons.<br />

6. Jupiter; it has the largest diameter of 142,796 km.<br />

7. Jupiter; it has the strongest gravitational force, therefore the<br />

spaceship must orbit at a fast speed to balance the<br />

gravitational force pulling the spaceship towards Jupiter’s<br />

surface.<br />

D. Convection zone<br />

E. Radiation zone<br />

F. Core


<strong>Skill</strong> Sheet 27.1: Arthur Walker<br />

1. You may wish to have students compare and contrast their<br />

definitions with those of a student who used a different<br />

source. Discuss with the class the value of using a variety of<br />

sources and the importance of crediting these sources.<br />

2. Walker didn’t allow prejudice to dissuade him from pursuit of<br />

his goals. As a result he made important contributions to<br />

science and society. He also spent time and energy helping<br />

other members of minority groups achieve their own goals.<br />

3. A spectrometer separates light into spectral lines. Each<br />

element has its own unique pattern of lines, so scientists use<br />

the patterns to identify the ions in the corona. Temperature<br />

can be determined by the colors seen in the corona. For<br />

example, red indicates cooler areas, while bluish light<br />

indicates a very hot area.<br />

4. Magazines and journals that may have one of Walker’s<br />

photographs can be found at public and university libraries.<br />

You might suggest that students contact a reference librarian<br />

for assistance.<br />

5. The committee found that the accident was caused by a<br />

failure in a seal of the right solid rocket booster. They also<br />

made nine specific recommendations of changes to be made<br />

<strong>Skill</strong> Sheet 27.2: The Inverse Square Law<br />

1. 0.25 W/m 2<br />

2. one-ninth<br />

3. 24,204 km (four times the original distance)<br />

<strong>Skill</strong> Sheet 28.1: Scientific Notation<br />

1. Answers are:<br />

a. 122,200<br />

b. 90,100,000<br />

c. 3,600<br />

d. 700.3<br />

e. 52,722<br />

<strong>Skill</strong> Sheet 28.1: Understanding Light Years<br />

1. 5.7 × 1013 km<br />

2. 4.3 × 1019 km<br />

3. 3.8 × 10 10 km<br />

4. 5,344 ly<br />

5. 1.16 × 10 –12 ly<br />

6. 1.16 ly<br />

<strong>Skill</strong> Sheet 28.1: Parsecs<br />

1. 1.84 pc<br />

2. 1.38 × 106 pc<br />

3. 1.23 × 10-3 pc<br />

4. 2.55 × 101 pc or 25.5 pc<br />

5. 4.85 × 10-4 pc<br />

<strong>Skill</strong> Sheet 28.1: Edwin Hubble<br />

1. Answers are:<br />

spectroscopy—a method of studying an object by<br />

examining the visible light and other electromagnetic<br />

waves it creates.<br />

cosmology—the astrophysical study of the history,<br />

structure, and constituent dynamics of the universe.<br />

Page 56 of 57<br />

to the space shuttle program prior to another flight. These<br />

steps included:<br />

a. Redesign the solid rocket boosters.<br />

b. Upgrade the space shuttle landing system.<br />

c. Create a crew escape system that would allow astronauts<br />

to parachute to safety in certain situations.<br />

d. Improve quality control in both NASA and contractor<br />

manufacturing.<br />

e. Reorganize the space shuttle program to place astronauts<br />

in key decision-making roles.<br />

f. Revoke any waivers to current safety standards, especially<br />

those related to launches in poor weather conditions.<br />

g. Open the review of a mission’s technical issues to<br />

independent government agencies.<br />

h. Set up an extensive open review system to evaluate issues<br />

related to each particular mission.<br />

i. Provide a means of anonymous, reprisal-free reporting of<br />

space shuttle safety concerns by any NASA employee or<br />

contractor.<br />

4. 55.6 N<br />

5. It is 9 times more intense 2 meters away.<br />

6. It is 16 times more intense at 1 meter than at 4 meters away.<br />

2. Answers are:<br />

a. 4.051 × 10 6<br />

b. 1.3 × 10 9<br />

c. 1.003 × 10 6<br />

d. 1.602 × 10 4<br />

e. 9.9999 × 10 12<br />

7. 1,200 ly<br />

8. 8.0478 × 10 14 km<br />

9. 4,280,056,000 km, or 4.28 × 10 9 km<br />

10. 0.000026336 AU, or 2.6 × 10 –5 AU<br />

11. 63,288 AU<br />

6. 54.8 pc<br />

7. 30,000 pc<br />

8. 770,000 pc<br />

9. 26 pc<br />

10. 1.44 × 10 -4 pc<br />

2. Outstanding students from around the world are nominated for<br />

the prestigious Rhodes scholarship. Rhodes Scholars are invited<br />

to study at the University of Oxford in England. Only about 90<br />

students are selected each year. This scholarship is awarded by<br />

the Rhodes Trust, a foundation set up by Cecil Rhodes in 1902.<br />

3. A larger telescope allows more light to be collected by the<br />

mirrors and/or lenses of the telescope. More light allows for a<br />

clearer image, which can then be magnified to show greater<br />

detail.


4. Example answer: Edwin was incredibly excited today. Albert<br />

Einstein came to visit him, and he even thanked him for all of<br />

his hard work. It’s almost unbelievable! The most celebrated<br />

scientist of our lifetime came to visit him. Just to be associated<br />

with Einstein is an honor, let alone be thanked by him. Edwin<br />

works very hard, and he must be very proud of his new<br />

discoveries that have changed the world of astronomy.<br />

<strong>Skill</strong> Sheet 28.2: Light Intensity<br />

1. Example problem: 4.8 W/m 2<br />

2. 0.0478 W/m 2<br />

3. 0.0119 W/m 2<br />

4. If distance from a light source doubles, then light intensity<br />

decreases by a factor of 4. Example: 4 × 0.0119 W/m 2<br />

approximately equals 0.0478 W/m 2 (see questions 2 and 3).<br />

5. Answers are:<br />

<strong>Skill</strong> Sheet 28.2: Henrietta Leavitt<br />

1. Leavitt discovered new variable stars by comparing<br />

photographic plates. The photographs showed the same<br />

regions in the sky, but at different times. This allowed Leavitt<br />

to examine the photos and identify stars that had changed in<br />

size over time.<br />

2. Leavitt studied Cepheid stars and found an inverse relationship<br />

between a star’s brightness cycle and its magnitude. A stronger<br />

star took longer to cycle between brightness and dimness. As a<br />

result, she developed the Period-Luminosity relation.<br />

3. The asteroid is called 5383 Leavitt.<br />

4. The observatory was founded in 1839 and currently conducts<br />

research in astronomy and astrophysics.<br />

5. Example Answers:<br />

Ejnar Hertzsprung:<br />

Danish astronomer born October 8, 1873<br />

Found the distance to the Small Magellan Cloud located<br />

outside of the Milky Way Galaxy in 1913.<br />

<strong>Skill</strong> Sheet 28.2: Calculating Luminosity<br />

1. luminosity = 30 watts<br />

power rating on bulb = 300 watts<br />

2. luminosity = 1 watt<br />

power rating on bulb = 10 watts<br />

<strong>Skill</strong> Sheet 28.3: Doppler Shift<br />

Part 1 answers:<br />

1. Graphic, right<br />

2a. B and D<br />

2b. A and C<br />

2c. C<br />

2d. B<br />

Page 57 of 57<br />

5. The fact that the universe is expanding implies that it must<br />

have been smaller in the past than it is today. The expanding<br />

universe implies that the universe must have had a beginning.<br />

This idea led to the development of the Big Bang theory,<br />

which says that the universe exploded outward from a single<br />

point smaller than an atom into the vast expanse of today’s<br />

universe.<br />

a. 0.005 W/m 2<br />

b. 0.05 W/m 2<br />

c. 0.5 W/m 2<br />

d. 5 W/m 2<br />

6. The watts of a light source and light intensity are directly<br />

related. This means that if you use a light source that has 10<br />

times the wattage, then light intensity will increase 10 times.<br />

Developed the Hertzsprung-Russell diagram which<br />

graphs the magnitude of stars against their surface<br />

temperature or color.<br />

Harlow Shapley:<br />

Astronomer born November 2, 1885 in Nashville,<br />

Missouri<br />

In addition to being an astronomer, Shapley was a writer.<br />

Shapley determined the size of the Milky Way Galaxy.<br />

Edwin Hubble:<br />

Astronomer born November 29, 1889 in Marshfield,<br />

Missouri.<br />

Earned an undergraduate degree in math and astronomy,<br />

and went on to study law.<br />

Developed a classification system for galaxies and created<br />

Hubble’s Law which helped astronomers determine the<br />

age of the universe.<br />

3. Challenge:<br />

luminosity = 1.370 × 10 3 (4π)(1.5 × 10 11 ) 2<br />

= 1.370 × 10 3 (4π)(2.25 × 10 22 )<br />

= 39 × 10 25<br />

= 3.9 × 10 26 watts<br />

Part 2 answers:<br />

1. The star is moving away from Earth at a speed of<br />

5.6 × 10 6 m/s.<br />

2. The star is moving toward Earth at a speed of<br />

8.6 × 10 6 m/s.<br />

3. Galaxy B is moving fastest because it has shifted farther<br />

toward the red (15 nm) than Galaxy A (9 nm).<br />

4. It supports the Big Bang. This theory states that the universe<br />

began from a single point and has been expanding ever since.

Hooray! Your file is uploaded and ready to be published.

Saved successfully!

Ooh no, something went wrong!